2020 Pacific KMLE: 1 Cardiology [1]
 9791158142773

Table of contents :
목차
심혈관계 질환
혈압측정(Blood pressure)
실신(Syncope)
흉통(Chest pain or chest discomfort)
심혈관계의 진찰
General considerations
동맥압과 맥박
경정맥 박동
전흉부 검사(시진 및 촉진)
청진
심음
심잡음
심혈관계 검사
심전도 기초
전해질 이상, 디지탈리스 중독에 의한 심전도 이상 및 기타
심초음파와 심장핵의학
심장도관술(Cardiac catheterization)
부정맥
부정맥의 개요
부정맥의 분류
Sinus node dysfunction
방실차단(AV block)
전도 장애 - 각차단(Bundle branch block)
조기 박동(Premature complexes)
심방세동(AF) / 심방조동(AFL)
방실 접합부 율동(Atrioventricular junctional rhythm)
발작성 심실상성 빈맥(Paroxysmal Supraventricular Tachycardia, PSVT)
조기흥분증후군(Pre-excitation syndrome, WPW syndrome)
심실빈맥(Ventricular tachycardia)
심실세동 & 심실조동
부정맥 한눈에 보기
EKG 보기
심부전
정상 심장 vs. 심부전
심부전의 원인, 증상, 진단
만성 심부전의 치료
Acute heart failure and acute pulmonary edema
디지탈리스 중독(Digitalis intoxication)
Prognosis of heart failure
폐성심 (Cor pulmonale)
판막질환
승모판 협착(Mitral Stenosis, MS)
승모판 역류(Mitral regurgitation)
승모판 탈줄(Mitral valve prolapse)
대동맥판 협착(Aortic stenosis)
대동맥판 역류(Aortic regurgitation)
삼첨판 역류(Tricuspid regurgitation)
폐동맥판 협착(Pulmonary stenosis)
심근질환
심근병증(Cardiomyopathy)
확장 심근병증(Dilated cardiomyopathy)
비대 심근병증(Hypertrophic cardiomyopathy)
제한 심근병증(Restrictive cardiomyopathy)
심근염(Myocarditis)
심막질환
급성 심막염(Acute pericarditis)
심장 눌림증(Cardiac tamponade)
교착 심막염 (Constrictive pericarditis)
심장종양
심장종양(Cardiac tumor)
감염 심내막염
감염 심내 막염(Infective endocarditis) - 분류와 원인
감염 심내막염 - 증상 및 진단
감염 심내막염 - 치료와 예방
죽상경화증
죽상 경화증 (Atherosclerosis)
허혈성 심장질환
허혈성 심장질환의 개관
협심증의 병태생리
허혈성 심장질환의 흉통(Anginal pain)
안정협심증(Stable angina)
운동부하검사의 해석
관상동맥조영술(Coronary angiography)
안정협심증(Stable angina) - 약물치료
변이협심증(Prinzmetal's variant angina)
급성 관상동맥증후군
NSTE - ACS(non ST-segment elevation acute coronary syndrome)
STEMI (ST - segment elevation myocardial infarction) - 진단
Atypical Ml
STEMI - 치료
급성심근경색 - 합병증
급성심근경색 - 예방
심장 응급 질환
심장성 쇼크(Cardiogenic shock)
급성 폐부종(Acute pulmonary edema)
급성심장사(Sudden Cardiac Death, SCD)
고혈압
고혈압(Hypertension)
고혈압 - 치료
치료저항성 고혈압
고혈압 - 고혈압의 응급
대동맥질환
대동맥박리 (Aortic dissection)
다카야수 혈관염 (Takayasu's arteritis)
마르판 증후군(Marfan Syndrome)
R-type
Index

Citation preview

KO REA M E D I C A L 니 CE N S I N G E X A M I N A T I O N

: : : : : : : : ;>=

PACIFIC

(2Q20^\ v

y

대비

KMLE 예 상

: : r

i문 제 풀 이

순 환 기

V'O 'L'U 'M 'E

S3 Pacific Book

N T R O D U C T I O N

의시-국가고시를 준비하는 여러분 모두에게 Pacific KMLE가 수월한 합격의 길잡이가 되기를 소망하며 글을 시작한니다. 여러분이 치르게 될 의시국가고시는 크게 필기시험과 실기시험으로 구성되어 있습니다. 싶기시험은 9월 중순부터 시작하여 약 3개월 동안 진행되며,수험생은 각자 지정된 시험 일자에 ‘한국보건의 료인국가시험원'을 방문하여 CPX(Clinical Performance Examination) 6문항,ᄋSCE(ᄋbjective Structured Clinical Examination) 6문항의 시험음 보게 됩니다. CPX 시험에서는 특정 주소로 내원한 모의환자와 면담을 하고 환자

-의사라는 상황 아래 신체검진 및 환자교육을 10분 동안 진행히-게 됩니다. OSCE 시험에서는 의사라면 기본 적으로 할 줄 일-이아: 하는 숨기들을 5분 동안 모형이나 모의환자에게 시행하게 됩니다. 필기시험은 의학총론, 의학각론, 보건관계의료법규 3개 과목으보 구성되어 있으며 이틀에 걸쳐 5교시 동안 총 360문항읍 풀게 됩니디-. 1교시에는 보건관-계의료법규 20문제와 의학총론 6()문제가 출제되며 나머지 교시들 에는 의학각론 문제가 70문제씩 출제됩니다. 하지만 실제로 의학총론과 의학각론은 명확하게 구분되어 있지 않 은 경우가 많습니다. 필기시험의 경우에는 전 괴-목 총점 60% 이상(216문항),각 과목 점수 40% 이상일 경우 합격입니다. 보건관계의료법규의 경우 문제수가 20문제로 적기 때문에 과락(8문제 미만) 되지 않도록 각별히 주의해야 합니다. 2019년도 필기시험은 전반적으로 평이했던 2018년 시험에 비해 난이도가 어렵고 생소한 문제가 출제되었다

는 의견이 많았습니디-. 최근 국가고시 출제 경향을 보더라도 국가고시 문제와 답안이 공개되면서 조금씩 난이 도가 높아지고 있는 추세이며, 산세로 점차 악기형 문항을 없애고 기존의 집병중심에서 임상표현중신으로 문제 를 히m 하는 방식의 문항이 늘어나고 있습니다. 이에 따라 기존의 기출 문제듀은 문제를 보고 바로 답읍 떠올릴 수 있는 “straightforward”한 문제들이 많았 던 반면에,최근에는 여러 단계의 사고를 거쳐 ''생각을 해야 풀리는'' 문제의 비중이 늘어나고 있습니다. 이름테 면' 점차 질환이 증상 및 징후. 검사 소견 등을 통해 간접적으로 표현되는 반면. 전형적인 건사나 치료보다는 이것이 불가능한 때 차선책-0 로 선택할 수 있는 처치와 그차 약제 능읍 꼼꼼히 공부하고 정리해 놓0빠 풀 수 있는 문제들이 舍제되고 있습니다. 이럴 때인수록 중요한 짓은, 지엽적이고 세세한 지식에 얽매이기보다 기본에 충실하여 꼭 악아두어야 한 필 수적인 의학지식을 확실하게 공부하는 것입니다. 이는 의사국가고시의 목표가 예비 의사들이 후!;자가 호소하는 증상. 나타나는 징후. 기본적인 검사 소견을 통해 기"능성이 높은 진단명을 떠올리고 가장 적합한 치료를 선택하 는 능력을 갖추었는지 평가하는 것이기 때문입니다. 즉. 실제 암상 상황에서 접하기 쉬운 질병 또는 즉각적인 처치가 요구되는 싱황에서 꼭 알아야 할 감별진단 및 치료방인-을 확실하게 제시할 수 있어야 합니다. 2020년 대비 Pacific KMLE에서는 침차 변회-해가는 의사국가고시 유형에 맞게 문제륜 편집하여 해설하였고

올해 분제에서 추가된 이론 부분의 내용을 강화하여 앞으로 어떤 부분에서 어떤 유형의 분제가 나오더라도 여 러분듬이 쉽게 풀어낸 수 있도목 하였습니다. 또한 각 파트름 시직-하면서 최근 경향 및 중요한 부분에 대해 언급하여 해당 파트에서 중요하게 공부해야한 내용이 무엇인지 파악함 수 있도록 하였습니다. 2()2»년 대비 Pacific KMLE는 최근 시험 경향의 분석을 통해 합격과 더불어 고득점을 원하시는 분들을 위해

다읍과 같은 부분에 중점을 두고 개정하였습니다.

머。리。 말 1. 국가고시 출제 유형이 변화함에 따라 점차 기존에 출제된 족보내용만 외우는 공부가 아닌,문제를 통해 impression을 잡은 후 진단에 필요한 검사를 떠올리고,해당 상황에서 가장 적절한 치: ■편 선택하는 논리적

사고가 중요해지고 있습니다. 이에 따라 2020년 대비 Pacific KMLE는 ‘이론학습 후 문제해결’ 순서를 유지 하여 이론을 충분히 학습한 후 문제풀이를 통해 이를 적용해보고 복습하면서 문제해결 능력을 다질 수 있도 록 하였습니다. 또한 이론학습 부분의 질적인 향상을 위해 정신과 DSM-V,응급의학과 2015 AHA guideline, 외과총론 2016 Sepsis-3 등 최신 가이드라인을 반영하였습니다. 그리고 검사와 치료들 단순히 나열하는 방식에서 벗어

나 특정 상황에서 어떠한 처치를 해야 하는지에 대한 기준을 명확하게 제시하기 위해 노력하였습니다. 추가 적으로 국가고시 문제에서 시용되는 최신 한글용어를 사용하여 실제 시험장에서 문제를 풀 때 의학용어 때 문에 정답선택에 차질이 생기지 않도록 하였습니다. 2. 출제빈도와 임상적 중요성을 고려하며 ‘ 중요항목’을 반영하였습니다. 2016년도 국가고시부터 기본항목이

폐지되고 임상표현 중심의 평가목표가 제시되면서 기존에 출제되지 않았던 부분에서도 문제가 많이 출제 되 고 있습니다. 이에 각 파트의 앞 부분에 출제경향과 중요사항을 제시하여 공부할 때 어떤 부분에 집중해야 하는지 파악할 수 있도록 히-였고,그동안 출제빈도가 낮았던 항목도 최근 국가고시 의 출제 경향을 고려하여 '중요항목’으로 표시했습니다. ‘중요항목’을 완벽하게 숙지하고 시험장에 들어간다면 합격을 넘어서 충분히

고득점까지 노려볼 수 있을 것입니다. 3. 국가고시 출제 경향을 이해하고 중요한 부분에 더욱 집중할 수 있도록 기출문제에 A, B, C 분류를 적용하였

습니다. 최근에 국가고시에 출제된 최신 경향이 반영된 문제를 A로 분류하였고,B와 C는 편집자의 재량에 따라 중요도와 출제 가능성을 고려하여 분류하였습니다. 특히 각 단원에서 2019년도 의사국가고시와 관련된 문제-ri 가장 앞쪽에 배치하였고 최대한 효율적으로 공부할 수 있도록 최근 출제 경향에 이 벗어나는 문제, 같은 내용이 반복되어 쉽게 풀리는 문제들은 삭제하였습니다. R type 문제들은 이러한 문제들의 출제경향에 대한 감을 잡고 각 과별 내용을 복습하고 정리하는데 도움을 드리고자 뒷 부분에 따로 묶어 놓았으니,R type 문제를 풀어보면서 공부한 내용들을 한 번 더 정리해 보기를 바람니다. 4. 문제에 충실하고 통합적이며,이해하기 쉬운 해설을 달기 위해 노력하였습니다. 또한 필요치 않게 긴 해설을

줄여 가독성을 높였고,최신 문헌을 참고하여 문제에 대해 정확한 해설을 하기 위해 노력하였습니다 \ 않이 바뀌었습니다. ■개열 질홧의 tgEI에리와 역학 등의 세부지식을 물어보면 에 ^과 달리 최근은 환지■가 호소하는 증상을 보고 정멸진단을 떠올린 뒤,적^ ^ 진단과 치료를 생각하는 임상에 면관된 지 식들을 더 묻고 91습니다. 특히 치료의 ᄀ辟 가장 않이 사용하는 일차치료 분41:아니라 이차치료나 상차치료 그리고 에% 법에 대해서도 묻고 있는 ^이 순환기를 비롯한 ^ 반적 국가고시의 출제 정^ 입니다. 순환기의 정우 R type으로



제되는 편이니,더욱더 진1it기준이나 치료법들을 꼼吾건I 않아야 형니다. 올해 국가고시의 정우 순환기 파트에서 총 26문제가 출제되91으며,자주 출제되면 문제 유형이 비교적 않앗습니다. 크 기ᅵ분류해보앗을 때,부왕맥과 심부7a에서 각각 6문제가 출제되있으며,허혈성 심7양질환에서 4'개의 문제가 출 제 도 습 니 다. 이외에 고혈ᄋ抑1서 1문제,대동1객 에서 vg•저I,상양1각 진%에서 4문제, 2문제가 출제되%습니다. 부정맥

팟막 진%oil서 2-£^1,

심근질% 에서

의 경우 진1난을 울어보는 경우가 ?iSJt으나,심 부 ^이 나 허혈성 심질%에서는 구체적

°J. ~B사나 치료를 묻는 경우가 앉았습니다. 각 진흰:마다 어병 임상상을 나타내는지,어1편 ~S사를 시 ^해 야 하는지’ 그리고 적용할 수 있는 치료는 에면 •것이 있는지 d jj/^ ^ o i 부분은 꼭 숙지하고 ~7抓다야^ 습니다. 특히 내 . 외과적 치료가 가능한 질환에 OfdH서는 수술 적용증에 대해서도 확^ O il o j지하고 기써야가습니다 순환기로 국가고시를 시작하는 분이 을

으리라 생 74 향니다. 그리고 방대한 Oj과 지식에 ^ |~ 7 ^ | 좌철을 1과는 분도 않

이라 생 74형니다. 순% 기는 내용이 않고 모든

을 이해하는 정은 어려울 수 9JL지41 막상 국시에 출제된 문제들은

어령지 앞■게 해■걸되는 정우가 앞기 매문에 너부 적정하지

으셔도 필 ᄀ X

습니다. 기본적으로 혀 ^ 째 공부에서는

적0J 내용을 OI6H하는 식으로 공부하시고,두 병째는 주요 질%의 진단과 치료를 정리하고 외우는 식으로 공부하면 좋을 '

길습니다!!

순 환 기 r各





심혈관계 질환 -

-• 9

감염 심내막염..........

345

심혈관계 검사 -

-46

죽상경화증................

358

부정맥 ............

-75

허혈성 심장질환.……

362

심부전............

181

급성 관상동맥증후군 -

393

판막질환 .........

229

심장 응급 질환 ........

441

심근질환.........

285

고혈압 .......................

456

심막질환.........

313

대동맥질환................

477

심장종양.........

342

P A C IF IC

KMLE

심혈관계 질환 순환기의 시작입니다ᅵ 이제부h 시작해 봅시다. 순환기학은 크게 나누면 성원;판기I 진찰과 점사, 부정맥, 심부전, 허혈성 심장7알환, j |

기타 이링게 5'개의 파트로 구성평니다. 이 중 심^ iTMl 진찰과 ~0사는 가장 기초적0d 단우보이며, 동시에 다른 단원들을 공부할 nj) 기본 내용이 되기도 현니다. 너우 꼼꼼하게 보실 필요는 없으니,헌을 WH시고 I&OJ; 하'게 임에-T7I■시기 바협니다. 2019면에는 혈0』측73에 관

j

하여 1-gr^nh 출제되었습니다.

*.....

1~

v»*«**uc^hc -■>•- v'iv» r j* » -

:*.* f: H*«nBfHrtc3• 상지 혈압 > 하지 혈알' Takayasu’s arteritis —►좌우 상지 혈압 차 >10m m E^;

4. 혈압이 실제와 다르게 측정되는 경우 1) 혈압이 실제보다 낮게 측정되는 경우

(1) 청진7ᅵ * 세게 누른 상탠0i 서 혈압을 측정한는 경우 (2) 팔을 심장보다 높게 위치S 을 때 (3) 공7삐

J 의 찰인가 긴 경우

(4) 가면고혈압(Masked hypertension) 2) 혈압이 실제보다 높게 측정되는 경우 (1) 동맥경화증 : 가성고혈압(PseudohypertensiorQ (2) 팔을 지지하지 않고 늘어뜨린 채 측정할 경우 (3) 공기주머니의 길이가 짧은 경우 (4)

백의고혈압(White coat hypertension)

2020년 대비 PACIFIC KMLE

순환기

* ᄈ 이 설제보다 닛계 측정a 는 경우 1) 청진기를 세게 누른 성:태로 측정하면 turbulent flow가 증가하여 Korotkoff음이 늦>11 나타1날 수 91고 이 로 9|dH 혈O』이 10시 5mmHg 정도 1낮>11 측정월 수 있습니다. 2) 팔을 심7당보다 높>11 우ᅵ치시=?1면, 혈0Jj이 심장o)l서 팔 높이77(■지 올라고 만큼 hydrostatic pressure가 I t 소하으로 혈o납이 낮게 측;영평니다ᅵ 아파트 고층에서 수(압이 낮은

을 생각하면 엽습니다.

3) 공기주머니의 길이가 !!• 정우 혈Qti이 실제보다 낮게 측정되는 이유는 임서 설명하였습니다.

완현qfoi 일제보다 높계 측정a 는 경우 1) 동0JP3화증의 정우 혈0』을 측정하는 형 1a의 73화가 말생* ™ a 어 실제 ^ 서망 열류가 차단되고 성ᅵ제 혈0』보다 월씻 높은

내_

보다 높은



으로 측정팅니다(1택작해서 진 Oh 막던다고 생각하

;1이 아니라 원a j 세요), 이를 가성고혈0li(pseudohypertension)이라고 형니다, ' ^ o ^ i 높■게 측정되는 ᄀ 혈0남이 높은 것 아닌가?' 할 수 앗지만,invasive*nil 측정한 혈ot4이 정확한 혈o옴이으로 이에 비해서는 혈법대를 이용해서 측정하는 혈Sioi H 높게 측정된다고 않할 수 있습니다. 2) (2), (3)은 잎서 설1경한

과 반대로 생각하면 평니다.

❖ 백의고^ ^(White coat hypertension)과 가면고연^ M a s k e d hypertension) 1) 진료실형0 이 140/90mmHg 이상이고, 가정혈빕 또는 명균 주'7跑 동 열 ^은 135/85mmHg QlHhRi ^ 우 를 백의 고혈o음이라고 하고, 반대로 진료실혈입은 ^o/gOmmHg

가정혈삠 또는 평균 주'안

활동열0』은 135/85mmHg 이상와 정우를 가면 고혈0和 I라고 형니다. 2) tijj의고혈o납이나 가면고혈o음이 의심되는 ^우 (1) 가정혈와을 측정해보고, 1)에 *H당*J: 다면 (2) 24시I t 활 동 던 ^을 측정형니다. 3) 두 진환 모두 완전히 早6H한 칫은 아닝니다, (1) 실제 고혈0』환자보다는 morbidity가 적고, 약울치료로 얻을 수 있는 이득은 없으나, (2) 혈0』이 정성유! 경우보다는 target organ damage의 위형이 높고,

후 고혈0』으로 진6跑 가 능 ^이

높습니다. (3) [다라서 고혈0』말현과 target organ damage의 '가능성을 영두에 두고, 추적 관 敬 필요가 91습니다. 임상진단학 4만,pp.165〜 166; 이영우 편저, 순환기학,pp.86〜 87 Harrison 2Q만,p. 1669, 1895

( 12)

#

01

문제해설



55세 남자가 집에서 잰 혈압이 높다며 병원에 왔다. 현재 복용하고 있는

약은 없다고 한다. 혈압 126/82mmHg,맥박 50회/ 분,호흡 16회/ 분, 체온 36.4°C이다. 가슴 청진에서 심음과 호흡음은 정상이다. 적어 온 가 정혈압은 다음과 같다. 심전도이다. 검人fe ? 가정혈압(mmHg) : 아침 152/100, 154/102, 저녁 142/94, 137/98

i ᅭL



L

_

심혈곤r계 질환

----------------------cA)

01 2018년 임종평과 2019년 국시에서는 백의 고혈압과 반대로 가정 혈압이 135/95 이상으 로 높은 반면,병원에서 측정한 혈압은 정상 으로 나타나는 가면고혈압이 의심되는 증례가 출제되었습니다. 가정 혈압과 병원에서 측정 한 혈압의 양상이 일치하지 않으면 24시간 활 동혈압을 추가적으로 측정해야 합니다. 참고 로 백의고혈압은 혈압이 정상인 사람에 가깝 지만,가면고혈압은 일반적인 고혈압에 가까 워서 더더욱 추적관찰이 필요합니다. Harrison 20만,p. 1895

L ^ᅳ L .

1)

기립혈압 측정

2) 24시간 활동혈압

3)

목동맥 초음파검사

4) 복부 컴퓨터단층촬영

5) 콩팥혈관 도플러초음파검사

02 실제보다 혈압이 더 높게 측정되는 경우는? 1) 청진기를 세게 누른 상태에서 혈압을 측정하는 경우 2) 혈압대의 감압을 빨리 시키는 경우 3) 혈압대를 심장보다 높게 위치했을 때 4) 성인용 혈압대를 소아에게 이용했을 때 5) 동맥경화증 노인

□ 0 2 ‘혈압’이 무엇인지 생각해볼 수 있었던 좋은 문제입니다. 보통은 측정상의 어려움으로 인 해 혈압대와 수은혈압계를 이용해 혈압을 측 정하지만,가장 정확한 혈압은 arterial line을 잡아서 invasive하게 측정하는 혈압입니다. 이와 비교했을 때 혈압대를 이용한 혈압계의 혈압 측정 원리와 약점이 무엇인지 알아야 풀 수 있었습니다. 자세한 설명은 해설에 추가해 놓았으니 해설을 참조하세요. 2) 적절한 cuff 감압 속도는 2〜 3mmHg/s입 니다. 참고문헌에 정확하게 나와 있자는 않았지만,빠르게 감압하면 혈압의 측정 단위가 커지면서 혈압이 실제와 같거나 낮 게 측정될 것으로 생각됩니다. 5) 가성고혈압(pseudohypertension)에 대한 설명입니다. Harrison 20만,p. 1669 Messerli FH (1986),"Osier's Maneuver, Pseudohypertension, and True Hypertension in the Elderly", Am J Med, p.907



Y

2020년 대비 PACIFIC KMLE

순환기





55세 여자가 건강검진에서 혈압이 높아서 왔다. 가져온 혈압 기록은 다

음과 같았다. 혈압 160/95mmHg, 맥박 70회/ 분, 호흡 18회/ 분, 체온 36.5도였다. 가슴 청진에서 심음과 호흡음은 정상이었다. 검사는? 측정 장소와 시간

혈압(mmHg)

검진기관, 2주 전

150/90

집,기상 후 1시간 이내

130/80

집,취침 전

125/85

1)

맥박파전파속도

2) 사지혈압

3)

24시간 활동혈압

4) 24시간 홀터검사

문제해설

^

0 3 고혈압을 진단큰ᅡ기 위해서 우선적으로 감별해 야 히는 상황 중 하나가 백의고혈압(white coat hypertension)입니다. 검진기관,병원에 서 측정했을 때만 혈압이 140/90 이상으로 나오며 가정혈압은 정상 범위에 있는 경우 백 의고혈압을 의심해야 합니다. 이러한 경우 24 시간 활동혈압을 측정하여 실제로 혈압이 높 은지에 대한 확인이 필요합니다. 백의고혈압은 향후 심장에 대한 합병증이 발 생할 위험이 다른 사람들보다 높아 일반적으 로 지속적인 경고[관찰이 중요합니다. Harrison 20만,p. 1669

5) 운동부하검사

0텬__________________________o 다음 중 혈압 측정 시 실제 혈압보다 낮게 측정되는 경우는? 1) 고혈압 전단계(prehypertension) 2) 팔 둘레에 비해 폭이 좁은 혈압대를 사용한 경우 3) 빗장밑동맥 협착(subclavian artery stenosis) 4) 백의 고혈압(white-coat hypertension) 5) 수면무호흡증

0 4 3) Subclavian artery stenosis가 있는 경우 동측의 팔로 가는 혈류가 줄어들기 때문에 brachial artery에서 혈압을 측정할 경우 실제 혈압보다 낮게 측정됩니다. 1) 고혈압 전단계란 SBP 120〜 139 or DBP 80〜 89인 경우를 지칭하는 것으로 고혈 압 진단기준에는 미치지 못하지만 혈압에 대한 경각심을 가지고 생활습관 교정을 통해 혈압을 낮추어야 할 필요가 있는 단 계를 의미합니다. 2) 팔 둘레에 비해 폭이 좁은 혈압대를 사용 한 경우 실제 혈압보다 높게 측정됩니다. 4) 백의 고혈압이란 집에서 측정한 혈압은 정상이나 병원에서 측정한 혈압은 고혈압 을 보이는 경우를 의미합니다. 실제 혈압 보다 혈압이 높게 측정되는 상황입니다. 5) 폐쇄성 수면무호흡이 혈압을 높일 수 있 다는 논문들이 축적되어 있습니다.

圓 _______________________ ^ᅳ 54세 남자가 건강검진에서 혈압이 높다고 해서 왔다. 혈압 151/94mmHg,

맥박 70회/ 분, 호흡 21회/ 분,체온 36.7도였다. 집에서 아침과 저녁 안정을 추I한 후 측정한 3 일 동안의 평균 혈압은 126/79mmHg였다. 조치는? 1) 경과관찰

2) 혈압강하제

3)

4) 혈장 메타네프린

대동맥조영술

5) 복부 컴퓨터단층촬영

0 5 백의고혈압(White coat hypertension)문제입 니다. 백의 고혈압은 약물치료로 얻을 수 있 는 효과가 없습니다. 하지만 향후 고혈압으로 발전할 수 있기 때문에 지속적으로 경과관찰 을 하는 것이 중요합니다. Harrison 2ᄋ만,p. 1669

ᅳ끝혈관겨r 질환

06

B\ Q \

50세 여자가 건강검진에서 혈압이 높다고 해서 병원에 왔다. 혈압 152/96mmHg, 맥박 80회/ 분,호흡 21회/ 분,체온 36.60C 이다. 키 164cm, 몸무게 57kg 이다. 가슴 청진에서 심음과 호흡음은 정상이다.

조치는? 경 ^^찰

2) 가정혈압측정

운동부하검사

4) 맥박파전파속도

0%

0 6 중년의 여자가 건강검진 상에서 측정된 혈압 이 높아서 내원한 증례입니다. 1회 측정만으 로 고혈압을 진단할 수 없으며,백의고혈압 의 가능성을 배제하기 위해서 보기 중에서는 가정혈압측정을 하는 것이 가장 적절한 답입 니다• 참:0 로 맥박파전파속도(Pulse wave velocity) 는 동맥의 경직도오ᅡ 관련된 검사로 고혈압에 으ᅵ한target organ damage를 나타내는 간접 적인 지표의 하나로 사용할 수 있습니다. Harrison 20판, pp.l669~1670

24시간 홀터검사

07

문제해설





45세 여자가 혈압이 높다고 해서 병원에 왔다. 진료실 혈압은 164/90 mmHg였다. 일주일 후 다시 측정한 진료실 혈압은 170/94mmHg였다.

진료실 밖에서 30분 안정하게 한 후 측정한 혈압은 134/80mmHg였다. 적절한 검사는? 1)

관동맥조영슬

2) 가정혈압 측정

3)

24시간 소변 VMA

4) 운동부하심전도검사

B

다음 중 혈압을 측정하는 방법을 맞게 기술한 것이 아닌 것은? 1) Korotkoff 음이 처음 들리는 지점이 수축기 혈압이다. 2) 측정 띠의 길이는 팔 둘레의 2/3가 적당하다. 3) 30분 이상 커피 섭취와 흡연을 금한 뒤 측정한다. 4) 혈압측정 띠(cuff)는 팔오금오목에서 2 〜 3cm 위에 갑는다. 5) 공기주머니의 중심은 요골동맥(radial artery) 부위에 위치시킨다.

©ANSWER

압을 측정하여 그 평균값을 얻어야 합니다. 또한 진료실에서의 혈압이 가정 또는 일상 활 동 중의 혈압보다 높게 나타나는 현상,즉 백 의고혈압을 주의해야 합니다. 백의고혈압의 경우 진료실이 아닌 상황에서 적어도 두 번 135/85 미만의 혈압이 나와야합니다. 백의고 혈압을 확인하기 위해 가정혈압을 측정해보[야 하겠습니다. Harrison 2Q만,p. 1669 The JNC 7 Report (2003) The JNC 8 Report (2013)

5) 가슴컴퓨터단층혈관조영술

08

0 7 고혈압의 확인을 위해서는 최소 2회 이상 혈

01.② 02.⑤ 03.③ 04.③ 05.① 06.② 07.② 08.⑤

0 8 5) 공기주머니의 중양에 brachial artery가 오 게 합니다. 1) Korotkoff 음이 처음으로 들릴 때의 압력 은 수축기 혈압,Korotkoff 음이 완전히 사라지는 때의 혈압은 확장기 혈압입니다. 2) 공기주머니(inflator bag)의 좁은 쪽(폭)은 팔 둘레의 40%, 긴 쪽(길이)은 팔 둘레의 80% 또는 2/3 정도가 되어야 합니다. 폭 이 너무 좁을 경우 환자의 혈압이 높게 측 정됩니다. 그 반대의 경우도 가능합니다. 3) 커피와 흡연을 30분 이상 삼가고 5분간 안정 후 측정합니다. 4) 팔오금오목(antecubital fossa)으로부터 2.5cm 정도 위에 위치시킵니다. 그래야 청 진기를 댈 수 있겠죠? 임상진단학 4^, pp. 165~ 166

2020년 대비 PACIFIC KMLE

순환기

험S 圓B



^^1 1.

Explanation

실 신 (Syncope) 실신의 정의 1) 뇌혈륜량 감소론 인한 일시적 의식 소실 2) Presyncope : 실신이 일어나기 전의 전구 증상(faintness)

어지럼증(vertigo와는 다름), 열감,식은땀,구역감,눈앞이 하얘지는 현상 등

2. 실신의 원인 및 감별진단



은 어1편 진병의 던 manifestation에 지나지 앞아요. ^

자체를 치료하는 7X이 아니라 그 원인을 찾아서

치료해야 하죠ᅵ 0 ^ 이 단우크에서는 사 심 0ii성 성신과 기립성 저혈0』에 대해 자세히 c찰아보려 협니다. 실/난을 주소로 한 흰:지ᅡ가 R type으로 나오고 %요한

사를 모두 고르라고 할 수 91기 매문에 표 향고헤서 진1단법들을

0 ^1 도륵 ᅵ할시다. 기본적으로 심장의 구조적 문제는 심초음파, 으로 기억하시고 약울력 조사는 기본인거 아시죠?

리듬의 문제는 EKG, 24hr holter monitoring

3. 신경성 원인 1)

신경성 원인 : 부교감신경 1 또는 교감신경 I 로 인한 저혈압으로 실신이 발생 - 부교감신경(미주신경> 1 :살장게 영향을 주어 심장 수축력 I 및 심박수 丄 -

신경 1 :혈관 평활근이 제대로 수축해주지 못하여 혈압 i

(1) 신경심인성 실신(neurocardiogenic syncope) : m/c (JJ Vasovagal syncope, vasodepressor syncope을 모두 포함히는 개념 • Vasovagal syncope : 교감신경은 저하되고,부교감신경은 항진 • Vasodepressor syncope : 교감신경 저하만 있을 때

② 유발인자 :매우 덥거나 혼잡한 환경, 음주, 심한 피로,극심한 통증, 공복 시,오래 서 있을 때, 감정적인 스트레스가 있을 때 ③ 전구 증상 :수초〜수분 간 앉거나 선 자세에서 허약감, 오심, 발한,어지러움,시야 흐려짐, 심계 항진 등이 나타남 ④ 의식 소실의 정도와 기간은 다양할 수 있으며 누운 자세를 취하면 수분 내에 회복,간질 발 작과는 달리 괄약근 기능은 유지 ⑤ 최근 연구에 따르면 단순히 자율신경 기능의 저하에 의한 현상만은 아닌 것으로 보임 ⑥ 진단 : 기립 경사 검사(tilt table test) —신경심인성 실신의 확진검시• 정상인의 경우 기립시킬시 다리 혈관으로 피가 쏠릴 경우 보상기전을 통해 혈압 유지 • 신경심인성 실신의 경우. 혈압과 심박수 감소

⑦ 치료 • 안전한 곳에 눕히고 기도를 확보하고 꽉 끼는 옷을 풀어주는 등의 안전 조치 . 약물치료 - Fludrocortisone - Vasoconstricting agent - /3-blocker (2) 기립성 저혈압(postural hypotension)

① 원인 • 순환 혈액량의 감소 • 혈관운동반사(vasomotor reflex)의 만성적 불안정성

② 눕거나 앉은 자세에서 2 과기 일어설 때 발생 cf. 신경심인성 실신의 경우는 주로 오래 서 있는 자세에서 발생

③ Tilt table test에서 자세 변화에 따른 갑작스런 혈압의 저하를 확인할 수 있음 ④ 노인층의 30%까지 경험,혈압강하제나 항우울증약물(TCA) 복용이 흔한 원인



2020년 대비 PACIFIC KMLE

순환기

⑤ 질단

• 누운 상태에서 5분 후에 측정한 안정 시 혈압에 비해 • 일어선 직후 3분 이내에 바로 측정한 혈압에서 수축기혈압 >20, 이완기혈압 >10m mHg

이상 떨어지는 경우 ⑥ 치료 • 유발 요인(주로 약물)을 제거 • 비 약물 치료 : Compression stocking 착용,침대 머리 높이기,수분 및 염분 섭취량 늘리

기등 • 약물 치료

一 Fludrocortisone - Vasoconstricting agent(midodrine, pseudoephedrine) - 위 약물로 直과 없을 시 추가 치료로 pyridostigmine(cholinesterase inhibitor), yohimbine, desmopressin, erythropoietin을 써 볼 수 있음 (3) 목동맥팽대 과민성(carotid sinus hypersensitivity) : 50세 이상의 남성에서 목동맥팽대 압력수용

기(baroreceptor)를 압박하는 경우 주로 나타난다. 9번 뇌신경(glossopharyngeal nerve)으로 전달 ᅳ 숨뇌를 거쳐 ᅳ 미주신경을 통해 심장으로 신호 전달 ᅳ 동정지,방실차단 유발 (4) 상황성 실신 : 기침,배뇨, 배변, Valsalva maneuver, 삼킴운동(deglutition) 둥의 상황에서 발생 2)

심혈관계 질환 (1) 대부분 부정맥에 의해 발생,HR < 30/min or >180/min인 경우 1회 박출량(stroke volume) 조

절에 의한 심박출량(cardiac output) 유지 실패 ᅳ 뇌 허혈 및 실신(실신 전구 증상이 없는 경우 가 (2) 서맥성 부정맥

① 동정지, 동기능부전증후군(sick sinus syndrome), 방실전도장애(AV block) ② 서맥ᅳ빈맥 증후군(bradycardia-tachycardia syndrome) ③ 약물에 의한 서맥 : antia订hythmics,^-blocker, calcium channel blocker, digoxin (3) 빈맥성 부정맥

① 상심실성 빈맥(supraventricular tachycardia) : 심방조동 / 세동,방실결절회귀(AV node reentry), W PW ^후 군

② 심실빈맥(ventricular tachycardia) : 구조적 심질환01 있을 때 ③ Torsades de pointes : QT간격이 길어지는 조건(long QT syndrome, hypokalemia, hypo­ calcemia -§■) (4) 폐색전,심낭 압전,원발성 폐동맥 고혈압 등

심혈관계 5 ¥ ᅳ

z

r : #

3) 뇌혈관-계 질환

목동맥협착(carotid artery stenosis), 척추기저동맥허혈(vertebrobasilar insufficiency) 등의 경우 다른 요소와 함께 syncope 발생에 원인 인자로 작용 4) 감별 진단(syncope like states) (1) 불안이나 공황 발척•(과호흡으로 재연 가능) (2) 저혈당,경련,급성 출혈 (3) acute intoxication(e.g. alcohol) (4) psychogenic syncope, sleep disorders

Vasoactive agents의 기전 및 효과에 관한 정리 ai

adrenergic receptor : 혈^ 011존재, vasoconstriction

0 1 adrenergic receptor : 심장에 존재, inotropy & chronotropy

02 adrenergic

receptor : 혈관에 존재, vasodilaticm Hemodynamic response Drug

Principal mechanism HR

MAP

CO

SVR

t t t

I

t

i t I t II

INOTROPIC AGENTS(MAY BE CHRONOTROPIC) Dob니tamine Dopamine(low dose)

/?i-adrenergic

i t

1

/?i-adrenergic : dopaminerigic

Epinephrine(low dose)

J3i and ^ 2 -adrenegic :less a

t 11

I t

11 i t i

t 1 f t t i

VASOCONSTRICTORS AND INOTROPIC AGENTS Dopamine(high dose) Epinephrine(high dose)

^-adrenergic :less dopaminergic a-adrenergic : less /?i and 曰 2 c?-ad「 ene「 gic :less /?i and P2

Norepinephrine

t T

I

* ^ 1-adrenergic effects are inotropic and increase contractility ** /?2 -adrenergic effects are chronotropic (*inotropics :contractility | , chronotropics : HR t , lusitrᄋpics : myocardial relaxation f )

❖ 교-7』^ ^ 의

작용을 잘 정리하고 있으면

의 기7a을 이해하는 데도 도움이 ^ 분 아니라 순환기 진반

이I서 자주 등장하는 dopamine, dobutamine, norepinephrine 등의 약울을 언제 쓰는지도 이해하기 쉬 우 _1진니다ᅵ 이 기회에 정리하고 ^ 어가세요(본문 마지막oil /날은 표 참고)! ❖

하'게 정리하면 ① 알파= 혈3a 수축= 혈와 상승,② 베타1 = 상당에 작용= c o . 심박수 상승,③ 베 타2 = 심장 의의 근육 0 1 % = ^ ^ 유 발 기3a 지 확7a 작용 등입니다.

방기본적으로 dopamine과 dobutamine은 C .0를 높이는 작용을 현니다. 다1만 dobutamine은 혈0』을 낮추 는 작용이 91으으로 저혈0음에서는 dopamine,고혈0남에서는 dobutamine을 성호형니다. ❖ Norepinephrine은 주로 a-adrenergic 작용에 의해 않 초 ^^을 수축시% 으로^

BP를 상승시% 니다.

Cardiogenic shockᄋII서 preload, BP의 ^승 을 우때 norepinephrine을 사용협니다. Harrison 20민:,pp. 122〜 128

c - 2020년 대비 PACIFIC KMLE !

순환기

01

B

34세 여;다가 출근길에 의식을 잃고 쓰러져 병원에 왔다. 지하철 안에 서

있는데 속이 메스껍고 울렁거리더니 머리가 어지러워지면서 쓰러졌다. 의식은 2분 후 돌Of왔다고 한다. 신경학적 진찰에서 이상소견은 없었다. 기립경사검사 결고I는 다음과 같다. 원인은?

혈 ^ XmmHg)

검사 전

검사 종료 시

130/90

120/85

1) 전부하 증가 3)

교감신경 억제

2> 부교감신경 항진

---------------------- cA,

01 신경심인성 실신 증례입니다. 사람이 북적거 리는 출근길 지하철 안에서(유발요인) 속이 메스껍고 울렁거리다가{전구증싱卜) 쓰러졌군 요. 의식소실은 수 분 이내로 짧게 끝나는 것 이 경련과의 차이점이죠. 신경심인성실신은 교감신경 억제,부교감 신경 항진이 단독,또 는 복합적으로 작용하여 나타납니다. 제시된 보기 중에서는 2번,3번이 해당됩니다. 이제 제시된 기립경사검사를 봅시다. 누워 있을 때 오 卜일으켜 세웠을 때 혈압^이가 크게 나지 않는군요. 그 말은 교감신경의 혈관반사가 적 절히 이루어지고 있다는 뜻입니다. 일어섰을 때 교감신경이 적절히 활성호m 으로써 혈관을 수축시키고 혈압을 유지시켜 주고 있는 것이 죠. 따라서 3)은 답과는 거리가 멀겠네요. 제시 된 답 중에서는 2)이 환자의 증상을 유발한 원 인으로 생각됩니다. 문제에는 심박수가 나와 있지 않습니다만, 부 교감 신경 항진으로 인한 실신 시에는 심박수 가 많이 낮아져 있습니다. Harrison 20만, pp. 122〜 128

4) 후부하 증가

5) 레닌一안지오텐신 억제

02

문제해설



25세 여자가 더운 여름 만원 버스에서 의식을 잃고 쓰러져서 병원에 왔

다. 쓰러진 직후 정신이 돌아왔다고 하였으며,이전에도 3 번 의식을 잃 은 적이 있었다. 혈압 120/80mmHg,맥박 70회/ 분,호흡 14회/ 분,체 온 36.5도였다. 가슴청진에서 심잡음은 들리지 않았으며 호흡음도 정상 이었다. 심초음파검人h 홀터검사 및 운동부하심전도검사 결고는 정상이 었다. 추가로 필요한 검사는? 1) 발목위팔지수

2) 기립 경사 검사

3) 관상동맥조영술

4) 전기생리학검사

0 2 실신을 주소로 오는 환자에서 기본적으로 기 립성 저혈압,신경심인성 실신,뇌전증을 의심 해야합니다. 이중 뇌전증은 모습이 특이적이 므로 여러 단서를 줍니다. 기립성 저혈압과 신경심인성 실신은 쓰러진 상황을 봅시다• ① 오래 서있다가 실신 ᅳ 신경심인성 ② 앉아있다가 서면서 실신 ᅳ 기립성저혈압 문제는 신경심인성 실신이 의심됩니다. 진단은 기립경사검人曝 통해 합니다. Harrison 2Q만,pp. 122〜 128

5) 24시간활동혈압 측정

03



30세 여자가 반복되는 실신을 주소로 병원에 왔다. 호ᅵ>나원으로 직장에

서 고I■도한 업무와 스트레스를 받고 있었다. 출근길에 사람 많은 지하철 에 서 있던 중 어지럽고 식은땀이 나며 욕지기를 하고 실신하였고,실신 후 의식이 곧 회복되었다. 진단은? 1) 기립성 저혈압

2) 경련(seizure)

3) 과다환기 증후군

4) 일과성허혈발작(TIA)

5) 신경심인성 실신(vasovagal syncope)

0 3 신경심인성 실신입니다. ① 혼잡하고 오래 서있는 환경,감정적인 스 트레스라는 유발인자가 있고 ② 오심,발한 등 전구증상을 보이는 전형적 인 증례입니다. Harrison 2Q만,pp. 122~ 128

€-

0 4



23세 여자가 실신을 주소로 내원하였다. 초ᅵ근들어 피로감을 자주 느꼈

으며 실신 직전에는 人|야가 좁아지고 흉부 불쾌감과 구역감을 느꼈다. 그 외에 실신에 따른 비정상 운동을 보인 것은 없었다. 수 분 후 저절로 회복되었으며 호|복 후 특이 증상을 보이지 않았다. 지난 1년간 지금과 비슷한 실신 경험이 3 번 있었으며 실신으로 인해 얼굴부위에 상처를 입 기도 했다. 환자의 혈압은 110/70, 맥박 90회/ 분이었다. 다음 중 우선 시행되어야 할 검사는? 뇌파

2) 24시간 생활심전도

기립 경사 검사

4) 심장초음파

뇌 CT

©ANSWER

01.② 02.② 03.⑤ 04.③

문제해설

심혈^

T i l 厂ᅳ

----------------------

0 4 신경심인성 실신입니다. 흉부 불쾌감과 구역 감의 전구증상을 보이며 기타 검사에서 특이 소견을 보이지 않았습니다. 전형적인 경우 맥박이 낮지만 가장 적합한 진단은 신경심인 성 실신으로 보Of야 합니다. 밝혀지지 않은 재발성 실신에서는 진단을 위해 실신발작을 일으키는 상황을 재연시켜야 합니다. 신경심 인성 실신이 의심되는 경우 기립 경사 검人卜 (tilt table test)를 실시합니다. Harrison 20만,pp. 122~ 128



2020년 대비 PACIFIC KMLE



_

순환기 I #

Explanation

------------------------

통 통 (Chest paiin or chest discomfort)

^

용통을 주소로 내우크던 환자•들을

줄 (갈아야 현니다. %기보다는 실기시험을 준비하는 데 더 중요한 내용

이지만, 기시형에도 충분히 출제철 수 91습니다. ^ 기시형에서는 주로 R type에서 강열진단을 물어보게 점니 다. 앙으로 uh울 진 ^! :들이기 때문에 굳이 여기서 다 보실 생74은 안 하셔도 점니다. 일1i± 진하'게 표시^ 질홧들 을 위주로 기본적인 '개병을 진고 명어가도록 해요八/\

흉통의 원인 동반 증상

악화 또는 완화 요인

지속 기간

위치 심 혈 관 계 원인

2 〜 10min

악화요인 : 운동 , 추운 날씨 . 감정적 S4 or pansyst이ic murmur of papillary muscle dysfunction during pain 스트레스 완화요인 : 휴식 . nitroglycerin

안정협심증

흉골 부위에 넓게 분포 , 경 계 불 분 명 ; 목 , 턱,윗배 , 어 깨 , 윗팔(특히 좌측)로 방사 . 종종 이 부위에서만 통증을 느끼기도 함

Pressure, burning, squeezing, heaviness, indigestion 조인다 , 누른다 , 답하다,

불안정협심증

안정협심증과 동일

안정협심증과 비슷하나 강도가 심할 수 있다.

Usually < 20min 안정협심증과 비슷하나 가벼운 운동에도 통증 유발되게 4 휴식 시

급성심근경색

안정협심증과 동일

안정협심증과 비슷하나 강도가 심할 수 있다.

^30m in but variable

휴식이나 nitroglycerin 으로도 통증이 Shortness of breath, sweating, weakness, nausea, vomiting 완화되지 않음

이형협심중

안정협심증과 동일

안정협심증과 비슷하나 강도가 심할 수 있다.

variable

운동과 무관한 안정 시 통증 , 흔히 한밤중이나 새벽에 발생

심막염

Usually begins over sternum Sharp, stabbing, or toward cardiac apex and knifelike ;pleuritic pain may radiate to neck or left shoulder ;often more localized than angina

Lasts many hours to days ; may wax and wane

악화요인 : 심호홉 , rotating chest, Pericardial friction rub - > 앉아서 앞으로 숙일 때 호기 시 증 개 AR과 유사) or supine position 완화요인 : sitting up and leaning forward

대동맥 박리

Anterior chest;may radiate to back

Excruciating, tearing, knifelike

Sudden onset, unrelenting

Usually occurs in setting of hypertension or predisposition, such as Marfan syndrome

폐색전증 U S E S )

Substernal or over region of pulmonary infarction

Pleuritic(with pulmonary infarction) or angina-like

Sudden onset ; 악화요인 : 심호흡 , or supine several minutes position to a few hours

폐동맥 고혈압

Substernal

Pressure ;oppressive

variable

악화요인 :i f

Pain usually associated with dyspnea ;signs of acute right ventricular failure

흉막액을 동반한 폐렴

Localized over involved area

Pleuritic, localized

Brief or prolonged

악화요인 : Painful breathing

Dyspnea, cough, fever, dull to percussion, bronchial breath sounds, rales, occasional pleural rub

자발성 기흥

Unilateral

Sharp, well localized

Sudden onset, lasts many hours

악화요인 : Painful breathing

Dvsonea ; hyperresonance and decreased breath and voice sounds over involved lung

근골격계 질환

Variable

Aching

variable

history of muscle exertion or injury Tender to pressure or movement 악화쇼인 : movement

대상포진

Dermatome을 따라 분포

Burning, itching

Prolonged

None

역류성 식도염

Substernal, epigastric

Burning, visceral discomfort

10 〜 60min

악화요인 : large meal, postprandial Water brash

뻐근하다 Similar to stable angina, but may be pronounced ;transient heart failure can occur

통증 발생 가능

sweating, weakness, nausea, vomiting

Murmur of aortic insufficiency, pulse or blood pressure asym metry ;neurologic deficit pericardial tamponade, acute AR 동반가능 Dyspnea, tachypnea, tachycardia ;hypotension, signs of acute right ventricular failure, and pulmonary hypertension with large emboli ;rales, pleural rub, hemoptysis with pulmonary infarction

비심혈관계 원인

vesicular rash appears in area of discomfort ( 단 , 피부 병변 없이도 통증이 있을 수 있음 )

recumbency

완화요인 : antacid 소화성 궤양

Epigastric, substernal

Visceral burning, aching

Prolonged

완화요인 : food, antacid

담도계 질환

Epigastric, right upper quadrant

Visceral

Prolonged

May follow meal

Right upper quadrant tenderness may be present

스트레스

Often localized over precordium

Variable ;location often moves from place to place

Varies ;often fleeting

Situational

Sighing respirations, often chest wall tenderness

€-

계 질환

:

t » • ■i

❖ 아래 열게卜 질% 들은 심근 허형을 유1질:하기 쉬운 진환들0』니다. 문제에서 용통과 형께 추가적인 정보를 주고 아a 질환 중 하나를 특정할 수 있는지를 묻기도 하으로, 진 알아둡시다ᅵ - H-CMP :용통 ± 심실 중격의 비대, 급사의 가족력 등 - Aortic stenosis :욤통 ± 화심실 비대,복장뼈 오른쪽 2번째 갈비사이공\ᅡ에서 수축기 ^음 - Aortic regurgitation : 용통 ± 이%기 혈0扣 1 비행상적으로 낮움, 복7^

원쪽 3먼째 ~y더lAhoi공'7抑 1

서 확장기 진음 - HI동DJj고열0쇼 :용통 ± 우심실 비대

❖ H-CMP, AS, 폐동맥 고혈와은 얻근 비관를 ° 말^ 니다. 비디떤 심근은 더 않은 혈ojj을 필요로 하으로 심근 허열이 오기 쉬우J진니다_

❖AR에서의 훙통은 위의 3가지 진환과는 기7S이 조금 다릅니다. 대동口J|의 oi윗기 형외이 유지되지 앞아, 관상동대(으로 가는 혈류가 김;소d ᅡ여 심근 허혈이 오기 쉬워진니다ᅵ

1. Guidelines and critical pathway for acute chest pain 1) 흉통 환자에서 반드시 시행해야 하는 검사 : EKG,C X R ± 심근효소(심근경색이 의심될 때) *

주의 : 내우크덩시

사한 성근효소 수치가 정상이라고 해서, 심근^색의 가능성울 배제할 수 없습니다!

2) 허혈성 심장질환이 의심되는 경우 (1) 병력에 따라 nitroglycerin 설하 투여를 시도해 본다. (2) 응급실 방문 당시와 4시간 이후의 심전도 심근 효소 검사 (3) C-reactive protein(CRP), Brain natriuretic peptide(BNP) 검사의 유용성은 연구 중 3) 관상동맥질환에 대한 유발검사 ᅳ 지속적 흉통 . 안정 시 흉통을 호소하는 환자나, 이상을 보이는 EKG 소견이 old infarct에 의한 것인지 확신하지 못하는 경우 글기 — 휴식 시 심근관류 스캔 (myocardial perfusion scan)으로 대체 4) 불안정 협심증 환자 모두를 입원시킬 필요는 없지만 심전도 집중 감시 필요 5) 대동맥박리가 의심되는 경우 line 잡고 최대한 빨리 contrast enhanced CT 시행!

6) 폐색전증은 의심하지 않으면 놓치는 질환으로 D-dimer 검사와 contrast enhanced chest CT 또는 폐 혈관 조영술 시행 7) 협심증 환자에서 특별한 금기가 없을 때 : 운동부하 심전도 부하 심초음파(stress echocardiography),

부하 심근관류스캔(stress perfusion imaging)

- i

c

2020년 대비 PACIFIC KMLE

* 용통을 일으^

순환기

수 91는 4디1 용급진환을 Hi드시 기억해 주시기 바렁니다.

급성 «>땐성 성 独 . 대동맥박HJ. 께여진증. ~U장 흐 기 율 0』니다(ACS, AD, PE, TP). 용통 환자와

문?:Hm 주어졌을 매, 환? ;ᅡ의 병력 상 아부리 위의 4대 용 급 이 외 의 다른 ⑩ 이

의심된다 하더라도, 반드시 위의 질환들을 하나하나 uHraidᅡ는 습관을 들이십시오. * 그 외에, 흥통을 동반하는 만성 진환으로서 방치할 정우 심■각한 1 과를 초래할 수 91는 진환으로

대동맥판 형착증, 안정협심증. 폐동OJ| 고혈와 등이 있습니다 Harrison 20민•,pp. 73~ 8ᄋ

01

B

운동 중에 흉통이 발생할 수 있는 질환은 다음 중 어느 것인가? 가. 안정협심증

나. 비대 심근병증

다. 대동맥판막 협착증

E-k 원발 폐동맥 고혈압

1) 기-,나,다

2) 가,다

4) 라

5) 가’ 나,다,라

3) 나’ 라

문제해설

----------------------a%

01 관상동맥질환 등 심장 질환이 있는 경우 운동 에 따른 산소 요구량이 늘어나는 만큼 혈류 예비량(blood flow reserve)이 충분하지 못해 허혈성 통증과 같은 증상이 발생합니다. 가.죽상경호I증에 의해 관상동맥이 좁아지면 심근 허혈이 발생해요. 어느 정도까지 좁 아지더라도 평소에는 증상이 나타나지 않 지만,운동과 같이 요구량이 증가할 때는 흉통이 나타나는데 이를 안정협심증이라 고 하지요. 나. HCMP는 비대된 심근에 의해 산소 요구 량이 증가합니다. 다. 대동맥판막 협착증의 3대 증상이 협심증, 실신,호흡곤란입니다. 라. 원발성 폐동맥 고혈압에서도 만성적인 압 력 부하에 으ᅵ한우심실 비대가 생기고 심 근 허혈에 의한 협심증 증상이 생길 수 있 어요. 이외에 호흡곤란,기좌호흡 심부전(S3 gallop 동반),LV 수축 기능의 심한 저하 시

교대맥(pulsus alternans) ' 정상에서도 흡기 시 수축기 혈압이 감소하나, 하강의 정도가 10mmHg 이상이면 이상 소견 ᅳ 심낭삼출{pericardial effusion), 심장눌림증(cardiac tamponade), 우심부전,폐 기종, 만성 천식 등

기이맥(pulsus paradoxus)

t • ■• i

* 표에 O0J:명! OldH가 잘 OJ;되시는 분들은 한면 임어보시고 생각* H보세요! 卿

% % 외우시지 않아도

평니다.

* 큰 맥박 1) Complete heart block 이 생기면 심방수축잊수가 심실수축잊수보다 驶卜져서 더 않은 혈액이 심실어 들어가' hi 되고 이로 q!dH preload가 않아?;다서 stroke volume이 증가t o i l 평니다. 2) LV의 preload가 병적으로 증가하는 상황들 : PDA, AVF, MR, VSD, AR

(1) MR의 ^ 우 - 정상적 대사를 위해 조J심실이 많초로 보내야 하는 활객의 ᄋ提 100, 승모1표을 통해 역류하는 형1 조혈되어 거의 동일회: 혈ᄋJ)렁이 RV로 들어오기 때문。 』니다(흡기 시 정11맥%류 t ASD shunt |

* Si, S2가 진움의



호기 시 정 响 류 1 ASD shunt f ).

시작이라면 d e cre s ce n d o sha p e

Si, S2가 ^ 음의 끝이라면 crescendo shape S!과 S2 사이에 기i속 들리는 진윰은 크기가 거의 일정 Si과 S2 사이에 시작되는 진음은 crescendo-decrescendo shape(다 이 이 드 모ᄋ궁) —> MS 제외

(2)

심음과의 관계

holosystolic murmur : MR, TR, VSD

Systolic murmur

aortic ejection murmur

systolic murmur : PS

① Systolic murmur • Holosystolic(pansystolic) m urm ur : TR, M R, VSD

一 Aorto-pulmonary shunt에서도 가능 - MR, VSD : 운동 시 f - TR : 폐동맥고혈압과 잘 동반,흡기 시 t ( ―

Pulmonary a.가 맹 ^ ! 폐에 눌a o辟 | )

• Midsystolic(systolic ejection) m u r m u r : AS, PS - Crescendo-decrescendo shape(다이아몬드 모양)

C 厂2020년 대비 PACIFIC KMLE

-

순환기

심실 압력이 valve를 열 수 있게 되는 S, 직후 시작,심실압이 감소해 판막이 닫히면 murmur도 중단

- Functional murmur의 경우 대개 폐동맥판 기원 • Early systolic murmur : S !에서 시작하여 수축중기에 끝남 - Large VSD w ith pulmonary hypertension : 심실 수축기 oil 피I혈류가 증가하다가 KI고형와으로 정처!되으로 RV ᄋ與이 높아져서 VSD shunt가 정지하■게 령니다- Small muscular VSD : 수축하1면서 구빙을 막'741죠.

一 TR without pulmonary hypertension :

41^ 당히고 I데동0J)oi 그 이후에 열된니다ᅵ 따

라서 TR이 생기다가 폐동다1) mpjp\ 열리면서 저항이 낮은 폐동1객 쪽으로 열0네이 wᅡ져나가면서 더 이상 TR murmur가 들리지 않게 법니다. • Late systolic murmur - M VP : 좌심방으로 승모판이 prolapse되면서 midsystolic click with late systolic MR murmur (2)

Diastolic murmur • Early diastolic m urm ur : AR, P R with pulmonary hypertension - S2 이후 시작,decrescendo shape - High-pitched AR murmur : left midstemal border, leaning forward position, holds a breath in full expiration

동맥압을 높이는 handgrip 시

,nitrite inhalation 시 丄

• Mid-diastolic m urm ur : MS, TS - Murmur의 intensity보다 duration이 협착의 정도를 더 잘 반영 - MS : opening snap에 이은 low-pitched, middiastolic murmur, left lateral decubitus position

에서 청진기의 bell로 maximal LV impulse site에서 듣는다 . Supine exercise나 nitrite inhalation 시 증가 - TS : left sternal edge, 흡기 시 } -

From MV : MR, PDA, VSD/from TV : TR, ASD — 상대적 혈류 f

- Austin-Flint murmur : severe 새에서 aorta에서 LV 로 역류하는 flow와 LA 에서 유입되

는 flow 가 만나면서 turbulence가 생기면서 anterior mitral leaflet가 진동하는데 여기에 서 발생하는 murmur • Presystolic murmur : MS, TS - Atrial contraction이 일어나는 경우에만 들림 : sinus rhythm에서만 - Atrio-ventricular valve 기원, crescendo shape, Si 에서 최대 - TS with sinus rhythm에서 특징적임,atrial myxoma에서 들리기도 - MS에서 AF 가 동반되는 경우 presystolic murmur가 사라짐

③ Continuous m urm ur ᅳ* PD A 가 대표적!! • Systole에서 시작하여 S2에서 최대가 된 후 diastole까지 이어짐 •

압력이 큰 대동맥과 압력이 작은 폐동맥 사이에 존재하는 PD A 와 같이 큰 압력차가 있는 경우 ^^생

• 혈압이 상승하는 경우 크기 증가 • 폐동맥 고혈압 동반 시 diastole phase | : aortopulmonary septal defect에서 잘 들리지 않

는이유 • PDA, aortopulmonary window, coronary arterial fistula 등 • Systemic/coronary AV fistula, pulmonary AV fistula(only systolic phase), CoA, pulmonary embolism, renal artery stenosis 등에서도 들릴 수 있음 • Cyanosis를 보이는 경우 collateral pulmonary vascular bed, 임신 후기 / 산욕기 breast(mammary souffle), cervical venous hum 등도 가능

④ 심낭 마찰음(pericardial

friction rub)

傘 AR murmur나 pericardial friction rub 둘 디' St으로 숙여서 듣고,호기 시 증가형니다. 생리학적 및 약리학적 조 작 심잡음 크기 T Valsalva 수기

Systolic murmur H-CMP(intensity만) Late systolic murmur MVP(length도)

심잡음 크기 i Most murm니「 (length & intensity) Right-sided murmur가 left보다 빨리 회복

심실조기박동(VPB) 또는 정상 혹은 좁아진 반월판 기원의 murmur는 length, MR, TR로 인한 수축기심잡음은 변화 없거나 감소 심방세동P4!죠). 판막이 당히면서 S,이 들린 이후 심실 수축기 Si력을 이기다■가 열리으로 midsystolic 또는 late systolic으로 들명(니다. 그리고 틈이 작으므로 crescendodecrescendo shape으 로 들않니다.

괴는 순 들 리 는 소리를 mid or late systolic click이라고 행니다.

* H-CMP와 MVP는 왜 다른 심진음들과 다른 양상을 보이는지 교과서들을 六 ^{아봐도 어'철 매 m u n w 가 증 가 / '7』소6J: 다고4 i 나와ᅵ9JL지, 기7U~m지 KMtdnn 나와ᅵ 있지는

II,m , aVF에서 높은 P파를 보여 우심방확 장이 있을 것으로 생각됩니다. 또한,I과 aVF 에서는 각각 ᅳ,+ 이므로 우측편위가 있고 V I에서 tall 요이 관찰되며,V!과 V2에서 in­ verted T wave가 관찰되고 있어 우심실비대 가 있을 것으로 생각됩니다. 심방중격결손(ASD, ostium secundum)인 경 우 LA ᅳ RA ᅳ RV로 혈류가 흘러가 RV 에 volume overload가 발생하여 심전도상 RVH가 생기게 됩니다. ASD 외에도 MS, 폐 기종,기관지확장증,좌심실부전, 심근병증, 폐동맥판협착, 팔로사징(TOF), 아이젠멩거 증후군(Eisenmenger syndrome) 등 다양한 원인에 의해 RVH가 생길 수 있습니다. 정상 인에서 LV wall이 RV보다 약 3배 정도 두껍 기 때문에 RVH로 인해 유의한 EKG signal 이 나타나기 위해서는 상당한 RVH가 진행되 어야 합니다. 대동맥축착,동맥관개존,심실중격결손의 경 우 조I심장에 부하가 나타나고 승모판협착증 에서는 조[심방확장이 우선적으로 발생하고 우 심실비대가 동반될 수 있습니다. 주어진 심전 도에 가장 II당한 질환은 심방중격결손입니다. 임상심전도학 51민:,p p .4 7 ~ 4 9

승모판협착

2) 대동맥축착

3) 심방중격결손

4) 동맥관개존

1)

5) 심실중격결손

0



'

2020년 대비 PACIFIC KMLE

02

순혼^기 I ®

- * •■咖 圖 1



*

____________________________________ g H B B

42세 여자가 2 년 전부터 시작된 운동 중 호흡곤란을 주소로 내원하였

다. 심첨부에서 확장기 잡음이 들렸다. 심전도 소견이 다음과 같을 때 다 음 심전도에 나타난 소견으로 옳은 것은?

1)

RVH

2) LVH

3)

RAE

4) LAE

5)

ST elevation

03

0 2 lead II에서 peak 人[이가 1칸(0.04초) 이상인 notched p wave와 Vi의 P wave에서 terminal negative deflection(P-terminal force >0.04) 이 보이므로 LAE(left atrial enlargement)에 해당히는 소견을 보입니다. 심첨부에서 확장 기 잡음을 보여 MS가 의심되고 이것과도 부 합하는 소견입니다. 임상심전도학 만,pp.40~41



_________________________________________________ 因•回 b 0 3 ① V5,V6에서 R파가 높고 ② V卜2에서는 S파가 깊고 ③ ST 하강과 함께 역위된 T파가 나타났으 니 LVH입니다. 좌심실의 용적 및 압력 부 하에 의해 나타납니다.

다음과 같은 심전도가 관찰되는 질환은? 가. 고혈압

나. 폐성심

다. 대동맥판 협착증

라. 승모판 협착증 T~W I

aVR

끼사ᅳ

aVt aVL

aVF

il

|



• •시 V 지/ —

가. HTN : LV pressure overload(afterload 증가) 나. 폐성심 : RVH가 특징적입니다. 다. AS : LV outflow tract obstruction에 의 해 LV pressure overload 라. MS : LAE가 나타나지만 LV에는 영향을 미치지 않습니다. 지속될 경우 RVH가 생 길 수는 있습니다. 임상심전도학 5ᅵ 만 ,pp.4 0 〜50

1) 가’ 나,다

2) 가,다

4) 라

5) 가,나,다,라

ooo

3) 나,라

iiin iiiin u n iiiifiiii^ -

0 4

0-SB

다음과 같은 심전도가 관찰되는 진환0 曰 막는 거은? 가. 팔로 사징(TOF)

나. 심방중격 결손

다. 폐동맥판 협착증

라. 승모판 역류증

Vi

심혈관계 검사

문제해설

~~

-------- 스

^

0

0 4 심전도상의 심실비대는 심실의 압력 부하 (pressure overload) 및 용량 부하(volume overload)에 의해 발생할 수 있습니다. 특징 은 심전도상 V,을 향하는 전기적 힘이 비정상 적으로 크고(\^에서 qK Rs 혹은 R/S >1) 왼 쪽에서 바라보는 심장유도인 V5,V6으로의 전기적 힘이 약해집니다. 제시된 심전도에서도 우축 편위가 있으며 V, 에서의 qR 소견이 보입니다. 우심실 비대가 의심됩니다.

v5

V4

1) 가,나,다

2) 가,다

4) 라

5) 가,나,다,라

v6

3) 나,라

가,다 모두 폐동맥협착으로 인해 압력 부하 가 생기고 우심실비대가 발생합니다. 심전 도상에서도 RVH로 나타나겠지요. 나. 심방중격 결손 시 조f심방에서 우심방으로 혈류가 유입되므로 우심실의 용적부하가 발생하여 우심실비대가 생길 수 있습니다. 라. 승모판 역류 시에는 용량 부하에 의한 조 卜 심실확장(LV enlargement)이 생깁니다. 이 두 경우는 심전도상 LVH로 나타날 것 입니다. cf. 심장의 생리적 변호[에서 사용하는 용어인 심실 확장(용적 부하),심실 비대(압력 부 하)는 모두,심전도상의 ‘심실비대’를 야 기할 수 있습니다. 폐동맥판 협착에 의한 압력 부하로 심근의 비후가 생겨서 심전 도상 ‘우심실비대(RVH)’가 생길 수 있 고,ostium secundum type ASD가 있어 서 우심의 용적 부하가 생길 때도 심전도 상의 ‘우심실비대(RVH)’가 생길 수 있습 니다. Harrison 20만,pp.l675~1682 임상심전도학 5만,pp. 4ᄋ〜 50

© ANSW ER

이 .③ 。2.④ 03.(D 04.0



0

(

5 5

)

2020년 대비 PACIFIC KMLE

순환기ᅳ 그 포

iim m u m m m m m m iim

Explanation

전해질 이상,디지탈£1스 중독에 의한 심전도 이상 및 기타

§3



Hyperkalemia 가 가 장 중 요 업 니 다 . 만 성 ^ 부진의 병 력 이 나 항 고 형 와 제 로 Ac 曰를 쓰 면 병력 등 특 진 적 0d 소^ 을 주 는 정 우 가 앞 으 니 , hyperkalemia 를 유 발 할 수 앗 는 대 표 적 0요 요 91들도 정 리 해 두 세 요 (진 장 내 과 , 소아과 에

91습 니 다 ). ACEi, ARB, spironolactone 등 hyperkalemia 유 말 섶!물은 꼭 정 리 해 두 세 요 .

작 변 국 시 에 서 는 digitalis(digoxin) 중독 환자에 서 1탈요dJ: ᄀ브시를 고 르 는 문 제 가 출 ?;'(I도J9오습니다. dH% 내용은 뒤 면 ■각론 파 트 에 , 문 제 는 R-type 파 트 에 접 부 하 엇 습 니 다 . 심 전 도 소 ^ 을 주면서 역 으 로 원 0요을 울 어 볼 수도

9^[으니 digoxin 중독 심 7ii 도 도 한민 보고 기 억 해 두 세 요 . 마지! 각으로 몇 가지 진환의 특 진 적 0i! 심 7a 도 를 녕 °있습니 다 . 중 요 6J:내 용 은 아 니 지 만 , 던 변 보 고 지나가시 면 종을 것

^ 습니다.

"T파는 K+ 이온을 당는 Tent다!1 1 라고 기억해 놈으세요. 고합롬혈증에서는 K+를 당기 위* « tall 꾸가 나타나H. 傘 저1알통혈증에서는 당울 K+가 5.5mEq/L) and HypokaIemia( Q T |

❖ ST-T wave가 1면1? (flattening)컨H지ᄀ사나 약7J: 역우l(minimal inversiᄋn)되는 헤 t(depression)이 나타나는 정우는 씽/날의 KH분극에 장에가 백 change가 협심증 등의 허혈성 심7표환을 뜻하는 정우가 않지만 ^

우,또는 ST segment의

경우를 뜻협니다. ST-T wave CNS disorder, 내분비 진환,

여러 가지 약저I,저성소증,거의 모든 형o 의 심피I진환,식사 후,자세 면화,과호흡,운동 등 다양한 % qioii 의해 이러한 병회■를 보일 수 9jL으으로 주의해야 현니다. ❖ 저할륨혈증이나 약제에 의천H 재■§■극이 지면되는 정우 torsades de pointes 와 '같은 상날민0'1)의



능성이 높아■진니다. 또紙 hypokalemia 는 digitalis toxicity 의 우때성을 높이구요.

4)

기타 몇 가지 특징적인 심전도

Hypothermia

Brugada syndrome

osborn wave

coved type ST elevation Harrison 2 0 잔;,pp.l679~1681; 홍칭■의 소아괴■학 11ᅵ 만 ,pp. 124~ 129

^

2020년 대비 PACIFIC KMLE |

순환기

01

Q 回 *I

60세 남자가 3 일 전부터 어지러워서 왔다. 3 년 전부터 만성콩팥기능상

실로 혈액투석을 받?^는데 최근 1주일 동안은 받지 못했다고 한다. 의식 은 명료하였고,혈압 92/62mmHg ,맥박 32회/ 분,호흡 21/분,체온 36.2도였다. 가슴청진에서 심잡음은 들리지 않았고,양쪽 등쪽 아랫부분

에 거품소리가 들렸다. 양쪽 정강뼈앞 오목부종이 있었다. 아래는 해당 환자의 심전도이다. 검사는?

1) 맥박파진파속도 3)

머리기울임검사

5)

홀터검사

----------------------01 만성신부전환자입니다. 투석을 받지 못하였으 므로 전해질 이상이 생겼으리라는 것을 예상 할 수 있습니다. 심전도를 보시면 다양한 lead에서 T파가 커 져있는 것을 볼 수 있습니다. 전형적인 hyperkalemia의 tall T소견입니다. 혈청 전해질 검人[를 통해 전해질 농도를 확인해보아이: 합 니다. Harrison 201만,pp. 1679〜 1681

2) 복부 컴퓨터단층촬영 4) 혈청 전해질

□ : ; : ; :

만성콩팥병증으로 혈액투석 대기 중이던 57세 남자가 어지러움을 주소 로 내원하였다. 심전도이다. 진단은?

0 2 만성 신부전환자가 어지러움으로 내원하였습니 다. 심전도에서 T wave가 높은 전형적인 고칼 륨혈증의 소견이 관찰되고 있고 심박수가 30 회 미만인 것이 어지러움의 유발원인으로 보입 니다. 실제로 만성신부전은 고칼륨혈증의 흔한 원인이고 고칼륨혈증에서 AV conduction 장 애 등을 유발하여 bradycardia가 동반될 수 있습니다. 참고로 2차성 QT 연장 증후군은 유전적 감수 성을 가진 사람에서 약물 등에 의해 아 가 연 장되는 경우를 말합니다. Harrison 2 0 ^ ,pp. 1679〜 1681

1) 2차성 QT 연장 증후군

2) 고칼슘혈증

3) 고칼륨혈증

4) 급성심근경색

5) 심장막염

심혈관계' 검사 ᅳ

37세 여자가 근위약감과 오심,구토를 주소로 내원하였다. BP 180/110, BUN/Cr은 45/7.3 ,Na/K은 132/7.5,Ca은 7.3,P 9.4, Hb 6.3 이었다.

심전도 소견은 다음과 같을 때 가장 중요한 처치는?

amiodarone

2) dobutamine

ACE inhibitor

4) 인 결합제제

0 3 칼륨이 7.5mg/dL로 굉장히 높아져 있네요. 심전도 역시 tall T wave를 보이는 hyperkalemia입니다. Cardiac arrest가 가능하므로 응 급으로 membrane excitabilty를 줄여주71위 해 Ca gluconate를 주사합니다. BUN/Cr이 높아져 있고 Hb이 떨어져 있는 것으로 보아 만성신부전으로 인해 hyperkalemia가 생긴 것으로 보입니다. Harrison 20팝,pp. 1679〜 1681

Ca gluconate

04 — ^—

n

____________________________________________________________________________

[A B € ] _

만성 신부전으로 1년 전부터 투석 치료를 받고 있는 환자의 심전도 소견 이 다음과 같을 때 진단은?

0 4 심전도를 볼 때는 rate와 axis를 파악한 다음 에는 우선 맨 아랫줄의 lead II 를 보세요. 혹 이 2개 달린 낙타 등도 아니고 뭐가 QRS complex이고 뭐가 T wave인지 구분할 수 없을 정도로 이상한 ECG 소견입니다. 문제의 심전도는 Tall T wave가 뚜렷하게 보 이는 고칼륨혈증에 해당합니다. 만성 신부전 환자라는 점도 고칼륨혈증의 가능성을 지지합 니다. 일반적으로 고칼륨혈증에서의 심전도는 정상 심전도에서 T파만 높게 솟은 소견을 보 이지만,이와 같이 K +농도가 더 높아지면 P 파는 시라지고 QRS파가 넓어지면서 T파와 합쳐지게 되어 문제에서와 같은 낙타 등 모양 의 심전도를 보이게 된다고 합니다. 이 상태에서 고칼륨혈증이 더 진행하면 sine 파 모양이 되고 심정지나 심실세동 및 무수축 이 초래됩니다. Harrison 20판,pp. 1679〜 1681

1) 고칼륨혈증

2) 저칼륨혈증

3) 고나트륨혈증

4) 저체온증

5) 고칼슘혈증

2020년 대비 PACIFIC KMLE

순환기

문제해설 : ' 0



ᄋ 'O

:

0 5 심전도에서 tall T wave가 보이고 3년 전부

70세 여자가 5일 전부터 몸이 붓고 저리다며 병원에 왔다. 4 년 전부터

고혈압, 심부전으로 약을 복용중이다. 혈압은 160/100mmHg,혈청 크 레아티닌 3.9mg/dL 이다. 심전도상 아래와 같은 소견을 보일 때 원인이 되는 약제는 무엇인가?

터 고혈압, 심부전으로 약을 복용 중이니까 복용하는 약의 부작용으로 인해 고칼륨혈증이 생긴 예가 되겠습니다. 제시된 약제 중 부작 용으로 고칼륨혈증을 유발할 수 있는 약은 ACE inhibitor인 enalapril입니다. ACEi는 aldosterone의 합성을 저해하므로 K 1 배설이 감소하여 고칼륨혈증이 나타납니다. 특 히 당뇨,신부전,유효순환량 저하,양측성 신 혈관 협착,NSAIDs와 K+-sparing diuretics 를 병용할 경우에 위험성이 높아집니다. ACEi 이외에 ARB, K+-sparing diuretics, NSAIDs, heparin 등도 고칼륨혈증을 일으킬 수 있어요. Harrison 2ᄋ만,pp.l679~1681 Washington manual 33:만,pp. 73〜 75

1) labetalol

2) enalapril

3) thiazide

4) digoxin

5) amlodipine

06

m„

_________________________________________________________________________________________________________因

( a

• 回

_

59세 여자가 전신 쇠약감 및 사지의 힘빠짐을 주소로 내원하였다. 내원

당시 심전도 소견은 심실빈맥이었으나 잠시 후 다시 시행한 심전도상에 는 sine 파형이 나타났다. 이 환자에게 가장 먼저 취해야 할 조치로 옳

0 6 사지 무력감,심전도상의 sine-wave 모양으로 미루어 보아 고칼륨혈증의 소견입니다. QRS interval이 길어져 tall T파오ᅡ 연결되어 sine 파형을 보이게 됩니다.

은 것은? 1) digitalis

2) Ca gluconate

3)

KC1

4) lidocaine

5)

0.9% saline

이 경우 우선적인 치료는 EKG monitoring하 면서 10% Calcium gluconate 10mL를 2〜 3 분에 걸쳐 주사하는 것입니다(소아는 0.5mIV kg, 7〜 8분에 걸쳐 주사). Ca은 역치 전위를 증가시켜 membrane excitability를 낮추고 항부정맥 작용을 나타냅니다. 수분 내에 작 용을 나타내나 30〜 60분밖에 효과가 지속되 지 않아 늘어난 extracellular K+을 cell 안 으로 들여보내거나 몸 밖으로 내보내는 조치 를 취해야 합니다. 처음 주사한 후 반응이 없 는 경우 5〜 10분 후 다시 한 번 사용할 수 있습니다. Washington manual 331 만, p. 77

S ANSWER

01.④ 02.③ 03.⑤ 04.① 05.② 06.②

심초음파와 심장핵의학 *

최근 국가고시나 임상종협평가에서는 거의 나오지 앞0났습니다. 심초음파의 M-mode 보는 법(승모1& 협착, 승모 판 ^ 출, 비후성 심근병증 등에서 또 나옵니다), 이면성 심초음파에서 심장 구조물의 위치와 방형, 정용부 심초 음파와 정식도 심초음파의 차이껍 정도는 일아 두시면 종습니다.

심 초 음 ᄑ k e c h o ca rd io g ra p h y}

傘 이용되는 초음파의 진동수가 높을수록 해상도는 종아지지만 투과도는 칠어진니다.

- 파동의 진동수가 높을수록 상란이 혈 일어나기 매문에 더 께끗한 영상을 얻을 수 있습니다. 파동의 진동수가 높을수록 울진을 투과하1면서 에너지의

-

I t 소H가 H 않이 일어나기 OH문0)1 투과도가

% 어진니다. 즉 ^ 숙이 자리진고 91는 구조울을 보기가 어령습니다.

1) M-mode echocardiography

서로 다른 굴절률을 가지는 두 매질의 경계면에서 소리의 일부가 발산되는 것을 이용하여 일직선 상의 구조가 시간에 따라 변화되는 양상을 파악한다. © = W IVSd

\

I ivSs

•難 ^

현^

!

^ PWTs

LVIDs

LVIDd

^

PWTdi



② 함^

^



a«g•n MtuirBMff게"th월HP3— *ᅵ

널 Biffiiiiiiiii날수회iiir활

그!; ᅳ 셔 RV^

_K 화

>*.

-r^



y^-

v>

람 ^ x



'产

^

••✓

■ >,

^

y

*

LA

표준 M-mode를 심초음파를 이용한 각 구조물의 측정 RV : right ventricle;AO :aorta;LA : left atrium;MV mitral valve;LVIDd :left ventricular internal dimension during diastole;LVIDs :left ventricular internaldimension during systole;IVSs : interventricular systolic septal thickness;IVSd :interventricular diastolic septal thickness;LVPWs :leftventricular systolic posterior wall thickness;LVPWd :left ventricular diastolicposterior wall thickness



2020년 대비 PACIFIC KMLE



순환기

종은 그^ 0』니다. Transducer의 위치를 보고 ②번 그 ^을 보면 직관적으로 이해할 수 91습니다.

II

Probe 위치(앞쪽} 좌심실

LV0T :LV outflow tract



R-mode ①





혁 ^ 균^ 레 ^ ^ !^

^ ^ 옳 ^ post, leaflet

수축기

이완기 ■

실습을 도실 매 심초음파를 보진 기억이 9JL으실 접니다. 그 때 승모1&이 이완기에 열렸다가 담d i a 고 하다가 다시 吧

후에 당히는 모습(두 먼 파닥파닥)을 보썼을 게이라 생74 함니다.

tlW f 파1닥이는 ᅳ%이 위의 그핑에서 D - E - F 로, 심실 이유!: 초기 LV 이%게 의해서 LA 혈0과이 이동할 매 승모 이 열리는 모습이며 두 민째 파닥이는 것이 위의 그평에서 AC로 심실 이완기1말 atrial kick에 의해 승모판이 영리는 모습잉니다. LV로 표시된 부분은 좌심실 명역, LV0T라고 표시한 부분은 심중겨과 승모팟의 ant. leaflet 사이의 공 니 다 , 심근의 모습으로 수축기와 이완기가 진 확와되지 않을 매는 심전도가 반드시 주어질 '삿있 니다. QRS파(심실 % & 극파) 후에 심/ i 수축이 일어나고 T파(심실 재분극파) 후에 심실 이완이 일어나는 ^을 꼭 기억하시고 수축기 이원;기를 나눠서 보시면 분명 진 보이실 섭니다ᅵ AA Ant. leaflet

월 —

고 1s t leaflet

수축기 이완기

MS의 M-mode 소'

니다. 우성 가장 아래의 심근 움직0나을 통해 이% 기를 찾습니다.

ᅳ심혈관겨 m

01% 기에서 ant. leaflet을

91는 ^을

—z

z

확0J화니다 . 현자! ant. leaflet이 두 41 파닥이지도 앞고 기울기가 둔하게 ^소하고

볼 수 91습니다. 이는 valve가 당히는 데 시신이 오래 그떤[다는 의이로 MS의 전형적와 소'7표입니다.

석회화 등으로 인dH slopeoi 두■법고 W?; 사분해 보이는 ^을 알 수 앗습니다ᅵ 또던 pc>stᅵ |eaf|et도 앞쪽으로 쏠a

91는 ^을 확0d할 수 04는데 ant.와 post, leaflet이 함께 유착되는 MS에 특진적0d 소'7크입니다. 앞쪽 (probe 위치) 앞쪽 (probe 위치)

MVP시 LA쪽으로 빠진 post, leaflet

정상수축기

위 그령은 MVP의 M-mode 소'현입니다. 오른쪽 그험에서 보시듯 MVP탓

이 c?t력을

디다가 한기I에 도달해서 l a 쪽으로 w m 는 ^

tatKlUl 되으로 M-mode에서는 원쪽과

^

MVP

수축기에 승모핑:의 post, leaflet

입니다. 이로 9 ^ h post, leaflet의 끝이 뒤쪽으로

이 나H세 니다. oit>H되시죠?AA 이 M-mode는 심근 수축을 진 확0ui하기

현듭니다. 아래를 보시면 심 ^도 가 보입니다_ 심 ^도 를 통해서 수축기와 이완기를 나누어 보시고 ig독하서I요.

o Aj.M .

비후된 심실중격----------승모판전첨 ----------SAM

좌심실후벽

H-CMP에서 systolic anterior motion(SAM)의 소경0』니다. SAM은 앞

계서 0날씀드혔듯이

중 '7 ᅧ

의 비후로 q[dH LVCrnᅡ좁아져서 열류가 1말라지고 t계르누이 법칙에 의dH 옆에 9Jfit 승모판의 ant. leaflet 이 쓸려 들어가면서 AS와

은 현상을 만드는 ^ 이라고 행!습니다. 즉 수축기에 ant. leaflet이 LVOT 쪽(그

명에서 심 ^중 격 쪽)으로 쏠리게 팅니다. 보이시죠? 八八

n

2020년 대비 PACIFIC KMLE

순환기

.暴

2) 이면성 심초음파(2D echocardiography)

초음파를 부채꼴 모양으로 주사해 2차원의 단층 영상을 얻는 방법,구조적 변화를 관찰하는 데 유 리하나 단층 영상을 얻는 데 일정한 시간이 걸려 구조물의 시간적 변화를 관찰하는 데 한계가 있을 수 있다.

2D echocardiᄋgraphy으I 표준 영상 구도 A, parasternal long axis view B, parasternal short axis view at the base C,

p a r a s te r n a l s h o r t

a x is v ie w a t

th e m itra l v a lv e

le v e l

D, parasternal short axis view at the papillarymuscle level E, apical 4 chamber view F, superiorly tilte d a p ic a l 4 c h a m b e r view(apical 5 c h a m b e r view)

©

OOO^

3.) 도플러 심초음파(Doppler echocardiography)

움직이는 red blood cel'l에 의해 반향된 초음파의 주파수가 cell 속도에 따라 달라지는 것을 이용해 움직이는 red blood cell의 속도를 얻을 수 있다. 특정 지 점의 혈류 속도와 방향을 알 수 있게 된다. 초음파를 내는 방식에 따라 간헐파형 도'R-러,연속파형 도플러,둘의 절충형인 high pulse repetition 도플러 방식이 있다. ' ♦

도플려 초음파는 도플a 효과를 이용하여 혈류의 도플a 立과육!:, ^어 지 는 파우보에서

法과 속도를 파악현니다.

파동은 파장이 lim , 가까워지는 파우크에서 말쟁던 파동은 파정이 짧

'게 관질;a 는 현상을 의 d i하죠. 傘 그령다면 도플a 초음파에서 필어지는 혈류는 파장이 가 적색으로, 가7가워? :ᅵ는 혈류는 파장이 짧은 청색으로 표시해야 할 ᄀ ;i ^ 은데, 이상d n ii도 국시에 출제도j 는 도플a 초음피■에서는 ^ 어지는 혈류를 청색으로, 가w

워지는 형■류를 적세으로. f i ■시 ^!니 다 ᅵ 교수님께 여罕4 보o p 니 "엑말은 임의로 정한 ^ 이며, 실제로 몇엿 기기|에서는

을 반대로 표시하기도 di■으

로 도플러 초음파는 판독에 주의가 요구독!다11 하시더군요!

C o lo r P robe

D o p p le r

e c h o c a r d io g r a p h y ( A o r tic

in s u f f ic ie n c y )

에서 멀어지는 것은 청색. 가까워지는 것은 적색. 속도가

빠를수록 밝은 색 표시

교착성 심낭염

( C o n s tr ic t iv e

호흡에 따라 승모판 유입혈류의 혈류의 역류현상이 관찰( H V )

E



peak

p e r ic a r d itis )

에서의 도플러 심조음파 소견

속도가 현저하게 변화하며( M V ) 간정맥 혈류상 호기 시 이완기

2020년 대비 PACIFIC KMLE

순환기

ᅳ »

Clinical Uses of Echocardiography 2면 성 심 초 음 파 (2D echocardiography) ♦

해부학적

구조



이상

도플러



관찰

1) 심방 및 심실 평가 (1) 심방,심실의 크기 및 기능 _卜= -------------------------------------------------------이 완 기 조 침 실 용 적 -수 축 기 조 ^심 실 용 적 —► 卜 이 완 기 조 심 실 용 적 (LVEDV) = Stroke volume(SV) / LVEDV ᅳ MR에 서 으 I EF 생 각 해 보 면 SV가 LVEDV의 매 우 큰 부 분 을 차 지 하 므 로 MR에 서 EF는 정 상 보 다 증 가 합 니 다 . 하 지 만 SV의

심 초음파

(Doppler echocardiography) 혈 류

속 도



용 적 ,압 력

등 을

알아냄

1) 판막 협착 압 력 경 人 Kpressure gradient) zlP = 4 X (peak velocity)2 Valve area= 220/pressure half time 또 는 Ai X Vi =A2 X V2 2) 판막 역류 반정량적 검사 가능(color doppler)

3) 심장 내 압력 계산 많 은 부 분 이 LAS 역 류 하 는 것 이 므 로 MR에 서 EF가 조 금 만 LA pressure + LA-LV pressure gradient 떨어져도(EF 1.5

E : early diastolic phase-ventricular relaxation A : late diastolic phase-atrjal contraction

대부분의 심장 질환에서 심실의 이완기능의 장애가 나타나는데 초기에는

줄고 LA 의 압력

이 증가하면서 A 부분이 커지게 된다(E : A ratio < 1). 하지만 이완기능 장애가 진행하면서 좌 심방쌉■이 상승하면 제한성 충만 양상으로 이행하게 되는데 이 중간에 마치 정상과 같은 승모판 유입 혈류 양상을 보이게 된다. E phase에 서 혈류가 줄어드는 시간을 뜻하는 deceleration time 이 삐를수록 ventricle의 compliance가 저하됨을 뜻한다. (2) 심장 내 압력 계산

Mitral regurgitation에서 얻은 심초음파

그림과 같이 승모판 역류혈류속도(V)가 5.2m/s라면 modified Bernoulli equation( Zl P = 4 x V2) 에 의해 LA 와 LV 의 압력차(ZIP) 는 108mmHg(4 x 5.22) 이고 좌심방압을 ᅵ OmmHg로 가정하면 LV 수축기압은 118mmHg이 된디-. 대동맥판협착증이 동반되지 않은 경우 수축기 LV 와 대동맥의 입ᅵ력은 동일하므로 대동맥수축기압을 U 8m m H g 로 추정할 수 있다.

I ®

2020년 대비 PACIFIC KMLE

2.

순환기

심 장 핵 의 학 (nuclear cardiology) 1)

Photon(대개는 y-ray)을 내놓는 isotope를 주사한 다음 image를 얻음 : isotope의 differential and inhomogeneous distribution을 이용(stress시 정상 관상동맥 혈류는 5배 가까이 증가) "^ c

201ᄁ : Thallium 201 (K+ analogue)

: Technetium " m Sestamibi

기전) 수동확산으로 세포막,mitochondria막 통과 기전) Na+/K+-ATPase를 통해 세포내로 유입 1) Taken 니다by viable myocardial cells coronary artery 1) Higher photon energy, shorter ti /2 than 201Tl blood flow(perfusion雇 반영 : 정상 심근과 ischemic 2) Better image quality than 201Tl (••• ① 때문에 더 많은 양을 주사할 수 있다) tissue를 구분 2) Redistribution 후 ischemic tissue는 정상 심근과 비슷해 3) Ventricular function assessment(first-pass or gated SPECT) 지나 fibrosis된 old infarct은 계속 low signal intensity 4) Shorter imaging time, shorter protocol length 5) Ac니te imaging in myocardial infarction(myocardi니m at 3) 약값이 싸다 risk) and 니nstable angina 4) Severe coronary artery disease나 LV dysfunctionᄋ| 있는 경우 PCWP가 증가해 lung uptake가 증가, post-stress LV 6) Resting perfusion defect 발견에 더 좋음(infarct size) dilatation —*• adverse prognosis 5) Detection of resting ischemia(hibernating myocardium)

7) 2이Tl에 비해 redistriWic们(mitcchondria에 irreversible binding)이 덜해 반드시 resting/stress 두 번 주사해야 함

허혈성 심7당질환 단원에 '부하 심근관류영상1이 9>1으니 자서던 내용은 거기서 공부현시다. 2) Stress (1) 운 동 로 treadmill) : 심박출량 증가(S V & H R 모두 t )와 관상동맥혈관 확장이 동시에 (2) 약물(dipyridamole, adenosine) : 주로 관상동맥혈관 확장에 의한 효과(H R 도

}

)

3) 심실 기능의 평가 (1) Equilibrium radionuclide angiography(multiple-gated blood pool imaging)

① 심한 부정맥이 없는 환자에서 좌심실 크기 및 기능을 비침습적으로 확인 ② 신호를 얻고 분석하는 데 EKG signal을 기준으로 삼아 정확 ③ 우심실 기능 및 크기,심방과 대혈관의 크기, 이완기 지표,판막 역류의 정도도 평가 (2) First-pass radionuclide Ventricular wall motion에 대한 해상도는 나쁘지만 주입된 동위원소(isotope bolus)가 대정맥에서 심

장과 폐를 지나 다시 대동맥으로 나오는 동안의 시간에 따른 평7Ktenporal assessment)를 할 수 있음 (3) 단일 양자방출 전산화 단층촬영 (single-photon emission computed tomography, SPECT)

① Ejection fraction과 regional wall m otion 평가, " ^ c 이 더 선호(higher count rate) ® 응급실에서 acute coronary syndrome을 의심했다가 퇴원해도 좋은가를 판정할 때 심근 S P E C IE 사용,장 단기간의 예후를 예측,SPECT 결과가 정상이면 관상동맥 질환을 배제해

도 됨(high negative predictive value)

수축력 휴식기

동면심근{hibernating myocardium)

기절심근(stunned myocardium)

^■소

^■소

길호

포도당 대사

정상 또는 증가

증가 또는 감소

수축력 회복

재관류 치료 후 가능

추적기에 회복

심 ^계 검 사

(4) 양전자방줄 단증 촬영(Positron Emission Tomography, PET)

① 양전자가 소멸하면서 (annihilation of the positron) — 2개의 감마선이 서로 반대 방향으로 방출됨(2y-ray in opposite directions) ② 13N-ammonia, 150-water 등을 이용하면 상대적인 blood flow를 보는 다른 방법과 달리 절대 값을 얻기 때문에 blood flow 나 myocardial metabolism의 정량 분석이 가능함 ③ 18F-FDG(fluorine-18 deoxyglucose) : 심근세포의 포도당 추출과 hexokinase 작용에 의한 phosphorylation 과정을 반영,심근세포의 당대사를 평가

④ Ischemic myocardium : FOG uptake | / perfusion

{

(“glucose / blood flow mismatch”) —»• 심

근의 대사가 fatty acid나 lactate보다 glucose에 주로 의존하게 됨 ᅳ revascularization 후 function의 호전을 기대할 수 있는 hibernating myocardium 탐색에 유효함 : gold standard for myocardial viability("mTc 등에 의한 경우 생존 가능 심근 탐색에 10〜20% 오류) Harrison 20만,pp. 1683-1685;이영우 편저, 순환기학,pp. 124〜 140,152〜 163



©



2020년 대비 PACIFIC KMLE

r

순환기 —

01

문제해설

Q

— 우 —

정상인의 심장초음파도(parasternal long-axial view)이다. 호ᅵ■살표가가 리키는 구조물은?



Parasternal long-axial view는 심장을 장축 으로 잘라 단면을 본 것입니다. 왼쪽 하단 수 축기시 좁아지는 것이 조!심실,화살표가 가리 키고 있는,오른쪽 하단의 수축기人I 오히려 확장되는 곳이 조1심방입니다.

대동맥

2) 좌심실

우심실

4) 좌심방

우심방

02

B

다음 중 경흉부 심초음파로 진단이 어려워 경식도 초음파를 이용하는 질

1) 심낭 삼출 2) 좌심방 점액종 3) 감염성 심내막염 4) 좌심방이 (It. atrial appendage) 혈전 5) 승모판 ^출 증

0 2 경식도 심초음파는 심장 후벽의 구조물(좌심

방, 승모판,대동맥 등)을 확인하는 데 매우 유용합니다. 따라서 보기 중 2) 조ᅡ심방 점액 종,3) 감염성 심내막염,4) 조[심방이 혈전 3 가지 경우에 모두 진단에 있어서 경흉부 심초 음파(TTE)보다 경식도 초음파 mitral valvulo plasty에서 흔히 사용

5. Cardiac angiography 1) Left ventriculography : LV 에 조영제를 보내 systolic/diastolic function을 평가하고 ejection fraction

까지 평가할 수 있으며 valve disease의 평가에도 쓰인다. 2) Aortography : aortic dissection, AR, PDA, aortic aneurysm 등의 평가 3) Coronary angiography : coronary artery / CABG graft patency 평/h intervention

o

0 ( 73)

-

c -

2020 년 대비 PACIFIC KMLE

순환기 그 룹

IIIIUIIIiUlillHiliillHIHIItliniililHfilUHfHlilililtililiilllllUllllllHlHfnUHlUiilllilllilllllllllllllllltUHniltHilillllilllll

Aorta P니Imonary tmnk

Superior — vena cava Anastomosis ---------(junction of vessels)

Left atrium Left coronary artery

Right atrium Circumflex artery Right coronary artery

Left ventricle

Right ventricle

Right marginal artery

Anterior interventricular artery Posterior interventricular artery

The major coronary arteries Harrison 20만,pp,1709~1713

o i :

® •回 -a

다음 중 심장 도관술로 측정할 수 있는 것은? 가. 심박출량

나. 우심방 내 압력

다. 폐모세혈관 쐐기압

라. 우심실 내 산소포화도

1) 가,나,다

2) 가,다

4) 라

5) 가,나,다,라

a © ANSW ER

©

lo o-

이屈

----------------------이

3) 나,라

Swan-Ganz catheter로 다음 사항을 측정하 거나 계산할 수 있습니다. ① 압력 : CVP, RA pressure, pulmonary artery pressure, PCWP(=LA pressure) ② 각 구간 산소포화도 ③ 심박출량(cardiac output) Harrison 20만,pp. 1709〜 1713

브 저 ᄀ 「 O _______

i^ _

매 ᅳ

^

I

____________ _____________ 画•画二j 画•画• _:麵



,



®

공부하기 버거워하시는 분들이 앞습니다. 부정맥은 기본적으로 심^도의 이생이으로 심진도를 일을 줄 알아야형니다. 吧 심^도 입는 ^ 을 면저 공부하시고 정상 심^도를 익히시' y_ 바럽니다, 그리고 나서 부정맥의 분류를 따라서 공부하시면 크계 어령지 앞을 이라 생각^ 니다. 기본적으로 심진도 일는 ~게어a 운 만큼 문제는 진단과 치료 위주로 엽>ii 출제평니다. 빠르게 여러 41 임는 보다 한4! 정독하시고,"부정1객의 분류"와 "W7gD]J 한눈에 보기"를 여러 먼 보시면서 머타속에 녕으시^ 바명니다. 2이9,2018면 국시에서 '각"각 6,8문제 출제되91으며,이외에도 소화기나 호홈기와 ■같이 다른 과와 면동도I어서 심^도를 준 정우가 ᅳ상^히 않았습니다. 자주 출제되는 심방서1동을 확^ 하'7■« 눈에 익히셔야 하며, 0J상 증상을 질; 기억해서 도를 제대로 판독하지 5 ■하여도 추론해 볼 수 91는 능력을 기르는 도 하나의 종은 방법이 필 수 9P41습니다.

M M M M M M m m— w 440ms에서는 금기 5) Dofetilide

日차단제

W

Ca2+ ch. blocker : # 력i 심^■수 i

신호 발생 및 전달을 동시에 차단 ᅳ 상심실성 빈맥에 효고려 Pheochromocytoma, hyperthyroidism과 같은 교감신경 활성 t 에 의한 빈맥에도 효고I적 항부정맥 약제 중 유일하게 심근경색 환자에서 'sudden cardiac death' 예방 Propranolol, nadolol, atenolol 심실 반응 i ,운동에 으ᅵ해 유발된 VT, hyperthyroidism에 의한 부정맥에 효고려 Esmolol(ultra-short-acting, 반감기~ 9 분) 심방세동에서 심실 반응을 늦춤 Congenital long QT syndrome에서 예방 목적의 유지 요법으로 사용

1) Verapamil(phenylalkylamine) 2) Diltiazem(benzothiazepine) (1) 심방조동/세동 환자에서 심실 반응 늦추기 위해 먼저 人f용{세동 상태를 동율동으로 전 환하는 작용이 아님!) (2) AV nodal reentrant tachycardia 예방 및 치료(verapamil) (3) 원발성 심실빈맥

방실결절의 전도 속도 1) AV node reentry에 의한 발작성 심실상성 빈맥(PSVT)의 TOC !! 2) 반감기 및 발현 시간이 매우 짧음 Adenosine 지연 i 3) Sick sinus syndrome, 2° 〜3° degree heart block에서는 금기(전도를 지연시키니까) 심 ^'# 수 i Digitalis

미주신경 tone t 방실전도 지연 | 불응기 연장우

1) 발작성 심실상성 빈맥(PSVT)의 치료 2) 심방세동에서 심실박동수 조절 — 전^ 히 꾸 〜욱 서nil 짜주'mi 하는 역^이라고 생각하시면 청니다.

순환기

2020년 대비 PACIFIC KMLE

t«■니

❖ Calcium channel blocker(CCB) - DHP CCB :형편:에 작용 — ^

이완 立과를 보임

- non-DHP CCB :성장에 작용 — 심근수축력 정소,심장진도 지면을 통한 심박수 강소 立과 위의 내용은 기본 중의 기본있니다. DHP CCB으로 nifedipine, amlodipine이 % 고,non-DHP CCB로 diltiazem, verapamil이 91다는 정도 축떠 외우J두세요.

一 "non-이HP는 一dipine이 아니다11라고 외우면 diltiazem, verapamil이라고 외워지죠八八

혈^■선택성

약 제

1000

Nisoldipine

100

Felodipine, nicardipine

10

Amlodipine, nifedipine

7

Diltiazem

심장전도

심^■수





I

i

i

1 I

I

i

심근수축력

DHP



non-DHP Verapamil

1

❖ Dihydropyridine(DHP) 기1°찰의 nifedipine, am lodipine^ 부730뤠 서 시■용 91형 v 심근보다는 는

근에 더 성되)적으로 작용하기 때문이다ᅵ 따라서 형관수축에 의한 증상을 나타내

에서 활용형: 수 9JL는^ I ,대표적으로 면이형 협심증(variant angina)과 91이노드 병(Raynaud

syndrome)이 91습니다. 하지Ht 2 0 1 8 ^ 국시에서는 면이형 협심증의 치료제로 non-DHP CCB가 나 황습니다. 보기 중에 DHP-CCB가 9 B P 면 더 적7!^ 덩이' 7/!i지Hi, non-DHP CCB도 DHP CCB에 비

tw ^

이완 효;;2n 卜작은

이고,立과 자처n ᅡ없지는 앞기 매문에 성•택지로 나온다면 ^으 로 고를

수 91어야 하'74!습니다. 꼭 기억하세요! ❖ DHP CCB의 부작용 중 하나로 않초 부종이 91습니다. DHP CCB는 vein보다는 arteri이e에 작용하여 鞭 을 이% 시키죠. 따라서 capillary bed로 유0』되는 解 는 늘어나는 반면 vein으로 빠져나가는 혈류 는 성;대적으로 적기 매문에 알초 부종이 생 7대 1 청니다. ❖ Aortic dissection의 치료에서 non-DHP CCB는 사용가능던 H K DHP CCB는 사용금기0』니다. DHP CCB를 쓰■ ᄀ'II 철 1혼우 arteri이e들의 vasodilatation이 ^어 나 서 심장oil서 일:초로 가는 혈류의 속도가 발 라지면서 dissecting flap을 더욱 크게 만들 우려가 있기 i재문0』니다. 나중에 다시 나오는 내용0』니다. Harrison 20만, pp.l716~1722;임상심전도학 5:만,pp.66~86 이영우 면저,순환기학,pp.273~ 298, 492~498, 503〜 508

8을 )

O

Explanation

부정맥의 분류 傘 부정대!은 기본적91 분류 처raR i oidHdH도 반은 이dHdᅡ고들어가는 단우크입니다, 정상과 부정1객을 구분하고, 부정 맥을 세객성과 민맥성으로 구분하는 ■섯이 기본o』니다! 습관을 들이시기 ui■않니n ■ᅵ

*

부정맥은 기본적으로 서1객성과 빈1객성으로 나뉩니다. 정확히 서0내성 부정0과은 서04 빈맥성 부정맥은 빈맥!

°d

~앗온 아님니다. *

서대j성 부정맥에서는 P wave의 위치를 첫아내는 ■섯이 중요현니다ᅵ P wave와 q r s complex와의 관기민 주의 하면 평니다. 치료는 pacemaker! pacemaker! pacemaker!

*

우각차단/ 조「각차단(Bundle branch block)은 행갈* I 알하면 는 특진적

*

성 부정1객에 *H당던니다. 각차단은 분류보다

도 형태를 기억형시다.

H!1객성 부정1객은 심실상성과 심실성으로 나뉩니다. 심/표상성은 narrow QRS / 심실성은 wide QRS의 큰 특진이 91습니다. 하지1만 심실성이 아님에도 wide QRS를 가지는 ᄀ 又이 있습니다. 바로 WPW syndrome과 각 대 tt (Bundle branch block;BBB)0』니다.

2020년 대비 PACIFIC KMLE

순환기

: {:부정맥 기억합시다! 1) P파

(1) 심방의 흥분을 의미 ᅳ 심실상성 부정맥에서는 보0남/심실 부정맥에서는 안 보임 (2) 리듬 — 규칙적이면 PSVT, 불규칙적이면 AF (3) 모。 쵸—> saw tooth°J.

우 AFL, irregularly irregular즉! : 경우 AF

2) QRS군 (1)

흥분■을 의몌 ᅳ 심성/상성 부7향객에서는 narrow,

부정D«ipf WPW syndrome에서는 wide

(2) 리듬 ᅳ 규칙적이면 VT, 불규칙적이면 VF (3) 모。 (J — sine waveᄋ d 경우 VFL, irregularly irregular학!: 경우 VF

傘 나중에

narrow QRS

wide QRS

regular

PSVT

VT

irregular

AF

VF, WPW with AF

더 나옵니다. 경조〜장조〜

傘 아래표는 참고용입니다. 지금 보고 다 알기는 어려우니 부정맥 공부 마치시고 표 는에 일라두세요!! 자극전도 장애

자극형성 장애 상심실성 yj°H

삼않성 보삐

(supraventricular arrhythmia)

(ventricular arrhythmia)

1) 동성 자극형성 장애 (1) 동빈맥(sinus tachycardia) (2) 동서맥(sinus bradycardia) (3) 동부정맥(sinus arrhythmia)

1) 심실조기박동 (ventricular premature beat) 2) 심실빈맥(ventricular tachycardia) 3) 다형성 심실빈맥

(4) 동정지/ 휴지(sinus arrest/sinus pause) (Torsades de Pointes) 2) 이소성 자극형성 4) 심실조동(ventricular flutter) 5) 심실세동(ventricular fibrillation) (1) 심방성 자극형성 ① 심^•조기^■동 (atrial premature beat) ② 심방빈맥(atrial tachycardia) ③ 심방조동(atriaᅵflutter) ④ 심방세동(atrial fibrillation) ⑤ 다소성 심방빈맥 (multifocal atrial tachycardia) (2) 방실접합부성 자극형성 ① 방실접합부성 이탈박동 (AV junctional escape beat)

6) 가속성 심실고유율동 (accelerated idioventricxilar rhythm)

1) 동방전도장애 (SA conduction disturbance) 2) 심방내 전도장애 (intraatrial conduction disturbance) 3) 방실전도장애 (AV conduction disturbance) 4) 심실내 전도장애 (intraventricular conduction disturbance) (1) 각차단 (bundle branch block) 혼합 장애 1) 조기흥분증^군 (pre-excitation syndrome, WPW syndrome) 2) parasystole 3) AV dissociation

② 방실접합부성 조기박동 (AV junctional premature beat) (3) 심실상성빈맥 임상심전도학 크ᅵ 만 ,pp. 87—88

ۥ

0 0 1 S

^

부정

_ _ _____________

Explanation

Sinus node dysfunction ♦

서1객성 부정맥에서는 치료가 가장 중요항니다ᅵ 빈DJj성 부정DJ)과 t ^ a 서Djj성 부TgDJj의 ■경우 치료가 단 1가지분 있니다. Pacemaker죠! 쉽지만 자■주 나오는 문제! 혈대 틀a 서는 안 도厂겠습니다. 2이8 년 국시에서는 민1맥一서

DJI 증伞군(Tachycardia-bradycardia syndrome)이 출제도ᅵ었습니다, 언뜻 보면 hjdj(성 부정맥으로 오해할 수 91는 심7a 도를 가지고 있는 만큼 꼭 기억협시다!

2019년 국시에서는

빈맥ᅳ서1맥 중伞군과 완7a 방실차단

(Complete AV block)이 각각 1문저쳐 출저& 9 i 습니다.

정상 동율동, 동서맥, 동빈맥 1) 동율동은 심박동수 60〜 100/min이 정상 2) 동서맥(sinus bradycardia) : 심박동수 < 60/min(운동선수나 노인은 vagal tone 높아 정상적으로 나

타날

_ A _

1

y~\

/J L /v



sinus bradycardia, HR 37.5/min

(1) 고령에서 흔함(idiopathic degeneration), 약제에 의한 경우도 아으 1건口 (2) Hypothyroidism, advanced liver disease, hypothermia, typhoid fever, vasovagal syncope, hypoxia, acute hypertension 등에서도 가능 3) 동빈맥(sinus tachycardia) : 심박동수 > 100/min

■ ——

L J

Sinus tachycardia, HR〜 105/min

⑴ 르판간 Q R S파 앞에 ^ 온다. PSVT와 구분하기 위한 중요 소견 •

sinus tachycardia에서 RR interval이 짷아지면서 P파가 앞의 T파에 ^

수 앗습니다. PSVT와 구분

이 헌들어지죠, 그a (도 숨은 p w 를 찾아봐야 행니다. 하지만 일반적으로 p파가 진 보이지 앞는다면, 동 ujojj의 가능성보다는 p s v t를 우성적으로 떠을a 야 협니다!

: 표



2020년 대비 PACIFIC KMLE

순환기

(2) fever, volume depletion, anxiety, exercise, thyrotoxicosis, hypoxemia, hypotension, 심부전 등에

대한 생리적 반응 (3) 목동맥팽대 자극(carotid sinus pressure>시 서서히(gradual return) 정상 동율동으로 전환 傘 발작성 야신/ i 성 HL^(PSVT)ᄋII서는 목동1객1?대 자극 시 ■집자기1 빈맥이 정상^ 되죠.

(4) 치료 : 기저질환 치료. 빈맥으로 인해 위험해질 수 있는 상황 시(ischemic heart disease 혹은 rate-related anginal symptom이 동반된 경우) beta blocker 투여를 고려할 수 있음 #

2.

기능 현진증에서 sinus tachycardia가 동4!:S! '경우 )&^ocker(propran으lol) 사용

동기능부전증후군(sick sinus syndrome) 1) 동결절의 자동능이나 동방 전도에 장애가 있어서 2) 심박출량 감소에 의한 증상(실신, 현기증, 운동시 호H

란>을 동반하면서

3) EKG 상 지속성 동서맥(sinus bradycardia), 동정지(sinus arrest), 동방 차단(sinus exit block), 빈맥 —서맥 증후군(tachycardia-bradycardia syndrome) 등으로 나타나는 질환군

* •중상+ 성 ^도 이 이 '살이 나타1i 11에라는 '개1a 0』니다. 따라서 증상이 91을



도를 확인하려고

24시' 7J: 성진도 경사(Holter monitoring)가 팡요하고,중상이 91으면 pacemaker가 치료가 되죠. 진1 과 치료가 정의에 향당현니다. 동기능부^증후군을 일으=n는 각 진%에 대해 알아봅시다.

3.

동 정 지 (sinus arrest) :동결절의 전기자극 형성 장애 1) P 파와 QRS 군이 모두 빠짐 2) 돌정지 동안의^PP 간격인 정상 PP 간 격 왼 수 가 안 됨

동정지(sinus arrest), 동율동이 지속되다 정상 동성 신호대신 접합부 조기 수축(Premature Junctional Contractions, PJC)이 나타남. 정상 동율동 간격의 배수가 되지 않음

€-

부;정맥

aVL

L



LX

동정지(sinus arrest). 방실접합부 이탈박동이 나타난다.

4. 동방 차단(sinus exit block) :결절 주위로의

신호 전달 장애

동방 결절에서 신호는 발생했으나 심방 등의 주위 조직으로 전도되는 과정에서 장애 1) 1° sinoatrial exit block : 동방 결절에서 주위 조직으로의 전도가 느려짐 2) 2느sinoatrial exit block : 간헐적인 전도 실패 3) 3°(complete) sinoatrial exit block : 동방 결절 기능 장애로 아예 P파 없거나 ectopic atrial pacemaker 4) Surface EKG 로는 2 ᄋ호판독 가능 : 휴지기의 PP 간격이 정상 PP 의 배수

동방 차단 Pause interval이 정상 sinus cycle의 2 배

5.

빈맥 -서맥 증早군(tachycardia-bradycardia syn^^ 발작성 심방빈맥( 예, AF)이 종료되고 동휴지가 지속되거나, 빈맥과 서맥이 교대로 나타날 때

빈맥 - 서맥 증早군 (Tachycardia-bradycardia syndrome) 상심실성 빈맥이 갑자기 멈추고 6초 정도의 휴지기가 나타남. 이후는 정상 동율동으로 전환됨

0(87

c

순환기 I #

2020년 대비 PACIFIC KMLE

6.

동기능부전증早군(sick sinus syndrome)의 녕가 1) 증상과 EKG 소견을 연관시켜 생각해야 함 2) 24시간 심전도 검사QHolter monitoring) : 대개의 증상이 발작적, 예즉 불가능 3) Implantable event recorder : 환자가 monitor 기기의 작동법을 몰라도 되는 장점 4) 목동맥팽대 자극 :3초 이상의 동휴지(sinus pause)를 유발하기도 한다. 5) 생리적/약리학적 조작 : 동방결절에 여러 가지 조건을 부여하여 평가 Vagomimetic

Valsalva maneuver

Sympathomimetic

Phenylephrine, isoproterenolol

Vagolytic

Atropine

Sympatholytic

0-blocker

傘 Intrinsic heart rate :118-(0.57 X age), 위 조작을 통해 부증생의 sinus bradycardia 혹! :자들을 primary sinus node dysfunction(slow intrinsic heart rate) or autonomic imbalance(normal intrinsic heart rate) 군으로 나눌 수 91평니다.

6)

Invasive electrophysiologic study : 증상이 있으면서 다른 부정맥에 의한 원인을 찾지 못하는 경

우 시행해야 함

7. MS. :증상 없으면 관찰,증상 있으면 심박동조율기 1) Permanent pacemaker (1) 증싱미 있는 환자에서 주된 치료 (2) 동기능부전증후군 환자에서는 동방결절 이외의 이상도 동반되는 경우가 많아 atrial pacing 이

외에도 ventricular pacing까지 가능한(dual chamber pacing) pacemaker를 삽입해야 함 2) 증상이 없는 경우 치료가 불필요 추적 관찰

8. 인*§^1 박동기(pacemaker) :symptomatic bradycardia으 I 치료 1) Temporary pacii^; (1) Permanent pacemaker 시술을 앞둔 환자의 즉각적인 안정화 또는 약물 중독이나 허혈 등 transient event가 있는 경우 (2) Central venous line을 통해 right ventricle에 pacing electrode를 삽입하나, 48시간 이상 경과 시 infection, thromboembolism risk } 2) D D D vs. W I code

傘 크-게 중요하지는 앞습니다. Chamber(s) paced

Chamber(s) sensed

Response to sensing

DDD

Dual(A+V)

Dual(A+V)

Dual(Triggered + Inhibited)

WI

Ventricle

Ventricle

Inhibited

離 Inhibited type에서는 자연말생 QRS군을 강지하면 조울은적은 억제되어 나타나지 앞습니다.

Class I indications for permanent pacing

Acquired AV block

Sinus node dysfunction

1) 3° or high-grade AV block at any anatomic level asso­ ciated with : (1) Symptomatic bradycardia

1) SA node dysf니nction with symptomatic bradycardia

or si 门니s pa니 ses

2) Symptomatic SA node dysf니nction as a res니It of es­ (2) Essential drug therapy that prod니ces symptomatic sential long-term drug therapy with no acceptable bradycardia alternatives

(3) Periods of asystole > 3 s or any escape rate 디곡신 투여 3) 직류 심박동전환(DC cardioversion) 4) 임시형 심장율동조율기 삽입 5) 관동맥 성형술

@ ANSWER



01.③ 02.② 03.(3) 04.① 05.② 06.® 07.④ 08.(3) 09.④

o ----------------- -------------------------------

그 부 정 5L _



Explanation

m m m ________________________________________

전도 장애 _ 각차단(Bundle

^^1

branch block)

유명한 다른 부정맥들과는 ᅵ날리 ■각차단의 심 ^도 소'건은 따로 는에 익혀두지 않으면 심7a도를 보고 ⑩ 하 실 수 9 i습니다. 우각차단,조P4차11卜심7a도의 대표적0d 소'"s과,각 소'견에 대현: 대처법(그1냥 관찰할지,추가적인 work up을 할지)에 대해 TgEJdH 두시면 되^ 습니다. I K d i 요약하자면,RBBB는

하는 반면,LBBB는 심 뛰 구조적 이상이 있을 수 있기 매문에 심초윰

파 경사가 추가로 ig•요현니다(2017 국시, 임종1평에 각각 출제된 내용입니다). 실 ^ 각 狀 ii(BBB,Bundle branch block)은 심실 내 전도 장에를 초래하기 매문에 주로 q r s 의 이상 등이 나타나'7il 不 텅니다. n \-^ m 쪽으로 신호가 늦■게

되니77卜상대적으로 오른쪽에 위치헌: V!과 왼쪽에 우^ I 헌: V6에서 특7망

적ᄋJ_ 모o和 | 나EH납니다ᅵ (계를 들어 우각 차iy :의 정우 v !에서 보기에는 마지막에 우심/ i 쪽으로 '오니까1 QRS 중 마지막이 위로 솟는(「 SR1) 모Oj이 되고,\/6에서는 QRS 중 마지1각에 우심실 쪽으로 ^ 어지니77F 싶고 멀어진 S파(qRS with deep S wave) 모ᄋ({이 나Ei•닙니다. oltg

Complete BBB —► QRS interval >0.128(3^), incomplete BBB —» 0.1 〜0.11s

각차단(bundle branch block)의 발생과 EKG 피■형 \h 좌각차단

우각차단(RBBB)

좌각차단(LBBB)

V, 의 토끼 귀, v 6의 slurred S ( ^ 프하기 위 *케 웠다a 로 도움당기가 필요하죠ᅵ Slurred 은파가 도움닫기하는 챗처령 보이시나요? 八八;) ᅳ 토^ R a b b it) — RBBB

傘 LBBB는 우 d a 보면 정상 상 a도의 V,, V6에서 QRS파가 wideningO^J: 이상. >0.12초) 되면서 욱직해진 것을 느7기시면 팅니다. i>fla

아a ᅵ설명은 크게 의DI 없어요ᅵ RBBB, LBBB는 눈에 익히는게 중요해요. 바로 잉상상으로 명어' ^ 시다! 궁금하 신 분들은 아at 내용 임어보세요.

객박수가 증가하는 동성부정맥이 밀:^하며 이는 정상0』니다-

심장으로 들어온 혈액을 다 내보나T7I 위한 형목적적 방법이라 생각^ 니다.

(2) 항부정맥제

① Adenosine IV : treatment of choice(방실전도 차단 —►reentrant circuit 차단) 乘 Supraventricular tachycardia(narrow QRS tachycardia)ᄋ ᅵ l서 PSVT를 강절하기 우태 목적으로 adenosine을 이용하기도 업니다. 진1i±과 치료에 모두 쓰일 수 91는

물0d 정이죠. Narrow QRS

tachycardia에서 adenosine IV push로 EKG 호진을 보이는 경우 PSVT, 호7S을 보이지 양는 경우 AF, AFL, AT 등을 시사현니다.

② IV ff-blocker or calcium channel blocker : 2nd choice, slow or terminate tachycardia

傘 Digitalis는 onset time이 느려서 급성기 치료로는 사용하지 않음 (3) Temporary pacing : 위 치료에도 반응이 없거나 재발하는 경우 (4) DC cardioversion : 위의 내과적 치료로 호전이 없게 +, 혈역학적으로 불안정한 경우 (5) On-going treatent

① Antegrade slow pathway :

-blocker, CCB, digitalis

② Fast pathway : IA, IC(flecainide, propafenone) ③ RFCA(Radiofrequency catheter ablation )로 회귀 회로 차단

傘 급성기 치료로는 Vagal maneuver와■ABC(adenosine, BB, CCB)주사치료가 91습니다. Ongoing treatment라 고 적어둔 부분은 재발이 uj행하거나,증상이 (있는 환자들에서 재일:을 앵지험: 목적으로 시컨於卜는 치료를 의이 하고,위oil서 언급원 다행한 항부정맥제제를 사용해 볼 수 앗습니다. RFCA는 95%의 성공률을 보이지Ht,1%에 서는 AV block이 ^

pacemaker가 필요할 수 91고,환자가 젊거나,증상이 심하거나,장기!!:의 약울치료를

진 경디지 못할 정우에 사용입니다.

傘 우리나라에 은헌; AVRT로 위의 치료를 보시면 더 엽'거1 o\t>\n\- 점니다. 혹시 아래 그많 기억나시나요?

활동전위에 미치는 영향에 따른 항부정맥제의 분류 I& I



:심실세포, I

: SA node, IV : AV node

-

IA ,

I C군은 phase 0 기울기를 장소시귀으로 ^ 도속도를 느리게 헌니다.

-

I A, I I 군 은 phase 3 을 면 장 시 귀으로 불용기를 ~U>ll 형니다.

-

I B군 은 phase 3을 줄이으로 불용기를 단축시% 니다. phase 0 기울기 장소 쇼과는 DIDI행니다.

-

I(B B ), ff(CCB)군은 그 ^에 서 보듯이 AV node에 주로 작용하여 흥분성을 낮추어 심박동수를 늦奇니다.

PSVT 중 AVRT는 slow pathway가 AV node(AVN), fast pathway가 accessory pathway(AP)0J(니다. AVN가 PR interval을 만들어 장시 pause시극다 야!/날의 춤분 한 이욕!;기를 확 보 행면 "짓을 기억하시면 ᄃ방면6| slow pathway 이^ 죠. AVRT의 치료나 에밍은 회귀회로를 끊기만 하면 되으로 두 pathway 중 하나41: 막으면 텅니다. 마라서 slow pathway의 정 우 AVN에 작용 해서 전도속도를 늦^

Adenosine, BB( I 군), CCB(m군), DD 등 을 쓸 수

9jL습니다. fast pathway는 node보다 심근과 ^기 적 성진이 유사하으로 phase 0 기울기를 늦웠면

I C군이 사용 틸 수 앗습니다.

소시며 지크도 속도를

2020 년 대비 PACIFIC KMLE

순환기 一 暴

❖ PSVT의 급성기 치료

혈역학적 불안정 시 ᅳ DC cardioversion 혈역학적 안정 시 ᅳ나① Vagal maneuver(carotid sinus massage, diving reflex, Valsalva maneuver) ② Adenosine IV push ③ BB, CCB ᅳ 약물은 쉽게 ABC(BCD 는 예방약)

* 위 표는 해리슨 20판에 나와 있는 표입니다 PSVT의 치료는 위의 정리된 내용이면 충분합니다. 여러 책들ᄋII서 PSVT의 치료로

h y p o te n s io n 이

없는 경우 carotid sinus massage와' '같은 vagal

maneuver 나 diving ref lex 를 시 ^ᅡ 면 약 80% 정도에서 PSVT가 중1i± ^ 다는 91급이 91습니다. 모두 vagus nerve의 parasympathetic tone이 을라가는 경우 HR는 칠어지'게 되는 7取 1적 반응을 이용하고 9人지요. Carotid sinus massage는 횡:자를 바로 놓측I 다음,한쪽 mandible angle 바로 이•래 부분의 carotid bulb을 약 5초 동0J: 꾹 〜 눌러주는 정을 0날화니다. Diving reflex는 한 울에 신 걸 •굴을 도얻디■는정을 않하구요(울에 a外져서 허우적거리는 사첨의 HR는 기대와 달리 처윰에는 월어진^ 니다). 하지0J: carotid sinus massage는 횡:지■의 EKG를 monitoring 하면서 maneuver 시 ^에 따라



9Jte asyst이e or VF 등에 즉'74 적91 처치를 할 수 91는 준비를 해야 하며 쎄 이하의 소아oil서는 사용할 수 없고 동口 JP3화• 등에 의해 carotid bruit가 들리는 환자에서는 해서는 안 ^ 다는 등의 복진한 진께 조 ᄀ뵤이 ᄋ 4기 매문에 adenosine W push가 더 성호되는 방법이라고 할 수 9P41어요. Valsalva maneuver 도 carotid sinus massage와 비슷즉!: 효과를 4 !당니다. 마지막으로 형: 가지! Non-DHP CCBᄋ d verapamil이 1세 이7a의 영아나 심한 심부진에서 금기 사덩이라 고

을 기억하세요ᅵ 소아과 공부할 oh 꼭 다시 며을a 주십시오.

^정맥 ᅵ ᅵ ᅵ

2.

방실 회구I 빈맥(AV *

Reentrant Tachycardia, AVRT) :우리나라에서 흔함

AP를 통과하는 않형에 따라 다른 심전도와 정과를 보o』니다ᅵ 우리나라에서 은한 tachycardia가 Orthodromic AV reentryᄋ ᅵ I 의한 PSVTᄋ 』니다. Antidromic AV reentry는 WPW syndrome의 mechanisnn0t!(니다. Antidromic AV reentry는 다음 장에 나오는 WPW syndrome에서 자세히 보도록 형시다.

1) Tachycardia circuit Antidromic AV reentry

Orthodromic AV reentry

Accessory pathway

부전도로(Accessory pathway)의 이해 A, WPW syndrome. 동율동을 보이는 환자에서 방실 결절 이외의 accessory bypass tract으로 신호가 먼저 전도되어 심실이 일부 pre-excitation 되는 WPW syndrome의 mechanism을 보여 주고 있다. B, AV reentrant tachycardia. 방실 결절과 His-Purkinje system을 통해 전도된 impulse의 일부가 bypass tract를 통하여 심방으로 역행성 전도(retrograde conduction)됨. AV reentrant tachycardia는 APC가 주로 작용하는 AV nodal reentrant tachycardia와 달리 ventricle 기원의 VPC가 bypass tract를 통해 역행전도 될 수 있어 VPC에 의한 reentry는 AV nodal reentrant tachycardia라고 하지 않는다. 실제 bypass tract는 이 그림과 달리 판막에 있는 것이 아니라 left-side AV groove 근처에 위치한다. WPW syndrome의 bypass tract는 AV ring 어디에나 있을 수 있다.

AV reentry, 호I■살표 AV reentry에서는 보통 ventricular activation이 retrograde conduction에 의한 atrial activation보다 먼저 나타나기 때문에 QRS complex 다음(예,ST segment)에 역위된 역행 P파(inverted retrograde P wave)가 보인다.

c -

2020 년 대비 PACIFIC KMLE

2)

순환기

Treatment (1) PSVT by orthodromic AVRT : 앞서나온 PSVT의 치료와 동일 (2) 전기생리학적 검사 후 RFCA(Radiofrequency Catheter Ablation) therapy로 90% 이상 성공률 (3) Antidromic AVRT는 다음 장,WPW syndrome에서 치료 확인 Harrison 20판,pp. 1739〜 1743; 임상심전도학 5판,pp. 112-118, 152〜 161,251〜 276

35세 남자가 2시간 전부터 시작된 지속적인 두근거림을 주소로 내원하

였다. 혈압은 100/70 이었고 EKG소견은 다음과 같았다. 치료는?

01 빈맥성 부정맥입니다. narrow QRS complex이므로 심실상성이죠. QRS ccmiplex가 regular하므로 PSVT를 가 장 먼저 의심할 수 있습니다. 시작과 종료가 돌연한 모습도 PSVT에 해당 히는 임상증상입니다. 급성치^ ^ ① Vagal maneuver ② Adenosine IV push ③ BB,CCB 혈역학적 불안정시 : DC cardioversion입니다. Harrison 2Q만,pp. 1739〜 1743

1) verapamil

2) adenosine IV

3) pacemaker 삽입

4) lidocaine

5) DC cardioversion 广

02

S 'B 1

40세 여자가 아침부터 갑자기 가슴이 두근거린다며 병원에 내원하였다.

그동안 이러한 증상이 가끔 있었지만 자연히 없어지곤 하였으며 당시 흉 부불쾌감도 나타났다. 심전도는 다음과 같다. 진단은?

1) sinus tachycardia 2) multifocal atrial tachycardia 3) paroxysmal supraventricular tachycardia 4) Torsades de pointes 5) ventricular tachycardia

o

a

02 갑작스럽게 발생하며 자연히 없어지곤 하는 가슴 두근거림,EKG 상 규칙적으로 발생하 며 narrow QRS를 보이는 빈맥을 통해 쉽게 PSVT를 진단할 수 있습니다. Harrison 2Q만,pp. 1739〜 1743



03

Q

20세 남자가 발작적인 가슴 두근거림을 주소로 내원하였다. 내원 당시

증상이 남아있었고 심전도는 다음과 같다. 가장 의심되는 질환은? 다/R

Vi

문 제 해 설

;g l 맥

ZZZ: ]»

---------------------- cA.

0 3 빈맥성 부정맥입니다. narrow Q RS complex 이므로 심실상성이죠. Q RS complex 가 regular하므로. PSVT 를 가

장 먼저 의심할 수 있습니다. 시작과 종료가 돌연한 모습도 PSVT 에 해당 하는 임상증상입니다.

V4

Harrison 2ᄋ핍;,pp. 1739〜 1743

1) 굴빠른맥 (sinus tachycardia) 2) 다발국소심방빠른맥(multifocal atrial tachycardia) 3) 심방잔떨림 (atrial fibrillation) 4) 심실빠른맥(ventricular tachycardia) 5) 발작위심실빠른맥(PSVT)

04

[ g •回 13

47세 여자가 30분 전부터 갑자기 가슴이 두근거린다며 내원하였다. 증

상은 1년 전부터 시작되었으며 갑작스럽게 발생했다가 사라지곤 했다. 심전도가 다음과 같을 때,재발을 막기 위한 치료로 가장 적절한 것은?

0 4 앞의 해설에서의 추론고[정을 따라 이 문제도 PSVT 입니다. 재발을 막기위한 치료는 고주파 도자 절제술 (RFCA) 입니다. Harrison 20ᅵ 만 ,pp. 1739〜 1743 Hurst's The Heart 11th ed, Chapter 30. Supraventricular Tachycardia.

1) propranolol IV

2) ICD 삽입

3) 고주파 도자 절제술

4) 심장 이식 수술

5) adenosine IV

o

U1



2020 년 대비 PACIFIC KMLE

순환기

05

因•□.回

37세 여자가 간헐적인 가슴 두근거림으로 병원에 왔다. 활력징후는 안

정적이었고 심전도는 아래 그림고ᅡ 같다. 가장 적절한 처치는?

I

그■ 그그. I

I

^

-c/\>

0 5 빈맥성 부정맥입니다. narrow QRS complex이므로 심실상성이죠. QRS ccmiplex가 regular하므로 PSVT를 가 장 먼저 의심할 수 있습니다시작과 종료가 돌연한 모습도 PSVT에 해당 하는 임상증상입니다. 치료는 ① Vagal maneuver (2) Adenosine IV push ③ BB, CCB 혈역학적 불안정시 : DC cardioversion입니다. Harrison 20ᅵ 만 ,pp.l739~1743

aVR'

j~ii ^__ i—^

니 ^ 1 !빠

^

aV F

에피네프린

2)

아데노신

4) 아미오다론

프로프라놀롤

DC cardioversion

06

Ea j B ® ]

57세 여자가 2시간 전부터 갑자기 가슴이 두근거린다며 내원하였다. 혈

0 6 위 문제와 비슷한 예입니다. Lidocaine만 빼

압은 120/80mmHg이었고 신체검진 상 특이 소견은 발견되지 않았다.

놓고 모두 사용할 수 있습니다만 우선적인 처 치는 adenosineO | 되겠지요. 여러 차례 강조

심전도는 다음과 같다. 가장 먼저 시행해야 하는 치료는?

되고 있는 문제입니다. Harrison 20ᅵ 만 ,pp.l739~1743

1) 심방조율

2) 아데노신 정맥주사

3) 베라파밀 정맥주사

4) 리도카인 정맥주사

5) 디곡신 정맥주사

7

|AJ'0 !

18세 남자가 심계항진을 주소로 내원하였다. 증상은 2 〜 3 년 전부터 간

헐적으로 있었고 혈압은 110/85mmHg이었다. 심전도는 다음과 같다. 진단 및 치료를 짝지은 것으로 맞는 것은?

문 제 해 설

----------------------cA)

07 이 ECG에서는 뚜렷한 retrograde P wave는 보이지 않습니다. 하지만 약 180/min의 rate 에 normal QRS에 regular RR interval이므 로 PSVT를 으|심해보아야 합니다. Harrison 2 0만,pp. 1739〜 1743

1) AF ᅳ DC cardioversion

2) AFL ᅳ propranolol

3)

PSVT ᅳ verapamil

4) VFL ᅳ amiodarone

5)

VF — lidocaine

_______________________________ E E K g K 26세 여자가 갑자기 숨이 차서 응급실로 왔다. 가끔 두근거림이 있었다

고 하였다. 혈압 97/65mmHg ,맥박 160회/ 분이었다. 으I식은 명료하고 빠른 심장박동 외에 다른 이상은 없었다. 심전도는 아래와 같다. 목동맥 팽대마사지(carotid sinus massage)에 반응이 없었다. 다음 치료는?

0 8 빈맥성 부정맥입니다. narrow QRS complex이므로 심실상성이죠. QRS complex가 regular하므로 PSVT를 가 장 먼저 의심할 수 있습니다. 시작과 종료가 돌연한 모습도 PSVT에 해당 히는 임상증상입니다. 치료는 ① Vagal maneuver ② Adenosine IV push ③ BB, CCB 혈역학적 불안정시 : DC cardioversion 입니다.

1) 관찰

2) 베라파밀(verapamil)

3) 아데노신(adenosine)

4) 아미오다론(amiodarone)

5) 직류 심장율동전환(DC cardioversion)

참고로 저혈압의 기준은 90/60mmHg입니다. Harrison 2Q만,pp.l739~1743

2020 년 대비 PACIFIC KMLE

순환기

09

r 꽤문 B

58세 남자가 5개월 전부터 가슴이 두근거린다며 병원에 왔다. 2 ~ 3 주에

한 번 정도 갑자기 시작되었다가 5분 뒤 갑자기 멈췄다. 운동이나 자세 와 상관없이 발생하였다. 혈압 122/79mmHg, 맥박 66회/ 분,호흡 19 含O O

회/ 분, 체온 36.7°C이다. 가슴 청진에서 심음과 호ᅵ

정상이다. 심

전도이다. 검사는?

f-

제 해 설

-kA )

09 갑자기 시작되는 가슴 두근거림을 주소로 내 원한 50대 남자의 증례입니다. 신체진찰 상 특별한 이상은 관찰되지 않고 있으며, 두근거 림의 증상도 2〜 3주에 한 번씩 나타나고 있 습니다. 시행한 심전도 상에서는 lead V ,에서 전형적 인 토끼 귀 모양의 rsR’이 관찰되고 lead V6 에서 slurred S 소견이 관찰되는 것에서 우각 차단(RBBB)이 있음을 알 수 있습니다우각차단의 경우, 구조적 심장 이상을 동반하 지 않을 가능성이 높기 때문에 경과 관찰하는 것이 일반적인 조치이며,환자의 경우 증상이 나타날 때의 심전도를 획인해보기 위해서 1달 이내의 부정맥 관찰에 유용한 활동 사건기록 기를 활용하는 것이 적절합니다. 활동 사건 기록기(ambulatory event recorder)는 1달 이내의 부정맥을 관찰히는 경우에 유용한 검사이며,삽입형 루프기록기의 경우 에는 최대 3년까지 기록할 수 있는 장치로 매 우 드물게 발생하는 부정맥을 잡0卜내기 위해 사용할 수 있습니다. 따라서 환자의 경우에는 삽입형 루프기록기보 다는 활동 사건기록기를 人[용하는 것이 더 적 절하겠습니다.

1) 머리기울임검사(head-up tilt test)

Harrison 2ᄋ:만,pp.l739~1743

2) 전기생리 검사(electrophysiologic study) 3) 심장동맥조영술(coronary angiography) 4) 활동 사건기록기 (ambulatory event recorder) 5) 삽입형 루프기록기 (implantable loop recorder)

广

10

0-0-0

갑자기 발생한 가슴 두근거림을 주소로 내원한 35세 남자의 심전도이 다. 진단은?

10 Ventricular tachycardia를 제외한 나머지 부 정맥들은 ‘심실상성,supraventricular’ 부정 맥에 해당합니다. 그래서 별다른 문제가 없는 한 normal QRS interval을 보입니다. ECG 모양만으로 기전을 판단하기는 어렵지만 이 ECG는 170/min의 rate에 regular RR inter­ val, normal QRS complex 후에 retrograde P wave가 나타났으므로 AV reentrant tach­ ycardia에 해당합니다. Harrison 201만,pp. 1739~ 1743

1) sinus tachycardia

2) Torsades de pointes

3) PSVT

4) AFL

5) AF

^A N S W E R

01.② 02.③ 03.⑤ 04•③ 05.③ 06.② 07.③ 08•③ 09.④ 10.③

_______________________

조기통분증早군(Pre ᅳ excitation *

Explanation

syndrome, WPW syndrome)

WPW syndrome은 증성이 없을 '개의 특진적면 심전도 소^( 1 〜 2mm) : STEMI, variant angina, Acute pericarditis (2) Depression(J point에서 0.08s 후 >lm m ) : ischemia

ST segment는 허혈성 심질환에서 굉장히 중요합니다. J point란 일종의 변곡점이라고 생각하시면 쉽

습니다. 특히 단순히 ST elevation만 보고 무조건 M I구나 하지 말고, reciprocal change는 없는지 확인해 보세요. 혹시 모든 lead에 모두 elevation이 있다면, pericarditis는 아닌지 잘 감별해야 합니다.

7) T wave (1) High voltage( > 15mV) : Hyperkalemia, hyperacute T (2) Inversion : Strain, Coronary T

학생 수준에서 높은 T wave가 보이면 우선 고칼륨험증부터 생각해야 합니다. Hyperacute T란 STEMI 의 시작으로 ST elevation이 일어나기 전 보이는 심전도 소견입니다. T wave가 inversion되어 있다면,

2020 년 대비 PACIFIC KMLE

순환기

혹시 심근이 힘들어하고 있는 것은 아닌지(ventricular strain) 생각해 봅니다. Coronary T란 STEMI에서 Q wave가 생기고 난 후 T wave가 뒤집히는 것을 말합니다. 8) QTc & U wave (1) long QTc : Hypocalcemia, Long QT syndrome (2) short QTc : Hypercalcemia (3) prominent U wave : Hypokalemia

수고하셨습니다. 험난한 EKG 보기의 마지막입니다. QTc란 QT time을 HR로 보정해준 값입니다. 이 시간은 혈중 calcium 농도와 관련이 깊습니다. 또한 U wave가 보인다면 저칼륨혈증을 생각합니다. ♦

요즘은 기기I가 EKG도 다 팟독해 주는데, 왜 이절 공부해야

■나 61■는 생각이 들 » 에요, 그래도 우^ 의사가

되기 익i)H 공부6H야 형니다. 훅훅… 보고 또 보고… 지 다 들 현내지■고요!

*

모든 EKG를 이면 순서로 다 보 어 령 습 니 다 . 시형 시'1ᅡ이 부족하니까요. 우성은 ^^에서 공부한 '부 정맥 한눈에 보기'에서와 -알이 typical한 EKG를 숙지하세요. 또 뒤에 '급 성 심근그쎄'에서 공부할 STEMI, Right Ml에 대한 EKG 소"73도 숙지하시면,

문제를 푸시는 데에는 어령지

으실 거에요. 하지1만,약' U 비전형적인 EKG를

주면 으a i ^황 하 기 험고, 문제에서 보여주는 함정에 mᅡ지기 엽습니다. 또한 소아 성 ^성 심기형에서의 EKG소 은 우리를 면제나 딩;% 스™ 하죠. 확실하다고 생각되는 EKG가 아니라면, 위의 바^ 니다. 분명 어령지 앞"게 문제에 접근하실 수 91을 것입니다. AA

법대로 차근차근 따져 보길

심부전 雾 심부7크은 심장이 지진 상태입니다. 심장이 과부하가 걸리는 여러 상^ CHI서 우리 몸의 대사Oil 필요한 혈QJj평을 보낼 수 없는 상EH가 되는 을 많형 니다. 즉 심부^은 하나의 진단명이라기보다는 "상Elf를 나타녕니다. 심부^의 원인은 여러 가지가 있습니다 팟막진환, tH혈성 심장진환, 심근질환, 고 혈 ^,심지어 민일 1^ 기능항진증, 41맥 등 심장의 과부하를 줄 수 91는 모든 에서 심부진이 유1^ 수 있습니다. 따라서 다른 파•트의 문제를 풀다가도 심부^을 ^상7 4해야 현니다. 복장하지Ht 중요한 ^은 맛성 심부^의 생존율을 높이는 치료0』니다. 또한,급성으로 악화된 심부^에서는 치료가 다르므로 이에 대해서도 숙지하 시기 바령니다. 올해 국시에서는 r 형 3문제를 포함하여 6문제가 출제되91습니다. 비슷한 유형의 문제가 않이 나오는 파트이므로, 치료에 대해서는 cg^dm i 알고 명어집시다! mmsmismtts



%

Explanation

0 0 i S „ _________ _________

정상 심장 vs. 심부전 離

m ^

생리는 총론에서도 충분히 나올 수 있는 중요한 내용들입니다ᅵ ᅳ상조원 부분을 위주로 차근차근 이해

하면서 공부해 보세요, 심부;"a의 기7a 부분은 뒤에 나을 심부전 W료제를 이dHdᅡ는데 uh^oi 되므로 삭r 辦■사서 봐 주시면 종습니다.

1. 심박출량의 조절 1) 심박출량(Cardiac Output, CO ) = 1회 박출량(Stroke Volume, SV) x 심박동수(Heart Rate, HR)

傘 Stroke 베니ᅡ阳二조성^의 이완기0날 용적-수축기 Dj; 용적 —► BP(Blood Pressure) - CO x SVR(Systemic Vascular Resistance) 2) Stroke volume을 결정히는 3가지 인자 : preload, contractility, afterload 寧 따라서 CO는 위의 3가지 91자+ 심박동수를 포항던 4가지 °d^oil 의해 #

정침니다.

심박출량표 진부하, 수축气 심박동수가 증가^ 수록. 伞부하가 강소^수록 즐가합니다! !

(1) 전부하(preload)

① 수축 직전까지 심장 내로 들어오는 혈액량 ᅳ 좌심실 이완기말 용적을 반영 • /?-adrenergic activation : 심근이완이 잘 일어남 ᅳ 전부하 증가 • Ischemia, ventricular hypertrophy, pericardial corrpression —*• 전부하 감소 ® 심근 수죽력이 일정한 경우 preload = myocardial performance (Frank-Starling의 법칙 : 이완기말 용적 증가 ᅳ 심근 섬유의 신장(stretching) —► actin과 myosin filament가 서로 더 많이 접촉, Ca2+에 대한 반응도 증가 ᅳ 수축력 丁) (2) 수죽력 -수축기말 용적(ESV> 2) Cardiac output = SV x HR 3) 구출률(Ejection Fraction, EF) =

SV - (정상치 67 ±8%) C/JJV

ᅳ Failing heart에서 ejection fraction은 stroke volume이 정상 범위에 있더라도 대개 감소하는 경 향을 보여 cardi쁜 output이나 stroke volume 보다는 pum p function 을 더 잘 반영하는 것으로 알려짐 4) 이완기말 용적(정상 : 75±20mL/m 2)이나 수축기말 용적(정상 : 25±7mL7m2)이 상승하는 경우도

좌심실 기능에 이상이 있음을 시사 5) 운동 시 산소 소비량을 측정하는 것도 심장기능 평가에 도움이 됨

6)

정상 심장

Failing heart

산소 소비 lOOmL 증가 시 cardiac output

500mL/min 이상 증가

12mmHg

최대 산소 소비량(Vmax02)

>20mL/min/kg

< 10mL/min/kg

이완 기능에 이상이 있을 경우 0 ) 초음파로 승모판의 혈류 속도를 추적 관찰하는 것이 도움이 됨

① 정상 심장에서는 초기 이완기(early diastole)의 속도가 후기 이완기(late diast이e)보다 삐-름 ② 심실의 이완 기능에 장애가 있는 경우 후기 이완기(late diastole) 속도가 더 빨라짐

€ - 2020년 대비 PACIFIC KMLE

순환기 그 조

4. Failing heart 1) 심부전(heart failure)

대사 요구링만큼의 혈액을 공급하지 못히는 심장의 수축 / 이완 기능과 관련된 구조적, 기능적 이상 상태 傘

% 부진은 여러 증상/협병증을 보이는 임상적 증후군을 뜻하는 ^ 이지, 진1난명이 아닝니다.

2) 수축기능의 이상 (1) Cardiac output [ (2) Ventricular emptying |

ventricular diastolic pressure }

3) 이완기능의 이상 傘

대표적인 에로 H-CMP,R-CMP, AS 등이 91죠.

離 Ejection Fraction(EF)이 50% 이 ᄋ d 심부진을 diastolic heart failure라고 부릅니다.

베타교감신경항진제 기저상태

200 mL 용적

용적

A , 교감신경계의 항진에 의한 심실 작업 효율의 변화. adrenergic activation으로 수축력의 향상이 일어나고 (‘ 가, 一 ‘ 나’ ) 이완기능 도 증가한다. 그 결과 기저 상태에 비해 외적 작업(external work; 혈액을 분출하는 작업)이 증가하고 내적 작업 internal work (심실에 가해지는 스트레스를 극복하기 위한 에너지)은 감소해 작업의 효율이 증가한다. B, 심부전 상태에서 심실 작업 효율의 변화. 정상 상태에 비해 수축력이 저하되고(가 ’- ‘ 나’ ) 이완 기능의 장애도 일어난다. 결국 외적 작업이 줄어 심박출량이 감소하고 대신 ventricular dilation에 따른 심근벽 스트레스의 증가로 내적 작업은 증가해 작업의 효율이 감소한다.

4)

심부전의 기전 傘 세장에 혈역학적 부1^(pressure/volume

overload)이 주어질 ^ 여러 가지 보상기7S을 통해 이를 극복해보

a 고 노력하지망 ᄀ표국 그 보상기진이 remodeling을 초래하여 씽/ i 기능의 비가역적 저하를 초래형니다. Neurohumoral/cytokine system의 병회■가 심부7il 여주는 여러 약제들의 background가 도H I 도 업니다.

0

O

| 부정맥 유발 T

Vasodilation

Skeletal m니scle catabolism, endotheli니m Unction i ,

심실

수 ^ •력

丄,LV

remodeling3)

Muscle fiber에 가해지는 압력 부하를 분산하 심근 세포의 기능 저하4\ Cell death, Cytokine/ 는 효과 neurohormonal activation

심근 섬유호Kcollagen }) 심근 이완 저하

심실 이완의 장애 초래

1) 심부전 환자에서는 혈중 순환 NE(norepinephrine) 농도 상승 T 2>교감신경계 활성 시 혈관 저항 증가, 심근세포에 손상을 주는 Ca2- overload 초래 : 신-blocker가 심부전에서 사용되는 이 유가 됨 3)심실이 더 둥근 모양이 되어 ejection에 비효율적으로 바뀌게 됨 4) Overload cardiomyopathy : 심근의 산소 요구량이 증가하면서 심근허혈의 위험성이 더욱 커진다.

P 2020 년 대비 PACIFIC KMLE

순환기

^

❖ Endothelin : powerful vasoconstrictor TNF-a : systolic dysfunction, myocarditis, ventricular dilatation, survival i 그러나 endothelin/TNF-a blocking은 임상적 효고n 卜없있윰

❖ V asod ilator peptide 1) 심7^ !서

기원하는 vasodilator peptide

(1) Atrial Natriuretic Peptide(ANP) :

SIADH : ADH | — Na 재홈수소x ),수 대 S■수 effective volume | — 우 확 대 — 우

*

상으로부 s ANP

출 — Na wh출 t 과 항7게수분도

(2) Brain Natriuretic Peptide(BNP) : ^

2) 소면령

Na

|,

출 | (solvent drag) —- hypᄋNa+euvᄋlemia

&

| ,reni n / aldosterone | 의 효과가 9J。 으나 다른 신경성, 호르본성 보

상기진을 중화시키지는 못항 3) 특히 BNP t 은 poor prognosis 시사 4) Acute pulmonary edemaᄋII서 BNP 치료가 면구 중(nesiritide)

❖ Renin-angiotensin-aldosterone system - 심부전 환자에서 활성화청

- Angiotensin I :과도한 현1S수축,영분. 수분 저류,심근 성유와 심장독성 - Aldosterone : 심근비대,ventricular remodeling - 심부진 환지■에서 ACEi, angiotensin I receptor blocker, aldosterone antagonist 사용의 이유 가 됨 Harrison 20만,pp. 1763〜 1768

01



B

다음 중 심박출량이 증가하기 우I한 조건으로 옳은 것은? 나. afterload 감소 라; 수축력 증가

가. preload 증가 다. HR 증가 1) 가,나,다

2) 가,다

4) 라

5) 가’ 나,다,라

3) 나,라

문 제 해 설

-

01 Cardiac output = stroke volume x HR 심박출량은 SV이 증가하거나 HR가 빨라지면 ^ 가합니다. 심박출량(cardiac output)이 증가하는 경우 ① Preload 증가 ② Myocardial contractility 증가 ③ Afterload 감소(① 〜③ — SV 증가) ④ HR 증가 Harrison 20만,pp. 1763~ 1768

문 제 해 설

다음 중,운동 시 증가하는 지표는? 가. 심근 수축력 다. 정맥 환류

나. 수축기 혈압 라. 피부 혈관 저항

1) 가’ 나, 다

2) 가,다

4) 라

5) 가,나,다,라

blastic anemia)

우 호흡곤란이 동반될 수 있음) • 알코올 중독의 병력 • 간염의 병력

• 목정맥압은 정상 • 만성 간질환의 신체검진 소견

ᅳ황달

간성

- Palmar erythema

(Hepatic)

- Spider angioma

• 요산 수치 상승 • 저나트륨혈증 • 간울혈에 의한 간효소 수치 싱씀

一Dup니ytren's contract니re - 여성형 유방 一퍼덕떨림(flapping tremor) - 간성뇌증

신성 (Renal)

• 동반된 만성 신질환의 병력이 있음 • 신부전에 의한 고혈압 동반 가능 • 요독증상 :식욕 감소, 수면장애, 사 • 고혈압성 신병증,당뇨성 신병증에 의 지 근경련, 집중력 저하 한 증상 나타날 수 있음 • 주로 눈 주위 부종 • 진행된 신부전의 경우, 요독성 심막염 (uremic pericarditis)에 으|한 perica­ rdial friction rub

• 단백뇨,저알부민혈증 • BUN, Cr 증가 • 고칼륨혈증, 대사성 산증 • 고인산혈증,저칼슘혈증 • 빈혈(주로 EPO 부족에 의한 normocytic anemia)

3

Source : From Chertow. Note : S , third heart sound.

難 Edema를 일으길 수 91는 대표적인 진% 3가I■ 지。 Congestive heart failure, Liver cirrhosis, Nephrotic syndrome (혹은 renal failure)의 심떨과 원면된 내용0』니다. 이 3가지



파트 Hyponatremia, 소회ᅵ기 파트의

Ascites의 정1절진1난에서도 또 如게 등7강향니다. - Edema가

는 혹!:자에서 Jaundice와 '살이 I t 의 이상을 나타내는 소 이

동1반되면 Liver cirrhosis

- Orthopnea, S3 gallop 등 심7당의 이상을 나타내는 소 이 - Uremic symptom, serum Cr의 증가 등이 동1만된

동41:되1면 Congestive heart failure

우 Nephrotic syndrome(혹은 renal failure)라고 기억해

두시면 면업니다. 참고하시라고 녕어둔 표이니 너우 자서!하'게는 보지 (않으셔도 접니다. 孝 나중on

^우크에서 나오지망 기억하세요.

- Hyponatremia(hypoNa)+edema+ 요중 Na(UNa) | — acute or chronic renal failure(ARF or CRF) - HypoNa+ edema+ UNa | (... effective blood volume "U소로 RAAS f 되어 Na+ 재흡수 } 울 재홈수 | ) —»• CHF(congestive heart failure), LC(liver cirrhosis), NS(nephrᄋtic syndrome)

4.

심부전의 진단 전형적인 증상을 보이는 환자에게서 HF의 진단은 비교적 쉽지만 sensitivity, specificity가 높지는 않다. 중요한 것은 고위험군 환자 fr 에게서 HF의 발생음 예견하고 의심하는 것이며,이러한 환자들에게서 HF 의 증상,징후가 발견될 경우 아래(5. 검사)에 니-열된 검사들을 시행하도록 한다. 傘 아래 진단 기준은 이국oil서 시6辟 Framingham cohort study에서 얻어진 "첫0』니다. H arrisonlS ^부S 는 WI■졌 고 a 우실 필요도 없지만, 모두 심부전에서 나타1날 수 앗는 증상이나 소경들을 포항하고 앗으으로 한 민 보시고 가벼운 마음으로 지나가세요(외우는 방법은 CRASHPIN0』니다. 굵은 글자로 표시해 농앗습니다).

Framingham criteria for diagnosis of congestive heart failure* Paroxysmal noct니rnal dyspnea Neck vein distension Rales Major criteria

Cardiomegaly Ac 니te p 니Imonary edema

S3 gallop Increased venous pressure( > 16cmH20) positive Hepatojug니lar reflux extremity edema night co 니gh dyspnea on exertion Minor criteria

hepatomegaly pleural effusion vital capacity red니ced by one-third from normal tachycardia( > 120bpm)

Major or minor

weight loss > 4.5kg over 5days treatment

* Major criteria 2개 이상 또는 major criteria 1개 이상+ minor criteria 2개 이상 해당되면 진단

5. 검사 1) 12-lead ECG : LV hypertrophy나 이전 M I의 흔적(Q waves)을 찾을 수 있음 2) Chest X-ray : cardiomegaly, pulmonary congestion 3) BNP(Brain Natriuretic Peptide) : 호흡곤란을 호소히는 환자에서 그 원인이 cardiogenic인지 primary pulmonary origin인지 감별하는 데 도움(민감도와 특이도가 높음), 심부전에서 높은 값을 보일수록

€-

심부전



poor prognosis. 그러나 지속적인 즉정은 현재 권고되지 않는다.

ᅳ 문제에서 환자가 호흡곤란을 호소하고 B N P 수치가 높은 경우 심부전이라고 생각한 채 접근하기 시작하면 됩니다. 4) Exercise testing : routine하게 시행되지는 않으나 밀ᅵ기의 심부전 환자에게 심장이식수술 필요성을

판단하기 우1한 보조 검사로 활용가능 5) LV 기능을 평가하기 위한 방법들 ᅳ EF의 측정 (1) 2 D 심초음파 검사(± 프_플러 혈류 검사) : 매우 중요. 진단,원인 감별,severity 판정

(2) M RI : LV 질량과 부피를 평가하는 데 있어 서 gold standard Harrison 2Q만,pp. 1763~ 1768

01



문 제 해 설

-c/\>

74세 여자가 한 달 전부터 걸으면 숨이 차서 병원에 왔다. 누우면 숨이

01 고령의 여자 환자가 한 달 전부터 발생한 호

차고^ 새벽에도 숨이 차서 장에서 깬다. 7 년 전부터 혈압강하제와 혈당강

흡곤란 증상을 주소로 내원한 증례입니다. 기 조F호흡 소견과 발작야간호흡곤란 소견이 관찰 되고 있으며,제3심음과 함께 양쪽 등아래쪽

ᅵ / 하제를 복용하고 있다. 혈압 122/기 mmHg, 맥박 87 회/ 분, 호흡 21 호 분, 체온 36.4°C 이다. 가슴 청진할 때 제3심음이 들리고, 양쪽 등쪽 아랫 부분에서 거품소리가 들린다. 가슴 X선 사진과 심전도이다. 혈액 검사는?

에서 들리는 거품소리는 심부전에 의한 폐부 종을 유추할 수 있습니다. 시행한 가슴 X선 상에서도 C/T ratio가 0.5를 초과하는 심비대 소견이 관찰되고 있으며,양측 폐야의 음영이 전반적으로 증가되어 있어서 폐부종에 합당한 소견을 나타내고 있습니다. 시행한 심전도 상 에서는 lead V I에서 토끼귀 모양의 rSR’0| 관찰되고,lead V6에서 slurred S 소견이 관 찰되어서 RBBB가 있을 가능성이 높습니다. 심부전에 의해서 발생한 증상이므로,심부전 의 중증도를 판별하고 예후 예측에 도움이 되 는 노|나트륨이뇨펩티드(BNP)를 측정하는 것 이 가장 적절한 검사입니다. Harrison 20판,pp. 1763〜 1768

1) D-이량체

2) 트로포닌-T

3) C-반응단백질

4) 크레아틴키나아제

5) 뇌나트륨이뇨맵티J

0

2020년 대비 PACIFIC KMLE i

순환기

13

® 75세 남자가 2일 전부터 숨이 더 찬다며 병원에 왔다. 2일 전에 감기 증

상이 있어 치료를 받았으나,점차 심해져 누워서 잠을 잘 수 없을 정도 로 숨이 찬다고 한다. 기침과 소량의 횐색 가래가 있었다. 6 년 전부터 만 성폐질환으로 익물치료를 받는다고 한다. 48 갑 년 흡연자이다. 혈압 132/86mmHg, 맥박 96회/ 분(불규칙),호흡 29회/ 분,체온 37.10C 이다.

목정맥이 늘어나 있다. 청진할 때 심장 박동이 불규칙하며,양쪽 아랫 가 슴에서 쌕쌕거림이 들린다. 가슴 X선 사진과 심전도이다. 혈액검사는 다 음과 같다. 검사는? 혈색소 15.7g/dL, 백혈구 6,300/mm3, 혈소판 220,000/mm3, 뇌나트륨이 뇨펩티드 580ng/L(참고치, 기관지내시경

3) 심초음파검사

4) 메타콜린기 관지유발검사

5) 기*슴 고해상컴퓨터단층촬영

@

€-

68세 남자가 3 개월 전부터 호흡곤란이 심해져서 응급실에 왔다. 40갑 -

년의 흡연자였다. 혈압 110/70mmHg,맥박 66회/ 분,호흡 24회/ 분, 체 온 36.5도였다. 가슴 청진에서 심음은 전반적으로 작게 들렸고 심장 끝 에서 제3 심음이 들렸다. 정강뼈앞 오목부종이 만져졌다. 가슴 X선 人유진 과 심전도이다. 감별을 위한 검사는?

심부^ᅳᅳ

0 3 호흡곤란을 유발할 수 있는 주된 장기 2가지 는 폐와 심장입니다만,주어진 증상과 징후로 는 환자의 호흡곤란의 원인이 폐인지 심장인 지조차 정확히 알 수 없어 다소 당황스러울 수도 있습니다. 그러나 ‘환자의 호흡곤란을 유발한 원인이 폐에 있는지,심장에 있는지를 어떻게 감별할 것인가?’가 바로 이 문제의 출 제 의도인 것으로 생각됩니다. 30갑• 년의 흡연력은 폐질환의 위험인자이기 도 하지만, 허혈성 심질환의 위험인자이기도 합니다. 심음이 작게 들리고, 제3심음이 들리 며, 오목부종이 있는 것, ECG 상 조[심실 비 대가 나타나는 것은 심부전의 가능성을 시사 합니다. 반면 가슴 X선 사진에서는 폐기종으 로 인한 CPA blunting이 관찰되고 있습니다. 더구나 환자가 보이는 심부전이, 이보다 선행 한 COPES 인해 이차적으로 발생한 cor pulmonale일 7|능성을 생각하면 머리는 더욱 복잡해집니다. Harrison에서 제시한 호흡곤란 환자의 진단 알고리즘에 따르면,충분한 문진 및 신체 진 찰만으로도 진단이 명백한 경우가 대부분이라 고 합니다. 그러나 진단이 여의치 않을 경우, 심초음파, 폐기능 검사 등 호흡곤란의 심장성 원인과 폐성 원인을 감별할 수 있는 추가적인 검사를 시행할 것을 권고하고 있습니다. 문제의 보기 중 호흡곤란을 호소하는 환자에 서 그 원인이 심장성인지,폐성인지를 구별하 는 데 도움이 되는 검사는 BNP뿐입니다. 1), 2),4)를 답으로 하기에는 환자의 병력 상 폐 색전증이나 심근경색(또는 심근염)을 의심할 근거가 매우 부족합니다. Harrison 20ᅵ 만 ,pp.l763~1768

1)

혈장 D-이량체

2) 혈청 미오글로빈

3) 적혈구침강계수

4) 혈청 트로포닌 I

5) 혈청 뇌나트륨이뇨펩티드

04 —

n _________________________________________________________________________________________________________ i a

a

_

67세 남자가 호흡곤란으로 왔다. 5 년 전 고혈압으로 진단받고 혈입약을

복용중이다. 흉부 X선 검人ᅵ서 심장은 커져 있었으나 폐는 이상소견이 관찰되지 않았다. 다음 검사 중 진단에 도움이 도ᅵ는 biomarker는? 1)

CRP

3) Uric acid 5) TNF receptor

2) Troponin-T 4) Pro-BNP

0 4 심비대와 호흡곤란이 있어 심부전이 의심되는 상황입니다. BNP와 pro-BNP는 EF이 감소 한 심부전에서 EF이 정상인 심부전에 비해 특이적으로 증가합니다. 호흡곤란을 호소하는 환자에서 원인이 cardiogenic인지 primary pulmonary origin인지 감별히는 데 도움이 도ᅵ고심부전에서 높을수록 poor prognosis를 보입니다. Harrison 20맏,pp. 1763~ 1768



2020 년 대비 PACIFIC KMLE

순환기

f"----

®



60세 여자가 운동 중 발생한 호흡곤란과 흉부 불쾌감을 주소로 내원하였

다. 15년 전 당뇨와 고혈압을 진단받았고 내원 당시 혈압 150/110mmHg, 맥박 94/min, 호흡수 22/min 이었다. 심장 청진에서 제3심음이 들렸고 심초음파에서 조ᅡ심실이 두꺼워져 있었으나 심실벽 운동의 이상 양상은 관찰되지 않았고 조h심실구출률은 55%이었다. 진단은? 1) 확장심근병증

2) 불안정 협심증

3) 심근경색

4) 급성폐부종

이 환자와 같이 폐부종과 같은 심부전 증상이 있는데 EF가 50% 이상인 경우 이완기 심근 장애를 생각해 볼 수 있습니다. 1) 확장심근병증은 수축기 심부전의 양상을 보이므로 태가 감소하는 경우가 많습니다. Harrison 20ᅵ 만 ,pp.l763~1768

0 6 J— _______________________________________________ 그



70세 여자가 계단을 오를 때 나타나는 호흡곤란을 주소로 내원하였다.

증상은 6개월 전부터 나타났고 5 년 전 고혈압을 진단받아 약을 복용 중 이었다. 심장 청진 상 이상 소견은 발견되지 않았고 검사 결고h는 다음과 같다. 원인으로 맞는 것은? EF 60%, septum 두께 12mm(

02 만성 심부전 약의 부작용에 관한 문제입니다. ACEi와 ARB는 기전이 매우 유사한 약제이 지만,ACEi 에는 대표적인 부작용이 있으며, 해당 부작용이 발생하였을 경우 ARB로 바꿔 투약하게 됩니다. ACEi만의 부작용으로 dry cough, angioedema가 발생할 수 있습니다이 환자의 경우에도 다른 이상소견 없이 마른 기침만 발생한 것은 1개월 전 바꾼 약제 중 ACEi인 enalapril의 부작용 때문이라고 판단 할 수 있겠습니다. 심부전이 발생하여 처방한 약물이기 때문에 이뇨제,베타차단제, ACEi(또는 ARB)의 약 물 조합은 변경할 필요가 없어 보입니다. 따 라서 ACEi를 쇼때로 Uf꾸는 것이 가장 적절 할 것으로 판단됩니다. 문제에서 제시된 각 약물에 해당하는 분류는 알고 있어야 합니다. ① Furosemide : 고리작용이뇨제 Carvedilol : 베타차단제 ②< ③]Enalapril : ACE inhibitor ④ ]Losartan : Angiotensin I Receptor an­ tagonist ⑤ Spironolactone : Aldosterone antago­ nist ⑥ Digoxin : 강심제 Harrison 2ᄋ만,pp.l769~1779

1) 푸로세미드 증량 2) 에날리-프릴을로살탄(losartan)으로 교체 3) 카베딜롤 증량 4) 스피로놀락톤(spironolactone) 추가 5) 디곡신(digoxin) 추가

03



58세 남자가 3 달 전부터 숨이 차서 병원에 왔다. 혈압 134/80mmHg,

맥박 98호 ᅵ / 분, 호흡 23회/ 분이었다. 목정맥이 늘어나 있었고, 청진에서 제3심음이 들렸으며, 간비대와 양쪽 정강이의 오목부종이 있었다. 흉부 X선 검人N 서 폐부종 소견을 확인할 수 있었고, 심 초음파 검사상 조빙

실 확대 소견, 조ᅡ심실 박출률 감소 소견을 확인할 수 있었다. 이 환자Oil 서 장기 생존율을 높이는 약제는? 1) 디곡신

2) 이뇨제

3) 도부타민

4) 칼슘통로차단제

5) 안지오텐신 전환효소 억제제

0 3 만성 심부전 환자의 장기 생존율을 증가시키 는 약제들 중 기억해야할 다섯 가지입니다. ① ACE inhibitor : captopril, enalapril, lisinopril, ramipril, trandolapril ② Angiotensin H receptor blocker(ARB) : valsartan, candesartan, irbesartan, los­ artan ③ /?-blocker : carvedilol, bisoprolol, me­ toprolol succinate CR ④ Aldosterone antagonist : spironolactone, eplerenone ⑤ Direct vasodilator : hydralazine+isos­ orbide dinitrate Harrison 20만,pp.l769~1779

7 #



2020 년 대비 PACIFIC KMLE

순환기

QA X - J _________________________________________________ B !내 1ᅵ 73세 남자가 6개월 전부터 서서히 숨이 차서 왔다. 혈압 120/80mmHg,

맥박 73회/ 분, 호흡 19회/ 분, 체온 36.5도였다. 가슴청진에서 거품소리는 들리지 않았고 제3심음이 들렸다. 정강뼈앞 오목부종은 없었다. 심전도 人h진이다. 심초음파검사 결괴는 다음과 같았다. 치료는? 확장기말 왼심실 내경 57mm(참고치,36〜54),왼심실박출률 36%{참 고치,>55)

0 4 6 개월 전부터 서서히 진행된 호흡곤란,제 3 심음,심초음파 상 L V ED D 의 확장,LVEF 감소 등을 확인할 때 전형적인 만성 수축기 삼부전 증례입니다. 가슴청진에서 거품소리가 들리지 않고 오목부종이 없어 fluid retention 이 없는 상태아므로 가장 먼저 ACEi+ BB 치 료를 시직하여 RAAS activation에 따른 심장 의 remodeling을 억제합니다(N Y H A I 〜 IV 의 모든 stage에서 효과). 심전도에서 확인되는 소견은 wide QRS 로써 심부전에 따른 심실비대로 발생한 심실전도지 연에 의한 것으로 생각됩니다. Harrison 201만, pp,1769~1779

1)

디지탈리스

2) 아미노필린

3)

알파차단제

4) 칼슘통로 차단제

5) 앤지오텐신전환효소 억제제





___________________________________________________________________________________________

62세 여자환자가 3 개월 전부터 숨이 차다고 내원하였다. 새벽이 되면

호흡곤란은 더 심해진다고 하였고 10년 전부터 고혈압약 복용 중이었 다. 혈압 150/90 맥박수 95회 규칙적인 박동이었으며,종아리에 오목부 종,경정맥 확장이 관찰되었다. 흉부 방사선 소견상 폐부종,심비대 관찰 되었으며 이뇨제 투여받고 증상 호전되었다. 치료는? BUN/Cr 11/0.9, Na/K 139/4.1 1)

헤파린

2) 안지오텐신 전환효소 억제제

3)

디지탈리스

4) 칼슘채널차단제

5) 도부타민

0 5 호흡곤란,오목부종,경정맥 확장을 보이는 증 상을 동반한 만성심부전 환자입니다. 아뇨제 투 여 후에 환자의 fluid retention이 해결되었으 므로 다음 치료로 장기 생존율을 향상시카는 ACEi 와 BB 를 투여해야 합니다. 환지는 陪 도 정상아고 renal function에도 문제가 없기 때문에 ACEi를 人|용할 수 았습니다. 참고로 CCB 는 심부전 환자에서 人1용이 권장 되지 않고 디지탈리스의 경우 A F ,AFL 이 동 반되어 rate control하는 목적 이외에는 drug level 조절 등이 어려워 현재는 많이 人[용되 지 않습니다. Harrison 20만,pp. 1769~ 1779

53세 남자가 호흡곤란으로 왔다. 증상은 1년 전 시작되었고_ 2개월 전 등

산 중 호흡곤란이 심해져 도중에 내려왔다고 한다. 혈압은 125/80mmHg, 심박수 115회/분이었고 청진 상 제3심음이 들렸다. 이 환자의 사망률을

06 심부전에서 장기 생존에 도움이 도|는 약제에 는 다음과 같은 것이 있다고 알려져 있어요. ① ACE inhibitor : captopril, enalapril, lisinopril, ramipril, trandolapril

② Angiotensin I receptor blocker(ARB) :

줄일 수 있는 약으로 맞는 것은?

valsartan, candesartan, irbesartan, losartan

③ jff-blocker : carvedilol, bisoprolol, metop­ rolol succinate CR

④ Aldosterone antagonist : spironolactone, eplerenone

⑤ Direct vasodilator : hydralazine+isosorbide dinitrate

답은 ACE inhibitor인 enalapril 입니다. Harrison 20 판,pp. 1769〜 1779

furosemide

2) nifedipine

enalapril

4) dopamine

digoxin

r

07



심부전증으로 functional class IE인 57세 남자에게 실시한 심초음파 검 人 서 심박출 계수는 28%이었으며,심전도에서 wide QRS(LBBB)소 견을 보였다. 약물치료에서도 증상호전이 없었다. 치료로 최근 가장 추 천되는 것은? 1) ACE 억제제 2) 심장이식 3) 심근 수축제 정주 4) 심장재동기화 치료(biventricular pacing) 5) 베타 차단제

07 문제에서 약물치료에 증상이 호전되지 않았다 고 하였으므로 1,3, 5는 정답이 아닙니다. 결 국 심장이식과 심장재동기화 치료 중 하나인 데 각각의 적응증은 다음과 같습니다. ① 심장 이식 :NYHA class IV이고 기대 수 명이 1년 이하인 심한 불응성 심부전 ② 심장재동기화 치료 : 약물치료에도 증상이 호전도! 지 않고 EF150ms일 때, LBBB 가 있 는 경우에는 QRS> 130ms에서 실시한다고 합니다. LBBB 가 동반되면 더더욱 심장재동 기화 치료를 생각해볼 수 있겠습니다. Harrison 20만,pp.l769~1779

^

2020 년 대비 PACIFIC KMLE

순환기

08

Q

68세 남자가 호흡곤란으로 입원하였다. 3년 전 심근경색으로 관상동맥 중재술을 받았다. 흉부 청진 상 수포음이 들렸고 심장 초음파에서 확장 기말 조ᅵ심실 안지름이 60mm이었고, 구혈률{ejection fraction)은 30% 이었다. 며칠 간 입원 후 증상이 호전되었을 때 장기생존율을 증가시키 기 위해 쓸 수 있는 약은? 가. 칼슘통로 차단제 나. 앤지오텐신 전환효소 억제제(ACE inhibitor)

0 8 심부전 환자의 장기 생존율을 증가시키는 약 제 다섯 가지 ① ACE inhibitor : captopril, enalapril, lisi­ nopril, ramipril, trandolapril ② Angiotensin H receptor blocker(ARB) : valsartan, candesartan, irbesartan, losartan

(D /?-blocker : carvedilol, bisoprolol, metoprolol succinate CR

④ Aldosterone antagonist : spironolactone, ⑤ Direct vasodilator : hydralazine +isosor-

라. 베타 차단제

bide dinitrate

1) 가,나. 다

2) 가’ 다

4) 라

5) 가,나,다, 라

(2 1 2 ) o o o ^

----------------------cA)

eplerenone

다. 디곡신(digoxin)

©ANSWER

문 제 해 설

3) 나,라

이•④ 02.② 03.⑤ 04.⑤ 05.② 06.③ 07.④ 08.③

가. 칼슘통로 차단제는 심부전에서는 人}용하 지 않습니다다. 디곡신은 사망률을 감소시키지 않아요. Harrison 201만,pp.l769~1779

심 ^ᅳ



Explanation

^

A

*

c

u

t

e

h

e

a

r

t

f

a

i l u

r

e

a

n

d

a

c

u

t

e

p

u

l m

o

n

a

r

y

e

d

e

m

a

심부7a의 치료를 단지 '4ᅳ 卜 성심부전'의 ■정우만 생74해서는 안 평니다. cgc莫 0 ! ^ 요cj[에 의해 심부전의 보상 작용이 붕괴되는 ^우 가 91•기 때문잉니다. 이러한 ^우 를 심 부 ^의 급성 악 ^라 하며’ 이 ^우 생기 생존율'을 높이는 치료보다, '증성 ih ^ 'o i 우성있니다. 병력 상 심부전이 의심되는데, m 부종으로 인한 호흡곤란 등의 증상이 b ᅡ르게 악^ 되91다면 원래의 심부^

치료와는 o p j; 다르계 험근하는 칫이 필요형니다.

특히 beta-blocker 의 정우 만성 심부7S 매와는 Normal

급성 성부7a이I서는 금기라는 정 기억해 두세요.

Chronic HF

Death

A = q e> e a H

c

#

만성 심부전은 진기\卜 중^ 중'7uᅡ

^

기능이 지속적으로 ^어지는 상태를 의Didᅡ고, 급성 심부전은 마치 통풍 방작처령

하는 attack이라고 생각하시면 쉽습니다. 만성 심부전의 치료 목표가 생존율 형생이라면’ 주로 급

성 폐부종으로 나타나는 급성 심부전은 증^ pH성이 우선이' 있죠.

1. 치료의 목표 1) 증상을 일으킨 혈역학적 상태를 안정화(즉, profile A로 만들어야 항!) 2) 가역적인 원인을 찾아서 치료 3) 질병의 진행 및 재발을 막기 위한 효과적인 약물 요법

2. 급성 심부전의 원인

*

이^ 상%에서 심부전이 말생할 수 9P41구나 정도로만 보시면 청니다.

1) Ischemic heart disease ~ 100 : Nitroglycerin(preload J,) - SBP 70〜 100 & shock 증상 없음 : Dobutamine(심실수축력 T, HR ᄂ 말초혈관이완 7]능성 ^ 의)

- SBP 나진이다. 치료는?

혈액요소질소/크레아티닌 18/1.0mg/dL,트로포닌 T 3ng/L(참고치,0 〜 14), 혈장뇌나트륨이뇨펩티드 730ng/L(참고치,pulmonary vein —> LA 의 순환은 매우 낮은 상태의 압력에서도 LV stroke volume과 동일한 양의 혈액을 감당할 수 있음 3) 운동이나 심호흡과 같은 RV 로의 혈액 유입이 늘어나는 상황쎄서도 아주 적은 압력 상승만으로도 pulmonary circulation 유지가 가능함 4) 그러나 pulmonary circulation의 afterload(pulmonary vascular resistance>가 상승하는 경우 RV enlargement가 일어나게 됨(COPD, lung resection, restrictive lung disease, hypoxia, acidosis)

2. Cor pulmonale의 원인 ♦

Cor pulmonale를 초a 하는 RV afterload의 상승은 크■게 두 가지로 분류할 수 91어요.

1)

Pulmonary vascular disease : pulmonary blood flow j 초래 (1) 일반적 특징

① Pulmonary hypertension0] parenchymal disease보다 심함 ② Pulmonary embolism, pulmonary vasculitis, 고지대에 사는 사•람에서 생기는 pulmonary vasoconstriction, L-to-R shunt(ASD, VSD, PDA)

③ 원인을 규명하기 어려운 폐혈관압 상승을 보이는 경우를 primary pulmonary hypertension이 함 (2) Acute cor pulmonale : pulmonary embolism 이 대부분

① 높아진 pulmonary vascular resistance 때문에 RV 가 폐로 혈액을 제대로 보내주지 못함 一 결국 cardiac output

[

② 임상양상 • Sudden onset of severe dyspnea, cardiovascular collapse, venous thrombosis history • Pallor, sweating, BP [ , 빠르고 낮은 pulse • Neck vein distention with prominent v wave • Liver : pulsatile, distended, tender • Systolic murmur of tricuspid regurgitation, S4 gallop • PaC>2 j (ventilation/perfusion mismatch), PaCCh | (hyperventilation)



심부전

③ 치료 • Acute RV infarction에 의한 RV failure와 비슷함 • Cautious blood volume expansion(IV saline infusion) : R V preload | ,C O • Anticoagulation : 보통 DVT 와 같이 혈류 정체로 생긴 red clot의 색전이 원인이므로 • 100% Oz inhalation : 산 소 포 화 도 | ᅳ >• vasoconstriction [ ~ *• afterload [

• 혈역학적 불안정 시 thrombolytics or thrombolytics 금기시 thromboembolectomy

傘 Pulmonary emb이ism의 이보다 더 자세한 치료는 2그헌호흡기를 참조하십시오. (3)

Chronic cor pulmonale

① Recurrent pulmonary embolism, primary pulmonary hypertension 등에 의함 ② Dyspnea, tachypnea, unproductive cough, anterior chest pain, hepatomegaly, ankle edema ③ Cyanosis, RV heave(left sternal border/epigastrium), S2 sound | (흡기 시 narrow, fixed S2 ) 가능 ④ 경정맥파 : prominent a/v wave, hepatojugular reflux ⑤ Chest PA : 폐혈관 및 h ik in g 확장 소견 ©

ECG : P pulmonale, right axis deviation, RVH

뾰족한 P

© Echocardiography : tricuspid valve flow velocity 등 평

⑧ Cardiac catheterization : pulmonary vascular pressure, congenital disease 여부 등 평가 2)

Parenchymal pulmonary disease 傘 상소포화도가 정소하면 폐서I동맥이 수축하여 pulmonary HTN을 일으1입니다. 따라서 산소포화도를 증가시 키는 0 2가 치료가 도P4[죠. (1) CO PD : hyperinflated lu 매:이 폐혈관을 압박 (2) Restrictive lung disease, lu 폐 resection, Tb sequelae(성인 TB 는 fibrosis) (3) RV failure보다 폐질환 자체에 의한 poor prognosis (4) 치료

① 급성 감염,hypoxia/acidosis 등의 치료 ② Long-term O 2 therapy : severe CO PD 환자에서 도음 ③ Bronchodilator, antibiotics, diuretics Harrison 20®];, pp. 1768~ 1769

2020 년 대비 PACIFIC KMLE

순환기

I 심부전 한눈에 보기 LV failure

RV failure 1) Abdominal (1) Anorexia, nausea (2) Abdominal pain, fullness (3) Jaundice —*• Hepatic congestion

증상

1) Jug니 lar venous distension

징후

2) Congestive hepatomegaly 3) Pitting edema of leg 4) Hydrothorax, ascites

1) Respiratory (1) Exertional dyspnea

progression

(2) Orthopnea (3) Paroxysmal nocturnal dyspnea — p니Imonary venous congestion 2) Fatigue, weakness, | exercise capacity — decreased cardiac ᄋ utp니t 1) 흉부 진^소견 (1) 청진 : moist, inspiratory, crepitant rale (2) 타진 :dullness — pulmonary edema 2) Prerenal azotemia : | urine output

Tachycardia, S3 gallop 1) 2D echocardiography(+doppler) : m/i 검사

2) Chest X-ray :cardiomegaly, pulmonary congestion 3) BNP(brain natriuretic peptide) 本 Chronic HF 1) General measure :Na 제한 등 2) ACE inhibitor or ARB(필쉬

치료

3) 4) 5) 6)

/0-blocker(필수) Triple therapy : (low dose) spironolactone, hydralazine + isosorbide dinitrate+ digitalis 중 하나 추가 Diuretics : thiazide, metolazone, loop diuretics(fluid retention시) ARNI :위의 기존 치료에도 증상이 지속되고 LVEF < 35%로 호전되지 않는 경우 ACEI, ARB를 대체 가능

7) Ivabradine : 위의 기존 치료에도 증상이 지속되고, LVEF < 35%로 호전되지 않고 HR가 70회 이상인 경우 本 Acute HF & pulmonary edema

Poor prognosis

1) Morphine 2) ACE inhibitor

5) Dopamine / d ᄋ b니tamine, digoxin 6) ᄋxygenation(산소공급, 양압 환기)

3) IABP :acute MR/VSD 시 4) Loop diuretics :f니 rosemide

ᅳ MAIL DONI 매일 도니까 폐부종이 생기지..?!!

1) 환자의 functional status(m/i) 2) 고령, 남성,낮은 LVEF, 기저 관상동맥질환

7) Nitrate :nitroglycerin, isosorbide dinitrate

IT :

64세 여자가 3 년 전부터 숨이 차서 왔다. 50갑 년의 흡연력이 있었다. 2층을 걸어 올라가기 힘들다고 하였으며 앉이숨쉬기나 발작성 야간호흡

곤란은 없었다고 하였다. 혈압 125/60mmHg, 맥박 103회/ 분, 호흡 23 회/ 분, 체온 36.6도였다. 가슴 청진에서 복장뼈 왼쪽 옆 다섯 번째 갈비 人때에서 3/6 도의 수축기심잡음이 들렸고 들숨에 증가되었다. 호흡음이 감소되어 있고 거품소리는 들리지 않았다.양쪽 정강뼈앞 오목부종이 있 었다. 가슴 X선 사진과 심전도이다. 진단은?



심부ᄊ전



문제를 처음에 보고 바로 감별진단을 생각하 기 어려운 증례입니다. 주소가 호흡곤란이라 는 점, X-ray에서 cardiomegaly가 관찰된다 는 점에서 heart failure를 생각해볼 수 있습 니다. 그러나 발작성 야간 호흡곤란이 없고 앉아숨쉬기 또한 없다는 점에서 흔히 Heart failure에서 생각히는 조f심실부전은 아니라는 것을 생각해볼 수 있습니다. 왼쪽 5th ICS에서 수축기 잡음이 들리고 들 숨에 증가되는 경우 가장 먼저 생각해볼 수 있는 것이 삼첨판폐쇄부전증0.12s), notched P wave(lead I ,I ) ② P-terminal force >0.04 in V,

40ms 80ms ^ 120ms MS에서 LAE 발생하여 LA의 v이tage가 우세해짐

(2) Pulmonary hypertension이 심한 경우 : r ^ h t axis deviation, RV H (3) Atrial fibrillation

o

o 233

2020 년 대비 PACIFIC KMLE

순환기

傘 MS로 0d해 심앵벽이 늘에남에 따라 성경 진도로가 ^ 어지-게 되면 순환회로가 진 생 \[다 고 현니다. 이 때문 에 AF가 장 동H ta w AF으로 인해 생앞이 효율적으로 수축하지 못하고 부들부들 ^고 망 0 ^ 1 되어 열B 정舶卜 생기고, 이로 0於 H 농도가 높아진 coagulation fact이•가 반응을 일으려 fibrin이 형성, 주 ^의 RBC 를 포협하여 red clot-§ 4t들ᄀ비 평니다. ° 도 1■서 anti-coagulation Tx.를 형니다. ♦

Afib의 발 ^은 고령의 환자ᄋ이 H 치명적행니다 그 이유에 디^ 서 l ^ d r n i 생각해보' 74!습니다. 조배태에서 조매/표로 열류가 가는 기 ^은 크-hi 2가지0옵니다. H ᅵ ^째 는 화Ac!피의 확장으로 0d해 아지면서 혈류가

이 1낮

가는 기 ^입 니 다 ᅵ 두 면째는 atrial 너改에 의해 상양이 직협 수축을 하면서 혈류가 흘

a 가는 기^ 0』니다. H. 먼째 기전은

이 않마나 진 吧 적 으 로 늘어나는 지에 의해 명항을 받고, 심4^에

서 심/표로 흘러가는 誓 뛰 상딩평이 H 먼째 기전에 의한 칫이며, 두 민째 기7aᄋ d atrial kick에 의해서 심 실로 i : a 가는 혈액은 비중이 상대적으로 작은 면입니다. 고령의 환지■일수록, 파 ᅳ먼™ 기7a을 가능하게 하 는 심실의 이완 기능이 벌어지으로, 두 41째 기전인 atrial kick에 의존을 하게 되는데, afib이 러한 atrial kick이 알어나지

아서 심실로 충분한 형QJj(preload)이 공급되지 않게 평니다. 가병"게 9A고 범

어가세요!

2) Echocardiography (1) M-mode echocardiography

① EF slope 감소(valve가 닫히는 데 시간이 오래 걸린다는 의미) \'l\ Valve

thickening & calcification (두껍고 지저분해 보이는 slope)

(,: i) Anterior m otion of posterior valve leaflet(아래쪽에 있는 echo 음영이 post, leaflet)

normal

thicken e d im m obile leaflets

M-mode에서 정상 MV의 움직임은 early LV dilatation(E), late LA contractbn(A)에 의해 구성됨

; 2 3 a)o



판막질환

(2) 2D echocardiography

2D echocardiography in MS

thrombus in LA(transesophageal echocardiography)

두꺼워진 MV, LA enlargement를 관찰할 수 있 음(LV는 상대적으로 정상)

Short axis view of MS planimetry for MV orifice area 1.09cm2. M-mode : restricted opening of MV in diastole, with classic diastolic rumbling murmur

3)

Chest PA (1) Straightening of the left cardiac borderCO) : LA appendage가 돌출되어 보이기 때문 (2) Prominence of the main pulmonary arteries (3) Dilatation of the upper lobe pulmonary veins(}) (4) Double contour by LA enlargement^/ ^ (5) Kerley’s B lines in lower lobe( ▼) : interlobar septal distention with lymphatic edema-pLA >20mmHg를 의미

c - 2020년 대비 PACIFIC KMLE

순환기

傘 사진에서 '각 소 ^이 부 엇 ^지 정도만 보고 녕어가세요.

6 . Severity of mitral stenosis 傘 MS에서 내과적 치료 이외에 interventional approach를 철:지의 여부는 severity가 '할정헌당니다. 아래 기준을 다 외우기는 형듭니다. severe의 기준1만 확 少 1 *알아두고 녕어가세요. valve area가 1cm2 d idJPI라는 정이 중 요형니다.

Mean gradient, mmHg

Valve area, cm2

PA systole pressure, mmHg

Normal

0

4.ᄋ〜6.0

0 5 MS 의 심초음파는 자주 나오므로 잘 익혀두

어야 합니다. EF slope의 감소(valve가 닫히 는 데 시간이 오래 걸린다는 의미)가 가장 특 징적이랍니다. MS의 특징적인 청진 소견 3가자를 꼭 기억해 ^ 세요.

뼈 전

① 이,P2 t ② Opening snap(OS) ③ Apex에서 들리는 Low-pitched, diastolic, rumbling murmur

^

\■서

참고로 5) 박출성 클릭(midsystolic click)은 mitral valve prolapse의 특징적인 심잡음입니 다. 수축기에 mitral valve가 심실의 입력을 견 디다 못해 심방 쪽으로 밀려서 발생하는 소리 입니다. Harrison 20판,pp.l813~1817

5)

저U 심음 증가

2) 제2 심음 감소

제4 심음

4) 연속성 잡음

박출성 클릭

06



호흡곤란을 주소로 내원한 24세 여자의 심도자 소견이다. 진단은?

심전도

0 6 정상적으로는 이완기 LA 와 LV 의 압력은 거

의 같습니다. 확장기에 조ᅡ심방과 조ᅡ심실이 큰 압력차를 보이므로 승모판 협착증을 의심해볼 수 있어요. 이완기 심방에서 심실로의 혈류가 방해를 받기 때문이죠. Harrison 2Q만,pp. 1813~ 1817 심장학교과서,대한순환기학회,pp. 332 〜 342

mmHg -1 0 0







매 ᄀ 기 아! !



시 口 좌

o h b d



확장기압교차

Catheter 검사 중의 압기록

1) ASD 3) AR 5) MVP

©

o

cy

2) AS 4) MS

07

문 제 해 설

Q

-------------------------------------------------- ᄋ요 。

37세 여자가 운동 시 호흡곤란을 주소로 내원하였다. 심도자 검사 결과

가 다음과 같을 때 가장 가능성 높은 진단은?

0 7 Femoral vein을 따라 집어넣은 catheter는 pulmonary artery# 지14 wedge pressure# 구하기 위해 끝이 폐 속 어딘가에 박혀 있을 것이고,femoral artery와 aorta를 따라 들어 간 catheter는 LV까지 들어가 있을 거예요. PCWP는 대개 LA pressure의 근사치를 보 입니다. 정상적으로는 이완기 LA와 LV의 압 력은 거의 같습니다. 확장기에 조I심방과 조심 실이 큰 압력 차이를 보이므로 승모판 협착증 을 의심해볼 수 있어요. 이완기 심방에서 심 실로의 혈류가 방해를 받기 때문이죠. 더군다 나 심전도상 승모판 협착에서 잘 동반되는 심 방세동이 보이고 있습니다. Harrison 2Q만,pp.l813~1817 심장학교과서,대한순환기학회,pp.332~342

승모판 역류

2) 승모판 협착

앱스타인 기형

4)

삼첨판 협착

대동맥판 협착

08

B

46세의 여자가 심한 호흡곤란으로 내원하였다. 과거 류마티스 열을 앓

았던 적이 있고 심도자 소견은 [ᅡ음과 같다. 이 질환에서 나타날 수 있 는 징후로 맞는 것은?

가. 개방음 청취

나. 제1심음 증가

다. 이완기 심잡음

라. 좌심방 비대

ID: 91 AD: 83

1) 가,나,다 4)



©ANSWER

2) 가,다

0 8 심도자 소견이 나왔을 때 이것이 어느 valve 와 관련되어 있는지를 먼저 파악하세요. aortic valve라면 LV와 aorta or radial artery, mi­ tral valve라면 LV와 LA 또는 PCWP와 같은 식이 되겠지요. 심도자에서 diastolic phase에 서 LA pressure(MPCWP)가 LV pressure보 다 높아 left atrioventricular pressure gradient가 상승한 모습을 보여주고 있어요. MS 의 특징적인 소견이죠. 류마티스 열을 앓은 병 력,호흡곤란도 MS임을 시사합니다. 가,나,다. diastolic murmur만 기억하지 마 시고 S,, ?21,OS도 구ᅵ여워 해주세요.八 八 라. 조ᅡ심방의 비대가 예상됩니다. 실제로 환자 의 심전도에서 조ᅡ심방의 비대 소견인 notched P wave(P-mitrale)가 보이네요. Harrison 201 만,pp.l813~1817

3) 나. 라

5) 가, 나,다,라

01.③ 02.② 03.① 04.③ 05.(1) 06.④ 07.② 08.⑤

-o

o

©

關 IS 승 모 판 역 류 (Mitral regurgitation)

Explanation

_________________________________________________________________



주로 류o r a 스영 매문에 일생하던 승모판 협착과 철리 승모판 역류는 여a 가지 원인에 의해서 일생할 수 9]•습니다. 따라서 원인이 되는 다른 심장 진환들과 면기咖서 출제철 수 9BJ1 죠. ■대표적으로 급성 심근경색의 mechanical complication으로 일생다!: 승모19:역류를 들 수 Si습니다(급성 심부전 등과도 °S기I 가능형니다). 승 모1ft 협착과 아찬가지로 중요한 소^ 위주로 공부하시는 ^이 좀습니다. 2019년 국시에서는 급성 심부전과 면 ᄀ혜 문제가 vg■제 출저ia 있습니다. 치료는 ① 심부전에 대던 적극적 치료와 @ AF 방생시 AF에 대한

를 현니다,

傘 승모판 역류의 정우에도 수술 적응증에 대해서 앞아두시면 종을 것 '살습니다. 제시된 수술 적용증의 수치를 기 본적으로 외우고 문제를 풀 매는 종형적9 1 1®단을 할 수 91는 능력을 기르면 칠 '찻0』니다.

Etiology 1)

Chronic rheumatic heart disease( 니 /3) : 남자에 서 더 많아 一- MR 一 Mr. — ^K M I요!

2)

Congenital anomaly(e.g., endocardial cushion defect)

3)

After ischemia/infarction (1) 특히 acute or transient M R 에서 : papillary muscle의 injury/fibrosis (2) Ventricular remodeling : LV 가 늘어나 MV annulus도 늘어나는 경우

4) Hypertrophic cardiomyopathy : 수축기 때 anterior leaflet의 anterior displacement —»• SAM이죠! 5)

Degenerative change : old age women에서, calcification 등

6)

Infective endocarditis : valve 침범하는 경우 acute MR

7)

Mitral valve prolapse(MVP)

8) Progressive property : severe M R 에서 LV 나 LA 가 확장되면 보다 바깥쪽에 있는 posterolateral leaflet

을 바깥쪽으로 밀어내 valve가 제대로 닫히지 못하게 된다(“MR begets MR”).

Pathophysiology 1)

LV contraction ~ > LA 로■regurgitation (1) LV tension이 일찍 떨어져 초기에는 LV enptying을 보다 더 많이 하려는 것으로 보상 (2) 그러나 결국 LV 확장이 일어남(LV ftm ction 1 에 비례) (3) Forward(effective) cardiac output J,

① Regurgitation • LV systolic pressure와 MV size에 따라 달라져 • LV ventriculography로 LV stroke volume을 구하고 Fick principle로 forward CO 을 구해 비교 • Severe M R : LV stroke volume의 50% 정도가 LA 로

룰 ^

존 i l

ᅳᅵ



• Color flow doppler를 이용해 구하는 방법이 가장 널리 쓰임

② Ejection fraction은 보통 증가 : M/VI) ① Apex에서 가장 잘 들리고 axilla로 radiation ② Isometric strain 시 murmur | , Valsalva maneuver 入1 murmur J,

preload } 시 심잡음 j

A2 I P2

normal expiration

pure mitral regurgitation

4)

MS와 M R 이 동반되는 경우도 있어 청진 시 주의를 요함

o

o

(& y

2020 년 대비 PACIFIC KMLE

순환기

5. Laboratory examination 1) EKG : 좌심방 확장, 심한 폐고혈압 시 RAE 도 보일 수 있음. 만성 M R 의 경우 AF 동반 가능 2) Chest PA (1) 중등도의 심장비대 傘

MR의

우 vc)lume overload가 7분리으로 부피를 찰 수용하고 효울적0 박출을 우ᅵ혜주머니 모0온으로

hypertrophy가 스갈어1■납니다. 이를 eccentric hypertrᄋphy라고 형니다. ᄐ지ᅥ요계서는진1만적으로 축 늘어 닌 주dm 모^의 심장 contour와 함께 apex가 왼쪽 아래쪽으로 더 내3 ^ 모습을 볼 수 91어요. (2) Double contour : LA enlargement

elevation of the left main bronchus and splaying of the carina

3)

Echocardiography

수축기에 LV ᅳ LA로의 regurgitant jet

©

o

oᅳ

tz z 4) Cardiac catheterization : late systole 때 좌 심 방 압 (는 PCW P)

MR 환자의 심도자술 소견

판막질환



그 조

]

MR의 LV ventriculography. LV에서 조영제를 주입할 때 수축기에 LA로 조영제가 역류함

離 심도자술 소^을 해석해봅시다. 승모1B: 역류의 정우에는 조카! /요이 수축하는 수축기Oil서 혈01(이 대동DJj으로DJ; 나가는 7;1이 아니라,좌심방으로도 나가기 매문에 좌심방의

이 수축기에서 증가하는 심도자술 소^을 1만칠:

% 수 있습니다. 그렁에서 색찰한 부분이 바로 증가한 할객명에 의한

증가를 나타내는 정0』니다. 따라서

심진음은 벙면히 수축기에서 들리'74!죠?

6. Treatment 1)

Chronic severe M R — 심부전에 대형: 적극적 치료와 AF빌:거생시 AF에 대현: 치료 (1) Medical approach



Warfarin(INR 2 〜3> : AF 발생



② Cardioversion도 고려할 수 있음 ③ Vasodilator : HTN이 없을 경우 ④

acute M R 에 비해 근거가 부족함

HF 에 대한 적극적 치료 : Diuretics, BB, ACEi,digitalis,

biventricular pacing 등

(2) Surgical approach

① 적응증 ᅳ 고대까진 "세세하■게 외우실 필요 없습니다. 참고4i하서I요 ;0그부분이있으나,수술적 치료의 적용증을 묻는 문제가 2017 국시에 출제되91습니다. 주의 기울여 ^ 펴봐 주세요. • 증상이 있으면서 LVEF >30%, LVESD(end-systolic dimension) 1든요 . 증성이 없으면 그닝 경과관찰을 하고 융통이 앗거나 VPB, PSVT, VT 동1반 시 신- b lo c k e r 를 /나용형니다.

1. Etiology and clinical features 1) MVP = systolic click-murmur syndrome, floppy-valve syndrome 2)

대부분은 원인을 모름, 여자에서 더 흔함(보통 14〜30세),대부분 무증상

3) Mitral leaflet tissue가 너무 많아 늘어짐(myxomatous degeneration, mucopolysaccharide의 침착) or redundant chordae 4) Marfan syndrome, osteogenesis imperfecta, Ehlers-Danlos syndrome 등의 collagen fiber 관련 유전

질환에서 병발하기도 하지만 대부분의 환자에서는 MV 만 침범되는 양상 5) Posterior leaflet이 더 흔히 침범

MV annulus dilation

傘 M \ f 니까 Post.!! 6) Papillary muscle, chordae 등의 이상을 유발 一 MR (북미 지역 MR 수술의 가장 많은 원인),arrhythmia (ventricular premature complex, PSVT, ventricular tachycardia) 7) Chest pain : substernal,prolonged, 운동과 상관없는 pain으로 angina pectoris와 다르다. 8) Transient cerebral ischemia attack, infective endocarditis

2. Diagnosis 傘 MVP는

날수축기 승모1ftoi LV의 0ii 력을 버EJ다가 Post, leaflet이 적하고(mid or late systolic click) 풀3 버

a 좌심방으로 쏠린다고 생각하시면 엽습니다. 따라서 승모팟 사이에 틈이 생기게 되고 MR이 생기'게 되죠. 하

지만 틈이 작은데다 심실 수축기 중^ 4Jpii 틈이 열리으로 late systolic crescendo-decrescendo 로 들혈니다. 1)

Auscultation

(1) Mid or late systolic click after Si (2) High-pitched, late systolic crescendo-decrescendo m urm ur

① At apex, ‘whooping’ sound

② LV volume 이 줄면 MV prolapse f : Valsalva maneuver, standing 시 j ③ Squatting, isometric exercise(hand grip) 시에는 丄

ᅳO

O

253

2020년대비 PACIFIC KMLE

2)

순환기 r 룰

2D echocardiography

수축기에 MV annulus보다 상부에서 판막이 접합, MV posterior leaflet이 LA쪽으로 2mm 이상 이동(하얀 화살표) 앞쪽 (probe 위치) 앞쪽 (probe 위치)

Ant. leaflet、

Ant.

MVP시 LA쪽으로 빠진 post, leaflet

Post. 정상수축기

normal

MVP

prolapsing posterior leaflet of the mitral valve

M mode echocardiography in mitral valve prolapse

3. Treatment 1) 무증상 :치료하지 않는다(안심시킴 2) Prophylaxis of endocarditis : 더 이상 예방적 항생제 사용 대상이 아님 3) 0-blocker : chest pain, palpitation 이 있을 경우 4) MV repair : severe MR 로 증상이 있을 경우 5) Warfarin : transient ischemic attack 시 고려

- .2 5 4 ) 0

O

Harrison 2Q만,pp. 1821~ 1823

w

01



26세 여자가 2년 전부터 시작된 흉부 불쾌감과 두근거림을 주소로 내원

하였다. 심첨부에서 수축기 심잡음과 클릭이 들렸다. 진단에 도움이 되 는 검사는?

판막질환

문제해설

---------------------- cA d

0 1 젊은 여자에서 심첨부에서 수축기 심잡음과 클릭이 들리는 소견은 MVP에 합당한 소견입 니다. 심초음파로 관찰시 MV displacement 를 볼 수 있어요. Harrison 2(>만, pp. 1821~ 1823

1) Holter monitoring

2) 심초음파 검사

3) coronary angiography

4) chest X-ray

5) SPECT

S ANSWER

01.②

2020 년 대비 PACIFIC KMLE

순환기

'•) Explanation

#

대동맥판 협착(Aortic stenosis)

^

傘 다른 팟막;일환들과 마찬가지로 특진적면 진찰 소정, 접사 소정을 위주로 공부하시면 평니다. 어않 때 어떤 치 료를 쓰는지도 중요업니다. Aortic valve는 mitral valve보다 종 H 조심스령>11 다루4야 해서, interventional approach보다는 수술을 하는 편업니다. 傘 2019년 국시에서는 출제되지 ^

지41, 이7a 기출문제들을 살펴보면 승모1S 진완에 비해 임상상만으로 -让단

di dff! 할 수 91는 문제들이 주로 출제되어 왕습니다.

1. Etiology 1) Chronic valvular heart disease의 I/4 ,S0%가 남자 2)

Cause (1) Age-related degenerative caIcification(m/c)

傘 AS는 퇴^성 진환이니까 AS를 받아야 던:다! 라고 외우면 쉬워요! ① 혈관의 atherosclerosis와 같은 risk factor를 공유 Age, male, smoking, diabetes mellitus, hypertension, LDL | , HDL | , CRP }

② Inflammation, calcification 등 조직학적 소견도 비슷함 (2) Congenital cause(bicuspid valve 등) : 유년기에는 이상이 없으나hemodynamic stress에 취약, 자라면서 valve deformity를 보이게 됨 (3) Rheumatic endocarditis of valve leaflet : commissure fusion

vulnerable for hemodynamic

stress { ,거의 항상 mitral valve disease나 AR 과 동반 (4) LV outflow obstruction을 유발하는 다른 질환

① Hypertrophic cardiomyopathy esp.,interventricular septum hypertrophy —> subaortic stenosis (2)

Discrete congenital subvalvular AS : membranous diaphragm, fibrous ridge —> endocarditis에

취약함 ③ Supravalvular AS : ascending aorta의 narrowing

2. Pathophysiology 1)

LV outflow tract obstruction — LV 와 aorta 시•이에 systolic pressure gradient

concentric LV

hypertrophy — 심실 내압을 높여 LV output | 를 보상하려고 함 ᅳ 보상 기전이 실패함에 따라 C.O. 1 , LV dilatation 등이 나타남

S)

ᄋ o



1

1

판급i 글

-1

2 ) Severe LV outflow obstruction

(1) Aortic orifice < 1.0cm2 or < 0.6cm2/m2(body surface area) : 정상 area의 〜 1/3 (2) Peak systolic pressure gradient > 40mmHg(normal C.O .을 보이더라도) 3) LV dilatation or hypertrophied LV compliance J, —►LV end-diastolic pressure } 4) C.O. at rest는 정상이나 운동 시 C.O. 상승이 일어나지 않거나 오히려 저하 5) 말기에는 C.O. at rest | , LV-aortic pressure gradient [ —► LA 압력이 상승하는 등 back-pressure

6)

Hypertrophy of LV : O 2 demand } , coronary blood flow 방해(coronary arterial obstruction이 없어도

심실 내압 상승 등의 원인에 의해) ᅳ tachycardia 등에서 ischemia에 취약해짐

3. Symptoms and natural history 1) 대개 valve area grade 1 /IV , LV failure 등으로 stroke volume 이 줄면 murmur도 soft and brief

6. Laboratory examinations 1) ECG (1) LVH 소견 : ECG 는 hemodynamic severity 반영하지 않음 (2) Advanced stage : ST segment elevation, T wave inversion(LV strain pattern) in I, aVL,V5 〜

6

2) Radiological findings

• Left : 초기에 AS의 보상기전으로 일어나는 LV hypertrophy는 LV dilatation과 함께 일어나지 않기 때문에 심장 음영이 커지기보다 chest PA에서 apex 가 둥글어지고(ᅳ} ascending a o rta 의 p oststenotic dilatation 소견(« flow chart와 어긋나는 부분이 않지 앞아서, 가1접게 보고 녕어가시면 H.

습니다. 전1안적으로 팟막 진환에 대해서 지나치게 숫자

기준으로 외우a 는 ᄀ X 보다는, 임상상을 보면서 판단하는 습곡! :을 기르셔야 현니다. Class I

A b n o rm a l a o rtic v a lv e w ith re d u c e d s y s to lic o p e n in g

Class lla

1

Class lib S evere AS

V max3m /s 〜 3.9m/s

Vmax>4m/s

APmean 20 〜 39mmHg

ᅀ Pmean>40mmHg

Asymptomatic (stage C)

Symptomatic (stage D1)

LVEF 60mmHg low s니rgicaᅵrisk

DSE with AVA 0 .3 m /s /y

low s니rgical risk

As likely cause of Symptoms

AVR

AVR

AVR

AVR

(I)

(Na)

(Nb)

(lla)

Harrison 20판,pp. 1802-1809;이영우 편저,순환기학,pp.31ᄋ〜 314 A Randomized Trial o f Intensive Lipid Lowering Therapy in Calcific Aortic Stenosis. Cowell S. J. et al, the Scottish Aortic Stenosis and Lipid Lowering Trial, Impact on Regression(SALTIRE) Investigators N Engl J Med 2005 :352 :pp. 2389〜 2397,Jun 9,2005

©

ooa-

tz 01

—^

.........................................................................................

n

H

문제해설

______

77세 남자가 혈압이 높아서 병원에 왔다. 12년 전부터 혈압이 높디는

말을 들었으나 혈압강하제를 복용하지는 않았다. 혈압 162/96mmHg, 맥박 74회/ 분, 호흡 17회/ 분, 체온 36.5 °C이다. 청진에서 심음은 규칙 적이고, 복장뼈 오른쪽 두 번째 갈비 人h이 공간에서 3/6 도의 수축기 잡 음이 들린다. 심초음파 결과는 다음과 같다. 치료는? • 대동맥판막 최고혈류속도(Vmax) : 3.0m/sec(참고치,40mmHg이므로 very severe이네요

5) 81mmHg

[5H1 B 58세 남자가 등산할 때 발생하는 흉통으로 내원하였다. 심기저부에서

수축기 잡음이 들렸고 목으로 방사되었다. 심초음파에서 조ᅡ심실 확장기 말 안지름은 60mm, 구혈률은 35%이고,대동맥 판막을 통한 혈류 속도 는 4.5m/sec 이었다. 가장 도움이 도I는 치료는? 1) statin

2) /^-blocker

3) ACE inhibitor

4)

이뇨제

5) 판막치환술

11

운동 시 발생하는 흉통,심7 ᅵ저부(cardiac base 는 apex의 반대쪽을 말합니다. 즉 2nd right intercostal space)에서 들리며 carotid artery 쪽으로 방사되는 수축^71심잡음,end- diastolic LV dilatation(정상 LV의 확장기말 안지름은 4.7 ± 0.4cm랍니다), ejection fraction 1 는 모 두 AS를 이야기하는 소견들입니다. 환자에게 필요한 적절한 치료는 무엇일까요? EF이 50% 미만이므로 aortic valve replacement 를 해주어야 합니다. Aortic valve를 통한 혈류 속도로는

A P = 4 V 2= 4 x 4 .5 2=81mmHg으

로 40mmHg보다 크기 때문에 ‘severe LV out­ flow obstruction’ 이 있다는 이야기도 할 수 있 답니다. Harrison 2ᄋ만,pp. 1802~ 1809

©ANSW ER

01.② 02.④ 03.① 04.① 05.① 06•③ 07.② 08•④ 09•⑤ 10.④ 11.⑤

c-

2020년 대비 PACIFIC KMLE

순환기

關 1 1 ____________ ______________



대동맥판 역류(Aortic

^^jmn

*

Explanation

regurgitation)

마찬가지로 특진적0d 진찰 소's, 점사 소^ 위주로 공부해 주시면 팅니다. 글성 스크시 혈입울 낮추면서 수술을 하고 IABP나 BB는 금기0』니다. 11편i A R 시 역류되는 혈억을 줄이기 위해 않초혈;!; ^항 울 I월어뜨리는 치료로 혈Si을 닛奋니다. 판막 질환도 일마 안 님았습니다. 헌내세요!

1. Etiology 1) Primary valve disease (1) Pure or predominant AR 환자의 3/4 정도가 남자,mitral valve disease가 동반된 경우는 대부분

여자임 (2) Rheumatic disease(2/3) : valve cusp deformity ᅳ 수축기에는 안 열리고 이완기에는 안 닫힘, AR 만 보이는 경우는 드묾 (3) Ankylosing spondylitis, congenital bicuspid valve (4) Ventricular septal defect(VSD) : 〜 15%에서 chronic AR 傘 소아과에서 subarterial type 은 Uu卜드시 조기에 수술학! :다고 빼우셨죠? subarterial VSD는 대동맥판 바 로 아래쪽에 defect가 생기는데 데우 벼ᅡ른 혈류가 1_\/에서 RV 쪽으로

하으로 베르누이 현상으로 대

동! 객판막이 VSD 쪽으로 쓸a 들어가면서 손상을 있게 되어 Ar 이 발 ^ n il 워니다. (5) Rheumatic disease, congenital anomaly가 있는 환자에서 infective endocarditis 발생 후 leaflet에 perforation or erosion (6) Traumatic rupture or avulsion of the aortic valve : nonpenetrating cardiac injury 이후

남은

환자에서 가장 흔한 형태의 AR 2) Primary aortic root disease (1) Aortic root disease —>• aortic annulus dilatation — AR (2) Cystic medial necrosis of ascending aorta(± Marfan syndrome), osteogenesis imperfecta, severe hypertension (3) Aortic dissection이 AV annulus 침범하는 경우 (4) Syphilis, ankylosing spondylitis

① Thoracic aorta의 media에 scar, aortic dilatation, aneurysm formation ② Syphilis는 aorta의 intima change —> coronary ostia를 좁혀 ischemic insult ^

AR은 류마HJ스코 정직성 척추영,bicuspid valve, 데독 등 평장히 다행한 원인이 있다는 ^

두세요!

©

o o-

기억해

C

판막질환



2. Pathophysiology 1) LV stroke volume } : forward stroke volume(to aorta) + regurgitant flow(back into LV) 傘 에를 들어 AR에서 수축기oil 좌심실 EF가 60%로 측73되있다 하더라도 그 중 일부는 다시 LV로 돌아오 는 regurgitant flow이기 매문에 실제 厂 ^ i 순환에 기여하는 EF는 측정되는 값보다 적습니다. 에를 들어 regurgitant flow가 1/ 3을 차지한다면 측정^

EF가 60%라고 해도 실제

순흰:으로 돌아가는 摇 는

40% 정도 빅;에 ᄋ J: 되는 셈이죠. 11다라서 AR에서는 LV dysfunction을 평가할 매 기준을 종 더 높'게 진아 50% 아래로 내려가면 좌심부진이 동반되(었다고 보고 수술의 적용증이 점니다. cf. MR의 수술 적용증 :LVEF < 60%

2) LV end-diastolic volume(preload) | (1) Major hemodynamic compensation

(2) LV dilatation and eccentric hypertrophy (cf. AS : concentric, pressure overload)

① 더 적은 힘으로도 더 많은 양의

s tro k e

v o lu m e 을

만들어 냄

② Severe AR이라도 normal stroke volume, normal ejection fraction 7 ^ ③ EF= total(forward+regurgitant) stroke volume/end-diastolic volume ④ LV end-diastolic pressure | 도 동반 ⑤ LV dilatation

myocardial wall tension도 증가 ᅳ LV end-diastolic volume은 더 싱승, forward

stroke volume과 EF는 丄(운동 시 두드러짐) ᅳ LV dilatation, LV function J, (resting state에

서도 EF,CO | ) ( V Laplace’s law : wall tension= intracavitary pressure x LV radius/wall thickness) 3) Diastolic m urm ur : decrescendo nature(... 이완기에 Ao —►LV flow 가 점차 감소) 4) Myocardial ischemia of subendocardium in A R (1) O2 demand | : LV dilatation, LV systolic wall tension | (2) Blood supply | : coronary perfusion pressure [

V aortic pressure [ in diastole

3. Symptoms of chronic aortic regurgitation 1) Long latent period : 10〜 15yr 2) 누울 때 이상한 심박동에 따른 불편감,운동 시 sinus tachycardia, palpitation(premature ventricular contraction), head poundi매:이 먼저 나타남. systole 때마다 몸이 흔들거리거나 고개가 끄덕끄덕

하기도 함 傘 AR에서는 M순%>il ^요 한 혈04과 역류하는 열객 모두가 대동口그관을 통해서 지나가으로 고속의 猫 가 ᄈ 안 다고 ^씀드했면 H 기억하시죠. 이 매문에 head pounding이나 봄이 은들리는 현상이 일어나^ 죠.

ᅳ o

o

©

2020년 대비 PACIFIC KMLE

순환기 I #

3) Exertional dyspnea (1) 1st symptom of reduced cardiac reserve (2) Orthopnea, paroxysmal nocturnal dyspnea, excessive diaphoresis 등으로 발전 4) Nocturnal angina with marked diaphoresis : nitroglycerin에 반응하지 않을 수도 5) Systemic fluid accumulation : 말기에 congestive hepatomegaly, ankle edema

4. Physical findings 1) Arterial pulse

(1) Confean’s pulse(water-hammer pulse) : late systole-diastole 때 갑작스런 arterial pressure | 반영 (2) Quincke’s pulse : severe 새에서 손톱 끝을 누르고 있으면 새 에 따라 손톱에 혈액이 찼다가

빠지는 것을 관찰할 수 있음 (3) Traube’s sign : ‘pistol-shot’ sound over femoral artery (4) Duroziez,sign : femoral artery를 청진기로 가볍게 누르면 들리는 to-and-fro murmur (5) 새에서의 혈압측정

① Systolic pressure가 심하면 300mmHg까지 될 정도로 크게 상승 ② 정상 Korotkoff sound에 systolic sound가 겹쳐 혈압즉정이 어려울 수도 ③ Arterial pulse pressure | : AR 의 severity를 반영하지 못함 ④ LV end-diastolic pressure가 상승하는 advanced 새 에 서 는 diastolic arterial pressure도 상승 : severe, acute AR 에서 맥압이 작은 이유 2) Palpation (1) LV dilatation : LV impulse의 lateral deviation (2) Sustained systolic thrill : LV 의 급격한 수축 / 확장 때문,jugular notch에서 carotid artery로 방사

됨, AS 의 동반을 시사하는 것은 아님 (3) Bisferiens pulse of carotid artery : 2 systolic wave 3) Auscultation 離 Left lower sternal border(3rd ICS)에서 둘리는 이완기 장윰 (1) A 2 (aortic valve closure) 소실 (2) S3, systolic ejection sound 離 Volume overload가 되어서 축 늘어1tt 풍성이라고 /생각하시면 독!다고 일;씀드혔죠? 초기에 급' 7



^이 확장하면서 형OJ(이 일a 들어오면서 나는 소리가 s 30ti니다. (3) High-pitched, blowing, decrescendo diastolic murmur 攀 야!/실이 이완하자■마자 대동DJpgoi 담히지 못해 심진음이 시작되죠, S2( ^ 확히는 P2) 후에 바로 시작되는 심진음으로,혈억이 역류하는

대동1객0 이 Si아진에 ^

적어7i "것0』니다. [다라서 decrescendo

판막질환

shape이 팅니다(cf. MR은 승모판의 크기가

defect가 큰데다, 심실이 수축기 동안 지속적으로 柄

을 가해서 역류건™ 되으로 일정하게 큰 심진윰이 pansystolic으로 들렁니다).

① Left sternal border, 3rd intercostal space(Erb’s area) ② A R 이 심할수록 더 커지고 오래 들림 ③ 앞은 자세에서 앞으로 기대고 숨을 다 내쉰 다음 청진 ♦ 앞으로 기대면 행지 대동맥이 가슴 앞쪽으로 기울어져 딱 붙'거I a 어서 a 잘 들^ i-\ 길죠? 호기 '훙'상

시에

OJ:

공기'라는 진OH물이 줄어들기 n«문에 1당면Ssi

들리겠죠.



© Right sternal border에서 잘 들리는 경우 : aortic root의 aneurysmal dilatation에 의해 AR 이

생긴 경우 Acute AR

Chronic AR Diastolic decrescendo

Basel

.



Si

I

Systolic ejection

murmur

I



J

Si S,

Systolic ejection S2

Apex

Diastolic decrescendo

murmur

murm 니「

I



S2

murm 니r



!

Si





Austin-Flint Murmur

^y\ /v\ (v Austin-Flint

S2

Si

Murm 니r

S2

Chronic AR과 acute AR의 청진소견 비교 雄 AS에서와 약7J: 성^이 다른 midsyst이ic ejection murmur가 들리기도 형니다(이완기 매 역류할

고려

(경 까 지

하면 LV에서 향d ^은 피를 뿜어내야 하기에 그러다 보니 상대적으로 AS가 되는 ■%). 傘 Austin-Flint murmur :

AR % 자ᄋil서 soft, low-pitched, Rumbling mid-diastolic murmur, AR streamᄋil 의

mitral anterior leaflet이 끌3 들어가 1밀:ᄉ 생 (fluttering),MS murmur외•

opening snap, loud Si이



5. Laboratory examination 1)

EC G (1) LV hypertrophy (2) ST segment depression, T wave inversion(LV strain) in I ,aVL,Vs~6 (3) left axis deviation, QRS interval } : poor prognosis(diffuse myocardial disease)



o i271 ;

2020년 대비 PACIFIC KMLE

2)

순환기 ::#

Radiologic findings

대동맥판 역류 Left : 우측으로 둘출된 a s c e n d in g a o rta , a p e x 가 좌하측으로 이동, Eccentric hypertrophy. 대동맥의 많은 혈류로 Ascending aorta의 bulging Right : AR의 cine-ventriculᄋgraphy, ascending aorta에 들어간 catheter로 주 입한 조영제가 LV로 역류하고 있다.

3)

4)

Echocardiography

M-mode in AR :숭모판 전엽의 flu tte rin g 소견 — AR로 인한 고속의 혈류로 옆에 있는 승모판의 전엽이 끌

C o lo r flow d o p p le r in AK serial 2D echocardiography는

려들어가 fluttering(Austin-Flint m u rm u r 발생)

행을 발견하는 데 유용함

LV performance를 평가하고 myocardial dysfunction의 진

Cardiac catheterization Mild



^

-木

Severe

스 ■_

AR 환자의 심도자술 소견 5)

Angiography : coronary artery 상태도 관찰해야 함

판막질환

6. Treatment 傘 수술과 알초 鞭

저항을 1낮춰 역류를 줄이는 vasodilat이•가 기본0』니다!

1) Acute AR

(1) 수술이 TOC! (2) 수술 전까지 IV diuretics and vasodilator(sodium nitroprusside) 사용 (3) 금기 : IA BP(Intraaortic balloon pum p), 신-blocker 傘 IABP는 이%기에 대동맥 근위부

력을 높이■게 되어 역류가 더 악회도P4[죠. AR에서 금기입니다.

2) Chronic AR

(1) Diuretics, vasodilator[ACEi, CCB(DHP), hydralazine] : 초기의 호흡곤란 개선에 도움 (2) 혈압 조절(SBP < 140mmHg)이 필요 : vasodilator가 TOC (3) 부정맥, 감염을 적극적으로 치료 (4) Nitroglycerin, long-acting nitrate : 흉통 감소에는 도움이 되지 않으나 시도해볼 수 있음 (5) /^-blocker, losartan : aortic root enlargement의 속도를 늦출 수 있음 (6) Severe 새 에 서 는 isometric exercise를 피한다. 3) Surgical treatment (1) 6개월 간격으로 심초음파 검사를 통해 LV dysfunction 여부를 관찰하다가 ‘after onset of LV dysfunction, prior to development of severe symptom’ 시기에 수술을 시도 (2) 수술의 적응증

① 증상이 있는 경우(LV fanctkm 과 무관) ② 무증상이라도 severe A R 이면서 • Progressive LV dysfunction(LVEF 50mm • LV end-diastolic dimension >65mm

(3) Aortic valve replacement with prosthesis : 특히 rheumatic cause 등에서 傘 다음 그령은 aortic regurgitatic)n의 치료CHI 대헌: flow chart0』니다. AS와 마六J;가지로 flow chart가 1극 !:순헌: 면이

며,위에 적헌 수술의 적용증과 아래 flow chart가 진방적으로 일치형니다. f|ow chart를 자세히 보시기 보다는 위에 적던 수술 적용증을 진 외우고 ■계시면 철 기 x

습니다.

o ( 273) —



2020 년 대비 PACIFIC KMLE

순환기 그 ^

Class I

Aortic Regurgitation

Class lla Class lib Severe AR (stages C and D) Vena contracta>0.6cm Holodiastolic aortic flow reversal Rv이>60mL/beat RF>50% ERO>0.3cm2 LV dilation

Progressive AR (stage B) Vena contracta< 0.6cm Rv이50% LVEDD>65mm low surgical risk

AVR (Da)

(nb)

AVR

LVEF >50% LVESD < 50mm LVEDD50 Large reg 니rgitant jet to LV large reg니rgitant jet to LA mmHg in most severe case M V ant. leaflet의 fluttering

C once ntric LVH

LV en largem ent

EF slope 감소

BB, CCB(non-DHP), DGX Diuretics for HF; Diuretics for HF for rate control of AF ; Vasodilators for ac 니te MR (탈수, hypovolemia 주의) symptom Cardioversion for newonset * Acute MR diuretics, control AF and HF ; vasodilator, IABP, 수술 Diuretics for HF(폐부종) * Chronic MR BADDS natural history

Warfarin for AF or throm­ Warfarin for AF or throm­ No proven treatment boembolism;ᅳ LAE로 AF boembolism ; LAE로 AF 호발하므로 호발하므로 PCN for RF prophylaxis

Vasodilators for HTN

* Acute AR — 수술(TOC), va sodilator / 금기 : IABP, BB * Chronic AR ᅳ 이뇨제,BB, vasodilator(BP 조절),AC曰 (Aortic root 확장 속도 丄) Vasodilators for HTN

순환기

2020년 대비 PACIFIC KMLE

:暑

Summary of Severe Valvular Disease

Moderate or severe MS

1) 증상이 있으면서 LVEF 1) 증상이 있는 severe AS(AVA 1) 증상이 있는 경우 (LV function과 무관) >30%, LVESD 65m m

percutaneᄋ니s mitral balloon

도 정상인데 new onset

AF or pulmonary h y - 4) Vmax >4, ZIP >40

시행하고 안 되면 수술

pertension이 있을 때

의 내과적 치료를 요 보 면 ,diuretics를 기본으로 시•용하고,regurgitation이 91을 경우 vaso-

dilator를 추가하며. MS, MR의 정우에는 AF에 대6소 치료를 추가하는 ♦

이면서 (1) Progressive LV d y s -

increase in size > 0.5cm /

valvotomy가 가능하면 이를

傘 판막

2) 무증상이리■도 severe AR

니다.

시^ a 시는 분들은 요즘 출제 ^형 상 수술 적용증을 일어 농으실 필요가 있어 보입니다.

傘 판막 진의:에서 심도자 소 ^의

정리

pressureonmHg)

MS에서의 심도자술 소견

MR 환자의 심도자술 소견

판막질환

€F

Murmur

o 4

volume(mL)

ᄋ 8

iSounds

0 o 6 o ᄋ 2

pressure(mmHg)

AO

0

2

4

6

8

10

AR 환자의 심도자술 소견

AS 환자의 심도자술 소견

문제해설

□ 52세 남자가 건강검진에서 우연히 진단된 고혈압을 주소로 내원하였다.

혈압은 170/80mmHg 이였으며, 왼쪽 복장뼈 옆 3 번째 갈비 사이 공간 에서 확장기 심잡음이 들렸다. 의심되는 진단은? 1)

대동맥판폐쇄부전증

2) 대동맥판협착증

3)

삼첨판 폐쇄부전

4) 승모판 협착증

5)

심방사이막결손

02



Erb’ s area(왼쪽 복장뼈 옆 3 번째 갈비 人KH 공간)에서 들리는 확장기 심잡음은 AR의 전 형적인 청진 소견입니다. 이에 더하여 수축기 혈압과 확장기 혈압 차이가 큰 것도 새 을 시 人변! ■는소견입니다. Harrison 2ᄋ만,pp. 1809~ 1813



54세 남자가 2 달 전부터 조금만 걸어도 숨이 차서 병원에 왔다. 혈압 150/60mmHg, 맥박 100회/ 분이었다. 복장뼈 왼쪽 가장자리 제3 갈비사

이에서 확장기 심잡음이 들렸고, 상체를 앞으로 기울여 숨을 내쉬게 한 후 심잡음이 더 커졌다. 심초음파 결고I는 다음과 같았다. 치료는? 왼심실이완기말 지름 6.1cm(참고치,4.0〜5.5), 왼심실박출률 46%(참

0 2 복장뼈 왼쪽 가장자리 제3갈비사이(Erb's area) 에서 들리는 확장기 심잡음으로 대동맥판 역 류임을 알 수 있습니다. 대동맥판 역류에서 증상이 있는 경우에는 좌심실 기능과 무관하 게 수술의 적응증이 됩니다. 따라서 치료는 대동맥판 대치술입니다. Harrison 20만,pp.l809~1813

고치,>55) 1) 승모판 대치술(replacement) 2) 승모판 성형술 3) 대동맥내풍선펌프(IABP) 4) 대동맥판 대치술 5) 직류 심율동전환(DC cardioversion)

O

o (277)

2020년 대비 PACIFIC KMLE

r

순환기

a

03

문제해설

-----------

82세 여자가 건강검진에서 심잡음이 들린다고 해서 왔다. 특별한 증상

0 3 고령의 여자 환자가 건강검진 상에서 발견된 심잡음을 주소로 내원한 증례입니다. 수축기

은 없었다. 20년 전부터 혈압강하제를 복용한다고 하였다. 혈압 140/50

혈압과 이완기 혈압의 차이가 크다는 점과 복 장뼈 왼쪽 옆 3 번째 갈비 사이에서 3/6도의 확장기 심잡음이 들렸다는 점에서 대동맥판 역류를 생각할 수 있습니다. 대동맥판 역류의 수술 적응증은 증상이 있거 나 severe AR이면서 LVEF50mm 혹은 LV EDD>65mm에 해

mmHg, 맥박 70회/ 분, 호흡 18회/ 분,체온 36.5도였다. 가슴 청진에서

복장뼈 왼쪽 옆 세 번째 갈비 人K>l에서 3/6도의 확장기 심잡음이 들렸 다. 심초음파검사 결고I는 다음과 같았다. 조치는? • 왼심살확장기말 지름 : 60mm(참고치,42〜59) • 왼심실수축기말 지름 : 40mm • 왼심실박출률 : 56%(참고치,>55) 1)

주기적 심초음파검사

2)

심방사이막결손가리개 삽입

3)

승모판클립 삽입

4)

카테터경유대동맥판막 삽입

5)

대동맥판막치환술

당하는 경우입니다. 환지는 수술 적응증에 해 당하지 않기 때문에 수술적 치료를 진행할 필 요가 없습니다. 현재 내과적 치료인 혈압강하제를 복용하고 있으며,혈압도 140/50mmHg로 추가적인 항 고혈압제 복용의 필요성이 크지 않기 때문에 주기적인 심초음파 검人S 진행하면서 경고層 살펴보는 것이 적절한 조치가 되겠습니다. 내과적 치료로 활용되는 항고혈압제는 주로 vasodilator가 이용된다는 점을 기억하시고 넘어가시면 되겠습니다. Harrison 20핍;,pp. 1809〜 1813

0 4

B 0 4 가. 왼쪽 3 번째 갈비人K)|공간(Erb’ s area)에

대동맥판 폐쇄부전에 대한 설명 중 옳은 것은?

서 decrescendo type의 이완기심잡음을

가. 이완기 심잡음이 들린다. 나. 발치 시 감염성 심내막염 예방을 위한 항생제를 투여해야 한다. 다. 증상이 없더라도 판막치환술을 시행할 수 있다. 라. ACB 등 혈관확장제는 금기이다. 1) 가’ 나,다

2) 가,다

4) 라

5) 가,나,다,라

3) 나,라

들을 수 있습니다. 나. 가이드라인이 바뀐 후 AR에서 더 이상 감염성 심내막염 예방용 항생제를 쓰지 않습니다(감염성 심내막염 파트 참고). 다. AR의 경우 증상이 없더라도 LV dysfunction이 진행하기 전에 수술을 실시하 는 것이 바람직합니다. 라. ACE inhibitor 등 vasodilator는 AS에서 금기이나 MR, AR에서는 시용 가능합니 다. AR에서는 systemic resistance를 줄 여 cardiac output을 증가시킬 목적으로 투여합니다. 이영우 편저,순환기학,P.316 Harrison 20ᅵ 만 ,pp. 1809〜 1813

^A N SW ER

(278)

o

0 1 .① 02.④ 03.① 04.②

판막질환



___

할확 삼첨싼 역류(Tricuspid

Explanation

regurgitation)

傘 앞의 4가지의 팟막진%에 비해 중요성이 ^어지는 파트입니다i그 J: ,2018^ 국시에 출제되9^습니다. 따로 봐두지 않으면 상청3환의 청진 우^ᅵ가 어디0그지 했Ty일 수 91으니,복습하고 명어가세요. Rt. side murmur라서 흡기 시 에 크기가 증가한다는 정을 알아두면 종'74!습니다.

1. Etiology 1) Functional TR & secondary to marked dilatation of the tricuspid annulus(m/c) RV enlargement가 일어나는 경우에 발생(예 : inferior wall infarction) 2) Rheumatic or congenital heart disease, ischemic heart disease, cardiomyopathy, cor pulmonale 3) Infarction of RV papillary muscle, TV prolapse, carcinoid heart disease, infective endocarditis, trauma

2. Diagnosis i뚜 낸9

Left lower sternal border(4th or 5th ICS)에서 둘리는 수축기 진움

1) 증상 : systemic venous congestion, reduced CO 에 의한 증상들이 나타남 (1)

: prominent v wave, rapid y descent

(2) Hepatomegaly, ascites, pleural effusion, edema, systolic pulsation of the liver, hepatojugular reflux

2)

진찰 소견 (1)

Prominent RV pulsation : 좌흉골연을 따라

⑵ Blowing holosystolic murmur : 좌홍골 하연(흡기 시 증가,호기,Valsalva maneuver 시 감소) 3) EKG, CXR : RA, RV 의 enlargement 4) Cardiac catheterization : Pressure assessment0]]

3. Treatment 1) Pulmonary hypertension0] 없는 경우 : infective endocarditis, trauma 등의 원인에 의해 발생한 TR

은 치료가 필요 없음 2) Severe TR이나 우심부전 증상 보이는 환자에서 Diuretic & Aldosterone antagonist 효과적 3) MV disease에 의한 경우 or pulmonary hypertension이 있는 경우 : 보통 mitral valve surgery 시에 tricuspid annuloplasty

시행 Harrison 2 0 잔■ , p. 1825

^

2020년 대비 PACIFIC KMLE

순환기

01

문제해설



47세 남자가 2 달 전부터 숨이 차서 왔다. 계단을 오르면 숨이 더 찼다. 12세 때 심방사이막결손으로 수술을 받았다고 히였다. 혈압 112/72mmHg,

맥박 74회/ 분, 호흡 26회/ 분, 체온 36.6도였다. 가슴 청진에서 복장뼈 왼쪽 옆 네 번째 갈비 人K>l에서 3/6 도의 수축기 잡음이 들렸다. 기슴 X선 人면이다. 폐기능검人정상이었다. 심초음파검사 결과는 다음과 같았다.

01

-0^0

Left 4th ICS에서 수축가 잡음이 들리는 TR 입니다. TR에서는 PA pressure가 높거나 pulmonary vascular resistance가 높을 때 치료를 고려하며 PA hypertension이나 vas­ cular resistance를 알기 위해 오른심장카테터 검入ᅡ해야 합니다. Harrison 20만,p. 1825

조치는? • 심방사이막 결손 : 관찰되지 않음 • 오른심실 및 오른심방: 확장 • 상첨판막폐쇄부전 최고속도 : 4.2m/s(참고치,11 되고 이곳을 통과하는 열류 속도도 더 빵라지 ~/il 되어 dynamic obstruction이 더

^p\ \ Km

병니다. 따라서 씽경음이 더 크게 들리게 되죠. ③ vasodilator나

DHP-CCB를 사용할 때 알 초 避 I:저항이 정소하여 afterload가 정소하면,대동맥 쪽으로 삐며나가는 혈류 속도 가 더 병;라져 dynamic obstruction이 심해진니다. 離 SAM을 더 심하게 41드는 ᄋ 4들은 모두 금기。 』니다. - > digitalis, /5-agonist, 이뇨제,-dipine, 혈그? !:확7당제 SAM 을 줄이는

은 치료제ᄋ』니다. — ^-blocker, non-dipine(verapamil, diltiazem), disopyramide

傘 cf. MVP(post. leaflet) 심실 수축력이 증가할 매와 preload가 정소험: 때는 H-CMP와 동일한 이유로 심진음이 증가입니다. 하지만 vasodilator를 사용하는 정우 대동1객 쪽으로 열류가 더

나가'게 되어 MVP로 인해 ^

틈으로 나가는 열류

는 오d ia 줄어들1 ᅥ | 되으로 심진음이 각소하게 윌니다ᅵ

4. Laboratory evaluation 1) ECG : LVH, prominent q wave(old infarction), arrhythmia(ex. Holter monitoring에서 AF, VT)

Septal HCMP ECG

Apical HCMP ECG Giant T wave

Dagger(deep-narrow) like Q wave

inversion

傘 HCMP 심진도oil 대해서 7드!:히 다뤄보젯습니다. HCMP는 asymmetric건™ 심근이 비대되어 있는

우가 않고,

이에 따라서 septum이 비대 되어 9!으면 septal HCMP로,apex가 비대되어 91으면 apical HCMP로 분류하게 팅니다. Septal HCMP에 비해서 Apical HCMP는 드문 면이며,심7S도 상에서 Giant T wave inversion이 나타 나는 ~%01 특7강적0』니다. H K ,Septal HCMP에서는 T wave inversion 이 보통 동4i되어 9JI으W,단도와 길이 날카로운 dagger(deep-narrow) like Q wave가 보이는 ᄀ ;1이 전형적0』니다.

2) Chest PA : 거의 정상(보통 hypertrophy에 dilation이 동반되는 것은 아니기 때문) 3) Echocardiography : m ain diagnostic tool

A , 부분이 MV가 열리는 구간입니다. 심실이완기입니다. 우측의 모식도를 보면서 anterior leaflet과 posterior leaflet이 열렸다가 닫히는 모습을 상상하시면 좋을 것 같습니다. SAM은 심실 수축기에 anterior MV가 앞으로 쏠리는 현상입니다. B ,부분이 위로 올라가는 것이죠, 그래서 C ,부분이 좁 아지게 됩니다. 마찬가지로 우측의 모식도에서 C부분의 위치를 생각해 보시면 LVOTO가 어디서 좁아지는지 이해하실 수 있습니다(병태생리는 LVOT obstruction ᅳ 혈류속도증가 ᅳ 베르누이효과 ᅳ

SAM입니다). 문제에 나왔던 심초음파를 다시 한 번 보면서 SAM을 찾아보세요 AA

확획류 ® ® 쁘 툭 ® ^^^획 효 ^ 험 ^ 향 황""공력 우 심 실 ------------------------------------------------ ^

_____3 ^ ..

비후된 심실 중격

.S

승모판전첨 — SAM

좌심실후벽

(1) ASH(asymmetrical septal hypertrophy) : 심실중격이 high posterior LV free wall 두께의 1.3

배 이상 (2) 심실중격의 음영 : ground-glass appearance(abnormal architecture 반영) (3) SAM(systolic anterior motion) of MV (4) Small LV cavity, vigorous posterior wall motion 4) Radionuclide scintigraphy : 무증상에서도 myocardial perfusion defect 발견 5) Cardiac catheterization : LV diastolic pressure | , LV cavity와 subaortic region 간의 systolic pressure gradient

심근질환

고혈압에 의한 concentric hypertrophy

Hypertrophic cardiomyopathy

Parasternal long-axis view의 오른쪽 점들 사이의 간 격은 10mm이다. Septum과 LV posterior waᅵ ᅵ 이 두께 가 약 13mm로 비슷한 모습을 보인다.

Septum 이 LV posterior wall보다 2배 이상 두꺼 워져 있다.

5. Treatment 1) Sudden death의 위험 때문에 격렬한 활동이나 dehydration(excessive diuretics)은 삼가야 함

2) g-blocker, verapamil / diltiazem : initial therapy (1) 심박수 감소 ᅳ 이완기 시간을 길게 함 (2) 수축력 감소 — 운동 중 심실 내 pressure gradient | 제한 (3) sudden death에 대한 예방 효과는 없음 3) Diuretics : fluid retention 있을 경우 사용. 그러나 hypovolemia 주의 4) Surgical myectomy/myotomy or Alcohol septal ablation (1) hypertrophied interventricular septum에 대한 수술적 치료 (2) 내과적 치료 반응하지 않는 환자에서 효과적

5) Sudden cardiac death예방 (1) 격렬한 신체활동과 경쟁적인 운동 금지 (2) 고위험군에서 Implantable cardioverter-defibrillator

6) Atrial fibrillation (1) 비대 심근병증에서 Atrial fibillation 흔히 동반 (2) 0-blocker, Verapamil / Diltiazem, Disopyramide, Amiodarone 효과적 (3) Anticoagulation필요

ᅳ*• LV outflow tract pressure gradient^ 있는 환지서]서 피해야 할 익1제 : digitalis, diuretics, nitrate, CCB(DHP), vasodilator,g-agonist, alcohol

傘 Preload를 줄이는 ^ d iu re tic s , nitrate)과 afterload를 줄이는 약(DHP-CCB, vasodilator, alcohol)은 사용하면 °J; 병니다. HCMP에서는 최대현: LV에 열ᅵ객명을 양이 유지하는

이 종다고 例 하 시 면 면협니다. 또헌: 수축력

을 증가시키는 약(digitalis, /?-agonist)도 주의해야 형니다. 심장 수축력이 증가하면 LV outflow tract의 혈류속 도가 월:라져서 베르누이의 업칙에 의해 SAM을 더욱 더 심회시키게 도P/J[죠.

o

o (301)

2020 년 대비 PACIFIC KMLE

Z

순환기

(주의) Digitalis는 DCMP에서는 치료제이지만 HCMP에서는 금기0』니다!! HCMP는 심근 수축력이 부족한 '게 문 저마 아니라,심근 이완이 진 oh 되는 -엇이 문제니771■요.

6. Prognosis 1) 환자의 clinical course를 바꾸는 합병증 (1) Atrial fibrillation : 비교적 말기에 나타난다. 심방 수축에 의한 ventricular filling | (2) Infective endocarditis : < 10%, resting obstruction이나 M R 이 동반된 경우 예방적 치료 필요 (3) Burnt out HCM P (심실비대에 의한 심근 손상과 remodeling이 만성적으로 진행되어 HCMP이

면서도 수축력의 심한 저하가 동반된 상황) 2) Sudden cardiac death (1) H-CMP 환자의 주요 사망원인 (2) 무증상에서도 나타날 수 있음 : 1 % /year의 사망률 Harrison 2Q만,pp.l793~1797

심근질환

___

Explanation

제한 심근병증(Restrictive cardiomyopathy) General consideration 1) 주로 이완 기능의 장애 : Rigid ventricular wall —► ventricular filling |

CO J,

2) Myocardial fibrosis, hypertrophy, infiltrative disease, heart transplantation, mediastinal irradiation, carcinoid syndrome 등과 연관 R-CMP의 원인 A m yloidosis

Eosinophilia(예 , Loeffler's syndrome)

H em ochrom atosis

S cleroderm a

Glycogen storage disease

Endomyocardial fibrosis

S arcoidosis

Fabry's disease

Neoplastic infiltration

3) Thromboembolic complication : 〜 1/3 정도에서

amyloid heart의 부검 소견 심실벽이 비교적 균일하게 두꺼워져있으며 A s (a tria l s e p tu m )도 두꺼워져 있는 모습을 보이고 있다.

Clinical manifestation 1) Exercise intolerance, dyspnea : CO J, 2) Dependent edema, ascites, hepatomegaly(tender, pulsatile liver) : 정맥압 丁 3) Kussmaul’s sign : 경정맥압이 흡기 시에 오히려 상승 難 RA의 부피가 7뼤되으로 대정dj)의 혈ojj이 정처평니다ᅵ [다라서 ■昏기 시에 용^ 음o』으로 Tgoj)환류가 증가하 ■게되면 RA로 들어가지 못하고 상이'7비되어 JVP가 오건IS 상승하'게 팅니다. 또6J: back pressure로 ᄋ 부종,복수,자비대가 例 형 니 다 . RV의 부피도 제^되므로 RV preload가 강소하여 forward로는 LV의 C.O.이 I t 소하■게 접니다. RV preload가 깁소하니까 폐부종은 도쏘3히 진 일어나지



2020 년 대비 PACIFIC KMLE i

4)

순환기

Auscultation : distant heart sound, S3 and S4 ♦ R-CMP의 특진은 이완기 초기에는 급'7북한 심방, 심/ i 확7^이 일어나다가 제한된 부피에 도^하는 순^ 확 장이 '압자기 명춘다는 것0』니다.(님작한 가죽 포대에 울을 녕는다고 例 해 보 시면 , 잘 들어가다가 제한된 부피에 다다르면 t닥 벙추'7/찾죠?) 그af서 '경7gtajp온이ᅵ서 prominent x&y, 심도자시 심실의 square root sign 이 나태났죠(앞쪽 심체社기| 진찰 파트 참고), 급'역한 심/표 확장 시 또가 들HJ고 제던된 부피에 도^ dRii 되 면 atrial kick시에 S4가 들^ 니다.

5)

ECG : low-voltage, non-specific ST-T change, various arrhythmia

6)

Echocardiography : symmetric LV wall thickness

7)

Cardiac catheterization : CO J,, LV/RV end-diastolic pressure |

},

ventricle cavity size ^

J,

Constrictive pericarditis

Restrictive cardiom yopathy Transvenous RV endomyocardial biopsy :

Chest PA에서 pericardial calcification

infiltration이나 fibrosis 관찰

CT/MRI에서 비후된 pericardium 관찰

Apex impulse가 잘 느껴지고,MR이 흔함

Pericardiectomy로 치료 가능

모두 diastolic dysfunction이 주된 병태생리 심■EXl' 검시Oil서 dip-and-platea니 pattern of ventric니lar diastolic phase

3. Treatment 1) 直과적인 치료법이 없으나 다음의 경우에서는 일부 直과 ( 1 ) Hemochromatosis : phlebotomy, deferoxamine(iron deposition J,) (2) Fabry’s disease : alpha galactosidase /? 보 중 2) Chronic anticoagulation : embolic risk •

[

소아과 책에는 울현 중상 완화에 이뇨제가 유용하며 심실의 유순도 중가를 위해 CCB를 조심스럽■게 사용 % 수 91다고 향니다. 그리고 (북I :치료 立과는 제다!:적이으로 심장 이식술이 최성의 치료로 떠오르고 9]、 다

고 % 니다. Harrison 20°V, pp. 1791〜 1793 홍창•의 소아고)■학 11민',pp. 779〜 783

" ii 근s i r

i 심근병증 한눈에 보기 Dilated

Restricted

약 50%에서 AD 형태 유전

코카인 비만 원인

Hypertrophic Amyloidosis

Hemochromatosis

Viral infection

Sarcoidosis

약(doxorubicin, cyclophosphamide) 1술

Glycogen storage disease

^•상선 질환

mi -

코 비 약 (은) 술값 임

흔한 초기 증상

진찰 소견

운동 제한

운동 제한,흉통 동반

운동 제한

중등도 내지 중도의 심비대

중등도의 심비대

경도의 심비대

제3. 4 심음

심첨부 수축기 진전 제4 심음

제3. 4 심음

방실판막폐쇄부전 특히 승모판

방실판막폐쇄부전

Valsalva maneuver 로

증가되

Kussmaul's sign

는 수축기 ^음 흉부 ^사 선 소견

심전도

중등도 내지 심한 심비대(LV)

중등도의 심비대

경도의 심비대

폐정맥 고혈압

조버방 확대

폐정맥 고혈압

동성 빈맥

좌심실 비대

저전압

심실 및 심실상성 부정맥

비정상적인 이파

심실 내 전도 장애

심실 내 전도 장애

ST-T change

^■실전도 장애

심실성 tachycardia

심실성 tachycardia

Sarcoidosis 외에는 심실 부정맥은 드물

Chagas' disease는 전도 장애

심^■세동

Sarcoidosis/amyloidosis어ᅵ서는 전도 장애

ST-T change

부정맥 위험

심^■세동

심 ^ ^파

심방세동

조!심실벽 두께 감소 좌심실 확장과 기능 이상

조h심실강이 작거나 정상

정상 수축기능

비정상적 승모판 이완기 운동

승모판 수축기 전방 운동

심낭 ^•출

55% )

좌심실 확장과 기능 이상 승모판,삼첨판 폐쇄부전

>60%

25 〜 50%

조벼실 탄성 감소 승모판 폐쇄부전

조1심실 탄성 감소 심실 압력 곡선 sauare root sign

조h 우측 충만압 상승

수축 기능 항진

수축기능 유지

심박출량 감소

조!■심실 유출로 동적 압력차

조h 우 충만압 상승

Anticoagulation

BB

심부전 치료

Non-DHP CCB

효과적인 치료 방법 없음 울 ^ 상 ^화에 이뇨제 심실의 유순도 증가를 위해 CCB 약물 치료 효과는 제한적이므로 심장 이식술

Resynchronization 치료

대칭적 좌심실벽 두께와 심근량 증가 조1심실강 정상이거나 작아짐

탄성과 충만압 이상으로 인한

ejection fraction

심도자

비대칭성 중격비후 좌심실 유출로 협착

심장이식

(verapamil, diltiazem) Amiodarone ICD

금기

: Nitrate 제제.

이뇨제.

DHP CCB(nifedipine). ACEi. jg-agonist, digitalis, dopamine



2020 년 대비 PACIFIC KMLE

순환기

01

문제해설



23세 남자가 축구를 하다가 쓰러져 응급실에 실려 왔다. 3년 전부터 운 동 중에 으I식을 잃은 적이 여러 번 있다고 한다. 형이 4년 전 갑자기 사 망하였다. 혈압 122/82mmHg, 맥박 60회/ 분, 호흡 18회/ 분, 체온 36. 4°C이다. 가슴 청진에서 복장해 왼쪽 옆 네 번째 갈비 人K)|에서 3/6도 의 수축기 잡음이 들린다. 호흡음은 정상이다. 심전도이다. 검사는?

01 반복적인 운동 중의 의식 소실이 있고, 급사 의 가족력이 있으며 흉골 좌하연의 수축기 심 잡음이 있고,피지에서는 %〜\성는 ST elevation과 함께 deep S wave가 관찰되고,Vs ~V6는 ST depression과 함께 large R wave가 관찰되어 조[심실비대 소견이 있습니 다. 급사의 가족력이 있고 반복적인 의식소실 을 보이는 젊은 남성에서 비대 심근병증을 가 장 먼저 생각해 볼 수 있습니다비대 심근병증에서는 승모판의 anterior leaftlet의 수축기 이동으로 수축기에 MR의 심잡 음,그리고 LVOT가 좁아지면서 나타나는 상 대적인 AS의 심잡음 등이 들릴 수 있습니다. ECG에서는 심한 조! 심실비대 소견을 보입니다. 심초음파를 통해 직접 심근의 비대칭적 비대 를 확인하고 SAM 소견을 관찰하는 것이 비 대 심근병증의 가장 중요한 진단방법입니다. Harrison 2Q만,pp. 1791〜 1793

십초음파검사

2) 머리기울임검사

심장동맥조영술

4) 오른심 장카테 터검사

99mTc-심근관류스캔

02

Q

29세 남자가 운동시 숨이 차서 병원에 왔다. CXR에서 심장 크기는 정상 이었고,심초음파에서 심실사이막이 심실 뒷벽의 2.8배 두꺼웠고, 승모 판막이 수축시 앞쪽으로 이동하였다. 치료는? 1) isoproterenol

2) digitalis

3) dobutamine

4) propranolol

5) furosemide

0 2 HCMP입니다. 진단의 핵심은 심초음파 소견 이며, septum이 posterior LV free wall의 1.3 배 이상인 것과,mitral valve의 SAM(systolic anterior motion)이 가장 중요한 소견 입니다. 치료로 anginal pain과 syncope에 대하여 beta blocker를 人[용할 수 있습니다. Propranolol은 베타차단제이지만 심부전 치 료에 시용되지 않는다는 것 때문에 답으로 고 르기를 망설였을 수도 있겠습니다만, HCMP 에서는 propranolol도 효과적이라고 합니다. 더 중요한 것은 HCMP에서의 금기를 외우는 것입니다. HCMP에서으I 금가는 digitalis(LV contractility } ),diuretics(preload j ), CCB (DHP), vasodilator, beta agonist(HR 丁 : isoproterenol)입니다. Harrison 20^, pp.l793~1797

ooa

심근질환

€ F

03



42세 남자가 1시간 전에 테니스 경기를 하다가 쓰러져서 응급실로 왔 다. 그의 아버지는 43세에 갑자기 사망하였다. 혈압 112/71mmHg,맥 박 74회/분,호흡 17회/분,체온 36.6°C이다. 가슴 청진에서 복장뼈 왼 쪽 옆 두 번째 갈비 사이에서 3/6도의 수축기 잡음이 들리고, 발살바조 작 때 잡음이 커진다. 심장끝에서 3/6도의 수축기 잡음이 들린다. 심전 도이다. 진단은?

문제해설

---------------------- cA d

0 3 급사의 가족력을 가진 중년의 환자가 운동 중 에 쓰러져 내원한 증례입니다. 신체검진 상 복장뼈 왼쪽 옆 두 번째 갈비 人K)|에서 들리 는 3/6도의 수축기 잡음 그리고 발살바조작에 서 커지는 소견은 강력하게 비대심근병증 (HCMP)을 나타내는 소견입니다. 발살바 조작 시행 시,흉강압의 상승으로 인 하여 systemic venous return이 감소하게 되 고,이에 따라 preload가 감소하게 됩니다. 따 라서 대부분의 심잡음은 발살바 조작 시에 감 소하게 되지만,특징적으로 비대심근병증과 승포판막탈출증(Mital valve prolapse)에서 는 심잡음이 증가합니다. 두 질환의 감별점으 로는 MVP의 경우에는 심장 끝에서 수축기 잡음이 들리는 것이 특징적이며,HCMP에서 는 심장끝 뿐만 아니라 복장뼈 왼쪽에서도 수 축기 잡음이 들리는 것이 특징적입니다. 비대 심근병증의 경우에는 left ventricle에서 aorta로 나가는 길목이 좁아지면서 빠르게 통

1) Q T 연장증후군

2) 브루가디증후군

3) 비대심장근육병

4) 확장심장근육병

5) 오른심실형성이상

고卜히는 혈류에 으I하여 주변의 구조물 또한 빨 려 들어가는 압력을 받게 됩니다. 베르누이의 원리에 해당합니다. 이에 따라서 mitral valve의 leaflet 또한 anterior motion을 보ᄋ| 게 도|고, AS와 MR이 동시에 유발되는 상황 이 발생합니다. 따라서 심장 끝과 복장뼈 왼 쪽 옆에서 수축기 잡음이 들리게 됩니다. 시행한 심전도 상에서는 lead V4, V5에서 ST depression 소견과 함께 inverted T wave 소 견을 보여서 HCMP에서 동반될 수 있는 ischemia 양상을 관찰할 수 있습니다. 따라서 답은 비대심근병증을 고를 수 있습니다. 참고로,급사의 가족력을 보이는 대표적인 질 환으로는 HCMP 외에도 브루가다 증후군과 QT 연장증후군이 있습니다. QT 연장증후군 은 심전도 상에서 QT interval이 확연하게 증 가한 것을 관찰할 수 있으며,브루가다 증후군 또한 심전도 상에서 coved type의 ST seg­ ment elevation을 보입니다. 다음은 Brugada syndrome type 1에서 특징적으로 보이는 심 전도입니다.

Harrison 20만,pp. 1793〜 1797

t

2020 년 대비 PACIFIC KMLE

"0

순환기

"

4

g B -B

29세 남자가 운동 중 호흡곤란과 흉통을 주소로 내원하였다. 가족력상 작

은아버지가 원인 불명의 심장 질환으로 급사하였다. 혈압 130/90mmHg, 맥박 80회/분, 심첨부에서 수축기 잡음이 들렸고 valsalva법으로 잡음이 증가되었다. 심도자 결과 조임실 심첨부 190/15mmHg, 조심실 유출로 120/13mmHg, 대동맥 115/75mmHg이었다. 다음 중 올바른 진단은? 승모판 탈출

2)

비대 심근병증

4) 대동맥판 협착증

제한 심근병증

0 4 호흡곤란, 급사의 가족력,Valsalva maneuver로 증가되는 수축기 심잡음,심도자상 좌 심실 심첨부와 유출로 간의 systolic pressure 차이가 큰 것(190-120 = 70)으로 미루어 보 아 HCMP를 의심해볼 수 있어요. 만약 AS였다면,조!심실 유출로와 대동맥 사 이의 압력차가 컸을 것입니다. 조I심실과 조[심 실 유출로 사이의 압력 차는 별로 없었을 거 ^ 요. Harrison 20면;,pp.l793~1797

승모판 역류

05 65세 남자가 1일 전부터 갑자기 숨이 차서 병원에 왔다. 5 일 전에 감기

증상으로 감기익을 먹기 시작하면서부터 고혈입약을 중단하였다. 혈압 180/110, 맥박 95 회/분이었고, 심초음파 소견 및 CXR와 심전도는 다

음과 같다. 치료는? 심실사이막 12(참고치,6~11 ),심실후벽 13(참고치, 6 〜 11),EF 58% (참고치,>55)

__^

--1 j~

aVR

-^\r

--

广…~

기p

V il

V4 I

1

■ 으 ~ aaVL vrp 4

vrir*

I

V5

ᄋ5 심근 두께가 큰 것으로 보아 HCMP구나! 라 고 생각하셨을 수도 있겠지만,답은 RCMP입 니다. 감별점은 바로 LV wall이 대칭적으로 두꺼워졌느냐,비대칭적으로 두꺼워졌느냐 하 는 것입니다. HCMP에서는 조ᅡ심실 비대가 비대칭적으로 발생합니다. 즉,LV septal wall이 LV free wall(=post. wall)보다 1.3배 이상 두꺼워집 니다. 반면 RCMP에서는 septal wall고!' free wall이 비슷한 정도로 두꺼워집니다. 본 환자에서는 심장 초음파 소견상 심실사이 막과 심실후벽의 두께가 비슷하게 증가하였 JL, systolic function이 정상이거나 경도로 감소된 것으로 보아 전형적인 RCMP 소견입 니다. 본 문제에서 제시된 EKG는 비특이적 인 ST-segment abnormality를 보이고 있습 니다. 실제 문제에서는 leads I ,I , aVL, aVF, V2〜 V6에 1mm 정도로 경도의 ST elevation이 있었습니다. RCMP에 의한 심부전 때문에 폐부종이 생긴 상황이며 폐부종에 대한 치료로 이뇨제를 사 용합니다. 사실 폐부종에서는 폐에 저류된 물 을 빼 주는 것이 급선무이기 때문에,문제의 환자가 HCMP였다고 해도 우선적인 치료는 diuretics였을 것입니다. 하지만 체액 저류가 심하지 않은 HCMP에서는 preload가 떨어지 면 오히려 조[심실 유출로가 더욱 좁아질 수 있기 때문에,diuretics는 금기죠. Harrison 2Q만,pp. 1793~ 1797

j ᄂ 1

aVF aVF ;

V3|

1) diuretics

2) digitalis

3) dobutamine

4) heparin

5) BB

(308 o

J ᄂ

o

V6

C

06

B

운동 시 호흡곤란과 가슴 두근거림을 호소하는 33세 남자의 심장초음파 사진이다. c ᅡ음중 예후와 관련이 깊은 소견은?



1)

3) 증상의 중증도

문제해설

심근^

---------------------- cA,

0 6 M-mode에서 두꺼워진 심장벽과 SAM이 관 찰되고 있는 것으로 보아 H-CMP에 합당한 소견입니다(막상 시험장에서는 저 두꺼운 벽 도 안보이고 MS에서 나타나는 EF slope의 감소라고 생각하신 분들도 많았답니다. 원래 人유람은 보고 싶은 것만 보기 마련이니까 요八 八 ). 하지만 문제는 예후와 관련된 소견을 찾는 것인데요,H-CMP는 무증상에서부터 급성심장사에 이르기까지 人h람마다 다양한 경고!# 밟게 됩니다. 후기 합병증으로 심방세 동,감염심내막염,확장성심근병증 등이 나타 날 수 있지만,H-CMP 환자의 人F망의 가장 중요한 원인은 급성심장사입니다. 문제는 아 무 증상이 없던 환자에서도 발생한다는 것이 죠. 그래서 H-CMP에서 급성심장사의 위험 인자는 중요하게 다루어지는데요, 소생술의 과거력,반복적인 실신,홀터나 전기생리학적 검人[에서 나타난 심실빈맥,심실벽 두께가 30mm 이상인 경우,운동 중 혈압 증가의 실

2) 고혈압

심방세동



패,급성심장사의 가족력,특정한 유전자변이 등이 해당됩니다.

4) 왼심실 압력

Harrison 20만,pp. 1793〜 1797

5) 급성 심장사의 가족력

07 30세 남자가 운동 중 호흡곤란으로 내원하였다. 가족 중 형이 원인을 알

수 없는 급사를 한 병력이 있다. 치료는?

07 H-CMP에서으| 금가는 digitalis(LV contra­ ctility f ),diuretics(preload [), CCB(DHP), vasodilator, beta agonist(HR | : isoproterenol) 입니다. Verapamil은 ncm-DHP이고 nifedipine은 DHP입니다. Harrison 20퀸•,pp. 1793~ 1797

f e |MM|L 8a|g[!|Bj !jp M ^ ^ M ^ ^ | f ■"은 二 .- ^ ^ . -~i"~V.一 iy..i■ 다 크 vh 了

證5 ^ . 3 輕 :•

훈洋^ ^ . 하

l) 베라파밀

2) 니페디핀

3) 디지탈리스

4) 푸로세마이드

5) 나이트레이트

广 •니 고^ "■ 으그아七



2020 년 대비 PACIFIC KMLE

순환기

08

0-O -©

33세 남자가 운동 중 숨이 차고 가슴이 답답해서 병원으로 내원하였다. 혈압은 125/80mmHg, 심박 수는 75호I, 호흡수는 18회였다. 청진에서 흉골 좌하연과 심첨부에서 수축기 잡음이 들렸다. 앉았다 일어설 시 심 잡음의 크기가 증가하였다. 심전도에서 조ᅡ심실비대 소견을 보였다. 이 환자에게 투여할 약물은? 1) 질산염 제제

2) 베타차단제

3) 디지탈리스

4) 이뇨제

5) 안지오텐신전환효소 억제제

문제해설 0 8 운동 중 dyspnea와 anginal pain을 주소로 찾아온 30대 젊은 환자입니다. 흉골좌하연고卜 심첨부에서 수축기 잡음이 들리고 결정적으 로 앉았다 일어서는, 즉 preload가 감소하는 상황에서 심잡음이 증가되는 소견을 통해 HCMP를 의심할 수 있습니다. HCMP의 치료로 조I심실 수축력을 약화시켜 outflow pressure gradient를 감소시키는 베 타 차단제,non-DHP CCB 등이 이용됩니다. 반면에 preload와 afterload를 감소시키는 질 산염 제제, 이뇨제,vasodilator(DHP-CCB, ACEi 포함) 등과 좌심실 수축력을 증가시키 는 디지탈리스는 HCMP에서 금기입니다. 참고로 대부분의 질환에서 preload가 감소되 는 상황에서 심잡음은 감소됩니다. 이와 반대 로 preload가 감소될 때 심잡음이 증가되는 질환은 HCMP, MVP입니다. Harrison 2(>만,pp. 1793〜 1797

09

®-®-b

비대 심근병증에서 심잡음의 크기가 증가하는 경우는? J\ .

니•. amyl nitrate

inotropics

라. squatting

다. exercise i) 가,나,다

2) 가, 다

4) 라

5) 가,나,다,라

3) 나, 라

0 9 Hypertrophic cardiomyopathy와 mitral valve prolapse는 다른 심장 질환과 다른 murmur .특성을 보입니다. 실제적으로 그리 중요한 내 용은 아니나 나름대로 pathophysiology를 이 해하는 데 도움을 줄 수 있다는 미명 하에 옛 날 KMLE 기출 문제에 심심치 않게 등장했 던 문제입니다. 너무 자세하게 아실 필요는 없겠지만 한 번쯤 왜 murmur가 증가하고 감 소하는지에 대해 생각해보시는 기회로 삼아

보세요•八八 dynamic pressure gradient \ : 심잡음 크기 증가 LV contractility |

exercise, sympathom im etic am ­ ine, digitalis, isoproterenol

preload \

V alsalva m aneuver, sudden sta­ nding, nitroglycerin, amyl nitrite inhalation, tachycardia

afterload \

alcohol(vasodilation 을 일으켜 LV outflow tract pressure gradient | )

다음의 경우에는 심잡음 크기 감소

preload |

leg raising, blood volume expa­ nsion

afterload j

phenylephrine, squatting, sust­ ained handgrip Harrison 20 만,pp. 1793〜 1797

^AN S W ER

(돈

0)0

O

01.① 02.④ 03.③ 04.③ 05.① 06.⑤ 07.① 08.② 09.①



離 i s ____________ 심근염(Myocarditis)

Explanation

離 소아과 ■각론도 항7게 참고하세요, u r i history 있은 후 pericarditis처령 보이는 증상(특진5 ST분혈 연술, 흡 기 시 pleuritic pain, low tidal volume)과 聊 I CK-MB '살은 cardiac marker 상승 소'73이 동반되는 7X이 특 진있니다. 최근 들어 여러 차ai 다루어졌습니다ᅵ

1. 정의 Myocarditis : 심장을 구성하는 조직(심근,간질,소혈관 등)의 염증

2 . Etiology 1) Infection : 대부분 virus(coxsackie virus B 등), 그 외에 HIV, bacteria(diphtheria, salmonella 등 ), trypanosoma infection(Chagas disease), Lyme disease(tick-bite) 2)

Drug hypersensitivity, Radiation therapy, chemical or physical agent

3 . Clinical m anifestation of viral m yocarditis 1)

무증상에서부터 전격적 진행(부정맥,실분절, 심하면 사망)까지 다양

2)

무증상인 경우 ECG 에 transient ST-T wave change

3)

심근염 환자에서 흉통,EKG 상 ST elevation, troponin의 증가를 보일 수 있음

4)

Myocarditis with pulmonary hypertension : high risk of death

5) 진찰 소견 : 대부분 정상,일부에서 S, sound i ,& 및 M R 의 심잡음,pericardial friction rub 6)

Viral myocarditis (1) 대부분 self-limiting한 경과를 밟으나 일부에서 chronic dilated myopathy로 이행 (2) U R I history(l 〜2주 전〉or viral nasopharyngitis/tonsillitis 등이 동반되기도 함 (3) Stool, pharyngeal washing 등에서 virus 검출,또는 specific antibody titer 측정이 도움 (4) Endomyocardial biopsy : lymphocyte infiltration, necrosis of myocytes (Bacterial cause는 PMN, hypersensitivity는 eosinophil dominant)

4. Treatm ent 1)

활동 제한 : ECG와 좌심실 기능이 정상화될 때까지

2)

심부전이 발생한 경우 : ACE inhibitor, diuretics, salt restriction 등,digitalis는 반응 |

3)

부정맥 등에 의해 환자의 상태가 나빠지거나 사망(예, complete heart block)할 수 있으므로 ECG monitoring

4)

전격성 심근염 (Fulminant myocarditis) : mechanical cardiopulmonary support 또는 심장 이식 Harrison 20핍:,pp. 1784—1787;이영우 귄저,순환기학,pp.384—389

O 0311

2020 년 대비 PACIFIC KMLE

순환기

Q

U >, _

01

30세 여자 환자가 호흡곤란과 가슴통증을 주소로 내원하였다. 1주 전부

1주 전 상기도 감염을 겪은 후 경정맥 확장, 제3심음,폐부종을 보이는 급성 심부전을 보 이는 환자입니다. troponin I 가 상승소견으로

터 발열,인후통이 있었다. 혈압은 105/75,맥박은 130회,호흡 27회, 체&은 38.0도였다. 앉은 자세에서 목정맥이 목의 윗부분에 보였고 제3

심근에 문제가 발생한 것을 알 수 있습니다. 따라서 제시된 보기 중에 가장 가능성 있는

심음,수축기 심잡음이 들렸다. 가슴에서 전반적으로 거품소리가 들렸다.

질환은 급성 심근염입니다. 가슴통증의 양상과 EKG의 ST분절 상승 양상 (concave or convex, reciprocal change 동반

EKG 검사 상 ST elevation 소견이 \A~Ve, 1 ,1 , aVL에서 있었다. Troponin I 검사 상 7.5(< 0.03)이였다. 진단은? 1) 급성 폐색전증

2) 급성 심장막염

3) 급성 감염성심내막염

4) 급성

여부)에 대한 정보가 구체적으로 주어지지 않 았기 때문에 이 정보 만으로는 acute coronary syndrome인지 pericarditis인지 가늠하기는 힘듭니다. 하지만 두 가지 질환 모두 my-

시그여 O •ᄂ • 口

ocarditis와 합병될 수 있습니다. 특히 심근염 환자에서 acute coronary syndrome이 촉발될 수 있고 이로 인해 흉통,EKG상 STE, tropo-

5) 급성 승모판역류증

nin의 증가를 보일 수 있다고 합니다.

Harrison 20^, pp. 1784〜 1787

________________________________________1八B

_

26세 여자가 내원 전날 시작된 누울 때 심해지는 호흡곤란과 흡기 시 흉 통을 주소로 내원하였다. 1주 전부터 감기 기운이 있어 약을 복용하였

0 2 Pleuritic pain을 직접 경험해본 人[람들은 숨 을 들이쉴 때마다 아프기 때문에 low tidal volume으로 숨을 얄게 쉴 수밖에 없게 된다고

다. 심전도는 비~6, n, n, aVF에서 ST절이 상승되어 있었다. 혈액 검

하더군요. 7일 전부터 viral pathogen에 의해

人[는 다음과 같다. 진단은?

면 심해지는 호흡곤란에 pleuritic pain, 게다 가 심전도의 upward concave ST segment

백혈구 12, 000/mm3,CK 947U/L(참고치,40-150U/L) 1) 심근염 3)

대동맥박리

발열과 피로감을 경험하였고 3일 전부터 누우

상승이 있어 pericarditis를 의심해볼 수 있어

폐혈전 색전증

요. 그런데 myocyte에서 기원하였을 것으로

불안정협심증

있어요. 보기에 제시된 질환 중 myocarditis에

보이는 CK가 참고치보다 현격하게 증가되어 의해 CK가 상승했다고 보0|야겠지요. myocarditis는 무증상에서부터 ST segment 변

5) 심근경색

호! 로만 추측해볼 수 있는 정도,더 심해지면 arrhythmia, heart failure에 이르기까지 다양 한 임상상을 보일 수 있어요. 실제로 심근염과 심막염이 합병되는 경우도 흔하구요.

Harrison 20만,pp.l784~1787

r ---예

!

____________________________________________________________

^ 0 3 가. Viral myocarditis가 가장 흔한데 이 경우

다음 중 심근염에 대한 설명으로 맞는 것은?

에 대부분 1〜 2주 전 상기도 감염을 앓은

가. 1〜2주 전 상기도 감염의 병력이 있는 경우가 많다. 나. 심근 효소치가 상승한다. 다. 대부분 self-limiting이다.

병력이 있습니다나. 일부에서는 심근 효소치가 상승해서 acute myocardial infarction과의 감별을 요하 는 경우도 있어요다, 라. 대개 self-limiting disease입니다. 일

라. 확장성 심근증으로 진행될 수 있다.

부에서는 dilated cardiomyopathy로 진 행하기도 한답니다.

1)

가, 나,다

2)

가,다

4)



5)

가, 나,다, 라

© ANSW ER

312)l o

o-

01.④ 02.① 03.⑤

3) 나,라

Harrison 20ᅵ 만 ,pp. 1784〜 1787

심막질환 심장을 둘 고

91는 막면 심막에는 업증이

Vi

수 9 R 영증 수 협착이 생기기도 하고. ^ ^ 1 영증 울진이나 %oj| 등이 과도하-Hᅵ찰 수도 앗습

니다. 그래서 각각 급성 심막eg, 교착 성막표 심장 눌렁증이라는 질병이 생 7』니다. 각각의 심장 진찰 소고 심전도, 심초음파, Chest x-ray 를 진 기억해 두세요. 심장 늘림증은 임상에서 농치면 ^ 4 % 1 과를 초래할 수 91기 때운에 « 우 중요6 ™ 다早어지는

니다. 심장 늘령증의 triad(Bp 강소,

심음 I t 소. 목정맥 울혈)를 꼭 기억해 주시고' 치료는 심장막^다라는 정 DIHJ ^ 조형니다. 과거에는 심정 눌^증이 생길 수 91는 멍력을 던트로 제시해 주는 T i 이 보통이있으나, 최근에는 병력에 대던 자Ml던 면급 없이 성헤침진 또는 가슴 X 선 소^ , 가슴 CT 소정만으로 심장눌림증을 진단하는 문제도 앞이 출r a a 고 91으니 주의6 M 요. 2019면에는 R형 1문■제를 포현하여 총 4문rar가 출raia 9 i 습니다.

mmImmmm酬wmmm



B B M B

^^1

Explanation

급성 심막염(Acute

pericarditis)

傘 Pleuritic chest pain을 0표으^는 대표적인 진환0』니다. 허열성 심진환과 비교해서 흉통의 행상은 에현지,에황 I재 악^/완화되는지를 정리해 두세요. 모든 주제'7ᅡ그령듯이 진1난과 치료가 중요형니다. 대표적인 진단법 3가 지로 chest PA, echocardiography, EKG가 91습니다. 치료는 bed rest, aspirirvj : 시•용하고 立과가 없을 때 NSAIDs. cd ch ic in e 을 사용형니다. 강영성이 아닌 정우 steroid를 사용형니다. 심장눌첨증의 위험 매문에 anticoagulation-S- 금기입니다. 傘 급성 심 빼 은 대부분 self-limiting하'거나,기저 우 d91(요독증 등)이 해소되면 윌-hi 치료윌니다. 그러나 드울게 약울치료oil 진 4J:응하지

1.

나 자주 厂빠하는 경우,근본적0il 치료는 pericardiectomy0』니다.

C lassification of pericarditis 傘 Visceral/parietal pericardium 으로 이루어진 pericardial space 에는 parietal pericardium 으로부티 나온 transudate 15〜 50ml_ 정도가 들어 91습니다. Pericardium 은 심장을 보호하며 대형곡!; 등의 우^ I 를 진oi■주며

심장과 주위 구조울의 마칠;을 줄여주는 역 ^을 현니다. Pericardium이 없어도 대개는 별다른 이상이 생71지 앞 아 pericardiectomy를 형:

수 9JL습니다.

1) 임상경과에 따른 분류 (1) Acute pericarditis( < 6 ^ ) : fibrous, effusive (2) Subacute pericarditis (3) Chronic pericarditis( > 6 개월) : constrictive, effusive, adhesive 2) 원인에 따른 분류 (1)

감염성 심막염 ① Viral( coxsackie viruses A and B, echovirus, mumps, adenovirus, EBV, HTV, influenza) ② M ycobacterium tuberculosis

③ Bacterial, fungal infection 등

O : 0(313'

2020 년 대비 PACIFIC KMLE

순환기

(2) 비감염성 심막염 ①

심근경색



신부전(요독성 심막염)

(1)

종양 : 원발성 / 전이성(폐,유방,림프종,백혈병,악성 흑색종)

④ 방사선 노출 ⑤ 외상 © 원인불명 (3) Hypersensitivity or autoimmunity

① Rheumatic fever ② Collagen vascular disease (SLE,rheumatoid arthritis, ankylosing spondylitis, scleroderma 등)

③ 약물 傘 SLE의 진1 기준 DOPAMIN RASH 중 하나로 serositis(pericarditis, pleuritis)가 9JL9i죠?

2.

Clinical m anifestation 1) Chest pain (1) Retrosternal/left precordial severe pain —►등이나 왼쪽 어깨로 뻗치는 referred pain (2) 앉거나 앞으로 숙이는 경우 1 , 누우면 심해짐 (3) Pleura가 침범되면 pleuritic pain 의 양상도 보임(호톱이나 기침 시 / 자세를 바꿀 때 심해짐) (4) 천천히 진행되는 Tb, postirradiation, neoplastic, uremic pericarditis에서는 없을 수도 있음 (5) Epicardium 이 침범되면 급성 심근경색과 양상이 비슷함

(쥐어짜는 것 같은 통증이 팔로 방사됨) ᅳ 심전도에서 extensive ST elevation이 보이고 CK^ troponin이 중등도로 상승하는 경우가 있어 감 별에 주의 2) Pericardial friction rub (1) Acute pericarditis에서 가장 중요한 소견 (2) 앉은 자세에서 호기 시에 diaphragm으로 left lower sternal border 부위를 누르면서 듣는다. 傘

Friction mb을 던트로 주는 정우가 핑장히 ^ 습니다. '마찰윰•,■바스락거리는 소리' 등으로 표현하곤 하죠. 단순히 pericardial friction rub이 라 고 외 우 지 마시고 어1전 식으로 표현도J는지도 알이■두세요!

(314)。 C

c-

3)

심 막질환

ECG

ST segment

Reciprocal change QRS complex T wave

A c u te pe ricard itis(A )

A c u te m yo ca rd ia l infarction(B)

U pw ard co n ca vity( U )

Upw ard con vexity( n ) —» 비석모양

2 〜3 개의 표준유도,V 2~ 6

Ischemia/infarct site 반영

두드러지지 않음

특징적임

(aVR, Vi 등에서는 가능)

Ischemia/infarct site 반영

별 변화



Q wave, R wave 의 변화

Pericardial effusiᄋn 이 많을 경우 amplitude 1

수일 후 ST segment가 정상화되고 나면

ST segment가 isoelectric하게 정상화되기

T wave inversion

수 시간 전에 inversion

cf. Normal variant 중 early repolarization이 일어나는 경우 widespread S T segment elevation이 있을 수 있으나 이 경우 tall T wave, S T/T ratio 만,pp. 1843~ 1844

13

Q

30세 여자가 식은땀과 숨이 차는 증상으로 내원하였다. 3년 전 SLE 진

13 Hypotension과 paradoxical pulse소견을 보 이고 폐울혈은 없는 R V 혈액 유입장애를 보

단을 받았고 내원 시 혈압 80/60mmHg였고 맥박 125회/ min이었다. 들

이는 환자입니다. RV 혈액 유입장애를 보이

숨 때 수축기 혈압이 60mmHg로 감소하였다. Chest X-ray에서 심장의

는 질환에는 협착심막염,제한성심근병증,심 장눌림증,RV infarct가 있습니다. 이중에 심 전도에서 QRS의 크기가 작아진 소견을 보이

확장소견은 있었으나 폐울혈은 없었다. 심전도에서 QRS으 ᅵ크기가 작았

는 질환은 cardiac tamponade입니다.

다. 진단은? 1) 급성 폐색전증

2) 심장눌림증

3) 긴장 공기가슴증

4) 제한 심장근육병증

5) 비대 심장근육병증

이 문제는 SLE induced pericarditis로 인해 cardiac tamponade가 발생된 증례입니다. Harrison 2Q만,pp. 1843~ 1844

k

문제해설



50세 여자가 숨이 차고 어지러워서 병원에 왔다. 3개월 전부터 서서히 숨이 차기 시작하였고 1주 전부터는 누워서 잘 수 없을 정도로 숨이 찼 다. 혈압 70/40mmHg, 맥박 104회/ 분,호흡 20회/ 분,체온 36.6도였 다. 목정맥이 늘어나 있었는데 들숨 때에 줄어들었다. 들숨 때에 노동맥 (radial artery) 맥박이 만져지지 않았다. 제3심음이나 심잡음은 들리지 않았다. 가슴 X선 사진과 심전도 사진이다. 진단은?

----------- c/^o

14 가슴 X선 사진에 water bottle 소견 보이고 심 전도에서 low voltage 보이므로 pericardial effusion을 쉽게 생각할 수 있습니다. 혈압이 72/40이므로 심실로 혈류가 들어오지 못할 정도로 effusion이 생긴 심장눌림증입니다. 목정맥이 늘어나 있다가 들숨 때 줄어들었다 는 것은 Kussmaul’s sign 없음을 으ᅵ미합니 다. 심장눌림증에서는 한정된 공간이지만 effusion이H로 우선 RA로 혈액이 들어오는대 로 공간을 선점할 수 있습니다. SVC에서 온 혈액이 RA로 들어갈 수 있으므로 JVP는 상 승하지 않아 Kussmaul’s sign이 잘 관찰되지 않습니다. 들숨 때에는 흉강이 음압이 되므로 SVC, IVC를 통해 정맥환류가 더 많이 일어 나게 도ᅵ고이에 따라 RV preload가 증가합니 다. 반면 흉강의 음압으로 폐정맥을 통한 LV 로의 혈액 유입은 감소히는데다 요부의 부피 가 늘어나면서 septum이 LV 쪽으로 쏠리게 되어 LV의 preload는 더욱 감소하게 됩니다. 이로 인해 0 ᄋ가 감소하게 도ᅵ어들숨 때 노동 맥 맥박이 만져지지 않게 된 것입니다. 기이 맥에 해당하는 소견입니다. Harrison 2ᄋ ᅵ 만 ,pp. 1843〜 1844

1) 심근염

2) 심장눌림증

3) 협착심장막염

4) 비대심장근육병

5) 제한심장근육병

5



44세 여자가 운전 중 추돌사고로 응급실에 실려 왔다. 가슴에 멍이 들었 고 얼굴이 푸르스름하며 경정맥의 확장이 관찰되었다. 심음은 희미하게 청진되었을 때 가장 가능성이 있는 진단은?

1) restrictive cardiomyopathy

2)

aortic dissection

3) tension pneumothorax

4)

flail chest

5)

cardiac tamponade

15 외상을 의심케 하는 가슴의 멍,경정맥 확장 (CVP |), 심음의 감소로 미루어 보아 cardiac tamponade가 의심됩니다. 얼굴이 푸르 스름한 것은 정맥 울혈에 의한 것으로,역시 cardiac tamponade의 소견입니다. 傘Beck’s triad in acute, severe tamponade : BP [, CVP |,heart sound [ Harrison 2Q만,pp.l843~1844

2020 년 대비 PACIFIC KMLE

순환기

16

Q

호흡곤란과 기침을 주소로 내원한 42세 남자이다. 열흘 전부터 숨이 차 서 누워있기 힘들었다. 혈압 80/60mmHg, 맥박수 분당 118회였고,흡 기에 수축기 혈압이 더욱 감소되었다. 심 초음파 사진이다. 진단은?

급성 심근경색

2) 심장눌림증

승모판협착

4) 비대 심근병증

----------- cA> 16 숨을 들이 쉴 때 radial artery의 박동이 느껴 지지 않은 것은 흡기 시에 systolic pressure가 lOmmHg 이상 감소히는 pulsus paradoxus 현상으로 볼 수 있고 초음파 사진상 pericardial effiision이 보입니다. 심장 주변의 배 경이 까매서 심장만 외롭게 바다에 떠 있는 것 같이 보이죠. 1주 전부터 누워서 잠자기 힘들고,기침이 났다고 했으니까 viral patho­ gen 등에 의한 acute(idiopathic) pericarditis 에 따른 pericardial effusionO |상대적으로 짧 은 시간에 일어나 ventricle로의 blood flow 유입이 현격하게 떨어지는,cardiac tamponade를 생각해야겠습니다. Harrison 20ᅵ 만, pp. 1843〜 1844

협착 십장막염

17



52세 남자가 dyspnea를 주소로 내원하였다. 2개월 전 SCLC 진단받고 항암요법 중이었다. 내원 당시 BP 70/30mmHg, HR 103회/분이었으며 말초의 청색증이 관찰되었다. 심초음파가 그림과 같은 때 적절한 조치는?

17 심초음파 소견이야 거의 항상 판에 박힌 모습 만 제시되지만 환자의 clinical situation은 문 제마다 바뀌는 경향이 있어요. 이 환자에서는 폐암으로 항암요법 중인 52세 남자 환자에서 의 호흡곤란이 제시되었습니다. neoplasm에 으ᅵ한pericarditis 증례입니다. 답은 peri-cardiocentesis. 주사기 tJ|늘을 심전도 precordial lead로 써서 ST segment elevation이 나 타나면 바늘 끝이 심장의 epicardium을 건드 리고 있구나 하는 것까지도 알 수 있대요. Harrison 20만,pp. 1843〜 1844



^급 으 로

RTx

2) pericardiocentesis

3

high dose CTx

4) steroid pulse therapy

5

supportive treatment

©ANSWER

01.① 02.④ 03.③ 04.④ 05.④ 06.② 07.⑤ 08.⑤ 09.① 10.③ 11.② 12.③ 13.② 14.② 15.⑤ 16.② 17.©

334

O

O

c•

립법観 @|

..................... ......................—

_ — ᅳ ᅳ ᅳ ᅳ —

교착 심막염(Constrictive

^^1





심막질환

Explanation

— —

pericarditis)

*

Chest PAC>il서 보이는, 설진막4 따라l ^ i I calcification으Iᅵ특7j땐 0』니다.

*

제한 심근병증(RCMP)과 햇갈a 지 마세외 교착 상각영은 심장막의 진%, 제한 심근병증은 심근의 진환04니다.

1. Etiology 1) 결핵이 가장 흔함 2) Trauma, cardiac operation, irradiation, purulent infection, neoplasm(lung, breast, lymphoma), acute pericarditis, RA, SLE, 장기 혈액 투석을 받는 CRF with uremia

2. P ath op h ysiology 1) 급성 심낭염이나 심낭 삼출이 회복되는 과정에서 심막에 육아조직 형성 — 섬유화 및 석회화 발생해

쪼그라듦 — 딱딱해진 심막은 ventricular filling

[

in diastolic phase(tamponade와 비슷한 기전)

2) Chronic constrictive pericarditis (1) Ventricular end-diastolic pressure | /stroke volume | (2) 심장의 4 chamber의 확장기압이 같이 상 ^하 고 평준화(equilibratiorp 됨

ᅳ RV 와 LV 의 end-diastolic pressure가 동일 (3) 심근 자체의 기능은 정상

3. Clinical 1)

&

lab oratory findings

Symptoms (1) Weakness, fatigue, weight gain(말라보이기도 함), increased abdominal girth, abdominal discomfort (2) Edema, exertional dyspnea, orthopnea (3) Acute LV failure(pulmonary edema)는 드물 離 RV로 들어오는 정0그 |%류•가 줄어들기 매문에 폐부종은 드울죠. (4) Congestive hepatomegaly(간기'^ 저히*), ascites, dependent edema 傘 Murmur나 심비대 등도 특진적이지 양고 "간비대에 "y:기능 저하,intractable ascites ᅵ패문에 liver cirrhosis(LC)로 오91도n i 도 측!:다네요. LCᄋ ᅵ I서는 neck vein distention이 나타나지 있지요.

9

2020 년 대비 PACIFIC KMLE

순환기

2) Sign (1) Neck vein distention (2) Kussmaul’s sign : 흡기 시에 jt 방ular venous pressure가 감소하지 않음 傘 iH 착 생 관 g 시에는 RA가 학 닥 한 생각에 둘러 ^

있 기 에 충분히 늘어나지 못해,음기시 증가하 는 정0抱

류를 다 수용하지 못하고 성여서 JVP가 증가하 '>11평 니다 . 거의 동일호!; 이유로 restrictive cardiomyopathy 에 서 도 나EJ■님 니 다 , tricuspid stenosis, right ventricular infarction ᄋ사서는 backward pressure 가

a 서 우 심 4 ^°』이 높아져서 나 타 나 죠 ,

(cf. 성7양놀원중에서는 Kussmaul1sign이 없었죠. c^ ^ i : 심막이 아니라 심닝내 effusion이으로 RA 는 충분히 늘어1날 정도의 공'7uᅡ이 91어서 흡기시 증가하는 정1객환류를 다 수용 가능하기 매문0』니다) (3) Paradoxical pulse(〜 1/3) : 흡기 시에 SBP가 1OmmHg 이상 감소 傘

제한^ 공!!: 안에서 RV, LV가 부피 경행을 하는 정우는 다 기이1객이 생7』니다.

(4) Apical pulse | ,distant heart sound, pericardial knock(=S 3 sound) #

씽/ i 이완기 초기에는 급'7 和 I 이완되다가【 닥표닥한 심막에 의해 제형:도j는 순'7J; 딱! 이%을 엄추는in 교착 심막영과 r-cm p의 특진0』니다. 따라서 급'7 ᅡ이완:시 S3가 들딩니다.

3) ECG (1) Low voltage of QRS complex (2) Diffuse flattening or inversion of T wave (T wave inversion은 acute pericarditis 에서 ST segment가 정상화된 다음에 일어나는 변화 八 八) (3) Atrial fibrillation 〜 1/3

Jl 느 Vi

^

■x

aVF

v4

v5

V6

A------

v3



aVL

/ᄂ 이

V2



aVR



^ ᅳ -

^

'■어

*

____ ^





'

…—

…*.......................................................................... ..—

- —

-비-

•- m

쇼j 막■ :밀환

z

z

z

4) Chest PA : pericardial calcification, normal or slightly enlarged heart

5) 심초음파(echocardiography> (1) 두꺼워진 pericardium (2) Atrium, IVC, hepatic vein0] 늘어남 (3) Ventricular filling 이 early diastole에 갑작스럽게 끝남 (4) 수축 기능은 정상 (5) Doppler에서는 cardiac tamponade와 같이 흡기 시 pulmonary vein/MV flow 속도 丄, IVC 와 TV flow 는 반대 6) 심도자(cardiac catheterization) (1) 심방(RA, LA) —

or

shaped(prominent x & y descent)

(2) 심실60mg/dL : negative risk factor) 2) 超 3) 고1 알(BP >140/90mmHg 또는 항고혈압제 복용 중 ) 4) 달논병 5) 조즈ᅵ CH D 의 가족력(직계가족 남성 45세,여성 之55세 이상 7) 생활습관 (1) 빈만30kg/m2) (2) 운동부족 (3) Atherogenic한 식이 8) Emerging risk factors Lipoprotein, homocysteine, prothrombotic factors, proinflammatory factors, impaired fasting glucose, subclinical atherogenesis Harrison 19핍;,pp.291el~292el

죽상경 화증

c

^■자

여자

나이

>45세

>55세

가족력

Inferior

상 Cardiac SPECT

(왼쪽)

short axis view

vertical long axis view

Ap

apex transverse long axis view septal ;

lateral

base

!

^

2020년대비PACIFIC KMLE

순환기



A

부하기

휴식기

세 주동맥분지에 각각 협착이 있었던 예의 휴식/부하 심근 SPECT소견 A,

LAD,

B,

RCA,

C,

LCX

傘 단일 양자방출 전산화 단층촬영 (single-photon emission computed tomography, SPECT) - Ejection fraction과 regional wall motion 평가, )9wTc 이

더 선호(higher count rate)

- 응급실에서 acute coronary syndrome을 의심했다가 퇴원해도 좋은가를 판정할 때 심근 SPECT를

사용, 장 단기간의 예후를 예측, SPECT 결과가 정상이면 관상동맥 질환을 배제해도 됨(high neg^ ative predictive value)

기절심근(stunned myocardium)

동면심근{hibernating myocardium) 수축력 휴식기



^■소 ^■소

정상

포도당 대사

정상 또는 증가

증가 또는 감소

수축력 회복

재관류 치료 후 가능

추적기에 호|복

양전자방출 단층 촬영 (Positron Emission Tomography, PET) - 18F-FDG(fluorine-18 deoxyglucose) : 심근세포의 포 도 당 추출과 hexokinase 작용에 의한 phosphorylation 과정을 반영, 심근세포의 당대사를 평가

376

o

o

€ 1 1



장질¥



— Ischemic myocardium : FDG uptake ] I perfusion | (“glucose / blood flow mismatch”) ~ »• 심근

의 대사가 fetty acid나 lactate보다 glucose에 주로 의존하게 됨 ᅳ revascularization 후 fimction의 호전을 기대할 수 있는 hibernating myocardium 탐색에 유효 : gold standard for myocardial viability Harrison 20만,pp. 1854-1857;이영우 면저,순환기학,pp. 124-140, 152〜 163

=5=기타 핵의학적 검사들 — 99mTc-pyrophosphate scan : 괴사된 심근에 섭취

一 Radionuclide angiography(multiple-gated blood pool imaging : ""Tc-RBC or albumin 이'§-) :심실기능평가0.2mV이면 검人® 중단합니다. 2) V5, V6 lead에서 2mm이상 ST segment depression이 있어 심한 관상동맥질환을 시사하는 소견입니다. 이외에도 I I ,I I I , avF lead에서도 ST segment depression 소견이 관찰되고 있습니다. 3) Lt. main or multivessel disease가 의심 됩니다. 4),5) Stress test상 심한 관상동맥질환을 시 人!■히는 소견이 있다면 early invasive strategy를 통해 관상동맥의 협착 정도를 정확히 파악해야 합니다. 따라서 관상동맥 조영술을 바로 실시합니다. Harrison 20만,pp. 1854~ 1857

1) 운동시 흉통이 발생하지 않더라도 다음단계를 중지시킨다. 2) 관상동맥협착이 경미할 것으로 추측된다. 3) 우관상동맥 협착이 심할 것이다. 4) SPECT 검사로 관상동맥조영술 검사여부를 결정한다. 5) 약물치료로 증상조절이 가능하며,베타차단제가 효과적이다.

---------I 따

L _________________________________________________________________ ^

stress test에서 심한 관상동맥의 이상이 의심되는 경우가 아닌 것은? 1) 운동부하 시 심박수 100회/분의 심실빈맥 발생 2) 운동부하 시 ST 하강이 2mV 이상인 경우 3) 운동부하 검사 중 혈압이 상승하지 못하는 경우 4) 운동부하 Thallium scan에서 폐의 Thallium 톱수 증가 5> 운동 중 ST depression이 lmV 발생하였다가 운동 종료 직후 사

라지는 경우

0 2 운동부하 심전도 검人[에서 심한 관상동맥질환 을 시사하는 소견 ① 2mm 이상 ST segment depression ② Bruce protocol 1st. stage에서 1mm 이 상 ST segment depression ③ 운동 종료 후 ST depression이 5분 이상 지속되는 경우 ④ BP가 lOmmHg 이상 감소하는 경우(또는 상승하지 못하는 경우) ⑤ 심한 ventricular arrhythmia with HR < 120bpm ⑥ Bruce protocol 6분을 완료 못하는 경우 * 부하 심근관류 스캔 검人[에서 심한 관상동맥 질환을 시ᄉ卜하는 소견 ® > 15% ischemia of LV ② Multiple perfusion defect(2개 이상의 vascular bed에서) ③ Large & severe perfusion defect @ LV dilatation ⑤ Lung uptake with exercise ⑥ Post-exercise LV dilatation Harrison 2Q만, pp. 1854~ 1857

^78

o

a-

허혈성 심장질환

03

____________________________________

B

60세 남자가 운동 중 발생한 흉통을 주소로 내원하였다. 적절한 처치 후 익물부하 및 휴식 시 심근관류 SPECT(single photon emission com­ puted tomography)를 시행하였다. 부하기와 휴식기의 전벽관류의 차 이에 대한 설명으로 옳은 것은?

가. 전벽심근의 허혈 소견이다. 나. 운동부하 심전도를 했다면 V2^4 ST분절의 하강이 나타났을 것이다. 다. 전벽심근관류가 운동 시 산소요구량의 증가에 미치지 못한 것이 흉통의 원인이다. 라. 좌전하행동맥의 단면적 50% 협착이 이 환자의 운동 시 허혈의 원인임을 나타낸다.

0 3 익물부하 심근관류영상에 관한 문제입니다. myocardial SPECT는 3가지 view로 영상을 보여줍니다.(short axis, vertical long axis, horizontal long axis) 사진은 vertical long axis view입니다. 그림의 위쪽벽이 LV의 anterior wall, 아래벽이 LV의 posterior wall에 해당돼요. 가. rest에 비해 stress 때 ant. wall의 관류가 감소한 것처럼 보이나요? ant. wall의 ischemia를 나타내는 소견이죠. 나. 이 부위에 혈류를 공급하는 관상동맥은 LAD이죠. reservoir가 부족해서 부하 시 충분히 혈류공급이 되지 않아 흉통도 생 기고 SPECT 사진에서 uptake가 감소한 것으로 보이는 거죠. 다. EKG에서 V2~4 는 LV anterior wall에 해 당되는 부분입니다. 라. L\D의 협착이 원인인데, 증상이 나타나 려면 단면적으로 75% 이상(= 지름 50% 이상) 협착이 있어야 합니다. Harrison 20만,pp. 1854〜 1857

1) 가,나,다

2) 가’ 다

4) 라

5) 가, 나, 다, 라

©ANSW ER

이 ■① 02.© 03.®

3) 나,라

2020 년 대비 PACIFIC KMLE



순환기

험@ 1 1 ^ 관상동맥조영술(Coronary

Explanation

angiography)

傘 기본적으로 PCI가 필요한 STEMICHI서는 가능헌: 멜:리 coronary interventiorri: 해야 형니다. Stable angina와

UA/NSTEMI에서 Coronary intervention이 ᅵ 1®•요한 경우를 일: 아봅시다. 세세한 적응증을 외우기보다는 어1헌 % 자에서 coronary angiography가 조 달 •요학! ;지 느77ci을 진도륵 형시다.

1. 적응증- Stable angina 1) Chronic stable angina pectoris 환자가 적절한 내과적 치료에도 불구하고 증상이 심하여, revascula산 zation(PCI, CAB G >을 고려할 때 2) 진단이 어려운 환자가 괴로운 증상을 호소하는 경우 관상동맥질환을 확진하거나 배제할 필요가 있

을때 3) 심장마비에서 살아남은 환자가 angina pectoris를 진단받았거나 혹은 의심될 때 4) Angina가 있거나 또는 noninvasive test에서 ischemia의 증거가 있는 환자가 임상적 소견 또는 검사

소견에서 ventricular dysfimction의 증거가 있을 때 5) Noninvasive test 결과 h ^ h risk 소견을 보인 angina pectoris 환자 傘 즉,앞에서 설명한 부 하 ^사 '할과 심한 관상동0과 진환을 시사하는 소'7크을 보이는 경우를 많^ 니다.

6) 기타 (1) Anginal pain이 있으나, stress test에서 ( ― ) 또는 nondiagnostic 소견을 보인 환자 (2) Acute coronary syndrome이 의심되어 반복해서 입원하지만 진단이 확실하지 않은 경우 (3) 타인의 안전을 책임져야 하는 직업(예, pilot, 경찰,소방관)을 가진 사람이 의심스러운 증상이

있거나 noninvasive test가 정상이 아니며,coronary artery의 상태가 의심스러울 때 (4) AS 나 HCMP에서 anginal pain이 있을 때 (5) Valve repair / replacement 등의 심장 수술을 받을 예정인 45세 이상 남자,55세 이상 여자(동반

된 무증상의 관상동맥 질환을 진단하기 위해) (6) MI 후 angina가 재발했거나 심부전이거나 ventricular premature contraction이 자주 있거나 stress test 결과 ischemia의 소견을 보일 때 (7) Coronary spasm이나 기타 myocardial ischemia를 유발하는 nonatherosclerotic cause가 의심될 때 : coronary artery anomaly, Kawasaki’s disease 眷 여기77卜지가 현재 Harrison 20xSoil 실탓 Stable angina에서 Coronary angiography의 적응증ᄋ』니다. 알깼지만 내용이 않으나 입어보면 coronary intervention(PCI)이

어보시면

내용들도 꽤 않습니다. 또한 STEMI에서는 哪 히 용급 Percutaneous

•요하며 coronary angiography도 도 !•요형니다. UA/NSTEMIᄋ ᅵ I서의

intervention의 적용증은 다윰과 길습니다. 이후 다시 나오'7셌지또卜 임어두고 념어'7』시다.

우 coronary

ᅳ Si" 성 ᅵ ᅳ '심^질환

2. 적응증- Unstable angina(UA)/NSTEMI 치료에도 불구하고 휴식시나 적은 활동에도 홈통이 나타나는 경우 TnT ^ 는 T nl 의 상승 : NSTE M I

2

ST segment depression이 새롭게 나타난 경우 4

흉통이 심부전 증상, rales, M R 과 동반되어 나타나는 경우 EF 3)

* TIMI 정수란 다음 7개 헌목 중

당하는 현목의 수로 정한다.

① 65서I 이상 ② 그 ? i 상동맥칠환의 위덩0요재고혈입, 흡면, low HDL, 1당뇨, 55세 대만의 님?;ᅡ 진족 또는 65세 몌만의 여자 하족에서 관상동다1(진환 발생) 중 3■개 이성; ③ 과거 50% 이성의 관1상동0내 형착

④ 심전도상 ST elevation / depression ⑤ 지4 : 24시ᅳ让 이내에 2회 이상의 anginal pain (6 )

W 4 7일 이내의 아스피^ i 복용

⑦ 심근효소 상승 Harrison 20만,pp. 1857-1858, p. 1871 이영우 퀸저,순환기학, pp. 172-173

Y

2020년 대비 PACIFIC KMLE

순환기

霞醒纖薦

안정협심증(Stable ♦

Explanation

angina) — 약물치료

만정협심증은 치료라기보다

되지 앞도록 유지하는 ^이 더 중요현니다. 약물치료가 복진6j :^ 은 아닝니다.

Aspirin과 Beta blocker와 Nitrate를 주로 사용하고 CCB를 추가할 수 91습니다. 네 가지 약제 모두 순환기에서 않이 나오는 약제이므로 기 ^에

이민 파트에서 다루도록 하74습니다.

* 동맥경화 위험요인 교정 - 금연, 체중조절 - 지질강하제,경구혈당강하제(DM),항고혈압꺅제(HTN),엽산ᅡ게 되는 ■것이으로 BB는 금기0』니다. 비교해서

보f두면 종은 것은 ' ■살색세포종의 수술 진처

치ᄋ』니다. 칼 SljAdl포종의 수술 전에 BB를 면저 사용해서는。 개 니 다 . DJ5갈 BB를 면저 사용하>11 되면 상 대적으로 a adrenergic receptor가 과활성회■되고 이로 oj^H a \ receptc)r의 작용으로 혈1 수축이 증가해 서 고형0음을 ᄋ 4^ 시려 crisis가

수 91습니다. 따라서 a-blocker를 면저 사용한 후 BB를 후에 사용하

죠. 심장에 대dH서는 C.O.을 높이는 inotropic 역 ^을 한다고 했죠? 따라서 C.O.이 너무 별어져 있거나 혈0故 1 '■낮으면 신-blocker를 쓸 수 없습니다. 그 박에 BB는 AV node의 ^도속도를 늦추는 항부정0과제이 으로 AV block이나 severe bradycardia에서 금기0』니다.

4)

분잠용 Fatigue, reduced exercise tolerance, nightmare, impotence, cold extremities, intermittent claudication, bradycardia, impaired AV conduction, LV failure, bronchial asthma, claudication & hypoglycemia의

^ •화

C a 一C h a n n e l b lo c k e r :nifedipine, verapamil, diltiazem 1) 작용기전 (1) Supply 증가 : coronary vasodilator (2) Demand 감소 : myocardial contractility, arterial pressure(afterload) |

myocardial O 2 demand |

傘 DHP CCB와' nonDHP CCB의 작용기^이 다르다는 기느기억하시죠? DHP CCB는 주로 형곡!:에 작용하여 열 관을 이완시키는 작용을 하고 nonDHP CCB는 주로 심장에 작용하여 심근 수축력을 강소시ᅦ고 심박수를 소시키는 立고H ᅡ있습니다.

2) Verapamil, diltiazem(nonDHP CCB) 傘 덩부생객저I로 사용되있S ^ 을 기억하세요. AV node 진도속도를 늦춰 심박수를 낮춤니다.

(1) 심장 내 전기전도의 이상과 bradyarrhythmia를 유발할 수 있음 (2) Inotropic action | —►LV failure 악화 3) Amlodipine, nifedipine(DHP CCB) (1) 강력한 혈관이완제 傘 cf) HCMP에서 -y 은 CCB라고 하더라도 鞭 을

이% 니혀 preload를 강소시키는 nifedipine(DHP

CCB)은 금기지4J: 심 例 1작용하여 LV 수축력을 강소시극1는 verapamil, diltiazem(nonDHP CCB) ■0:치료로 시■용정니다.

(2) 협심증과 고혈압을 동시에 치료하는 데 효과적 4) g-blocker와의 병용 투여 (1) Verapamil : 금기( V HR와 contractility가 너무 떨어지기 때문) (2) Diltiazem : 주의하면서 병용 5) 적응증 (1) 우blocker와 nitrate를 병용 투여했는데도 효과가 없는 경우 (2) 신-blocker의 부작용이 심한 경우 (3) Asthma 또는 chronic obstructive pulmonary disease의 병력이 있는 경우 (4)

Sick-sinus syndrome 또는 중요한 AV conduction disturbance가 있는 경우

허혈성 심장질혼r ᅳ — 경

(5) Variant angina 離 tgEJ 기진이 cc)rc)nary a.의 spasm0』니다. DHP-CCB를 시■용하면 혈그?i smooth m. 세포 속의 Ca2+을 줄여서 spasm을 억제할 수 91습니다. 그령다고 non-DHP CCB가 치료로 사용될 수 없는 7; 1은 아님니 다.

효과가 ᅵ 1 어지기 때문에 DHP-CCB를 않이 % 용■하는 7X 분0』니다. 2018^ 국시에서 출제

되(있기 oh문에 c갈아두시기 바협니다ᅵ

(6) Symptomatic peripheral arterial disease 傘 한마디로 않해 CCB는 신-blocker의대체재 또는 보완재 역 ^을 회:다고 보면 돼요.다4J; CCB는 심부7a 에서 금기라는

4.

A ntiplatelet

: a s p ir in ,

을 명심하시구요. 신-blocker는 급성 심부7&에서0소 금기죠.

c lo p id o g r e l

1) Aspirin (1) 작용기전 : irreversible inhibitor of

platelet cyclooxygenase activity ᅳ 혈소판 활성화 억제

(2) 지속적으로 투여할 경우,coronary

event를 예방하는 효과가 있음

(3) 부작용 : 출혈,알레르기, 소화불량 2) Clopidogrel (1) 작용기전 : ADP receptor-mediated platelet aggregation 차단 (2) Aspirin과 같은 효 과 & 부작용은 적음 難 UA/NSTEMI,STEMI의 기진은 atheroma의 손상으로 인해 방출된 물진들로 91한 혈소1&의 활성화로 thrombus가 형성되는

니다. 따라서 SA가 Ml로 진 ^하 지 앞기 우 ^서 는 혈소1S 활성화를 억제하는

이 기본이 병니다. 따라서 anti-coagulation Tx.가 아닌 anti-platelet Tx.가 중심이 ^ 니다.

5. Other treatment 1) ACEi : 고혈압,chronic IHD(stable angina 등), 혈관 질환(당뇨 등)이 있을 때 渗 Stable angina에서는 unstable angina나 STEMIᄋ right IMA ~ radial artery ~ gastroepiploic artery > saphenᄋ니s vein Atheroma 가 주로 관상동맥의 근위부를 침멍6 ᅡ기 매문oil bypass graft를 원위부에 면 ^하 는 CABG가 가능 행니다.

-o

o 397) 一

2020 년 대비 PACIFIC KMLE

순환기

6. Summary 반복되는 심근 허혈 증상 ST s e g m e n t s h ift o r d e e p T in v e rs io n ± c a rd ia c m a rk e r >1 A s p irin /9-blo c k e r Nitrs ite s H epa rin ± GP I b / I a

in h ib ito r

적극적인 모니터링

T nT 또는 T n l 의 상승 :NSTE-MI ST s e g m e n t d e p re s s io n 이 새롭게 나타난 경우 New b u n d le b ra n c h b lo c k o r 지속적인 V T

흉통이 심부전 증상,ra le s , MR 과 동반되어 나타나는 경우 혈역학적으로 불안정 한 경우(BP 의 감소,서맥

)

6 개월 전 PCI률 했거나,전에 CA BG 를 시행 받은 환자

Ischemia-guided strategy

Early invasive strategy 丄 Im m ediate

E arly o r d e la y e d

a n g io g ra p h y

a n g io g ra p h y

가능하면 재관류 치료

가능하면 재관류 치료

반복되는 증상, 허혈 심부전 쇼j 긱■힌■ 부정5与

안정화

LV fu n c tio n 평가

s tre s s te s t

丄 Y EF < 0 .4

Y

y

EF >0.4

>f

v

not low risk

low risk

내과적 치료 지속



7.

장기적 치료 :2차

1

급성 ᅭ관:상동ᅵ맥¥끟군

예방

효과가 입증된 5가지 약 물 ! 1) 욘-blocker 2) High dose statin 3) ACE inhibitor or ARB 4) Aspirin & clopid(^reI : 9〜 12개월 이상 병용 투여 후, 장기적 aspirin 단독 투여 傘 cf) AMI에서 생존율의 증가는 ABA 아바(아t바〜!) (aspirin, BB, ACEi)

AMI의 급성기 치료가 끝나고 퇴우cl시에 하는 치료 ABAS(antiplatelet, BB, ACEi, statin)

* 기억형시다 : NSTE-ACS -

: resting, new onset, crescendo angina

-

: plaq니e rupture, coronary spasm, progressive obstruction, 2° UA

- 진단 : 증상,EKG, cardiac marker, stress test(or coronary angiography) - 급성치료 : A'BCN(aspirin+ heparin+ clopidogrel, 크-blocker, CCB, nitrate) (coronary intervention) -

및 장기치료 :ABAS(aspirin & clopidogrel, 크-blocker, ACE inhibitor, statin)

—^ 내3 오는 ᄋ ct기1업은 c w ■비 시 (ACE inhibitor, statin, aspirin & clopidogrel, ^ -b lᄋcker)ᄋ 』니다스八

❖ 정리하면 NSTE-ACS(UA& NSTEMI)는 진단과 동시에 입우크을 통해 심근의 손성;을 최소화하는 가이 가장 중요입니 다. dHEJ슨oil서도 이제 더 이상 UA와 NSTEMI를 크>11 구분하지 ᅵ않아 UA와 NSTEMᅵ를구분하는

이 크'게

의이가 있지는 있을 기ᅡ^ 습니다. 즉시 우선 안정시키는 ^이 중요형니다. 또 심근 허혈을 조장하는 빈ᄐ맥,

HTN, 집상성기능항

진증,1말열 등의 이차적 원0그을 제거하는 h i 중요하죠ᅵ 약물적으로는 anti-platelet treatment와 anti­ thrombotic drug를 통해 심근손상을 최소화•하고,장기적으로 /5-blocker,Statin, AC曰 or ARB 등을 사용 하는 가을 기억형시다. 그리고 고위험군으로

매 시행하는 ear|y invasive strategy의 적용증도 알아농도록 힘시다. Harrison 20민•,pp.l866~1872

c-

2020년 대비 PACIFIC KMLE

01

순환기

____________________________________________________ B 册

^ _

68세 남자가 3시간 전부터 가슴이 아프다며 응급실에 왔다. 니트로글리 세린을 혀 밑에 투여한 후 통증이 없어졌다. 10년 전부터 혈압강하제와 혈당강하제를 복용하고 있다. 혈압 124/76mmHg, 맥박 78 회/ 분, 호흡 20 회/ 분, 체온 36.2°C 이다. 가슴 청진에서 심음과 호흡음은 정상이다.

심전도이다. 치료는?

혈청 트로포닌

T

17ng/L( 참고치,0 〜 14)

01 3시간 동안 지속되는 흉통을 주소로 내원한 환자로 니트로글리세린에 의해 흉통이 호전되 었습니다. 혈청 심근효소는 상승했으나,심전 도에서는 ST elevation은 없어 NSTE-ACS 의 증례입니다. 가장 먼저 해야 하는 것은 anti-ischemic treatment로 증례에서 이미 실시 한 니트로글리세린고卜 anti-thrombotic therapy로 aspirin, clopidogrel, heparin을 투여 하는 것입니다. 심장동맥중재술은 우선 antiischemic, anti-thrombotic treatments 통히] 환지를 안정시킨 후에 해야 합니다. 따라서 답은 LMWH에 해당하는 3)이 되겠습니다. Harrison 20판,pp. 1866〜 1872

1) 도파민 3)

에녹사파린

5)

조직플라스미노겐활성제

2)



4) 일차 심장동맥중재술

02

因•□•回

2주 전 시작된 흉통을 주소로 내원한 49세 남자의 흉통 발생 중에 찍은 심전도(가)와 니트로글리세린 설하 투여 후 흉통이 사라진 상태의 심전 도(나)이다. 환자는 어제부터 안정 시에도 흉통이 나타났다고 하였고 심 근 효소는 정상이었다. 진단은?

02 3주 전 발생, 안정 시 흉통 등은 모두 UA에 합당합니다. 심전도를 보면 (가)에서 ST depression이 보이는데, nitroglycerin을 투여한 뒤(나) 정상으로 바뀌었습니다. 또한 심근 효 소가 상승되지 않았으므로 NSTEMI는 가능 성이 낮습니다. 물론 심근 효소가 한 번 정상이었다고 해서 NSTEMᅵ를 배제해서는 안 됩니다! 3〜6시간 후에 한 번 더 재검해야 합니다! Harrison 2 0민;,pp. 1866~ 1872

1) aortic dissection

2) acute pericarditis

3) unstable angina

4) ST elevation MI

5) non-ST elevation MI

급성 관상동맥증후군

03



--------------------

55세 남자가 가슴이 아파서 왔다. 1년 전부터 등산 중 가슴 통증이 생겼

0 3 가슴 통증이 10분 이상 지속되며,이전보다

는데 2주 전부터는 집안에서 움직일 때도 발생하였다. 통증은 가슴 가운

더 자주 생기고 있으며, 심전도에서는 ST elevation의 소견은 없으므로 NSTE-ACS로

데를 짓누르는 듯한 느낌이었고 10〜15분 동안 지속되었다. 15년 전부 터 당뇨병으로 혈당강등ᅡ제를 복용하였고, 하루 1갑 씩 30년 간 담배를 피웠다. 혈압 152/96mmHg,맥박 87회/ 분, 호흡 19회/ 분,체온 36.5도

생각할 수 있습니다. 빠른 대처가 필요한 상 황에서 홀터검사는 옳지 않으며,NSTE-ACS 이므로 3)〜 5)의 부하 검사도 답이 될 수 없습 니다. 앞에 있는 coronary angiography의 적 응증 부분을 참고하면 환자는 당뇨가 있으므 로,관상동맥조영술이 가장 옳은 검사입니다. Harrison 20만,pp.l866~1872

였다. 심전도와 가슴 X선 사진은 정상이었다. 검人!는? 1) 홀터검사

2) 관상동맥조영술

3) 운동부하심전도검사

4) 아데노신부하심근스캔

5) 도부타민부하심 장초음파검사

r

0 4

S-B-B

53세 남자가 1시간 전 텔레비전을 보던 중 앞가슴 통증이 발생하였다. 통증의 양상은 짓누르는 양상이었으며 20분간 지속되다 사라졌다. 내원 하여 측정한 혈압은 140/90,맥박은 89이었다. 흉부 X-ray, 심전도,심 장초음피는 정상이었다. 다음으로 해야 할 것은? 1) 귀가

2) 식도내시경

3) 트레드밀 검사

4) 심전도 재검

5) 응급 coronary angiography

05

:j-Q

57세 여자가 1개월 전부터 심해진 흉통을 주소로 내원하였다. 환자는 3 년 전부터 같은 증상으로 약물 투여(propran이이고[■ nitroglycerin) 중이 었으나 1개월 전부터는 익물 투여에도 흉통이 나아지지 않았다. 이 환자 의 대한 치료로 가장 적절한 것은? 1) 특별한 문제는 없으므로 안심시킨다.

0 4 논란이 있는 문제입니다. 짓누르는 느낌의 앞 가슴통증은 전형적인 anginal pain입니다. 휴 식 상태에서 통증이 생겼고 20분 정도 후 사 라져 NSTE-ACS를 의심할 수 있습니다. 현 재 증상이 없으므로 트레드밀 검人* 할 수 있겠으나,심전도 재검과 같이 보기에 나왔다 면 심전도 재검을 우선 시행하고 트레드밀 검 사를 시행하는 것이 우선 순위상 맞습니다. GERD 의심 하에 식도내시경을 시행해야 한 다고 주장하는 분들도 있었지만 통증의 양상 이 GERD오[fe 맞지 않습니다. GERD가 의 심되더라도 일단 UA에 대한 검사가 모두 끝 난 후에 식도내시경을 해 볼 수 있습니다. Harrison 2ᄋ만,pp. 1866〜 1872

0 5 치료 중이던 stable angina 환자가 1개월 전 부터 진행하는 anginal pain을 주소로 내원했 습니다. UA가 의심되지만 M3와 감별을 위해 정밀검人! * 해야 합니다. 입원을 권유하는 것 이 좋겠습니다. aspirin을 추가하는 건 맞는 말이지만,지금 이 상황에서 가장 적절한 조치는 아닙니다. Harrison 2Q만,pp. 1866~ 1872

2) aspirin을 추가한다. 3) 입원을 권유한다. 4) lorazepam을 추가한다. 5) nitroglycerin을 nifedipine으로 바꾼다.

ᅳo

o

g- 2020년 대비 PACIFIC KMLE

순환기

06

문 제 해 설

@ - 0 '©

47세 남자가 1개월 전부터 가슴이 아파서 병원에 왔다. 가슴통증은 10

V느

0 6 New onset, Resting pain, Crescendo pattern 으로 불안정협심증(Unstable angina)을 시사

분 이내였고, 운동 도중에 발생하였으며, 5일 전부터는 평지를 조금만

하는 소견입니다. 불안정협심증과 심근경색증

걸어도 발생하였다. 고혈압과 당뇨병을 잃고 있었다. 가슴통증을 호소하

(Myocardial infarction)으로 구분하였던 과 거와는 다르게, 현재는 Acute coronary syndrome이 는 이름 아래에 불안정협심증과 NSTEMI 라는 두 질환을 포함하여 생각하고

였을 당시 심전도와 니트로글리세린을 혀 밑에 넣은 후의 심전도이다. 진단은?

있습니다. 두 질환의 명확한 구분을 위해서는 cardiac enzyme 상승 여부를 확인해야 하나,

이런 실험실 결과 수치가 주어지지 않은 경우 에는 전형적인 임상 증상으로 감별 진단을 내 려야 합니다. Harrison 2ᄋ만,pp. 1866〜 1872

1)

안정협심증

2) 불안정협심증

3)

급성심근경색증

4) 폐색전증

5)

급성십장막염

07



관상동맥 병변 치료를 위하여 약물 스텐트 시술을 하였다. 시술 후 반드 시 入ᅡ용해야 할 약제는? 1) atenolol

2)

nitrate

3) diltiazem

4)

clopidogrel

5) quinidine

07 Stent 시술은 재협착이 발생할 수 있으며 이를 예방하기 위해 dual-antiplatelet therapy가 필요합니다. aspirin+clopidogrel를 人[용합

니다. Harrison 2Q만,pp.l866~1872

급성 관상동맥증후군

08

® -B I

48세 남자가 1일 전부터 조금만 움직여도 흉통이 발생하여 병원에 왔 다. 4일 전에는 등신하다가 1시간 정도 흉통이 발생하였고 자연 소실되 었다. 심전도는 다음과 같을 때 해야 할 조치는?

coronary angiography

2) 24시간 생활 심전도

3)

도부타민부하 심장초음파

4) treadmill test

5)

myocardial perfusion scan

1)

09 I

불안정협심증을 진단^는 데 가장 중요한 것은? 1)

임상증상

3) 관상동맥 조영술 5) EKG

2) 심근 효소 검사 4) echocardiography



---------------0 8 ① 휴식시 흉통 ② 심하고 4 〜 6주 안에 새로

발생한 흉통 ③ 점차 심해지는 흉통에 해당하 는 불안정협심증(UA)입니다. UA가 의심되 는 환자가 내원한 경우 부하검사보다는 치료 를 먼저 해야 합니다. 이 환자의 경우 early invasive strategy의 indication은 아닙니다. 그러므로 내고ᅡ적치료를 한 뒤, 증상이 지속될 경우 coronary angiography를 시행해야합니 다. 하지만 보기에 내과적 치료가 없으므로^ 나머지 검사 보다는 coronary angiography를 하는 것이 좋아 보입니다. Harrison 20만,pp. 1866~ 1872

B

0 9 Unstable angina의 정의가 무엇인지 확실히

알지 못하면 얼핏 당황할 수 있는,중요한 문 제입니다. Unstable angina는 심전도나 심근 효소,관상동맥 조영술에 의해 진단되는 것이 아니라,anginal pain 이 다음의 3 가지 양상 중 적어도 하나 이상에 해당하는 경우입니다. ① 쉴 때 나타나고 10분 이상 지속 ② 4〜 6주내 새로 발생한 심한 흉통 ③ 점차적으로 심해지는 흉통{강도 빈도 시간) 이런 UA의 임상양상에 cardiac enzymeO | 증가되어 있으면 NSTEMI라고 합니다.(현재 는 두 개를 크게 구분하지는 않습니다.) 즉,불안정협심증을 진단하'는데 있어서 가장 중요한 것은 임상증상입니다. Harrison 20만,pp. 1866〜 1872

O

%

€ ■

2020년 대비 PACIFIC KMLE

순환기 그 3 1

10

Q

60세 남자가 1주 전부터 휴식 시에도 하루에 몇 번씩 가슴통증이 발생 하여 내원하였다. 3년 전 안정협심증으로 진단받고 익물치료 중이었다. 다음 중 가장 적합한 처치는? 1)

심장동맥조영술

2) SPECT

3)

부하심장초음파

4) 항응고제 추가

5)

운동부하 심전도

문제해설 10 휴식 시에 발생하는 angina는 unstable angina입니다. 가장 중요한 것은 흉통에 대한 병력 청취 및 EKG, cardiac marker가 우선이지만 보기에는 없습니다. Stress testing을 하고 싶 지만 resting pain이 있기 때문에 안 됩니다. 이미 stable angina로 치료 중이었으므로 aspirin은 복용중이라고 생각하면 헤파린 등 다 른 항응고제의 추가도 답이 될 수는 있으나, stable angina에 대해 치료를 받고 있었음에 도 불구하고 unstable angina로 진행히는 양 상을 보이고 있으므로 현 상황에서 가장 적합 한 처치는 coronary angiography 및 필요 시 PCI입니다. Harrison 20민■ ,pp.l866~1872

11

I? I Q [5]

5년 전 협심증으로 진단받고 약물 복용하며 잘 조절되던 63세 남자가 3주 전부터 안정 시에도 흉부 통증이 발생하여 입원하였다. 입원 후 nitroglycerin, 베타차단제,칼슘길항제,heparin으로 치료하였으나 전혀 증상이 호전되지 않았다. 혈청 심근 효소치는 정상이었다. 다음으로 해

Harrison 2이만,pp. 1866~ 1872

야 할 검사는? 1)

관상동맥조영술

3) 도플러 심초음파

2)

심장 자기공명영상

4)

부하심근관류영상

5) 흉부 X 선 검사

^AN SW ER

11 불안정협심증 환자의 증례입니다. 내과적 치 료에도 불구하고 증상이 호전되지 않는다면, coronary intervention이 필요합니다. PCI 또 는 CABG 등의 invasive therapy를 시행하 기에 앞서 coronary angiography를 실시해 야 하죠.

01.③ 02.③03■ ②04.④ 05.③ 06.② 07.® 08.(5) 09.① 10.① 11.①

급성 관^ ¥ ᄆ내증후^"ᅳ

E

Explanation

0 1 S _

S T E M I(ST-segment elevation myocardial infarction) ᅳ 진 단

^^1



급성 심근^색증은 NSTEMI와 STEMI로 나뉘W, 심근의 괴사가 진 *职 고 91음을 의Didᅡ는용어잉을 않아두시면 종'74!습니다. 하지만, StW 0췌 ^처 령 NSTEMI는 UA와 진1균, 치료를

하« STEMI는 다른 치료를 현니다.

이 파트에서는 심근^색증이라는 용어도 나오지0』대부분은 STEMI에 디한 설명이라고 생각하시면 도n^[습니다_ 초기 진되:, 처치를 않고’ 정색 부위에 따른 ^ 병증, 관리에 OfdH 숙지하도록 입시다.

1 I-

A H

시 그 겨 즈 이 저 이 口 I_ O —I O 一 I o — I

1)

Ischemia에 의 한 myocardial ‘necrosis,

2)

Troponin 등 검사의 발 달 로 세 가 재정의되어 감

ᅳ STEMI : myocardium의 큰 영역을 침 범 한 profound(transmural) myocardial ischemia

^

_______________________________________________________________________________________________________ CTID

❖Definition of acute MI Typical rise and fall o f biochemical markers(cardiac enzymes) of myocardial necrosis with at least one of the following — Ischemic symptoms — EKG changes indicative of new ischemia(new ST-T changes or new LBBB) 一

Development of Q waves on the EKG

— Imaging evidence of new loss of viable myocardium or new regional wall motion abnormality

o n人 h C-m 이口ᄉOK O O Chest pain 1) 30분 이상 지속되며,anginal pain보다 심한 pain 2) 휴식이나 nitroglycerin에도 완화되지 않음 3) 왼쪽 팔,목,턱,등,occipital area로 방사통(배꼽 아래로나 아래턱 위로 보통 방사되지 않음) 4)

3.

Weakness, sweating, N/V, dizziness, anxiety 동반

병태생리 1) Erosion, fissure, rupture of vulnerable atherosclerotic plaques 2)

Rupture 호발 : large lipid-laden plaque, thin fibrous cap, macrophage 등에 의한

3)

Complete occlusion : ST elevation이 일0^ '

염증

—»• UA & NSTEMI : partial occlusion, collateral circulation이 있는 경우,distal coronary embolization

o

o

r

2020년 대비 PACIFIC KMLE

순환기

4. 진단 1) EKG 2) Serum cardiac biomarker : cardiac-specific troponin T(cTnT), cardiac specific troponin I(T nl), CKM B 3) Cardiac imaging : 2-D echocardiography, radionuclide imaging 4) Nonspecific indexes of tissue necrosis & inflammation 離 STEMI는 워낙 중요다!:

이기 매문에 에■른 진1례 큼 정확한 진면도 요구^ 니다. Washington manual에 따르

면 STEMI는 30분 이상의 훙통 또t

용부 불3때 ,2기 ᅵ이상의 면속된 EKG lead에서 1mm 이상의 ST 분^

분상승, cardiac marker의 상승 중 두 기* oi^oi^g. 진1y;가능하다고 현니다. Washington manual 33만,p. 138

5. 심전도 1) 전형적인 STEMI의 EKG 소견 (1) Hyperacute T wav

ST elevation —»• Q 1

T

inversion ᅳ ST 정상화(Q wave는 수년 또는 평생 지속) (2) Reciprocal charge

2) ST segment (1) Elevation : epicardial artery가 완전히 폐쇄된 경우 ᅳ STEMI ♦

ST elevation

위로 볼록한 비석 모0(편 ST분^ 상승이어야 STEMI0』니다. 볼록한 STS정 상승0 1w 부분의 lead에서 보일 °}t는 pericarditis의 상 1a 도 소^ 이있습니다!

(2) No elevation : 완전히 폐쇄가 안 되었거나, 일시적으로 막힌

Q wave & T inversion ST 정상화& Q wave 지속 " V ᄂ ^

경우 또는 측부 혈관이 발달한 경우 — UA 또는 NSTEMl(cardiac markei•상승하면 NSTEMI!) 3) Q wave (1) 존재 : STEMI의 대부분

Q-wave MI

(2) 없을 수도 있음 : STEMI의 일부 또는 NSTEMI의 상당수 — non Q-wave MI 여 ^ 뿌

유의 Di 한 Q피는 AMI의 심전도 소ᄀi 연화 중 가장 마지1각oil 나타나며 ~HI속 지속^ 니다. 따라서 학생 수준에서는 유의미한 Q n n ᅡold infarct의 소경이라는 정도1만 1알아두시면 협니다. ᄋ1ᅵ전에는 unstable angina와 acute N/II를 cardiac marker로 구분하였는데, EKGᄋIIN/I1ᄋ!ᅵ 평하는 연화 0 .0 4 초

라도 H 그표고 '갈' Ml 지속a 는 경우를 abnormal Q wave라고 하지요. 보통 피서에서 Q wave가 보

--------

이거나 1 , 1 , aVL, aVF, V4~6에서 30ms 이상 지 속되는 Q wave가 존자I d a , 2 '개 이성의 면속된 lead에M 1mm 이생의 ᅳ난이로 존재할 매 QwMᅵ를 의Dldh는 abnormal Q로서의 의이가 91습니다.

R

'酬

6.



……⑴'…削 * € 1 1

ᅳ 글 공 ᅳ 국 군 —

- 1

§

혈청 심장 표지재 cardiac marker) - 심근경색의 정량 검사, myocardial necrosis 발견의 ‘Gold standard’ - 하지만 검출에 증상 발현 후 1〜 12시간 정도의 지연이 있으므로 긴급한 재개통 시술이 필요할 때에

는 환자의 병력과 EKG 만으로 판단하는 경우가 많다. 수

x upper lima it



normal

0

20

40

60

80

100

120

140

160

1) Troponin : cardiac-specific troponin T(cTnT), cardiac-specific troponin I(cTnl) (1) 정상인에서는 검출되지 않지만,STEMI에서는 20배 이상 증가 (2) M I의 진단에 선호되는 biomarker : most specific (3) STEMI 후 7 〜 1ᄋ일 동안 지속 2) Creatinine phosphokinase(CPK) (1) 4 〜 8시간 이내 상승,48〜 72시간 이내 정상화 (2) M I 진단의 특이도가 떨 어 짐 골 격 근 손상 시 증가) (3) 다른 질환에서도 상승 :근질환, 전기심장율동전환,갑상선기능저하증,뇌졸중, 수술. 골격근 손

상t 외상,경련,부동자세로 오래 지속한 경우) 3) Myoglobin (1) M I onset 후 수 시간 이내에 검출(가장 먼저 상승하는 cardiac biomarker) (2) 24시간 이내에 정상화94%) (3) Nitroglycerin

M :Morphine

0 :Oxygen N : Nitrate A 1 :Aspirin, Heparin B : BB R :Reperfusion

① 대부분의 환자에게 sublingual로 투여 ② Preload 낮춰 myocardial demand 낮추고 관상동맥 넓혀 myocardial supply 높임 ③ 금기

• SBP 특히 Bradycardia, Heart block, 일부 Inferior infarction의 부작용시 IV atropine 0-로 치료

③ IV route로 투여(피하로 투여할 경우 흡수량을 예측할 수 없음) (5) ff -blocker(심근의 산소요구량 i )

① 심근 산소요구량 감소시켜 통증완화 및 재발과 심실세동 위험 낮춤 ② 금기 : HF, severe LV dysfunction, hypotension, 심한 bradycardia^ AV blocks asthma, severe COPD (6) 저혈압이 나타난다면, nitroglycerin, morphine, 0-blocker 등의 약물을 중지하고 fluid 등을 투여

하며 hemodynamic stability를 유지해야 함 (7) Atropine : morphine등에 의한 excessive vagotonia로 syrrptomatic bradycardia, 저혈압 발생 시 傘 CCB는 STEMI에서 쇼과가 거의 없어요. 傘 Glucocorticoid, NSAIDs(aspirir)은 제외)도 금기에요. 경색 부위의 치유를 지면시키고,심근파열의 위

형을 높이거든요. 傘 Nitroprusside는 원상동0"^1의 arteriolar dilatation 유1말하으로 쓰 면 。 J: 도사요.

좁아진 그 ?1:성동1객의 coronary steal 유1길: 傘 NG는 venous dilatation이 주^ 작용이으로 coronary steal 현상

음,방면 nitroprusside는 arterial

& venous dilatation 둘 다 작용하으로 cc)ronary steal을 유1말 傘 Coronary steal 이릿; vasodilator 투여 시 정상

상동(과 atheroma가 진컨興 비W 상관상동0후 1 동시

에 늘어UKTII 되는데 이에 정상 원상동맥이 상대적으로 더 맥이 아 닌 정 상 관 상 동 맥 쪽 으 로 혈 류 가 오 히 려 쏠 a

傘 비슷한 현상으로는 Subclavian steal syndmme이 subclavian a.가

늘어나게 윌니다. 따라서 비정상 관성동

심 근 허 형 p i 더 악 화 되 는 현상입니다ᅵ

습니다. 예를 들어 Takayasus arteritis에서 It.

혈류가 강소하는 경우 그 곳에서 기시하는 it. vertebral a.의 혈류가 줄어들고

이oil 따라 rt. vertebral a.에서 basilar a.로 올라가는 혈류가 거꾸로 it. vertebral a.를 통해서 역류하게 윌니다. 이로 인해서 syncope와

3.

은 증상이 나타닝니다.

재관류 치료 乘 증상이 일생회:지 12시'7J: 이내인 STEMI 환자는 방드시 재그? 용 치료를 실시해야 하며,12~24시'7J:이 지1d 환자 는 임상적으로 혹은 심전도상에서 진행되고 앗는 허혈의 소경이 앗는 경우 t: H원류 치료를 해야 현니다. 아래 성명에 등7당하는 FMC-to-device time이라는 용어는 first medical contact, 즉, ■응■급 구조사 등의 의 료 。 J력이 처음으로 환자와 껍촉한 시점부터 device, 제1it류 치료를 하는 장치나 기구에 도착하는 시정77띠의 시'7그을 의이 현니다.

1) Primary percutaneous coronary intervention( P C I) (1)

Fibrinolysis를 하지 않고 먼저 PCI를 하는 것을 의미

-€Z:

급 성 ^^^맥 증 후 군



(2) PC I가 기능한 의료시설과 인력이 있단면 fibrinolysis보다 primary P C I가 추천

(3) FMC-to-device time 90분 이내로 PCI가 7]능한 병원으로 환자를 이송하는 것이 이상적 (4) PCI가 불 7]능한 병원으로 이송된 경우,FMC-to-device time 120분 이내로 PCI가 7)능한 병원으

로 이송이 가능하다면 fibrinolysis를 하지 않고,이송하여 primary PCI를 실시

(5) 재관류 치료 대상이고 fibrinolysis의 금기에 해당하는 경우 primary PCI를 실시 (6) Post-PCI management : Intracoronary stent restenosis의 예방과 치료

① PC I 시행 후에는 long-term dual antiplatelet therapy(aspirin and clopidogrel)이 필수적 ② Bare metal stents보다 drug-eluting stents의 경우 stent restenosis 발생 가능성이 더 적지만, late stent thrombosis에 위험으로 dual antiplatelet therapy 해야함

③ 재발 시 치료 • intervention의 적응증은 이전에 P C I 받지 않은 환자와 동일 2)

Fibrinolysis

심근세포가 비가역적으로 손상되기 전에 사용 — Infarct 크기 줄이고 LV dysfunction J,

심각한 합병증(septal rupture, cardiogenic shock, 악성 심실부정맥) 발생률 i

(1) 아래의 금기에 해당하지 않고 초기 PCI가 불가능한 병원으로 이송되었고 FMC-to-device time 120분 이내로 PCI가 7]능한 병원으로 이송할 수 없는 경우에 실시 (2) 적응증에 해당하는 경우,병원에 도착하고 30분 이내로 시술을 하는 것이 이상적 (3) 혈전용해제 종류 : tissue plasminogen activator(tPA), streptokinase, tenecteplase(TNK), reteplase(rPA) (4) 절대적 금기증 傘 Fibr이ysis가 치료로 등7양하는 진혹!:에서(에,폐색7S) ^

주의해야 형니다. 외우J두서1요八八

① 뇌출혈의 과거력 ② 지난 1년 이내:B]출혈성 뇌좋중 또는 다른 뇌혈관 질환의 과거력 (3)

증상이 나타나는 동안 언제라도 SBP > 1 8ᄋm m t병, D BP >110mmKfe

© 대동맥박리가 의심되는 경우

내출혈이 있는 경우(월경은 예외) *

고령은 금기에

되지 (않아요. 비록 효과가 조금 I!어 져 도 fibrinolysis로 사망률 각소가 9 ^ 니다. 오

d ia 고령은 PCI에서 효고n f 달 떼 |지

어요(benefit was not established for patients older than

75 years of age). 또 한 가지 주의할 ^ 은, 훙통 등의 증상이 산 르 나 ST elevationoi 없는 ^우 (UA, NSTE-MI)에 는 — fibrinolysis를 쓰 현 해 서 ^0소 된다는 정0은니다.

(5) 합병증 (I)

출혈 : 가장 흔하고 가장 심각한 합병증

②1 Allergic reaction(streptokinase) : 심한 저혈압 증상

O

o(A2l)



2020년 대비 PACIFIC KMLE

순환기

(6) Fibrinolysis 이후에 PCI를 해야 하는 경우

① Cardiogenic shock이나 심한 acute heart failure가 있는 경우 ② 재관류 치료가 실패하거나 다시 막히는 경우 ③ Fibrinolysis 이후 혈역학적으로 안정적인 경우에도 3 〜 24시간 이내 coronary angiography를 위해 PCI가 7]능한 병원으로 이송 傘 기본적으로 처음에 환자가 PCI가 불가능한 병우크으로 이송되9 i고,PCI가 가능한 병우cl으로 이송할 때, FMC-to-device time이 120분을 1■ 참으1겨,금기에 해당하지 앞는 ' 경우이PJ: 他如이乂如를 성•시하고 나머지는 primary PCI를 실시한다고 생 74 하시면 병니다!! 하지4t, 국시 문제에서는 PCI와 fibrinolysis가 모두 보 기에

는 경우는 대부분 fibrinolysis의 금기에 혜11용하여 primary PCᅵ를11점으로 고르는 문제들 ■뿐■이으로

fibrinolysis의 금기에 대컨H서는 않아두어야"국 }!고,그 o i외chi 둘 중 어1헌

을 헤야 하는가에 대헤서는 적1광

di 넘어가셔도 철 ᄀX '살습니다!! 또한,어1전 쟁법으로 辦 S류를 할 ^ 9 1 가도 중요하지만,최대한 바른 시기 oil 해야 헌;다는 점이 더 중요현니다!!

3) Evidence of Reperfusion (1) Pain relief (2) ST normalization : STE 소견이 50% 이상 감소 (3) Reperfusion arrhythmia : Accelerated idioventricular rhythm(AIVR)

(4) Early peak of cardiac markers(10〜 15hrs) (... infarcted zone에 있던 biomarker들의 washout) 傘 에를 들어 AMI의 natural course 에서 CK-MB가 24시I t 이

지나서 peak를 이루는 방면에 reperfusion

치료에 성공하여 혈류가 WT7H통^ 경우에는 12시'7J: 맛에 peak를 보이고 그 이후 강 소하는 소 ^을 보。 』 니다ᅵ



(4 2 2 )0

V £ r i h l l l l . r ;Xl » -T I* ^





^

STEMI환자의 재관류 치료 DIDO, d o o r- in - .d o o r o u t, F M C , first m e d ic a l c o n ta c t; LO E, L e v e l o f E v id e n c e ; Ml, m y o c a rd ia l in fa r c tio n ; PCI, p e rc u ta n e o u s c o r o n a r y in te rv e n tio n ; a n d STEM I, S T -e le v a tio n m y o c a rd ia l in fa rc tio n .

2013 ACCF/AHA Guidelines for the Management of ST-elevation Myocardial Infarction

4.

병원 처치 Coronary care unit에서 management ᅳ 적극적 감시로 사망률을 50% 감소 1)

.J

CCU care의 목표 (1) Continuous EKG monitoring과 항부정맥 치료 (2) 재관류 치료를 시작하거나 유지 (3) 다른 내과적 치료를 시작하거나 유지 ᅳ 약물요법 참조 (4) Hemodynamic monitoring (5) A M 의 합병증을 관리 및 치료

-o



423

14 *

그 그 J上!그 그 =a■ 그

2020년 대비 PACIFIC KMLE

2)

순환기

General care (1) Activity (2) Diet (3) Bowel management (4) Sedation

5.

입원 후 약물요법 離 있원 수의 치료는 A'BA+tf 있니다. 寧 기Qj하시죠?

Post-MI 사D< if 닛추는 약물은 아바(아W H ) ABA(aspirin, ^-blocker, ACEi) 토! 욱1시에는 ABAS(antiplatelet, BB, ACEi, statin) 1) Antiplatelet and anticoagulant therapy (1) Antiplatelet : aspirin, clopidogrel, prasi期 *el, ticagrelor, glycoprotein 1 b/IEa receptor inhibitor (2) Anticoagulant agent : UFH, LMWH(enoxaparin), Fondaparinux, Bivalirudin (3) Aspirin은 STEM 의 사망률을 27% 감소 2) Beta blocker (1) 사망률 15% 감소(fibrinolysis를 하지 않은 경우) (2) 금기가 아닌 모든 환자에게 투여 (3) 금기증 : 심부전, 심한 LV dysfimction, 심장전도차단, 기립성 저혈압, 천식 등 3) ACE inhibitor

(1) 사망률 감 쇠 aspirin과 beta-blocker를 같이 투여하면 사망률을 더 감소시킴) (2) Angiotensin receptor blockers(ARBs) : ACE inhibitor의 부작용이 심하고^ 심부전의 소견이 보이

는 ^자에서 유용 4) IV nitrate

사망률 감소 효과 약함. 응급실에서 통증 감소에 효과적이라는 것 기억 5) Ca channel blocker는 권장되지 않음 STEM 에서 사용은 권장되지 않음. 오히려 위험할 수도 있음 6) 혈당조절

당뇨 환자에서 사망률을 낮춤 7) Mg : 모든 입원 환자에서 검사해서 부족하면 교정(ᅳ 부정맥의 위험을 낮춤)

傘 Post-MI 시밍률 낮추는 *약울 : aspirin, /?-blocker, ACEi — 심부전에서 aspirin은 주의 / 금기 약제였죠. 나머지 2개는 동일하네요. 비교해서 봐두세요.

"관 상 동 맥 증 후 급 ᅳ ᅵ ᅳ * 3 1

6. 우심실 경색(RV infarction) 傘 우심실 정색은 치료가 특이하고 자■주 나오는 내용0』니다. 꼭 알아두도록 형시다.

1) Inferoposterior infarction 의 1/3에서 RV infarct 동반 2) 심한 RV failure의 증후(경정맥 확장, Kussmaul’s sign, hepatomegaly>를 유발 離 Kussmaul sign은 흡기 시 오천IS JVP가 상승하는

으로 constrictive pericarditis나 restrictive

cardiomyopathy에서 나타1납니다.

3) EKG : 오른쪽 precordial lead(주로 V 4 R ) 의 ST segment elevation 攀 I ,I ,aVF는 inf. p이lio n 이지,RV가 아님니다. 다만 둘 다 RCA에 의해 혈류공급을 받으으로 형7게 진 동 t{및 뿜이죠ᅵ 그래서 표,艮 aVF ST elevation이 확인되어 RV가 의심되는 정우 일단 오른쪽 precordial lead EKG(특히 V4R)로 확인해 봐야 현니다.

4) 치료 (1) Volume expansion(IV N/S-적절한 R V preload 유지) (2) LV performance 개선(eg. dopamine, dobutamine) (3) Temporary A V sequential pacir벌(h ^ h grade A V block0] 흔함)

ᅳ»• diuretics, NG, morphine는 preload 감소시키므로 금기 Harrison 2Q민;,pp. 1872〜 1881

찾 ^ ^ _______________________________________________________________________________________________________ criD

* STEMI 치료 이^ 4 1 은 기억현시다.

1) 용급실 처치 aspirin, heparin, nitroglycerin, morphine, ff-blocker, oxygen(MONA'B) 저혈0 이

는 ᅳ경우 nitroglycerin, morphine, |S-blocker는 금기)

2) STEMI는 reperfusion을 고a ! ! - p c i or Fibrinolysis. Fibrinolysis의 혈대적 금기증 (1) 되출혈의 과» 력 (2) 지난 i ua 이내 비출혈성 되졸중 또는 다른 뇌혈관 진환의 고n 사력 (3) y , 척수 내에 종0 이나 AVMOI 있는 정우 (4) 증상이 나타나는 동0그 면제라도 SBP >180mmHg, DBP >110mmHg (5) 대 동 맥 떠 가 의심되는 정우 (6) 활동적 내출열(월경은 에외) 3) 생존울을 높이는 0措

치료 : aspirin, ACE inhibitor, (S-blocker(ABA)

4) RV infarction :I , I , aVFCHl ST elevation, volume expansion이 치료의 혜심

5 2020년 대비 PACIFIC KMLE

순환기

01

B -回•回

66세 남자가 1시간 전부터 앞가슴이 아프다며 응급실에 왔다. 숨도 차다 고 한다. 7년 전부터 혈압강하제를 복용하고 있다. 혈압 94/62mmHg, 맥박 106회/ 분, 호흡 24회/ 분, 체온 36.0°C이다. 가슴 청진에서 심장끝 에서 제3심음이 들리고 심잡음은 없다. 양쪽 가슴 아래쪽에서 거품소리 가 들린다. 심전도이다. 치료는?

01 1시간 전 갑자기 발생한 흉통과 호흡곤란을 주소로 내원한 환자입니다. a 압이 낮고 청진 상 apex에서 S.,가, 양쪽 하폐야에서 거품소리 가 들리는 급성 심부전의 증상과 징후를 보이 고 있습니다. 심전도에서는 V ^ V ,,에서 모두 ST elevation을 보이며 그 중에서도 anterior wall에 해당하는 V2〜 V4 lead에서 가장 뚜렷 하게 나타납니다. 심전도의 ST elevation, 급 성 심부전의 임상상, 그리고 우리나라에서 가 장 흔한 급성 심부전의 원인이 허혈성 심질환 임을 생각한다면 anterior wall MI일 것으로 생각되고 primary PCI를 해야 합니다. Anterior wall MI는 reciprocal area가 pos­ terior wall이므로 V6까지의 심전도 상에서는 관찰되지 않을 수 있습니다. Harrison 2 0 ^ , pp.!872~1881

1) 왼심실보조장치

2) 체외막산소공급

3) 리도카인 정맥주사

4) 아데노신 정맥주사

5) 일차 심장동맥중재술

02

D-0-0

72세 여자가 1시간 전부터 어지러워서 응급실에 왔다. 2시간 전부터 명 치에 심한 통증이 있다고 한다. 10년 전부터 혈압 강하제와 혈당강하제 를 복용하고 있다. 혈압 82/46mmHg,맥박 76회/ 분, 호흡 22회/ 분, 체 온 36.5°C이다. 가슴 청진에서 심음과 호흡음은 정상이다. 오른쪽 유도 심전도이다. 치료는?

0 2 갑자기 시작된 흉통과 어지럼증을 주소로 내 원한 고령의 환자입니다. 심전도에서 II,III, aVF lead에서 ST elevation이 관찰되고, I, aVL lead에서는 reciprocal change로 ST depression을 보이고 있어 inferior wall MI 로 생각됩니다. Inferior wall MI에서는 RV infarction, AV block, sinus bradycardia가 생길 수 있습니다. HR는 76회로 50회보다 높습니다. 반면 혈압은 90/60 아래로 떨어져 있고 오른쪽 유도 심전도에서 V4R lead에서 ST elevation을 보이고 있어 RV infarction 이 동반되어 있을 것으로 생각됩니다. 치료로 는 preload를 늘리기 위한 N/S을 주어야 하 ^ 습니다. Harrison 2Q만,pp. 1872~ 1881

i) 디곡신

2) 도부타민

3) 테오필린

4)

5) 0.9% 식염수

푸 6) O

푸로세미드

-

03

o

i.fQi

55세 남자가 2시간 전부터 가슴이 아파서 왔다. 식은 땀을 흘리며 어지 럽다고 했다. 혈압 80/50mmHg, 맥박 82회/ 분, 호흡 22회/ 분, 체온 36.0도였다. 목정맥이 늘어나 있었다. 가슴 청진에서 거품소리는 들리지 않았다. 심전도이다. 치료는?

급성 관상동맥증후군

-------------- cA, 0 3 환자의 증상 및 심전도 소견으로 STEMI 임 을 진단할 수 있습니다. Inferior wall에 해당 하는 표, 1 , aVF lead의 ST elevation과 함 께 anterior wall에 해당하는 V2 〜 V4 lead의 ST depression이 보입니다. RCA가 막히면 서 inferior wall, posterior wall에서 경색이 발생하여 inferior wall에서는 ST elevation 이,anterior wall에서는 posterior wall에 대 한 reciprocal change로 ST depressionᄋ| 나 타난 것으로 해석할 수 있겠습니다. RCA가 막히는 경우 RV infarction가 동반되 지 않았는지를 확인해야 합니다. 목정맥이 늘 어나고, 혈압이 90/60mmHg보다 낮은 상황 이므로 RV infarction이 으ᅵ심되고, LV preload를 늘리기 위해 N/S을 주입해야 합니다. Harrison 2이만,pp.l872~1881

칼슘통로차단제

2) 혀밑 니트로글리세린

베타차단제

4) 도부타민

0.9% 식염수

r

04



57세 남자가 2시간 전부터 가슴이 아파서 왔다. 혈압 92/62mmHg,맥 박 118회/ 분,호흡 22회/ 분,체온 36.4도였다. 양쪽 등쪽 아랫부분에서 거품소리가 들렸으며,목정맥이 늘어나 있었다. 심전도는 아래오ᅡ 같다.

늘어나 있군요. 치료는 심장동맥중재술입니다.

치료는?

Harrison 2Q만,pp. 1872〜 1881

1) 디곡신

2) 0.9% 식염수

3) 심장동맥중재술

4) 임시박동조율기

5) 직류심장율동전환

0 4 환자의 증상과 심전도 소견으로 STEMI임을 알 수 있습니다. 전중격부와 전벽부의 심근경 색을 보이고 있고 LAD의 영역입니다. LV dysfunction으로 폐부종이 생기고, 목정맥도

^^•XT~crx7x:X7XXxvx^.-r::x::ir^^^ :

2020년 대비 PACIFIC KMLE

순환기

05

0 *0 -1

~ 있= 스

70세 남자가 한 시간 전부터 발생한 흉통으로 응급실에 왔다. 흉통은 가

슴을 전반적으로 조이는 듯한 양상이었으며,휴식으로 호전되지 않았다. 4 개월 전에 뇌경색으로 치료를 받았다고 하였다. 혈압은 122/86mmHg

--------------------- cA, 0 5 환자의 증상과 심전도 소견으로 STEMI임을 알 수 있습니다. 재관류치료를 해야겠습니다. 4개월 전 노ᅵ경색 고F거력은 fibrinolysis의 금 기증입니다. 치료는 심장동맥 중재술입니다. 離fibrinolysis의 절대적 금^ ᅵ증

였으며,호흡음은 정상이었다. 심전도이다. 조치는?

① 뇌출혈의 고ᅡ거력 ② 지난 1 년 이내 비출혈성 뇌졸중 또는 다 른 뇌혈관 질환의 과거력 ③ 증상이 나타나는 동안 언제라도 sBP 〉 180mmHg, dBP> llOmmHg ④ 대동맥박리가 의심되는 경우 ⑤ 활동적 내출혈(월경은 예오ᅵ) Harrison 20면;,pp.l872~1881 2013 ACCF/AHA Guideline for the Management of STEMI, p. 19

1) 혈전용해요법

2) 관동맥우회로술

3)

피부경유관동맥성형술

4) 임시 심장박동조율기 삽입

5)

0.9% 식염수 정맥내주입

06

□ • 回•回

67세 여자가 2시간 동안 가슴이 아파서 병원에 왔다. 10년 전부터 혈압

강하제와 혈당강하제를 복용하고 있었다. 혈압 72/40mmHg,맥박 52회 / 분,호흡 24회/ 분,체온 36.0도였다. 목정맥이 늘어나 있었으나,가슴청

진에서 거품소리나 심잡음은 들리지 않았다. 심전도 사진이다. 치료는?

0 6 환자의 증상과 심전도 소견으로 STEMI임을 알 수 있습니다. Inf. lead인 I ,I , aVF에 ST분절 상승, 그리고 해부학적으로 이들 반대 편에 있는 lat. lead인 I , aVL에서 ST분절 하강(reciprocal change)이 보이므로 inferior wall AMI입니다. 담당히는 동맥인 RCA에 문제가 생긴 것을 알 수 있습니다. 목정맥이 늘어나 있으나 폐부종은 없는 것으로 보아 RV infarct를 시사합니다. RV infarct에 서는 LV preload가 감소해 혈압이 낮아지기 쉽습니다. LV preload를 늘려주기 위해 N/S 치료가 필수적입니다. 여담으로 환자의 서맥이 심할 시 atropine or temporary pacemaker를 사용합니다. 만일 환자가 이보다 더 서맥이었다면(< 50회/분) anticholinergics인 atropine 혹은 termporary pacemaker가 답이 되었을 것입니다. Harrison 2Q판,pp.l872~1881 Cecil 25핍:,pp. 441〜 456

스28

1) 아트로핀

2> 베타차단제

3)

0.9% 식염수

4) 니트로글리세린

5)

칼습통로차단제

O O

급성 관상동맥증후군

rz

07

문제해설

EH1HD

48세 남자가 2시간 전부터 명치부우ᅵ가 조이듯 아파서 병원에 왔다. 2주

전에 전에 혈변이 있어서 식도위내시경검사에서 십이지장궤양으로 진단 받고 약물치료를 받고 있는 중이다. 혈압 116/74mmHg, 맥박 100호 ᅵ / 분이었다. 심전도이다. 치료는?

07



I ,n ,aVF에서 ST분절이 상승된 STEMI 증례입니다. 증상 발현 이후 3시간 이내에 내 원한 환자이므로 혈전용해술, Primary PCI (Percutaneous Coronary Intervention) 모두 생각해 볼 수 있으나, 십이지장궤양으로 인한 혈변의 병력이 있어 출혈의 위험이 높을 것으 로 생각됩니다. 따라서 PCI가 더 옳은 치료법 입니다. Harrison 2Q만,pp. 1872~ 1881 2013 ACCF/AHA Guideline for the Management of STEMI, p. 19

경 ^^찰

2) H.pylori 제균치료

리도케 인

4) 혈전용해제

관상동맥중재술

08

B

64세 남자가 2시간 전부터 발생한 앞가슴의 통증을 주소로 내원하였다.

흉통은 왼쪽 팔로 방사되는 양상이었고 nitroglycerin 투여에도 불구하 고 호전되지 않았다. 이 환자는 열흘 전 위궤양 출혈이 발견되어 치료 중이었으며 혈압은 155/100mmHg, 맥박수는 90 호I/ 분이었다. 가장 적 합한 치^ 는?

0 8 \자~ 부4에 ST elevation이 관찰되고, I , K, aVF에서 reciprocal change가 관찰되는 전 중격부 급성심근경색의 EKG입니다. 2시간 전 발생한 ST-elevation MI0 ᅵ므로thrombo­ lytic therapy나 PCI와 같은 적극적인 치료를 고려해 볼 수 있습니다. 하지만 이 환자는 2 주 전 십이지장궤양 출혈의 병력이 있으므로 thrombolytic therapy의 절대적 금기증에 해 당합니다. 따라서 심장동맥중재시술로 재관류 를 시키는 것이 가장 우선입니다. Harrison 2Q만,pp. 1872~ 1881

1) 유로키나아제 투여

2) 헤파린 정주

3)

도부타민 정주

4) 디지탈리스 정주

5)

심장동맥 중재시술



O

스29



0® 54세 남자가 3시간 전부터 시작된 가슴 통증으로 응급실에 왔다. 환자

0 9 PCI stent 삽입 후 restenosis에 으ᅵ한A M [재

는 2주 전 급성 심근경색으로 관상동맥 조영술 후 LAD에 익물방출 스텐

발이 의심되는 상황입니다. 문제에서처럼 drug eluting stents를 시용한 경우 bare metal

트를 삽입하고 약을 복용해왔다. 그러나 대장 내시경을 위해 1주간 모든

stents보단 restenosis에 의한 위험성이 떨어

익물 복용을 중단했다. 응급실에서 곧바로 시행한 심전도에서 정상 동율 동을 보이나 \h〜\U 유도(lead)에서 ST 상승 소견을 보였다. 시행해야 할 치료는? 1) Digitalis 2) Percutaneous coronary intervention 3) Pericardiocentesis 4) Calcium channel blocker 5) NSAIDs

지지만 문제의 증례처럼 aspirin과 clopidogrel복g 을 중단한 경우 restenosis가 발생할 위험이 높아집니다. PCI stent 삽입 후 AMI 재발 시 intervention 적응증은 AM I 첫 발병 시와 동일합니다. 가슴통증이 3시간 전부터 시작된 경우 혈전용해술(fibrinolysis)과 피부 경유관동맥성형술(PCI)을 모두 고려해 볼 수 있습니다. 참고로 이러한 PCI로 stent 삽입 후 restenosis를 예방하기 위해 시행 후에는 aspirin과 clopidogrel을 장기간 복용하는 것이 필수적 입니다. Harrison 2 0 ^ , pp.l872~1881

10 56세 남자가 2시간 전 발생한 흉통을 주소로 내원하였다. 환자는 급성

병색이었으며, 혈압은 80/55 이었다. 치료는?

1 0 환자의 증상과 심전도 소견으로 STEMI임을

알 수 있습니다. Inf. lead인 표,1 ,aVF에 ST분절 상승,그리고 해부학적으로 이들 반대 편에 있는 lat. lead인 I , aVL에서 ST분절 하강(reciprocal change)이 보이므로 inf. AMI 입니다. 담당하는 동맥인 RCA 에 문제가 생긴 것을 알 수 있습니다. RCA 에 문제가 생길 경우 RV infarct을 의심

할 수 있습니다. RV infarct에서는 LV preload가 감소해 혈압이 낮아지기 쉽습니다. LV preload를 늘이기 우ᅵ해 N/S치료가 필수

적입니다. Harrison 2Q만,pp.l872~1881

1) 베타차단제

2) isoproterenol

3) heparin

4) 0.9% 생 리 식 염 수

5) nitroglycerin

급성 관상동맥증후군

11 55세 남자가 10시간 전에 발생한 전통부 통통으로 내원하였다. 혈압은

11

90/60mmHg이고,목 정맥은 확장되어 있었고 심잡음과 거품소리는 들

리지 않았다. 심전도는 아래와 같다. 가장 먼저 해야 할 처치는? aVR

a\L

iVF

........

JL r



L

환자의 증상과 심전도 소견으로 STEMI임을 알 수 있습니다. inf. lead인 표,m, aVF에 ST분절 상승, 그리고 해부학적으로 이들 반대 편에 았는 lat. lead인 I ,aVL에서 ST분절 하 강(reciprocal change) 이 보이므로 inf. AMI 입니다. 담당히는 동맥인 RCA에 문제가 생 긴 것을 알 수 있습니다. 여기에 목정맥이 확장되어 있고 폐부종이 없 는 것으로 보아 RV infarct를 시사합니다. RV infarct에서는 LV preload가 감소해 혈 압이 낮아지기 쉽습니다. LV preload를 늘이 기 위해 N/S치료가 필수적입니다. Harrison 2Q민;,pp. 1872〜 1881

V5

V2

V

1

!

니™ nitroglycerin

2) digitalis

normal saline

4) diuretics

morphine

12

B

57세 여자가 3 시간 전에 시작된 전흉부 통증을 주소로 내원하였다. 심

전도 소견이 [ᅡ음과 같을 때,환자의 예후를 향상시키는 약으로 맞지 않 는 것은?

12

급성심근경색에서 예후를 향상시키는 약물은 ABA입니다. 기억합시다. ® Aspirin ② 신-blocker @ ACE inhibitor Harrison 2Q 만,pp.l872~1881 2013 AHA Guideline for the Management of STEMI, p. 30

1)

베타차단제

2) 혈전 용해제

3)

칼숨통로 차단제

4) 안지오텐신 전환효소 차단제

5)

아스피린

广

_______ 2020년 대비 PACIFIC KMLE

순환기

13

문제해설

61세 남자가 8 시간 전 人I작된 극심한 통통을 주소로 내원하였다. 응급

13

많은 문제를 통해 RCA에 문제가 생긴 것을 알 수 있겠습니다. 심도자에서 측정한 C.I.가 2.2 미만아고 PCWP 가 18 미만이면 hypovolemic state을 시사합 니다. 따라서 N/S을 줘야 합니다.

실에서 측정한 혈압은 90/60mmhg 이었고 흉부 청진에서는 특이 소견 없었다. 목정맥이 확장되어 있었고 심도자법으로 측정 폐모세혈관쐐기 압이 9mmHg,심장 박출 계수가 1.6l7min/m2이었다. 심전도 소견은

Harrison 2Q만,pp,1872~1881

다음과 같다. 이 환자에게 가장 먼저 해야 할 치료로 맞는 것은? aV R

aV l_

수:

V서

니 3 ) 5 )

urokinase

2) primary PCI

normal saline

4) dobutamine

diuretics

U

因 •回 -B 60세 남자가 1시간 전부터 계속되는 흉통으로 왔다. 응급 검사에서 혈

청 CK-MB가 증가되어 있었고 nitrate 주입 후에도 호전되지 않다가 갑 자기 의식을 잃었다. 당시 심전도 사진이다. 가장 적절한 치료는?

1) morphine

2) pacemaker 삽입

3) 심^ >>동맥

4) dopamine 투여

5) ICD 삽입

14

2시간 전부터 계속되는 흉통,심근 효소치 증 가,질산염제제로도 가라앉지 않는 흉통 등은 급성 심근경색증을 으ᅵ심하게 합니다. 그 후 의식 소실이 된 것은 심근경색의 여러 합병증 에 의한 cardiogenic shock 상태임을 말합니 다. 부정맥,기계적 합병증 등이 원인으로 가능 합니다. EKG를 봅시다. 초반에 나타나는 정신없는 심전도는 심근경색 후 나타난 VF를 보여줍니 다. 그 후 큰 wave가 나타난 후 한동안 wave 가 없더니 normal sinus rhythm이 나타납니 다. VF 후 defibrillation을 통해 정상 리듬으 로 돌아온 EKG인 것이 가장 합당합니다. 이 후의 치료 중 가장 옳은 것은 재관류 치료입 니다. 부정맥이 생긴 것도 허혈에 으ᅵ한mem­ brane^ 불안정성에 의한 것이므로 재관류를 시켜주는 것이 무엇보다 중요합니다. Harrison 2 0 잔■ ,pp. 1872〜 1881

©ANSWER

O

O

0 1. ⑤ 02.⑤ 03.⑤ 04.③ 05.③ 06.③ 07.⑤ 08.⑤ 09.② 10. ④ 1 1 . ③ 12. ③ 13. ③ 14.③

ItliilllliiflUitHlliififHUitltiUiWlliiltiflliilfttlSfiHiHiUiiliiliililiHlliillitlltilUiiilllllllllllllHnHilUiililiMilUfUti



%

_________

증후군

: 그J

Explanation

급성심근경색_ 합병즈 * 급성심근^색증부작용 -

부정1맥

-

Ventricular dysfunction

- 심장성 쇼크 - 기계적 형1병증 : acute MR, VSD, free wall rupture, LV aneurysm -

급성폐부종

ᅳ 기타 : 용통의 재일,pericarditis, thromboembolism

1. 부정맥 - STEM 이早에 자율신경계의 불균형,electrolyte disturbance, ischemia, 허혈심근 부위의 전도지연으 로 인해 ^■생

- Infarct 발생 후 첫 몇 시간 내에 부정맥 발생률 높음 - VPB, VT, VF, accelerated idioventricular rhythm(AIVR), IVR, supraventricular arrhythmia, sinus bradycardia(inf. M I 에서 rn/c)

- 전^장애 1) Inferior wall M I 에서 발생시

(1) 일시적으로 vagal tone 증가 및 adenosine 분비에 의해 발생 ᅳ anterior wall MI에서보다 예후 훨씬 좋다. (2) 1st degree AV block 이 m/c (3) Tx : IV atropine, temporary pacemaker

ᅳ Anticholinergic therapy(atropine) : 증상있는 sinus bradycardia(저혐 압,허혈, 또는 escape ventricular arrhythmia가 동반된 H R 갑작스럽게 발생해서 점차 악화되는 혈역학적 변화 - Morphine - ACE inhibitor - IABP :acute MR/VSD 시 - Loop diuretics : furosemide 一 D opam ine/dᄋb니tamine,digoxin

- Oxygenation(성:소공급, °SC& % 기) - Nitrate : nitroglycerin, isosorbide dinitrate —^ 이 중ᄋII서 Furosemide, Morphine, Oxygen, 그리고 systolic BP 100 이성;chi서 ACEi가 중요협니다• IABP는 다른 ^ 을 다 시도현: 후에도 어려운 용급혹!:자나,장기적으로 surgical implanted device를 유치하기 위^]: bridge로 /생74 하시고,Nitrate나 Dopamine, Dobutamine은 심장성 쇼크에 나온 표에 1따라 시■용항니다. Harrison 2Q민;,pp. 2057~ 2059

—J ____________________________________________________ O f -

60세 남자가 3일 전부터 숨이 차서 응급실로 왔다. 혈압 140/80mmHg,

맥박 72회/ 분,호흡 24회/ 분,체온 36.3도였다. 가슴 청진에서 심박동은 규칙적이었고 심음은 작게 들렸으며 양쪽 등쪽 아랫부분에서 거품소리 가 들렸다. 양쪽 정강뼈앞오목부종이 있었다. 혈액검사 소견은 다음과 같았다. 가슴 X선 人변이다. 치료는? 혈액요소질소/크레아티 닌 18/1.0rag/dL,트로포닌 T 3ng/L(참고치, 0〜 14),혈장뇌나트륨이뇨펩티드 730ng/L(참고치, 12

P wave가 잘 관찰되지 않으며 narrow QRS

가 규칙적으로 빠르게 나타나는 것으로 PSVT

외의 다른 만성질환 없는 49세 남자 환자의 항고혈압 약제로 가장 적합

를 생각할 수 있습니다. PSVT 가 있는 환자에서 고혈압의 치료에 관한 문제입니다. PSVT의 치료제로 A V node의 conduction을 늦춰주는 adenosine, /3-blocker

한 것은?

등을 人[용할 수 있으며,이 환자와 같이 고혈 압이 있는 경우 beta-blocker가 가장 적합한 항고혈압제가 될 수 있습니다. Harrison 2ᄋ만,pp. 1900~ 1906

1)

칼슘길항제

2) 베타차단제

3)

이뇨제

4) 알파차단제

5)

앤지오텐신 전환효소 억제제

13

Q 13

안지오텐신 전환효소 억제제에 대한 설명으로 옳은 것은?

ACE inhibitor의 부작용은 다음과 같습니다. leukopenia, cough,

hyperkalemia,

acute

renal failure, bilateral renal artery steno­

1) 신부전 시에 사용할 수 있다.

sis, teratogenic effect

2) 임신 중 사용해도 안전하다.

ACEi 는 단백뇨,당뇨를 보이는 환자에서는

3) 마른기침이 잘 생긴다.

쓸 수 있지만, 정작 급성 신부전을 보이는 환 자에서는 사용할 수가 없습니다!

4) 백혈구증다증이 발생할 수 있다.

Harrison 2이만,pp,1900~1906

5) 한쪽 신동맥 협착이 동반된 고혈압에서는 금기이다.

U

a -o

g

63세 남자가 운동 시 흉통을 주소로 내원하였다. 증상은 10분간 계속되다

가 니트로글리세린 설하정을 투여하면 소실되었다. 혈압은 170/110mmHg 일 때 이 환자에게 적합한 혈입약은? 1) vasodilator

2)

aldosterone antagonist

3) /5-blocker

4)

thiazide

Stable angina pectoris가 의심되는 환자의 고혈압 치료에 관한 문제입니다. 이 경우 신-blocker가 D O C 입니다. 혈압을 낮 출 뿐만 아니라,angina의 치료 효과도 있기

때문입니다. 다시 한 번 상기해 봅시다. Stable angina의 치^제는 ABCN(Aspirin, BB, CCB, Nitrate) 입니다.

5) ACE inhibitor

^A N SW E R

|ᄋ

o-

01.① 02.③ 03.⑤ 04.① 05.③ 06.② 07.② 08.(3) 09.④ 10.⑤ 11■② 12.② 13.③ 14•③

Harrison 201 만,pp. 1900~ 1906





___________________

Explanation

치료저항성 고혈압

^ ♦

치료? H항성 고 열 ^의 정우 2하성 고 혈 吧 가능성을 항상 eg두에 두어야 협니다ᅵ 특

1.

di 젊은 나이에 밀 생한다면 더욱 그령헹죠?

치료저항성 고 s 압(resistant 1)

hypertension)

정의 이뇨제를 포함한 3개 이상의 적절한 항고혈압계를 최고 농도로 사용하여도 혈압이 140/90mmHg보 다 높게 측정되는 것

2) 60세 이상의 노인에서 더 흔하다. 3) 관련된 요인

(1 ) Pseudoresistance : 병원에서 잰 혈압이 집에서 잰 혈압보다 높은 경우 (2) 약을 제대로 복용한진 ^을 결운 (3) 비만, 과음 등 (4) 약제 : estrogen, adrenal steroid, decongestant, 식욕억제제 , cyciosporine, TCA, M A 이 , erythropoietin, NSAIDs, cocaine (5) 가성고혈압(pseudohypertension) : 동맥경화가 있는 노인에서 혈압이 실제보다 높게 측정됨

傘 침습적 방법으로 동맥 내 열입을 측정하여 평가할 수 있습니다ᅵ Harrison 2 0 ^ , pp.l900~1906

广

01

B

고혈압 환자의 혈압이 항고혈압제로 잘 조절되지 않는 경우는?

문제해설 ---------------------- cA> 0 1 모두 항고혈압제로 잘 조절되지 않는 경우입 니다.

가. 염분섭취 과다

나. 경구피임약 복용

다. 이차성 고혈압

라. 부적절한 투여용량

1) 가’ 니% 다

2) 가, 다

4) 라

5) 가. 나,다, 라

©ANSW ER

01.⑤

3) 나,라

Harrison 20^, pp. 190ᄋ〜 1906

2020 년대비PACIFIC KMLE

순환기

!{niiUliHUSi!iiUHi3nUlifHHl!fUlU!USf11!H!U!HHlH$iniHyUiHU!i!in!UillilIUi!i!iUijlMlII!ilitii

B m m ___________________________



Explanation

고혈압-고혈압의 응급 傘 대동맥질환, 성경과의 stroke, 성:과에 전자공증에서 고혈SI의 관리를 또 배우시게 평니다. 이곳에서는 어1편 정 우를 고혈_

용급상황 이라 부르는지 알아두고 어1전 약제가 주로 쓰이는지 던번 임어보고 명어가시면 철 ^

'같습니다. 각 partoil서 다시 세부적인 약제들을 공부하도록 던시다. 고형0t5의 용급에서 혈0』7^>ᅡ는 정진적으로 이루어진다는 ᄀ;i 기억해주세요, 너부 ^리 '낮추면 머리로 피가 안가요.

1. 정의 1) Hypertensive emergency

진행하는 target organ 기능 장애 질환별 항고혈압제의 선택은 그 질환의 치료익제와 별로 다르지 않다.

W drug Hypertensive emergency

Immediate Delayed

Nitroprusside, nitroglycerin, diazoxide, fenoldopam, esmolol Hydralazine, nicardipine, enalapril, labetalol

Z

®

2020년 대비 PACIFIC KMLE

순환기

[01

g ]-0 |-[c ] _

27세 여자가 갑작스런 두통,시력저하와 경련으로 응급실에 왔다. 으I식

은 혼미하였고 혈압 240/160mmHg,맥박 100회/ 분이었다. 양쪽 다리 에 오목부종이 있었다. 안저검人빼서 유두부종이 관찰되었다. 검사 결과 는 다음과 같았다. 치료는? • 혈액 : 혈색소 9.6g/dL, 크레아티닌 3.0mg/dL, Na+/K+ 135/4.2meq/L • 소변 : 단백질(3+ ),적혈구 20〜29/고배율시야,적혈구원주(+)

문제해설

■CTᄋ'O

0 1 갑작스런 두통과 함께 심한 고혈압(>220/ 140mmHg), 시력저하, 경련,의식저하 등 target organ 기능장애까지 보이고 있는 고혈 압성 응급 환자의 전형적인 증례입니다. 뇌출 혈을 의심할 수 있는 상황입니다. 치료로는 Labetalol, Nitroprusside를 쓸 수 있습니다. 고혈압성 응급,Stroke, Aortic dissection, 산과의 Preeclampsia의 치료제를 헷갈리지 않게 정리해 두세요! Harrison 20=1;, pp. 1900〜 1906

1) 디곡신(digoxin) 정맥주사 2) 라베탈롤(labetalol) 정맥주사 3) 니페디핀(nifedipine) 설하투여 4) 푸로세미드(furosemide) 정맥주사 5) 안지오텐신수용체차단제(ARB) 경구투여

02 니广

____________________________________________________g ] ■回 .B .

53세 남자가 1시간 전 두통,구토와 의식 소실을 주소로 응급실에 실려

왔다. 혈압 220/120mmHg이었으며 흉부 X선 상 심비대가 나타났다. 뇌 CT 촬영에서는 특이 소견이 발견되지 않았다. 치료에 대한 설명으로 옳은 것은?

제입니다. 나. aspirin은 고혈압의 치료에 人[용되지 않습

가 . diazoxide 정 맥 주 사

니다. 라. 혈압을 응급으로 낮추되 점진적으로 떨어 뜨리는 게 중요합니다.

나 . aspirin 경 구 투 여 다 . nitroprusside 정 맥 주 사

Harrison 으ᄋ그만,pp. 1900~ 1906

라. 혈 압 을 최 대 한 빨리 정 상 화 시 킨 다 .

1)

가, 나,다

2) 가,다

4)



5>

©ANSWER

0 2 심한 혈압 상승과 함께 혼수,구토 증상0 1 Lf타 났습니다. Hypertensive emergency입니다. 치료는 항고혈압제를 IV로 투여해야 합니다. onset이 빠른 약을 써야합니다. nitroprus­ side, diazoxide는 onset이 빠른 IV 항고혈압

이.©。2.②

가,나,다, 라

3) 나, 라

대동맥질환 o ut

작년77ᅡ지는 대동DJj^% 에 대동다1)박리와 대동맥류에 원한 내용을 녕있습니다. 하지맛 대동1객류■의 정우에는 외과쪽으로 옮기는 ^이 적혈하다고 吧 하 여 순흰:기에서 대동1맥박리Duᅡ다루게 되%습니다. 대동0과류■의 경우 외과적 수술이 주라는 을 기억하시고,대동1객 박리에 원한 내과적 치료를 위주로 기억하시면 종'!}!습니다. 대동맥 박리의 경우,증상이나 영상소^이 확^하으로 진단하는데 어a 움은 없을 것 ~y습니다. 진 단 ᄍ 치료를 진 @ ᅡ두시고,특히 내과적 치료와 수술적 치료의 적응증을 질; 않고 91으셔야 형니다. 2019면 국시에서는 대동1객 박리에 관해서 1문제가 출제되었습니다.

Explanation

0@ E M __

^

대동맥박리(Aortic dissection) 傘 고열0음이 ^

환자에서 성자기 용통이 /냄기면 의심협시다. 일반적으로 용통은 찢어지는 행상으로 뻗어나 다

는 임상상을 제시현니다. 최근 정형에 따라 수술 적응증과 약울적 치료를 꼼꼼히 살펴봅시다.

분류 1) DeBakey classification (1)

Type I

: ascending aorta, aortic arch, descending aorta 모두 침범

(2)

Type E : ascending aorta만 침범

(3) Type III : descending acMta만 침범 2) Stanford classification (1)

TypeA : Ascending aorta 침범

(2)

TypeB : descending aorta에만 국한

Type A

Type B

classification of aortic dissection. Stanford classification Top panel illustrates type A dissections that involve the ascending aorta independent of site of tear and distal extension; type B dissections involve transverse and/or descending aorta without involvement of the ascending aorta. DeBakey classification : Type I dissection involves ascending to descending aorta(top left);type I dissection is limited to ascending of transverse aorta, without descending aorta(top center +top rights);type I dissection involves descending aorta onlyfbottom left).

-o

o

unl

2020년 대비 PACIFIC KMLE

순환기

離 A가 B보다 심하고. I > 1 > 1 순서로 심한

엽게 기억에 님는데,어1헌 분류법이 A,B이고 어^ 분류법이

I ,I ,11^ 지가 데변 했P i렸습니다. 그래서 저는 Stanford university는 A. B로 학정을 해그요단고 외웠습니다ᅵ

2.

원인 1) 고혈압 t시 ■ o o

O.

1)

Pain a ) 갑자기 시작되고 매우 심함, 찢어지는 듯한 양상의 통증 (2)

가슴의 암쪽이나 뒤쪽에서 발생, 종종 interscapular region, 박리 진행 방향에

따라• 이동

2) Syncope, dyspnea, weakness 3) 고혈압 또는 저혈압, 맥박 소실,AR ,폐부종 4)

신경학적 증상

: hemianesthesia, hemiplegia(carotid artery 침범),paraplegia(spinalcordischemia)

5)

대동맥의 분지 침범 ᅳ bowel ischemia, hematuria, myocardial ischemia

6)

주위조직 압박

7)

Acute A R : 대동맥박리가 AV annulus를 침범 시 나타남

— Homer’s syndrome, SVC syndrome, hoarseness, dysphagia,airway압박

Bounding pulse, wide pulse pressure, diastolic murmur, CHF -n-^*

4. 진단 1)

C X R : widenii페 of superior mediastinum(mediastinal widening^ pleuralefftision

2)

EKG : 정상,myocardial ischemia의 감별진단

3)

싶흐을파 (1) Transthoracic echocardiogram(TTE) : proximal ascending AD 의 진단에 유용 (2) Transesophageal echocardiogram(TEE) : ascending & descending AD 의 진단에 매우 유용

4)

CT, MRI

5) Aortic angiography : gold standard, 확진 검사 本 AD 의심할 Du향 상황(이7a HTN의 history가

면 ^

환자,가슴이 멀어지는 듯한 ^

CXR, CT 찍고 진단은 TEE, chest CT로 형니다.

pain, 현재 BP 높은 끼이

t ■■■1 ❖Aortic dissection의 TEE와 Color doppler

- Ao(Aorta), Fl(Flap), La(Left atrium) — True lumen(open arrow)0] false lumen(solid arrow)과 separation

5. 치료 1) 약물치료

離 약물 종류•까지 꼭 외우I두세요!!! (1) 진단 즉시 ICU 에 입원하여 약물치료를 시작 (2) S -blocker +sodium nitroprusside

① /?-blccker(propranolol, metoprolol, esmolol) — HR 60/min 유지 ② Sodium nitroprusside — SBP 120mmHg 이하로 유지 (3) Labetalol(nonselective a & ^-blocker) 단독 傘 서I IHHt 알아두어요.

攀 BB와 nitroprusside는 방드시 창이 사용해야 해요. nitroprusside가 혈0홈을 넘어뜨리지만 LV ccintractiHty를 증가시키기 때문에 대동dj(박리가 더 심해질 수 있기 매문이지요ᅵ 하나 더,대동맥 박리 의 중요한 우 p jp i 고형o』이기 매문에 대부분의 환자는 내% I당시 dh우 높은 형와을 보일 수 있어서 hypertensive emergency와 의fy않 수 9JL으니 주의하세요(diazoxide가 hypertensive emergency의 치료이기 매문이죠)_ (4) CCB(verapamil, diltiazem) : 위의 치료에 효과가 없을 때 사용 가능 (5) ACE inhibitor(enalaprilat) : 추가할 수 있다. (6) Direct vasodilator(diazoxide, hydralazine)는 금기 : tear가 propagation 될 수 있다. 雄 대동1객이 박리되•으으로 HR과 contractility를 1낮추나주는 치료를 형니다. vasodilate)「 는 금기0』니다. 그 서 DHP CCB와 direct vasodilator 모두 금기0』니다.

2) 수술 (1) ^술 적 응 증

① Type A(ascending aortic dissection)

2020년 대비 PACIFIC KMLE

순환기

② Complicated type B • Propagation • Compromise of major aortic branches • Impending rupture • Continued pain (2)

수술 방법 : incision of the intimal flap, obliteration of the false lumen, placement of an inter­ position graft Harrison 2Q만,pp.l919~1922

01

o

70세 여자가 2시간 전부터 가슴이 심하게 아프다며 응급실에 왔다. 통

증은 등으로 뻗치고 점차 아래로 내려갔다고 한다. 혈압 72/42mmHg, 맥박 150회/ 분, 호흡 24회/ 분,체온 36XTC이다. 의식이 호리고 오른쪽 팔 맥박이 만져지지 않는다. 가슴 X선 사진과 심전도이다. 치료는?

문제해설

----------------------

0 1 고령 여성이 갑작스런 심한 등으로 뻗치는 흉 통을 호소하고 있고 가슴 X선 사진에서는 종 격동이 확장된 소견을 보이고 있어 대동맥박 리를 의심해볼 수 있고,S T E M ® 배제하기 우ᅵ해 심전도를 보면, ST elevation은 관찰되 지 않아 배제할 수 있겠습니다. 오른 팔의 맥 박이 만져지지 않는 것으로 보아 aortic arch 에 있는 branch를 침범한 것으로 생각되며, 통증이 점차 아래로 내려가는 것으로 보아 박 리가 Of래로 진행되고 있음을 알 수 있습니다. 현재 활력 징후가 매우 불안정하고 의식 저하 도 나타나고 있습니다. 응급 수술이 필요한 상 황이며 답은 대동맥대치술이 되겠습니다. Harrison 2ᄋ ᅵ 만 ,pp.l919~1922

1) 도부타민

2) 라베탈롤

3) 대동맥대치술

4) 심장동맥성형술

5) 대동맥내풍선맞박동

대동맥 질환



02

—9— ^____________________________________________________________IA; E l gj ᅳ

57세 남자가 갑자기 가슴이 아파서 병원에 왔다. 통증은 2시간 전부터

시작되었다고 한다. 통증은 가슴이 찢어지는 듯하고 지속적이며 등쪽으 로 뻗치는 양상이다. 2 년 전 고혈압 진단을 받았으나 혈압강하제는 복용 하지 않았다고 한다. 혈압 98/79mmHg, 맥박 112회/ 분, 호흡 26회/ 분, 체온 36.7°C이다. 가슴 청진에서 심음과 호흡음은 정상이다. 가슴 컴퓨 터단층 촬영 사진이다. 혈액검사 결과는 다음과 같다. 진단은? 혈색소 13.0g/dL,백혈구 9,500/mm3,혈소판 240, 000/mm3

1) 폐색전증 3)

심장눌림증

5)

말로리一바이스증후군



0%

0 2 2시간 전부터 지속된 가슴이 찢어지는 듯하고 등쪽으로 뻗치는 통증을 주소로 내원한 중년 의 남성 증례입니다. 임상상을 통해 대동맥 박리를 의심할 수 있습니다. 고F거에 진단받고 제대로 조절하지 않은 고혈 압의 병력도 대동맥 박리의 가능성을 높여줍 니다. 시행한 가슴 컴퓨터단층 촬영 사진에서 descending acjrta에 flap 소견이 관찰되기 때문 에 대동맥 박리로 확진할 수 있습니다. Harrison 20만,pp.l919~1922

2) 가슴막삼출 4) 대동맥박리

___________________________________________

B

50세 남자환자가 1시간 전부터 시작된 흉통과 등으로 방사되는 통증으

로 응급실에 내원하였다. 내원 당시 BP 180/100 HR 90회/분으로 체크 되었다. 환지는 5 년 전 진단받은 고혈압으로 투약하다 1개월 전 자의로 투약을 중단한 상태였다. 환자의 진단은?

1) 폐색전증

2) 급성심막염

3) 심내막염

4) 대동맥 박리증

5) 급성 전벽부 심근경색증

문제해설

0 3 고혈압의 병력과 등으로 방사되는 급성 흉통, 흉부 CT상 보이는 intimal flap 등에서 대동 맥 박리가 가장 의심됩니다. Harrison 2ᄋ민■,pp. 1919〜 1922

2020년 대비 PACIFIC KMLE

[04

순환기

______________________________________ B

_

63세 남자가 2시간 전부터 가슴이 아프다며 응급실에 왔다. 통증은 가

슴 가운데에서 찢어지는 듯하게 생겨 등과 배쪽으로 뻗쳤다. 12년 전부 터 혈압강하제를 불규칙하게 복용하고 있다. 혈압 186/103mmHg, 맥 박 84회/ 분,호흡 18회/ 분, 체온 36.6°C이다. 목정맥은 늘어나 있지 않 고 가슴 청진에서 심음과 호흡음은 정상이다. 혈액검사 결고h는 다음과 같다. 가슴 X선 사진과 심전도이다. 검人!'는? 트로포닌-T 8ng/L( 참고치,0~14)

0 4 2시간 전부터 시작된 가슴 가운데에서 시작하 여 등과 배로 뻗치는 양상의 통증은 대동맥 박리(Aortic dissection)를 강하게 人I사하는 임상상입니다. 높은 혈압과 불규칙한 혈압 복 용의 과거력은 이를 뒷받침합니다. 시행한 가슴 X선 사진 상에서 상부 종격동이 확장된 소견을 관찰할 수 있습니다. 시행한 심전도 상에서는 lead V5, V6에서 높은 R wave가 관찰되며, lead Vi에서 P-terminal force 으0.04이 관찰되어서 LA enlargement 와 함께 LVH가 동반되어 있음을 유추할 수 있습니다. 대동맥 박리에 대해서 시행해야 하는 검사는 대동맥 박리의 범위를 구체적으로 알 수 있는 가슴 및 복부 컴퓨터단층촬영입니다. 약물 치료의 기본은 labetal이을 단독으로 사 용하거나 beta blocker+ sodium nitroprusside를 사용하는 것이며,ascending aorta를 침범한 경우나 complicated type B 에 대해서 수술을 시행할 수 있겠습니다. Harrison 2Q만,pp. 1919~ 1922

1) D -이량체

2) 관상동맥조영술

3) 운동부하 심전도 검사

4) 약물부하 심근관류스캔

5) 가슴 및 복부 컴퓨터단층촬영

들르iir —Z Z I® 05

o

57세 여자가 3시간 전부터 가슴이 아파서 왔다. 3년 전부터 혈압이 높

다는 말을 들었으나 혈압강하제는 복용하지 않았다. 통증은 목에서 등까 지 뻗친다고 했다. 혈압은 오른쪽 팔이 182/104mmHg,왼쪽 팔이 174/96mmHg, 맥박 62회/ 분,호흡 17회/ 분, 체온 36.0도였다. 가슴 CT 人KJ과 심전도이다. 치료는?

문제해설

--------------------

0 5 고혈압의 과거력을 가지고 있으면서 갑자기 가슴통증을 호소^는 환자입니다. Aortic dissection을 의심합시다. 등으로 뻗치며 심전도 상 ST elevation이 없는 것으로 보아 STEMI 는 아닙니다. CXR과 CT로 확인하고 필요할 경우 Aortic angiography로 확진합니다. 치료는 ① 약물치료 • !3 -blocker 十nitroprusside • Labetalol ② 수술치료 Ix • Type A(ascending aortic dissection) • Type B w propagation, rupture, major a. branch침범, 지속되는 흉통 Harrison 2ᄋ만,pp. 1919~ 1922

25mm/sec 10mm/mV

1) 질산염(nitrate)

2) 아스피린(aspirin)

3) 라베탈롤(labetalol)

4) 니페디핀(nifedipine)

5) 히드랄라진(hydralazine)

ᄋ은 83;

2020년 대비 PACIFIC KMLE

순환기

그 화

06



70세 여자가 3시간 동안 가슴이 아파서 왔다. 통증은 지속적이고 등까

지 뻗치며 니트로글리세린을 혀 밑에 투여하여도 좋아지지 않았다. 10 년 전부터 혈압이 높았으나 치료를 받지 않았다. 혈압 180/100mmHg, 맥박 90회/ 분,호흡 20회/ 분,체온 36.8도였다. 가슴 X선 사진과 심전

문제해설

니A )

0 6 고혈압의 고ᅡ거력을가지고 있으면서 갑자기 가 a 통g 을 호소하는 환자입니다. Aortic dissection을 의심합시다. 등으로 뻗치며 심전도상 ST elevation이 없는 것으로 보아 STEM!는 아닙니다. CXR와 초음파, CT로 획인하고 필 요할 경우 Aortic angiography로 확진합니다. 치료는 ① 약물치료 • jS-blocker+ nitroprusside • Labetalol ② 수술치료 Ix • Type A(ascending aortic dissection) • Type B w propagation, rupture, major a. branch침범, 지속되는 흉통

도 사진이다. 치료는?

Harrison 2ᄋ만,pp. 1919~ 1922

1) 헤파린(heparin)

2) 아스피린(aspirin)

3) 라베타롤(labetalol)

4) 푸로세미드(furosemide)

5) 히드랄라진(hydralazine)

대동맥 질환

07

문제해설



58세 남자가 가슴이 아파서 병원에 왔다. 약 2시간 전부터 등 한가운데

가 심하게 아왔다가 지금은 앞가슴에서 통증이 느껴졌다. 왼쪽 팔 혈압 180/90mmHg, 오른팔 혈압 80/40mmHg, 맥박은 90회/ 분이었다. 가

슴 X선 시•진은 다음과 같다. 다음 검人!는?

r : z

----------------------

0 7 CXR에서 상부종격동이 확장된 것이 보이시 나요? 통증의 전파 양상을 고려하면 aortic dissection 증례인 것을 알 수 있습니다. 양쪽 팔 혈압이 크게 차이나는 것으로 보아 intimal flap에 의해 aortic arch의 branch인 subclavian artery가 involve되었다는 것을 알 수 있습니다. 다음 검사로 CT를 해야겠습 니다. false lumen과 true lumen을 나누고 있는 intimal flap을 확인하면 진단이 확실해 질 뿐 아니라, aortic dissection이 발생한 위 치를 확인할 수 있어 수술적 치료의 적응증이 되는지를 판단할 수 있기 때문입니다. Harrison 2Q만,pp. 1919~ 1922

1) 관상동맥조영술

2) 혈청 트로포닌 I

3) 운동부하심전도

4) 가슴컴퓨터단층촬영

5) D-이량체(D-dimer)

r

08

B

55세 남자가 심한 가슴통증 때문에 내원하였다. 흉통은 무거운 짐을 옮

기던 중 발생하였고 가슴이 벌어지는 것 같은 양상이었으며 식은땀이 났 다고 한다. 흉부 CT 소견은 다음과 같았다. 이 환자에게 적절한 치료는?

0 8 CT소견에서 대동맥이 찢어진 것이 명확히 보 입니다. aortic dissection입니다. ① 약물치료 • /?-blocker+ nitroprusside • Labetalol ② 수술치료 lx • Type A(ascending aortic dissection) • Type B w propagation, rupture, major a. branch침범,지속되는 흉통 Harrison 2 0 잔■ ,pp. 1919~ 1922 심징■ 학교과서,대한순환기학회,pp.509~510 Braunwald's Heart Disease, 7th ed., 2005

nifedipine

2) labetalol

hydralazine

4) captopril

diazoxide

@

^

c-

2020년 대비 PACIFIC KMLE

순환기 그 포

09

Q

15년 전 고혈압을 진단받았으나 치료를 받지 않은 62세 남자가 오늘

목욕탕에서 갑자기 전흉부에 심한 통증이 발생하였고 2시간이 지나도 통증이 나아지지 않아서 응급실에 내원하였다. 이학적 검사상 혈압은

0 9 고혈압의 과거력을 가지고 있으면서 갑자기 가슴통증을 호소하는 환자입니다. Aortic dissection을 의심합시다. CXR과 초음파, (그로 확인하고 필요할 경우 Aortic angiography로 확진합니다.

180/112mmHg, 청진 상 심잡음은 없었다. 심전도는 정상이었고 흉부

Harrison 2 0 ^ , pp.l919~1922

X선 사진은 다음과 같았다. 진단과 치료 결정을 위해 우선 시행하여야

할 검사는?

1) 운동부하 심전도검사

2) lung scan

3) 폐동맥조영술

4) 경식도 초음파검사

5) catheterization

10



65세 남자가 3시간 전부터 人I작된 가슴통증을 주소로 내원하였다. 통증은

기슴이 벌어자는 것 같은 양상이었으며 식은땀이 많이 났다. 15년 전 고혈 압을 진단받았으나 특별한 치료는 받지 않았다. 혈압은 180/11 OmmHg, 호흡수 20회/ 분, 가슴 X선 人卜진은 다음과 같았다. 가장 으I심할 수 있는 진단은?

급성 심근경색

2) 대동맥자루

비후 심근증

4) 폐동맥색전

대동맥박리 ( ^ ) o

ᄋ—

10

CXR상 mediastinum이 넓어져 있으며 가슴 이 벌어지는 것과 같은 통증,높은 혈압은 급 성대동맥박리를 시사합니다. Harrison 2 0 ^ , pp. 1919〜 1922

대동맥 질환

11

문제해설

B

35세 여자 배구선수가 2시간 전부터 人|작된 극심한 가슴통증을 주소로

11

내원하였다. 혈압은 180/110,심박수 80회/ 분, 키 190cm, 몸무게 65kg 이었다. 이에 시행한 초음파사잔은 다음과 같다. 가장 우선적으로 해야 할

0^0

매번 X-ray나 CT로 주어졌던 aortic dissectionO| transthoracic echocardiography로 주어졌습니다. 혈압도 높고,키가 큰 마른 운 동선수라는 병력에서 왠지 Marfan 증후군의 냄새가 나지 않나요? 그렇지만 이건 심증에

치료는?

불과하고 확실한 증거는 심초음파에 있습니 다. 심초음파만 나오면 책을 덮으시는 분들을 위해 다른 논문에 실린 그림으로 잠시 설명을 드리면,시는 조침방, VI는 조[심실, AoA는 대동맥궁을 나타냅니다. 맨 위 동그라미는 RV이고 호!살표가 친절하게 가리키고 있는 부분이 바로 flap입니다. 문제의 그림에서도 이제 flap과 false lumen이 이제 보이시나요? 대동맥박리가 확인되었으니, 이제 즉시 ICU 에 입원하여 sodium nitroprusside+ selective /?-blocker ^ 0 - nonselective a , /?-blocker인 labetalol 단독으로 약물치료를 시작해야 합 니다. 그리고 어떤 type인지 확인한 후에 수 술 여부를 결정해야 합니다.

1) ACE inhibitor

2) /^-blocker

3) Nifedipine

4) Nitrate

5) Hydralazine

Harrison 2Q만,pp. 1919〜 1922 Arturo E et al., Role of Transoesophageal Echocardiography in the Diagnosis of Acute Aortic Syndrome

12

Q

고혈압이 있는 65세 여자가 갑자기 발생한 가슴통증과 일시적인 으|식소 실로 응급실에 내원하였다. radial a 의 맥박이 촉지되지 않았고, NG 투 여 후에도 가슴통증은 소실되지 않았다. 제3도의 초기 decrescendo diastolic murmur가 흉골연 상부에서 청진되었으며, 흉부 X선상 종격동

비대 소견을 보였다. 진단은? 1) AMI

2) aortic dissection

3) unstable angina

4) 심내막염에 의한 acute AR

5) pulmonary embolism

12

흉통, 의식소실, 맥박 소실, CXR에서 종격동 확대,diastolic murmur(AR)으로 미루어 보 아 대동맥박리입니다. Harrison 2 0 만,pp. 1919~ 1922

빼 * 2020년 대비 PACIFIC KMLE

순환기

r --------

1131

ra

고혈압 치료를 받고 있던 60세 남자가 갑자기 발생한 찢어지는 듯한 심 한 가슴통증을 주소로 내원하였다. 혈압 170/11 OmmHg이었고 흉부 CT 소견은 [ I 음과 같았다. 人^용할 수 있는 약은? J\. labetalol

나. propranolol

다. sodium nitroprusside

己h diazoxide

13

고혈압의 과거력을 가지고 있으면서 갑자기 가 □ 통o 을 호소하는 환자입니다. Aortic dissection을 의심합시다. 등으로 뻗치며 심전도상 ST elevationOl 없는 것으로 보아 STEMI는 아닙니다. CXR와 초음파,(그로 획인하고 필 요할 경우 Aortic angiography로 확진합니다치료는 ① 약물치료 • j3-blocker+ nitroprusside • Labetalol • Direct vasodilator(diazoxide, hydrala­ zine) 금기 ② 수술치료 Ix • Type A(ascending aortic dissection) • Type B w propagation, rupture, major a. branch침범,지속되는 흉통 Harrison 20만, pp.l919~1922

1) 가,나,다

가,다

4) 라

가,나,다,라

©ANSW ER

3) 나,라

01.③ 02.④ 03.④ 04.⑤ 05.③ 06.③ 07.④ 08.② 09.④ 10.(1) 11.® 12.② 13.©





aortic arch 에

대동맥 질환

Z



분 지 하 는

glucocorticoid 를

/서 볼



현 표 들 을

91습

니 다

막 는

.

병 입 니 다

수 술 적

.

침 t향

Z

I T

Explanation

-------혈관염 (Takayasu's arteritis)

다카야수 傘

H

^ 의

pulse 가



들 리 계 죠

bypass 나 endovascular intervention 이

.



1?i 조

영 술 로

필 요 하 기 도



진 단 하 고

'74!습

니 다

.

역학 1) 정의 : Medium and large-sized arteries, 특히 aortic arch와 그 분지를 주로 침범하는 inflammatory & stenotic disease 2) Adolescent girl, young women에서 호발,Asia에서 호발 3) 주로 침범하는 혈관 : subclavian > common carotid > abdominal aorta

O

스.

O L M O t시 ■ 口 CD O C5

1) Malaise, fever, night sweat, arthralgia, anorexia, weight loss 등 전신 증상이 혈관 증상이 나타나기

수개월 전에 나타남 2) 침범된 혈관의 pulse가 소실됨 3) 고혈압 4) ESR | ,anemia 霧;|청 ggj빼 |

Takayasus arteritisCHl서는 Subclavian steal syndrome이 동방 가능협니다. 에를 들어 It. subclavian a.를 참1권 해서 혈류게 줄어드는 경우 그 곳에서 기시하는 it. vertebral a.의 혈류가 줄어들고 이에 따라 rt. vertebral a.에 서 basilar a.로 ■§라가는 ^류 가 거꾸로 It. vertebral a.를 통건떠 역류베ᅵ 병니다. 이로 0抑 서 syncope와

은 증상이 나타님니다.

3.

진단 : 혈관조영술(arteriography) 젊은 여성에서 말초의 pulse가 감소하거나 없는 경우, 양쪽 혈압이 차이 나는 경 위 lOmmHg 이상 차이), bruit가 들리는 경우 시행 —> irregular vessel wall, stenosis, poststenotic dilatation, aneurysm formation, occlusion, collateral circulation

4. 치료 1) 사망원인 : CHF, cerebrovascular event, MI, aneurysm rupture, renal failure 2) 병의 경과 : 다양함,대개 chronic & relapsing 3) 치료방법 : glucocorticoid, 면역억제제(MTX),수술(bypass or endovascular intervention) Harrison 2Q만,p. 1922

^ I

2020년 대비 PACIFIC KMLE

순환기 I ' ®

01

O ' @ H I]

20세 여자가 2개월 전부터 갑자기 눈앞이 캄캄해지며 정신을 잃는다며

병원에 왔다. 오른팔 혈압 100/70mmHg, 맥박 70회/ 분, 호흡 19회/ 분, 체온 36.5도였다. 오른쪽 목동맥에서 잡음이 들렸고 왼쪽 노동맥(radial artery)은 맥박이 만져지지 않았다. 검사는?

1)

흘터검사

2> 관상동맥조영술

3)

대동맥조영술

4) 식도경유심장초음파

5)

운동부하심전도검사

문제해설 ------------------------cA> 0 1 젊은 여자가 한쪽 맥박이 만져지지 않습니다. 바로 Takayasu’ s arteritis를 으ᅵ심합시다. 진단은 혈관조영술입니다. 치료는 glucocorticoid / immunosuppressive agents를 써보거나 bypass 수술 혹은 stent를 넣을 수 있습니다. Harrison 2 0 ^ , p. 1922

rz

02

Q

29세 여자가 4개월 전부터 시작된 어지러움을 주소로 내원하였다. 오른

쪽팔 혈압 114/70mmHg, 왼쪽팔 혈압 70/40mmHg였고, 맥박수는 74 호l/min 이었다. 왼쪽 목에서 잡음이 들렸다. 검사는? 1) 24시간 홀터검사

ᄋ2 젊은 여자가 양팔의 혈압차이가 많이 납니다. Takayasu’s arteritis을 의심합시다. 목에서 잡음이 들리는 경우도 의심할 수 있습니다. 진단은 혈관조영술(angiography)이지만 최근 에는 영상기법이 발달되어 CT angiography 를 시행할 수 있습니다. Harrison 20=1;, p. 1922

2) 기립 경사 테이블 3) 트레드밀 테스트 4) 목 전산화단층혈관조영술(CT angiography) 5)

n

폐관류 스캔

03

Q (

30세 여자가 혈압이 낮다고 병원에 왔다. 오른쪽 팔에서 측정한 혈압은 120/80mmHg, 왼쪽 팔에서 측정한 혈압은 70/40mmHg이었다. 왼쪽

손은 7 년 전부터 설거지나 청소를 하면 통증이 발생했다. 혈액검사 결과 는 다음과 같다. 진단은? 백혈구 4700,Hb 10,ESR 60( t人h a

o

o

o

1) Skeletal effects (1) 키가 크고 팔다리가 길다(상지보다 하지가 길다). (2) Arachnodactyly : 손가락이 길고 spider like appearance를 보임 (3) Chest deformity : pectus excavatum(depression), pectus carinatum(protrusion) (4) Scoliosis and kyphosis (5) Enlargement of nerual canal, thinning of pedicle and laminas 2) Cardiovascular features : m ajor source o f m ortality and m orbidity

(1) MVP (2) Dilation of root of aorta —►AR, aortic aneurysm 3) Ocular features : dow nw ard displacem ent o f lens ᅳ »• 백 내 장

4) Other features (1) Striae : shoulder, buttocks (2) Spontaneous pneumothorax, inguinal and incisional hernia

3. 분류 및 진단 Slit-lamp examination, echocardiogram, amino acid analysis(homocystinuria) 1)

Type 1 Marfan syndrome —^ Major criteria(Ghent standards) (1) 4개 이상의 skeletal abnormality (2) Ectopia lentis (3) Dilation of ascending aorta w/ or w/o dissection (4) Dural ectasia (5) 같은 진단기준을 만족하는 blood relative 傘

Berlin classification 과 Ghent cla ssifica tio rri: 진 단 형 니 다 0 그 자 •세 던

^9 2 )o

O

진 단

기 준 은

묶 어 서

ᅳ 생략하'고 쌌 습 니 다 .

2 M ajor+1 Minor

또 는

1 m ajor+ 4 minor

이 상 이 면

一ᅵᅢ동맥질~

傘 Type 1 Marfan syndrome% ^ 의 90% 이상에서 fibrillin-1(FBN1) mutation이 말'7d월니다.

2) Type 2 Marfan syndrome : absence of ocular changes 3) Congenital contractual arachnodactyly : maifan syndrome의 feature외* 함께 contractures과 osteogenesis imperfecta의 일부 증후가 동반되는 경우 Congenital contractual arachnodactyly를 시사함

4. 치료 1)

Propranolol : 혈압강하로 대동맥 박리를 방지

2) Surgical correction :aorta, aortic valve, 3) Physical & emotional 4)

mitral valve

stress, pregnancy : cardiovascular risk를 높이므로 조심할 것을 교육

Scoliosis ( 1) > 20° : brace, physical

therapy

(2) >45° : surgery 5) Dislocated lens : 일반적으로 surgical removal은 필요하지 않지만 retinal detachment 주의 Harrison 20만,pp. 2975〜 2976

C 72020년 대비 PACIFIC

01

— S—

KMLE

순환기

____________________________________________________________ B

19세

_

남자가 6개월 전부터 계단이나 언덕을 오르면 숨이 차서 병원에

왔다. 혈압

120/50mmHg,

맥박

70회/ 분,

호흡

18회/ 분,

체온

이다. 가슴 청진에서 복장뼈 왼쪽 옆 세 번째 갈비 사이에서

36.5°C

3/6도의 확

장기 심잡음이 들린다. 가슴 사진과 손 사진이다. 진단은?

0 1 Left 3rd ICS 에서 들리는 확장기 심잡음만으 로도 대동맥판역류를 의심해볼 수 있습니다. 추가적으로 그림에서는 pectus carinatum,

arachnodactyly와 같은 골격계의 이상소견을 보여주고 있어 말판증후군을 의심해볼 수 있 겠습니다. 말판증후군에서 대동맥판역류가 잘 동반될 수 있습니다. Harrison 20판,pp. 2975~ 2976

1) 승모판협착 3)

대동맥판협착

5)

심방사이막결손

02 —

2) 대동맥판역류 4)

동맥관열림증



____________________________________________________________B

이 ' '

.

21 세 여자가 계단 오를 숨이 차서 병원에 왔다. 초등학교 때 양 눈에 인 공수정체 수술을 받았고 키 181 cm에 혈압 140/50mmHg, 맥박 86회/

0 2 Marfan 증후군입니다. 렌즈의 하방전위가 흔 하고 이로 인하여 백내장이 잘 발생합니다. 이 문제의 경우 백내장이 발생하여 양 눈에

분이었다. 복장뼈 왼쪽 세 번째 갈비 사이에서 확장기 심음이 들렸다. 진

인공수정체 수술을 받은 것으로 보입니다. 환

^■은?

자는 팔다리가 길고 키가 큽니다. 심혈관 기 형이 가장 중요한 사망 원인입니다. MVP,

1)

Coartation of aorta

3) PDA

4) Marfan syndrome

5) Takayasu’s disease

^ANSWER

2) Ebstein anomaly

01.② 02.④

MR, AR, aortic dissection, aortic rupture 등이 생길 수 있습니다. 복장뼈 왼쪽 세 번째 갈비사이에 들리는 확장기 심음은 새 입 니 다 . Harrison 2ᄋ1^ ,pp. 2975~ 2976



/



R -T ype

\

I ■■ R-Type I 순환기 R ty p e 은 각 주 요 진환들의 ° J 상0於 A 청 진 소 ^ 않고

^ 은

찌 며 진 소^ ,

형 적 0d 심 ^ 도 ,영 상 '7감사들을 진

私다야 형 니 다 . 문 제 에 주 어 진 정 보 가 않지 앞 을 73우 주 어 진 보 기 에 서 하 니 썩 r/o 해 나 기 는 과 73 을 통 해 문 제

에 껍 근 할 수 밖 에 없 는 데 이 매 74 질% 의 특 7강적인 정 에 대 해 서 않 고 1# 피야 하기 때 문 입 니 다 하 지 DJ; 문 제 가 너무 어협ᄀ비 출 제 되 지 는 ^ 기 에 겨 정 마 세 요 八八 아 래 문 제 를 푸 시 다 가 혹시 모 르 는 보 기 가 나 오 시 면 바 로 dH당 단 원 으 로 가 서 다시 학! ; 번 복 습 을 하시기 t i (■명 니 다 . Q foii 대헌; 문 제 도 자 주 다루어진니다ᅵ 각 ^% 에 서 어1헌 외 우 는 방 업 을 쎄 농 9jt■면 '것을 기 억 하 시 면 도움이 않이 되 실 거 에 요 . ^ 하 는 게 면 저 0』니 다 . 그 후 에 약의 종 류 가 앞 지 는

어 떤 진 %에 서 어^

을 쓰는지 ^ ᄀ11

약을 쓴다는

가불 기억

으니 약의 기 7i i ,적 용 증 ,금 기 에 대 해 서 정 리 6 H두 세 요 . 순환기

약 은 기 ^ H i 이 해 하 시 면 어1헌 정 우 에 7너야 하고 쓰 면 °J:되는지 ^ 1

수 9JL습 니 다 AA 2 0 1 9 ^ 에 는 R ty p e 36

문 제 중에서 7문 제 가 순 %기 에 서 출 제 되 었 습 니 다 . 2019년 문 제 들 을 가 7양 S t에 배 치 하 였 으 니 참 고 하 세 요 .

_______________________________ Questiori>A

V\>

각 문제에서 가능성이 큰 진단을 문제마다 지시하는 수만큼 답가지 중에 서 고르시오.

r

1) 폐렴

2) 심부전

3) 폐색전증

4) 공기가슴증

5) 폐동맥고혈압

6) 기관지확장증

7) 사이질폐질환

8) 만성폐쇄폐질환

01



72세 여자가 3개월 전부터 숨이 차다며 병원에 왔다. 빨리 걸으면 숨이

차고 가끔 새벽에 숨이 차서 잠을 깬다고 한다. 15년 전부터 혈압강하제 를 복용하고 있다. 비흡연자이다. 혈압 148/82mmHg,맥박 104회/ 분, 호흡 22회/ 분,체온 37.2°C이다. 가슴 청진에서 제3심음이 들리고 양쪽 등쪽 아랫부분에서 거품소리가 들린다. 양쪽 정강뼈앞 오목부종이 있다. 가슴 X선 사진이다. 혈액검사 결고I는 다음과 같다■(한 가지) 혈색소 11.8g/dU 백혈구 8, 900ten3(중성구 65%), 혈소판 320,000/W

0 1 호흡곤란에 대한 감별진단입니다. 호흡기질환 에 의한 것과 순환기질환에 의한 것을 감별해 야 합니다. 누워있을 때 숨이 찬 것, 청진에서 S3가 들리는 것 등은 심부전을 시사하는 소 견입니다. 가슴 X 선 사진에서는 양쪽으로 effusion이 보이고 폐혈관 음영이 증가해있으 며 좌심실의 비대소견이 보입니다. 정답은 2) 심부전입니다. 폐동맥고혈압은 조ᅡ심장에는 영 향을 미치지 않으므로^ 2) 대신 5)를 더 적절 한 답으로 고르기 위해서는 심실이 커져 있을 때 나타나는 S3 나 가슴 X 선 소견 등이 없어 야 합니다. Harrison 20민■,pp. 1763〜 1779

0(4 95

: €

: 2020년 대비 PACIFIC KMLE

순환기 I



0 2 -0 3



Jk u e s tim i^ A

2_

문제해설

각 문제에서 진단에 필요한 검사를 문제마다 지시히는 수만큼 답가지 중 에서 고르시오. 1) 24시간 홀터검사

2) 가슴 컴퓨터단층촬영

3) 관상동맥조영술

4) 심초음파검사

5) 오른심장카테터검사

6) 운동부하심전도검사

7) 혈장 D-이량체

8) 혈청 뇌나트륨이뇨펩티드

02

Q

68세 여자가 2개월 전부터 걸을 때 숨이 차다며 병원에 왔다. 혈압 124/70mmHg, 맥박 110회/ 분, 호흡 18회/ 분, 체온 36.3°C이다. 가슴

청진에서 빠른 심음과 제3심음이 들리고, 양쪽 등 아랫부분에서 거품소 리가 들린다. 정강뼈앞 오목부종이 있다. 가슴 X선 사진과 심전도이다. (두 가지)

ooo-

0 2 2달 전 시작된 호흡곤란을 주소로 내원한 고 령의 여자환자로, tachycardia, S3와 부종소 견을 보이고 있습니다. 이런 환자에서는 호흡 기질환,순환기질횐을 감별해야 하며,환자는 가슴 X 선 사진,그리고 심전도에 나타나는 좌 심실비대 소견(VI, 2에서 deepS wave를 보 이며,전반적으로 QRS오F 반대 방향으로 ST change가 일어남)을 통해 호흡기보다는 심장 의 문제일 것으로 생각되며 이를 확인하기 위 해 심초음파검사와 BNP측정을 해보는 것이 좋겠습니다. Harrison 20®[, pp.l763~1779

65세 여자가 1개월 전부터 숨이 차다며 병원에 왔다. 혈압 93/62mmHg,

맥박 122회/ 분, 호흡 26회/ 분, 체온 37.CTC이다. 가슴 청진에서 심음이 희미하게 들리고, 호흡음은 정상이다. 정강뼈앞 오목부종은 없다. 가슴 X선 사진과 심전도이다.(한 가지)

0 3 가슴 X선 사진상 심한 water bottle shape의 cardiomegaly를 보이며,호흡곤란 증상, 심음 의 감소,그리고 혈압의 감소로 심장 눌림증 이 의심됩니다. 심전도에서도 QRS complex 의 진폭이 매우 작고,크기가 커졌다 작아졌 다 번갈아 나타나는 electrical alterans를 보 이고 있는 것 또한 심장 눌림증에서 나타날 수 있는 소견입니다. 가장 먼저 심초음파로 확인해보아이^ 합니다. Harrison 2 0 ^ , pp. 1841〜 1847

^ n n r ~ r ᄀ



~ t~ tᄀ ' ᄀ 가

2020년 대비 PACIFIC KMLE

@ 11

순환기

04 〜 05

Q uestiorN w

문제해설

각 문제에서 진단에 필요한 검사를 문제마다 지시하는 수만큼 답가지 중 에서 고르시오. 1) 심초음파검사

2)

혈장 D-이량체

3) 혈청 트로포닌 T

4)

심장동맥조영술

5) 폐관류환기스캔

6)

가슴 컴퓨터단층촬영

7) 운동부하심전도검사

8) 혈청 뇌나트륨이뇨펩티드

□ 35세 남자가 4시간 전부터 가슴이 아프다며 병원에 왔다. 5 일 전에 몸

살 증상이 있었다고 한다. 통증은 지속적이었고 숨을 깊게 들이마시거나 기침을 하면 심해졌다. 혈압 122/72mmHg, 맥박 80회/ 분, 호흡 18회/ 분, 체온 36°C이다. 가슴 청진에서 심음과 호흡음은 정상이다. 혈액검사 결고느는 다음과 같다. 심전도이다.(두 가지) 혈색소 l4.0g/dL,백혈구 11,200/mm3,혈소판 174,000/mm3,혈액요소

질소/크레아티닌 14.1/0.8mg/dL,Na+/K+/C r 139/4.0/100meq/L

0 4 선행하는 감염증상이 있고,지속적이며,들숨 에 으|해 악호(•되는 흉통,심전도상 reciprocal change가 없는 아래로 볼록한 ST elevation 이 전반적으로 나타나 STEMI보다는 acute pericarditis가 의심되는 상황입니다. 심초음 파검사가 중요하겠습니다. 비교적 가능성이 낮아 보이지만,4시간 동안 심한 흉통이 지속 도I고 있으므로 반드시 심근경색증을 배제해 주어야 하고, 심전도 이외에 심근효소검人[를 해볼 수 있겠습니다. 따라서 답은 1),3)이 되 겠습니다. Harrison 2ᄋ만,pp.l841~1847

U98

ooa^

落.

0 5





R -T y p e

_ _

52세 남자가 6 개월 전부터 가슴이 아프다며 병원에 왔다. 통증은 2주에 1회 정도로 5층 계단을 오를 때 발생하고 약 3분간 지속된다고 한다. 30갑 년의 흡연자이다. 혈압 132/74mmHg, 맥박 70회/ 분,호흡 18회 / 분,체온 36.TC 이다. 가슴 청진에서 심음과 호흡음은 정상이다. 혈액

검사 결과는 다음과 같다. 심전도이다■(한 가지) 혈색소 13.3g/dL, 백혈구 6, 170/mm3,혈소판 171,000/mm3,혈액요소질

0 5 운동시에 발생하는 흉통으로 지속시간은 3분 정도로 짧고 통증의 빈도나 정도에 큰 변화가 없는 것으로 보아 안정협심증이 의심됩니다. 혈액 검사 결과 및 심전도에서는 특이소견이 없어 보입니다. 따라서 운동부하심전도검人['를 통해 운동시에 심전도의 허혈성 변호ᅡ가 나타 나는지를 확인해보아야 하겠습니다. Harrison 20ᅵ 만 ,pp. 185ᄋ〜 1866

소/크레아티닌 13.5/0.9mg/dL, Na+/K+/ Q ' 141/4.4/107meq/L

______________________________________ Q.U£sti.Q.ri>A

다음 문제에서 가능성이 큰 진단명을 문제마다 지시하는 수만큼 답가지 중에서 고르시오. 1) atrial flutter

2) atrial fibrillation

3)

ventricular parasystole

4) multifocal atrial tachycardia

5)

ventricular tachycardia

6) ventricular fibrillation

7) paroxysmal supraventricular tachycardia 8) atrioventricular junctional rhythm

06

__________________________________________B - B

Q_

58세 남자가 흉통을 호소하다가 갑자기 정신을 잃었다. 가장 가능성 있 는 심전도는?(두 가지)

0 6 흉통을 호소하며 갑작스런 의식소실을 보인 것 으로 MI에 으ᅵ한부정맥 또는 cardiogenic shock 을 생각해 볼 수 있습니다. MI는 심근 허혈, adrenergic tone의 증가,세포 내 전해질 불균형 등의 원인으로 부정맥이 유발되기 쉽습니다. 이 경우 다양한 부정맥들이 발생할 수 있는 데,의식이 소실되었다는 것을 통해 hemodynamic compromise를 유발 할 수 있는 VF, Polymorphic VT, monomorphic VT 등을 생각할 수 있습니다. 이 경우 빠른 electrical cardioversion이 필요하며 unsynchronized shock을 주지만,특히 monomorphic VT의 경우는 synchronized shock을 줍니다. Harrison 2 0 ^ , pp. 1757〜 1763

c - 2020년 대비 PACIFIC KMLE

순환기 I #

니 3 사 E

©

07 〜 08

QuestionNv8>

문제해설

-

각 문제에서 가능한 진단을 문제마다 지시하는 수만큼 답가지 중에서 고 르시오.

r

1) 심방빈맥

5) 심실성빈맥

2) 심방세동

6) 방실접합부빈맥

3) 심방조동

7) 가속심실고유리듬

4) 심실세동

8) 발작심실상성빈맥

11



54세 남자가 25분 전부터 가슴이 두근거리며 어지럽고 기운이 없어 응

급실로 왔다. 1주일 전에 가슴이 두근거리며 어지럼증이 생겨 주저앉았 으나 2 〜 3 분 뒤에 저절로 멈췄다. 3 년 전부터 걸을 때 숨이 차는 증상이 있었고 2 개월 전부터는 계단 한 층을 올라가기 힘들 정도가 되었다. 혈 압 84/59mmHg, 맥박 159회/ 분이다. 으ᅵ식은 명료하다. 심전도이다■(한 가지)

11

심전도가 나왔을 때,가장 먼저 서맥인지 빈 맥인지를 구분하는 것이 필요합니다. 빈맥성 부정맥이라면,QRS가 narrow한 지 혹은 wide 한지를 구분해야하며(QRS interval 짧 은 칸 3칸),또한 QRS 가 regular하게 나오는 지, 혹은 irregular하게 나오는 지를 구분해야 합니다. 이에 따라서 크게 4가지로 빈맥성 부 정맥을 분류하는 것이 가능합니다. 본 문제의 경우에는, QRS intervalOl 3칸 이상 으로 wide QRS이며,규칙적으로 QRS가 나타 나기 때문에,심실성 빈맥(ventricular tachycardia)임을 알 수 있습니다. 심실성 빈맥은 3개 이상의 VPC(ventricular premature complex)가 연속으로 나오는 것으 로 정의된다는 것을 잊지 마세요. Harrison 2Q만,pp.l750~1759

0

O

O—

€-

R -T ype

:#

r

12

문제해설

o

69세 여자가 1시간 전부터 가슴이 두근거리면서 가슴이 답답하다며 응

------------------------ cA) 12

급실로 왔다. 1년 전부터 두 달에 한 번 정도 두근거렸는데,1〜2시간 정도 지속되다가 멈추곤 하였다. 15년 전부터 혈압강하제를 복용하고

빈맥성 부정맥을 묻는 문제입니다. 심전도를 해석해보면, QRS intervalOl 3칸(0.12ms)이 하로 narrow QRS임을 알 수 있으며,QRS가 irregular하게 나온다는 것을 알 수 있습니다. 이런 경우에는 심방세동(Afib)과 다소성심방 빈맥(Multifocal atrial tachycardia)을 감별 해야 합니다. 심전도의 baseline을 보았을 때,

있다. 혈압 108/65mmHg, 맥박 118회/ 분이다. 의식은 명료하다. 심전 도이다■(한 가지)

자글자글 떨리는 것을 관찰할 수 있으므로, 심방세동으로 진단을 내릴 수 있겠습니다. 다소성심방빈맥의 경우,시험에 출제되지는 않 았지만,심방세동고1는 다르게 baseline이 떨리 지 않는다는 것을 감별점으로 생각해두시면 될 것 같습니다빈맥성 부정맥 정리해보겠습니다. 심전도 해석이 어려우신 분은 먼저 이렇게 크게

I

분류를 외워두시면 문제 푸는 것이 수월할 것입 니다. Narrow/Regular : PSVT, ST, AT, Aflutter Narrow/Irregular : Afib, MFAT Wide/Regular : VT Wide/Irregular : Afib with WPW, TdP Harrison 2 0 만,pp. 1756〜 1760

25mm/s 10mm/mV

_____________________________________QuestiorNfe

각 문제에서 필요한 조치를 문제마다 지시하는 수만큼 답가지에서 고르 시오. 1) 5% 포도당

2) 도파민

3) 농축적혈구

4) ^ ᅵ•트로핀

5) 노르에피네프린

6) 임시심장박동조율기 삽입

7) 기관^■관

8) 0.9% 식염수

13

Q

63세 남자가 2시간 전부터 어지러워 응급실로 왔다. 4시간 전부터 가슴

이 아프고 숨이 차다고 하였다. 혈압 85/50mmHg, 맥박 115회/ 분, 호 흡 25회/ 분, 체온 35.9도였다. 가슴청진에서 양쪽 등쪽 아랫부분에서 거 품소리가 들렸다. 중심정맥압 19cm H20 였다. 혈액검사소견은 [ᅡ음과 같았다.(두 가지) 혈색소 13g/dL,백혈구 8, 700/mm3,트로포닌 T 52ng/L( 참고치,1~14)

13

가슴이 아프면서 트로포닌이 상승한 경우 AMI를 가장 먼저 생각할 수 있습니다. 그러 나 이보다 환자의 혈압이 85/50이기 때문에 이에 대한 처치가 우선일 것입니다. 환자가 호흡곤란을 호소하고 있고 CVP 가 19라는 점 에서 AMI에 의한 Cardiogenic shock을 진 단할 수 있습니다. Cardiogenic shock에서는 심장의 수축력을 강화시키기 우|한 강심제와 함께 혈압 상승을 위해 승입제인 노르에피네프린을 투여할 수 있 습니다. 만일 익물치료로 교정이 되지 않는다 면 IABP 또한 고려해 볼 수 있으며,쇼크 교 정 후에 원인 질환에 대한 치료가 필요합니다. Harrison 2Q민;,pp. 2052~ 2059

O

0 (5 0 3 )

c -

2020년 대비 PACIFIC KMLE

( o X lM

순환기

M

Q uestiorN fil 2_J

문제해설 -0^0

각 문제에서 투여할 약제를 문제마다 지시하는 수만큼 답가지 중에서 고 르시오. 1)

모르핀

2) 헤파린

3)

아스피린

4) 베타차단제

5)

혈전용해제

6) 양성자펌프억제제

7)

비스테로이드소염제

8) 비디히드로피리딘 칼슘차단제

U

□ 45세 남자가 새벽에 가슴이 5분 정도 0 ® 다가 좋아지기를 두 번 반복

하고 1시간 후에 병원에 왔다. 1년 전부터 한 달에 한 번 정도 한밤중 또는 새벽에 가슴이 터질 것 같은 통증이 5분간 지속되었다가 저절로 소 실되었다. 술을 마시고 자다가 아파서 깬 적도 있다. 운동할 때는 가슴이 아프지 않았고 자세와도 상관이 없었다. 혈압 124/72mmHg, 맥박 79 회/ 분, 호흡 16회/ 분, 체온 36.6°C 이다. 병원에 와서 가슴 통증이 있을 때 찍은 심전도와 3분 뒤 가슴 통증이 가라 앉았을 때의 심전도이다.(한 가지)

14

활동과는 상관없이,술을 마신 다음 날 새벽 에 발생하는 가슴을 짓누르는 양상의 흉통은 특징적으로 변이협심증임을 알려주는 임상상 입니다. 변이협심증에서는 증상이 있을 때는 ST seg­ ment elevation을 보이며,흉통이 시리•진 뒤에 는 ST segment가 정상화되는 것을 관찰할 수 있습니다. Ergonovine을 이용한 관상동맥조영 술이 확진 검사입니다. 문제의 심전도 상에서 흉통이 았을 때,inferior lead인 lead II, III, aVF에서 ST segment ele­ vationOl 관찰되며,흉통이 사라진 뒤에는 정상 화되는 것을 관찰할 수 있습니다. 변이협심증의 치료는 크게 CCB오 卜Nitrate이 며,beta blocked 금기입니다. 문제의 보기에 서는 nonDHP CCB가 답입니다. DHP계열의 CCB는 주로 혈관 이완 작용이 있 는 항고혈압제로,amlodipine, nicardipine과' 같이 -dipine으로 끝나는 익제입니다. nonDHP 계열의 CCB는 주로 심장에 작용하는 항고혈압 제로 혈관이완작용은 부가적인 효과이며,심근 수축력과 수축 빈도를 주로 조절합니다. 만약 DHP계열의 CCB가 있었다면,더 적절한 답이 겠지만,보기에서는 nonDHP CCB만 있으므 로 이를 고르는 것이 가장 적절하겠습니다. Harrison 2Q만,pp. 1871〜 1872

25mm/s 10mm/mV

R -Type

r ----

Q s j ______________________________________________ 57세 여자가 6 개월 전부터 가슴이 아프다며 병원에 왔다. 1주일에 한

번 등산을 할 때 가슴을 짓누르는 듯한 통증과 호흡곤란을 느꼈고,10분 정도 앉아서 쉬면 없어졌다. 통증의 빈도가 증가하지는 않았다. 혈압 129/71 mmHg, 맥박 74회/ 분,호흡 16회/ 분,체온 36.6°C이다. 병원

계단을 올라오다가 가슴 통증이 생겨서 찍은 심전도와 5분 뒤 가슴 통증 이 가라 앉았을 때의 심전도이다■(세 가지)

15

6개월 전부터 운동 시에 가슴을 짓누르는 양 상의 흉통을 느낀 57세 여자 환자입니다. New onset, Resting pain, Crescendo pattem의 양상을 보이지 않기 때문에 불안정협 심증(Unstable angina)보다는 안정협심증 (Stable angina)에 합당한 증례입니다. 심전도를 보겠습니다. 흉통이 있을 때 심전도 상에서 lead V 4, V 5에서 저명한 ST segment depression이 소견이 관찰됩니다. 흉통이 사라 진 뒤에는 ST segment가 정상화되는 것을 관 찰할 수 있습니다. 안정협심증의 치료는 AB(C)N입니다. 따라서 3),4),8)을 답으로 고르는 것이 가장 적절하겠 습니다. 모르핀의 경우, STEMI에서 급성 치료 로 활용하는 진통제이며, 헤파린의 경우에는 Unstable angina, NSTEMI, STEMI에서 사 용합니다. 혈전용해제의 경우에는 STEMI에서 급성 치료로서 PCI가 하기 어려운 경우나 PCI 보다 아득이 았을 것으로 생각도[는 경우에 사용 하는 reperfusion therapy의 하나입니다. Harrison 2Q만,pp. 185ᄋ〜 1872

25mm/s 10mm/mV

0^-

2020년 대비 PACIFIC KMLE

순환기

문제해설

16〜 17

-0^0

각 문제에서 투여할 약제를 문제마다 지시히는 수만큼 답가지 중에서 고 르시오. 1) 0.9% 식염수

5) 아스피린(aspirin)

2) 도파민(dopamine)

6) 카르베딜롤{carvedil이)

3) 헤파린(heparin)

7) 테오필린(theophylline)

4) 도부타민(dobutamine)

8) 푸로세미드(furosemide)

16



68세 남자가 2주 전부터 걸을 때 숨이 찬다며 병원에 왔다. 6 년 전부터

고혈압으로 발살탄(valsartan)을 복용하고 있다. 혈압 122/71 mmHg, 맥박 118회/ 분, 호흡 23회/ 분, 체온 3 W C 이다. 가슴청진을 할 때 제3 심음과, 심장끝에서 3/6 도의 수축기 잡음이 들린다. 양쪽 등쪽 아랫부분 에서 거품소리가 들린다. 정강뼈앞 오목부종이 있다. 가슴 X선 사진과 심전도이다. 심초음파검사 결과는 다음과 같다.(세 가지) 왼심실박줄률 : 25%(참고치, >55)

1 6

2

주 전부터 시작된 숨이 차는 증상으로 내원

한 환자입니다. 심부전의 증상인 숨이 치는 증상과 함께 가슴 청진상 제 3심음 소견 그리 고 양쪽 등쪽 아랫부분에서 들리는 거품 소리 는 전형적인 심부전의 임상상입니다. 왼심실 박출률의 감소 또한 이를 뒷받침해줍니다. 문제에서 주어진 소견을 하나씩 살펴보겠습니 다. 심장끝에서 들리는 3/6도의 수축기 잡음은 승모판 역류가 동반되어 았음을 나타내며, 승모 판 역류와 함께 왼심실박출률 감소로 인하여 left sided backward failure로 인하여 폐부종 이 동반되어 거품소리가 들리는 것으로 이해할 수 있습니다. 정강뼈앞 오목부종은 right sided backward failure의 소견입니다. 시행한 기슴 X선 상에서는 양측 폐야의 음영이 증가된 소견에서 폐부종이 동반되어있음을 다 시 한 번 획인할 수 있으며,人I행한 심전도 상에 서는 narrow QRS와 함께 irregular한 양상을 보이는 심방세동을 관칠할 수 았습니다. 승모판 막협착에서 심방서I동이 잘 동반되기는 하지만, 이외에도 승모판 역류에서도 충분히 관찰될 수

aVR

있는 소견입니다. 치료를 살펴보면,현재 폐부종이 동반되어 호흡 곤란을 호소하고 있기 때문에 이에 대한 증상 완호[를 위해 furosemide가 필요하며,만성 심 부전의 예후 개선을 위해 carvedil이를 사용해 야 겠습니다. 그리고 동반된 심방 세동에 대하 여 CHA2SD2-VASC에 따라 2점 이상이므로 헤파린을 투여해 볼 수 있겠습니다. Harrison 2Q만, pp.l746~1750

一 ^ 06

o

aᅳ

c

17



고혈압으로 발살틴(valsartan)을 복용하고 있다. 혈압은 125/71 mmHg, 맥박 93회/ 분, 호흡 23회/ 분, 체온 36.6°C이다. 가슴청진에서 호흡음은 정상이다. 정강뼈앞 오목 부종은 없다. 가슴 X선 사진과 심전도이다. 심 초음파검사 결과는 다음과 같다.(한 가지) 왼심실박출률 : 30%(참고치,스55)

문제해설

IB] M: |

78세 남자가 4개월 전부터 걸을 때 숨이 차다며 병원에 왔다. 9년 전부터

:

17

R-Type

----------------------c/\)

심부전의 진단은 전형적인 증상과 징후,그리 고 박출률(Ejection fraction)의 감소를 바탕 으로 하게 됩니다. 본 증례에서는 걸을 때 숨 이 차는 증상과 함께 감소된 왼심실박출률을 통해 만성적인 경과의 심부전을 의심해볼 수 있겠습니다. 제시된 X-ray 상에서는 C/T ratio가 0.5보다 큰 cardiomegaly 소견이 관찰되고 있으며,심 전도 상에서 lead II에서 notched P wave와 lead Vi에서 P-terminal force 之0.04소견이 관 찰되어서 LA enlargement가 았음을 유추할 수 있습니다. 또한, lead V6에서 R>25m m 로 좌 심실비대가 았음을 유추해볼 수 있겠습니다. 오 래된 고혈압으로 인하여 심장 기능이 떨어졌음 을 추론해볼 수 있습니다. 만성 삼부전의 치료는 증상에 대한 치료와 예후 를 개선시카는 치료로 크게 나누어 볼 수 있는 데,본 환자의 경우에는 신체검진에서 특이시항 이 없고 걸을 때 호흡곤란을 호소할 뿐이기 때 문에 ftirosemide를 투여할 필요는 없습니다. 즉,증상에 대한 차료는 현재 필요하지 않은 상 황입니다. 다음으로 예후를 개선시키는 치료를 생각해봐 야 하는데, neurohumoral activation을 차단하 는 약제가 해당됩니다. SAS를 막는 beta blocker와 RAAS을 막는 ACEi 가 구체적인 약 물이 되겠습니다. 제시된 보기 중에서는 carvedilol0| 적절한 답이 되겠습니다. Harrison 2Q만,pp. 1769〜 1779

-O

O

(507

__________

2020년대비 PACIFIC KMLE

순환기



18〜 19

@1

QuestionN«Ml

니 i 문제해설 -c/\>

각 문제에서 필요한 검사를 문제마다 지시하는 수만큼 답가지에서 고르 시오. 1) 혈청 디곡신 농도

2) 혈청 갑상샘자극호르몬

3) 24시간 홀터검사

4) 심근관류스캔

5) 뇌컴퓨터단층촬영

6) 혈청 전해질

7) 심초음파검사

8) 혈청 크레아티닌

18 70세 남자가 3일 전부터 어지럽고 기운이 없어서 병원에 왔다. 입맛이

없고 머리가 아프다고 하였다. 심방잔떨림으로 디곡신과 아스피린을 복 용하고 있었다. 혈압 102/64mmHg, 맥박 42회/ 분, 호흡 18회/ 분, 체온 36.0도였다. 가슴청진에서 심음은 규칙적이었다. 심전도이다.(세 가지)



r

(508 o

u 「

—I



a -

t — W -4—



'•人 ^--**"-—

ᄂ^ ''

18

어지럼증, 기운 없음, 두통 등의 증상은 비특 이적인 증상이기 때문에 환자가 Afib으로 디 곡신을 복용하고 있다는 점에 주목해야 합니 다. 해리슨 18판부터 빠져있는 내용인데 갑작 스럽게 출제되었습니다. 디곡신 중독이 의심 되는 경우 serum digoxin level, serum potasium, serum magnesium, serum crea­ tinine level에 대한 측정이 필요합니다. 본문 에 있는 디지탈리스 중독 파트를 참고하세요. Cecil 25만,p. 700

^

O - M .

40세

남자가 3개월 전부터 어지러워 왔다. 무거운 물건을 들거나 갑자

기 움직일 때 어지럽다고 하였다. 혈압 112/70mmHg, 맥박 60회/ 분, 호흡 14회/ 분,체온 36.2도였다. 가슴 청진에서 복장뼈 왼쪽옆 세 번째 갈비 사이에서 3/6도의 수축기 심잡음이 들렸다. 심전도이다.(한 가지)

19

3개월 전부터 어지러운 증상을 주소로 내원한 40세 남자의 증례입니다. 이러한 증^^은 무거 운 물건을 들거나 움직일 때 더 심해진다고 하였습니다. 또한 복장뼈 왼쪽옆 세 번째 갈 비 人K>l에서 3/6도의 수축기 심잡음이 들리는 것은 VSD나 HCMP 등을 으ᅵ심할수 있는 소 견입니다. 임상상과 맞추어보았을 때,가장 합 당한 진단은 HCMP가 될 것입니다. 임상상과 맞추어서 설명해 보겠습니다. HCMP으| 경우, dynamic LV outflow track obstruction이 발생하기 때문에 어지러운 증 상을 호소할 수 있으며,실신의 임상상을 나 타낼 수도 있습니다. 이러한 dynamic obstruction은 preload가 감소한 경우 더 심하 게 발생하는데,무거운 물건을 들거나 갑자기 움직일 때(일어나서 움직이는 경우) preload 가 감소할 수 있습니다. 또한,HCMP의 경우 복장뼈 왼쪽옆 세 번째 갈비 사이 공간에서 수축기 심잡음이 들립니다. 마지막으로 심전도를 해석해보면,lead V4, V5, V6에서 T wave inversion 소견을 관찰 할 수 있고 V5에서 R wave가 25mm 이상 이므로 LVH 소견이 관찰됩니다. HCMP에서 LVH 소견의 심전도가 관찰되는 것과 일치합 니다. 따라서,systolic anterior motion을 관찰하 고,dynamic obstruction 소견과 함께 혈역 학적인 관찰을 위해 심초음파를 시행해 볼 수 있겠습니다.

Harrison 20만,pp. 1793〜 1796

C

7 2020 년대비 PACIFIC KMLE

순환기

@ ) l 2 0 -2 1

■ Q u estio jD b A

문제해설

V 느

각 문제에서 가능성이 큰 진단을 문제마다 지시하는 수만큼 답가지에서 고르시오. 1) 공기가슴증

2) 대동맥박리증

3) 가슴막염

4) 폐색전증

5) 불안정협심증

6) 급성심근경색증

7) 변이협심증

8) 역류식도염

20

B •回•(!)

40세 남자가 2주전부터 새벽에 가슴이 아프다며 병원에 왔다. 주로 술

마신 [ᅡ음 날 새벽에 가슴이 타는 듯하여 잠을 깼다고 하였다. 건물 2층 계단을 올라가도 통증은 생기지 않았다고 하였다. 비흡연자였다. 혈압 120/80mmHg, 맥박 68회/ 분,호흡 18회/ 분, 체온 36.5도였다. 가슴 X

선 사진과 심전도이다.(두 가지)

2 0 흉통의 감별진단에 관한 문제입니다. 계단을 올라가도 통증이 생기지 않는다는 점,심전도 소견이 정상이라는 점에서 변이협심증을 제외 한 허혈성 심장질환은 배제할 수 있습니다. 주어진 가슴사진에서 이상 소견이 없으므로 공기가슴증,대동맥박리증,가슴막염 등은 배 제 가능합니다. 주로 새벽에 0[•프고 술 마신 다음날 아 프 므 로 변이협심증과 역류식도염을 감별진단으로 생각해볼 수 있습니다. Harrison 20°}-, pp.73~ 80, 18기〜 1872

51^0

O

B

53세 남자가 1시간 전부터 가슴이 아프다며 병원에 왔다. 통증은 왼쪽목

과 왼팔로 뻗친다고 하였다. 40갑 • 년 흡연자였다. 혈압 120/80mmHg, 맥박 93회/ 분,호흡 20회/ 분, 체온 36.5도였다. 가슴 X선 사진과 심전 도이다.(한 가지)

—-------------------- c A ,

1T IS ]. 21

왼쪽 목과 왼팔로 방사되는 흉통이 있[ᅵ는 점 에서 ischemic heart disease를 으ᅵ심할수 있 는 상황입니다. 심전도를 보았을 때 !!,I , aVF lead에서 ST elevation이 보이고 있는 것으로 보아 inferior wall STEMI라고 진단 할 수 있습니다. Harrison 20만,pp.l872~1885

2020년 대비 PACIFIC KMLE

순환기 I #

____________________________________________Q uestion^니Si

각 문제에서 필요한 치료를 문제마다 지시하는 수만큼 답가지에서 고르 시오. 1) 심장동맥중재술

2) 베타차단제

3) 도파민

4) 니트로푸루시드

5) 고리작용이뇨제

6) 0.9% 식염수

7) 도부타민

8) ^ᅡ스피린

22

— n_J__________________________________________EHS.El. 63세 남자가 30분 전부터 가슴이 아파서 왔다. 안절부절 못하는 상태로

식은땀을 흘리고 있었다. 40갑 • 년의 흡연력이 있었으며,10년 전부터 불 규칙하게 혈압강하제를 복용하고 있었다. 혈입(오른쪽팔) 180/110mmHg, 맥박 85회/ 분,호흡 23회/ 분, 체온 364 도였다. 왼쪽 노동맥의 맥박이 잘 만져지지 않았다. 가슴청진에서 심음과 호흡음은 정상이었다. 가슴 X 선 사진 및 심전도이다. 혈액검사 결과는 다음과 같았다.(두 가지) 트로포닌 T 6 ng/L(참고치,0〜14)

2 2 고혈압의 고ᅡ거력을 가지고 있고 불규칙하게 혈압강하제를 복용하여 현재 혈압이 180/110 으로 높은 상황입니다. 고혈압 환자에서 갑자 기 가슴통증을 호소하는 경우 가장 먼저 aortic dissection을 의심해보아야 합니다. chest X-ray 상 mediastinal wideningᄋ| 보ᄋ|는 소 견, flap에 의해 왼쪽 팔로 가는 혈류가 막혀 왼쪽 노동맥의 맥박이 잘 안 만져자는 소견 또 한 aortic dissection을 시사하고 있습니다. 트 로포닌이 상승하지 않은 점, 심전도에서 ST segment가 정상인 점에서 ischemic heart disease는 배제 가능합니다. Aortic dissection의 경우 바로 ICU 입원 후 혈압 강하치 료가 필요하며 혈압약의 TOC는 beta block­ er + sodium nitroprusside 또는 labetal이입 니다. Harrison 20만,pp.l919~1922

i

avK

-^^*1—

—I, aVL

I

/ ᄂ

(512)0

vi



___A—

R -Type

23

Q T;

56세 남자가 4시간 전부터 가슴이 아파서 왔다. 30갑 • 년의 흡연력이

있었다. 혈압 128/74mmHg, 맥박 74회/ 분, 호흡 22회/ 분,체온 36.7 도였다. 가슴 청진에서 거품소리는 들리지 않았고 심장끝에서 제3심음 이 들렸다. 가슴 X선 人卜진 및 심전도이다. 혈액검사 결과는 다음과 같았 다■(세 가지) 트로포닌 T 48ng/L(참고치,0~14)

: 그I

I #

문제해설 ------------------------cA> 2 3 환자의 임상증상과 심전도 소견,트로포닌이 상승한 lab 소견을 보았을 때 V2, V3, V4, V5 lead에서 ST elevation을 보이는 ant. wall STEMI 인 것을 알 수 있습니다. 본문 에 기술한 바에 따르면 STEMI의 경우 재관 류 치료가 필요하며,생존율을 높일 수 있는 약물인 ABA(Aspirin, Beta blocker, ACE inhibitor)를 시용할 수 있습니다. 따라서 보 기 중에서 1 ,2’ 8번 치료가 가장 적절합니다. 실제 국시에서 폐부종을 의심할만한 chest X-ray가 제시되었고 제3심음이 들렸다는 점 에서 급성 심부전에 대한 치료도 해아하지 않 을까 생각할 수 있지만,원인이 있다면 원인 교정을 위한 치료가 우선시되어야 하므로 AMI에 대한 치료를 우선적으로 시행한 후 심부전이 지속되면 이에 대한 치료가 필요합 니다. Harrison 2ᄋ만,pp. 1872〜 1885

1 5

•J

3

o o

2020년 대비 PACIFIC KMLE

@ 11

순환기

2 4 -2 5

Q u e stio n ^ A

l

니^

각 문제에서 가능성이 큰 진단명을 문제마다 지시큰ᅡ는 수만큼 답가지 중 에서 고르시오. 1)

폐심장증(c o r

pulm onale)

2) 고혈압심장병

3) 동맥관열림증

4) 승모판협착증

5) 비대심장근육병

6) 심방사이막결손

7) 대동맥판폐쇄부전증

8) 폐동맥판폐쇄부전증

2U 47세 여자가 8개월 전부터 걸을 때 숨이 차서 왔다. 가슴 통증은 없었

다. 혈압 110/84mmHg, 맥박 82회/ 분,호흡 21회/ 분, 체온 36.8도였 다. 가슴 청진에서 심장끝에서 3/6도의 확장기 잡음이 들렸다. 가슴 X선 사진과 심전도이다.(한 가지)

2 4 Apex에서 이완기 잡음이 들립니다. Apex에 서 잘들리는 심잡음은 Mitral valve에서 기원 하는 MS와 MR입니다. 이중 이완기에 들리 는 잡음은 Mitral stenosis입니다. ① AS : right upper sternal border ② AR : left lower sternal border(3rd ICS) ③ PS : left upper sternal border ④ MS, MR : apex ⑤ TR : left lower stenal border(4th ICS) 각 심잡음이 들리는 위치입니다. 기억합시다. 여기에 더하여 심전도 상에서는 심방세동 (AF)이 보이고 있습니다,MS에서는 심방세 동을 자주 동반합니다. 또한 chest X-ray에서 는 LA의 확장으로 LA appendage부위가 튀 에4와 left cardiac border가 평평하게 보입 니다. Harrison 20:만,pp. 1813〜 1818

V5

R -Type

53세 남자가 4개월 전부터 걷을 때 숨이 차고 가슴이 아파서 왔다. 혈압 142/94mmHg, 맥박 85회/ 분,호흡 23회/ 분,체온 36.5도였다. 가슴 청

진에서 제3심음이 들렸으며 심장끝에서 3/6도의 수축기 잡음이 들렸다. 가슴 X선 사진이다■(두 가지)

2 5 호흡곤란과 anginal pain을 보이는 질환은 많 은 순환기 질환들의 특징입니다. 심장 끝에서 수축기 잡음이 들리는 것으로 보아 MR이 있 음을 알 수 있습니다. 보기에서 나오는 판막질환과 심장의 구조적 질환들은 잡음의 패턴이 다르게 나와야합니 다. 폐심장증의 경우 X ray에서 폐에 이상이 보여야 하나 X ray상 크게 폐의 이상이 보이 지는 않습니다. 순환기 질환 중 호흡곤란과 Anginal pain을 보이는 대표적 질환이 심근질환입니다. 특히 비대 심장근육병은 MR을 동반하는 경우가 많 다고 알려져있습니다. 고혈압 심장병 또한 비 대 심장근육병과 비슷한 형태로 나타납니다. Harrison 2 0 ^ , pp. 1818~ 1821

Questiori>A

각 문제에서 필요한 검人*

문제마다 지시하는 수만큼 답가지 중에서 고

르시오. 1) 항핵항체

2)

콩^■생검

3) 혈액배양

4)

가슴 고해상컴퓨터단층촬영

5) 혈액펴바른표본

6)

심초음파검사

7) 소변 단백질전기이동검사

8)

혈장 D-이랑체

rz

26

Q

64세 여자가 2주 전부터 열이 나서 응급실에 왔다. 혈압 104/62mmHg,

맥박 104회/ 분,호흡 22회/ 분,체온 38.4도였다. 가슴 청진에서 호흡음 은 정상이었고 심장끝에서 수축기 심잡음이 들렸다. 검사 결고I는 다음과 같았다.(두 가지) • 혈액 : 혈색소 K ^g /d U 백혈구 13, 20Q/mm3, 혈소판 125,000/mm3,혈

액요소질소/크레아티닌 25/1.4mg/dL • 가슴 X 선 사진 : 정상 • 소변 •. 잠혈( ᅳ),단백질( ᅳ),적혈구 5〜 10/고배율시야, 백혈구 1〜 3/

고배율시야

2 6 일단 열과 심잡음이 들립니다. 바로 감염 심 내막염을 의심합시다. 진^•은 1) 혈액배양 상 ‘ 양성 2) 심^음파 상 ■ 심내막 침범 소견 입니다. Harrison 2Q만,pp. 921~ 933

2020년대비 PACIFIC KMLE

순환기



27 〜 28 각 문제에서 필요한 검人»

QuestiorN&i p

문제해설 -cA ,

문제마다 지시하는 수만큼 답가지 중에서 고

르시오■ 1) 발목위팔지수

2) 심초음파검사

3) 운동부하검사

4) 맥박파전표육도

5) 혈장 D-이랑체(D-dimer)

6) 24시간 홀터검사

7) 가슴 컴퓨터단층촬영

8)

뇌타트륨이뇨펩티트(BNP)

27

B

74세 남자가 3개월 전부터 숨이 차서 왔다. 4일 전부터는 자다가 숨이

차서 일어났다고 하였다. 20년 전부터 혈압강하제와 혈당강하제를 복용 하고 있었다. 35갑 년의 흡연력이 있었다. 혈압 142/72mmHg,맥박 90회/ 분,호흡 22회/ 분,체온 36.6도였다. 가슴 청진에서 제3심음이 들

렸다. 가슴 X선 사진과 심전도이다.(두 가지)

'516)

o o-

2 7 호흡곤란은 호흡기질환과 순환기질환에서 공 통으로 보일 수 있는 증상이다. 이때는 다른 증상이나 고卜거력,기본 X-ray, ECG 등으로 판단하는 것이 중요하다. 해당 환자는 고혈압을 가지고 있고, 흡연력이 있다. 그리고 가슴 청진상 제 3심음이 들린다. 이중에서도 청진상 제 3심음은 심장의 문제임 을 시사한다. 실제 X-ray상은 폐쪽에는 이상 이 없어보인다. 심장의 문제임을 확인하기 위해 심초음파와 BNP를 측정해 보는 것이 좋다. Harrison 20^, pp. 1763~ 1779

B

45세 여자가 하루 전부터 숨이 차서 왔다. 2일 전 하루 종일 쪼그리고

앉아 밭일을 했다고 하였다. 혈압 92/74mmHg, 맥박 85회/ 분, 호흡 23 회/ 분, 체온 36.5도였다. 양쪽 목정맥이 늘어나 있었다. 가슴청진에서 복장뼈 왼쪽 옆 네 번째 갈비 人띠ᅵ에서 3/6도의 수축기 잡음이 들렸고, 호흡음은 정상이었다. 왼쪽 종아리가 붓고 열감이 있었다. 가슴 X선 사 진과 심전도이다.(세 가지)

-0^0 2 8 호흡곤란을 호소하면서 하루종일 앉아있었으 므무 pulmonary embolism을 가장 먼저 의 심해야한다. 왼쪽 종아리가 붓고 열감이 있는 것 또한 DVT를 人|사하면서 pulmonary embolism을 의심하게 한다. 또한,왼쪽 옆 네 번째 갈비 人ᅡ이에서 수축기 잡음은 심장의 구조적 이상을 시사한다. 그래서 pulmonary embolism에 대한 검사로 D-dimer & CT검사가 필요하고,심장의 구조 적 이상을 확인하기 위해 심초음파 검사가 필 요하다. Harrison 20 만, pp. 1910~ 1917

2020년 대비 PACIFIC KMLE

순환기

•":

문제해설

29 〜 30

@1

각 문제에서 필요한 처치를 문제마다 지시하는 수만큼 답가지 중에서 고 르시오. 1)

베타차단제(beta blockers)

2)

티 아 지 드

3)

고리작용이뇨제(loop diuretics)

(thiazide)

이 뇨 제

4)

칼 륨 보 전 이 뇨 제

5)

앤 지 오 텐 신 전 환 효 소 억 제 제

(K + -sparing diuretics) (ACE inhibitors)

6)

앤 지 오 텐 신

(angiotensin receptor blockers, ARBs)

수 용 체

차 단 제

(dihydropyridine) 칼 슘 통 로 차단제

7)

디 히 드 로 피 리 딘

8)

비 디 히 드 로 피 리 딘

(non-dihydropyridine) 칼 슘 통 로 차단제

29



70세 여자가 3일 전부터 기운이 없어서 병원에 왔다. 한 달 전부터 항고

혈압제를 복용하였다. 맥박은 34호I/분이었고 규칙적이었다. 심전도이다.

2 9 혈압강하제를 복용한 환자에서 나타난 동서맥 (sinus bradycardia) 입니다. 증상까지 동반되 었으므로 sick sinus syndrome이군요. 위의 약제들 중 심한 서맥을 유발할 수 있는 익제는 Class II(BB)와 Class F(non-DHP CCB)

혈액검人卜 결고[는 다음과 길았다.(두 가지) BUN/Cr 14/1.0mg/dL, Na+/K+ 140/4.2meq/L

항부정맥제입니다. 위의 두 약제들은 SA node오 卜AV node의 흥분성을 낮추어 서맥을 유발합니다. Vi lead에서 나타난 토끼 귀 (RSR’) 모양의 심전도는 RBBB를 시사하는 소견으로서,역시 AV node의 흥분성이 저하 되었을 가능성에 힘을 실어 줍니다. 이뇨제는 hypovolemia를 유발하므로 서맥보 다는 빈맥이 나타났을 것입니다. ACEi, ARB는 고칼륨혈증이 주 증상입니다. DHP CCB는 혈관이완 작용이 주로 나타나고 심 장에는 별로 영향을 미치지 않지요. Harrison 20만,pp. 1722〜 1727

^ —> ----------- ---——

v_ 25mm/sec 10mm/mv

o a

I

R-Type

문제해설

____________________________________________ B i g - B , 50세 남자가 이틀 전부터 기운이 없어서 병원에 왔다. 두 달 전부터 항

고혈압제를 복용하였다. 맥박은 74호1/분였고 규칙적이었다. 심전도이다. 혈액검人卜 결고는 다음과 같았다■(두 가지) BUN/Cr 20/1.5mg/dL, Na+/K+ 139/2.5meq/L

9

------------------------ c A , 3 0 심한 저칼륨혈증과 그로 인한 피로감을 동반 한 환자입니다. 심전도에서도 저칼륨혈증의 특징인 낮은 T파가 나타납니다. 보기 중 저칼 륨혈증을 유발할 수 있는 약제는 이뇨제,그 중에서도 Loop diuretics와 Thiazide diuretics입니다. 이 문제와는 반대로,고칼륨혈증을 유발하 는 이뇨제로는 칼륨보존이뇨제(K +- Sparing diuretics), ACEi, ARB가 있습니다. Harrison 2Q만,pp. 1722〜 1727

Q u e s tL o n ^ M

다음 문제에서 가능성이 큰 진단명을 문제마다 지시하는 수만큼 답가지 중에서 고르시오. 1) sinus tachycardia

2)

sick sinus syndrome

3) Torsades de pointes

4)

atrial flutter

5) atrial fibrillation

6)

ventricular premature beat

7) ventricular tarchycardia

8)

ventricular fibrillation

r 31



55세 여자가 자다가 왼쪽 팔과 다리의 마비 증상을 보여 내원하였다. 3

년 전부터 운동 중에 호흡곤란 증상이 발생하였다. 심장박동은 불규칙했 고 제1심음이 항진되어 있었으며 심첨부에서 확장기 심잡음이 들렸다. 이 질환에서 가장 흔히 관찰되는 심전도 소견은?(한 가지)

31

갑자기 오른쪽 팔과 다리를 움직일 수 없는 것은 stroke를 시사합니다. atherosclerosis에 의한 thrombosis일 수 있고 또는 어디가 embolic source가 있어 embolism이 생긴 것도 추측해 볼 수 있겠네요 또한 심장 박동이 빠르 고 불규칙하며 apex에서 확장기 심잡음이 생기 는 것은 MS에 합당합니다. 이 경우 가장 가능 성이 있는 심전도는 atrial fibrillation입니다. Harrison 2Q만,pp. 1746〜 1750

O

0 (5 1 9

2020년 대비 PACIFIC KMLE ;

순환기 그 포

32 〜 33

@ |

Q y £ S .tiQ n > A

jZ

문제해설

각 문제에서 가능성이 큰 진단명을 문제마다 지시하는 수만큼 답가지 중 에서 고르시오. 1) 동정지

2) 동방차단

3) 심방세동

4) 심방조동

5) 심실세동

6) 1도방실차단

7) 2도방실차단

8) 완전방실차단

32 70세 여자가 어지럼이 반복되어 왔다. 혈압 110/70mmHg, 맥박 97회/

32

분,호흡 18회/ 분, 체온 36.7도였다. 실신 당시의 심전도 모니터링이다. (두 가지)

興 25mm/sec |

33

심전도를 보면 우선 동정지를 획인할 수 았습 니다. 동방차단의 경우 간헐적인 전도 실패로 나타나야하며 휴지기의 PP간격이 정상 PP의 배수가 도는 것으로 획인할 수 있지만 현재 심 전도는 더 이상의 전기활성이 나타나지 않는 것으로 보아 동정지에 더 합당합니다. 동정지가 나 ᅡ 기 전의 심전도를 보면 QRS 파가 불규 칙하게 나타나며(불규칙한 RR간격), baseline 또한 흔들리고 있습니다. 따라서 AF임을 알 수 있습니다. Harrison 20만,pp. 1722〜 1733,1746〜 1750



58세 남자가 어지럼이 반복되어 왔다. 혈압 116/74mmHg, 맥박 45호I/

분,호흡 20회/ 분,체온 36.8도였다. 심전도이다.(한 가지)

.j., ,

j



-A___



^ X /니

. i y ' c 국!남i jy v

그 -

L짜 25mm/sec f

33

먼저 HR 45회로 bradycardia입니다. 이때 p wave와 QRS complex가 각각 따로 일정하게 나타나는 것을 확인할 수 있습니다. 따라서 complete AV block입니다. HR 45회이고 narrow QRS파이므로 junctional escape beat(HR 40〜 60)임을 알 수 있습니다. 현재 어지럼증의 증상이 있으므로 심장율동조율기 삽입이 필요합니다. Harrison 2Q만,pp. 1727〜 1733

9 -

■ ,

벨Fᅳ

34 〜 35



R -Type

문제해설

Questiori^A

각 문제에서 필요한 처치를 문제마다 지시하는 수만큼 답가지 중에서 고 르시오. 1)

비스테로이드소염제

2) 베타차단제

3)

질산염

4) 칼슘통로차단제

5)

혈전용해제

6) 헤파린

7)

양성자펌프억제제

8) 신경 차단술

45세 남자가 2일 전부터 가슴이 아파서 병원에 왔다. 통증이 운동고는

34

관련이 없었으나, 기침을 하거나 상체를 움직일 때 더욱 심해졌다. 혈압

병력상 근 골격계 질환에 의한 흉통이 으ᅵ심됩 니다. 따라서 단순 해열 진통제인 NSAID를 人[용합니다. Harrison 2Q민;,pp. 73~ 81

114/76mmHg, 맥박 82회/ 분, 체온 367 도였다. 오른쪽 제 4,5 갈비연

골 관절부위에 압통이 있었다. 가슴청진에서 심음과 폐음은 정상이었 다■(한 가지)

35 55세 남자가 2개월 전부터 발생한 한밤 중 및 새벽에 심해지는 가슴 통

증으로 병원을 방문하였다. 통증은 5분 정도 지속되었고, 운동과는 관련 이 없었다. 통증이 심하여 응급실 방문했을 때 니트로글리세린을 혀밑에 넣으니 증상이 빠르게 없어졌다. 혈압 134/78mmHg, 맥박 80회/ 분, 체 온 36.8도였다X두 가지)

35

운동에 의해서 악화되지는 않으나, 니트로글 리세린으로 완화되는 Anginal pain을 통해 변이협심증을 의심할 수 있습니다. 변이협심 증의 치료로 질산염과 칼슘통로차단제를 쓸 수 있습니다. 베타 차단제는 금기입니다. Harrison 2Q만,pp. 1817〜 1872

2020년 대비 PACIFIC KMLE

순환기 I #

36 〜 37

m 각

문 제 에 서

르 시 오

1)

■■■Quest ionN&i 2_J 필 요 한

처 치 를

문 제 마 다

지시초ᅡ는 수 만 큼

답 가 지

-



.

심 장

초 음 파

2)

심 장

효 소

3)

홀 터

검 시

4)

심 장

자 기 공 명 영 상

5)

심 장 동 맥 조 영 술

6)



검 사

검 사

{Holter monitoring) (cardiac MRI)

A

각 문제에서 필요한 처치를 문제마다 지시하는 수만큼 답가지 중에서 고 르시오. 1)

라미프릴(ramipril)

2) 라베타롤{labetal이 )

3)

로살탄(losartan)

4) 베라파밀(verapamil)

히드랄라진(hydralazine)

6) 프라조신(prazosin)

5)

_

7)

스표|로놀락® spironolactone)

8)

히드로클로로티아지드(hydr(x:hlorothiazide)

38 우

_______________________ ■___________________________________B

0 •回 ■

30세 남자가 6개월 전부터 머리가 아파서 왔다. 응급실에 도착해서 의

식이 흐려졌다. 혈압 200/142mmHg,맥박 90회/ 분,호흡 24회/ 분, 체 온 36.7도였다. 뇌컴퓨터단층촬영 결고ᅡ는정상이었다. 검사 결고h는 다음 과 같았다(한 가지). • 혈액 : 혈액요소질소/크레아티닌 l4/1.3mg/dL • 소변 : 잠혈(1+), 단백질(1 +),적혈구 2〜 3/고배율시야

3 8 6개월 전부터 있었던 두통과 의식 소실을 주 소로 내원한 환자로 혈압이 200/142, 뇌 CT 상 이상이 없었다는 점에서 고혈압으로 인한 ischemic stroke을 의심해볼 수 있습니다. 또 한 소변에서 잠혈 1 + ,단백질 1 + 로 신장 손 상이 동반되고 있음을 알 수 있습니다. 이는 고혈압의 응급에 해당하며 stroke 치료로는 Labetalol, Nitroprusside, Nicardipine을 사 용할 수 있습니다. Harrison 2Q만,pp. 1905~ 1906

'r

I F

2020년 cHH! PACIFIC KMLE

5 환기

39 〜 40

-fluestiofhA

문제해설 -c/\>

각 문제에서 필요한 처치를 문제마다 지시하는 수만큼 답가지 중에서 고 르시오. 1) 심폐소생술 2) 관동맥성형술 3) Nitrate

정맥내주입

4) Lidocaine

정맥주사

5) Adenosine 정맥주사 6)

제세동(defibrillation)

7)

임시 심장박동조율기(temporary pacemaker) 삽입

8)

대 동 맥 내 풍 선 맞 박 동

A t

문제해설 만,pp. 1881 〜 1885

^

2020년대비PACIFIC KMLE

순환기 그 룰

54〜 55



QuestiorhA

문제해설

각 문제에서 가능성이 큰 진단명을 문제마다 지시하는 수만큼 답가지 중 에서 고르시오. 1) AR

2) AS

3) MR

4) MS

5) TR

6) TS

7) VSD 9) PDA

8) ASD 10) H-CMP

a - 0

64세 남자가 1개월 전부터 빨리 걸으면 가슴통증이 생겨 병원에 왔다.

복장뼈 오른쪽 두 번째 갈비뼈 사이에서 수축기 심잡음 IV/VI이 들렸고, 목동맥으로 전파되었다. 진단은?(한 가지)

5 4 복장뼈의 오른쪽 위에서 들리는 수축기 심잡 음은 AS일 가능성이 높습니다. AS에서 valve level의 stenosis인 경우 심잡음이 2nd right intercostal level에서 잘 들리며 left carotid artery쪽으로 방사됩니다. Severe obstruction인 경우 대부분으ᅵ 환자에서 심잡 음은 H/VI 이상입니다. 이 증례에서는 운동 시 흉통이 생기는 것으로 보아 합병증으로 협 심증이 생긴 것을 의심해 볼 수 있습니다. Harrison 20만,pp. 1802—1809

55



58세 남자가 호흡곤란을 주소로 내원하였다. 17년 동안 만성기침, 가래

가 있었고 4개월 전부터 다리부종이 발생하였다. 복장뼈 우하연에서 수 축기 심잡음이 들렸고, 이는 명치로 전파되었다. 심잡음은 들숨 시 증가 하였고, 경정맥 확대와 간비대가 있었다. 진단은?(한 가지)

5 5 폐성심이 있는 환자에서 생긴 마 일 가능성이 높습니다. 병력을 요약하면 COPD로 추정되 는 병력,우심실부전, 복장뼈 우하연 수축기 심잡음,흡기시 심잡음 증가입니다. 수축기 심잡음이 있다는 점에서 MR, TR, VSD, AS, HCMP를 생각할 수 있습니다. 흡 기시 심잡음이 증가만, pp. 1823〜 1826

O

o

56 〜 57



___________________________________________Q u s s .t io .n b A

각 문제에서 가능성이 큰 진단명을 문제마다 지시하는 수만큼 답가지 중 에서 고르시오. 1) 심방사이막결손

2) 방실사이막결손

3) 대동맥축착

4) 폐동맥판막협착

5) 대동맥판막협착

6) 대동맥판막역류

7) 승모판협착

8) 승모판역류

39세 여자가 1년 전부터 시작된 호흡곤란으로 왔다. 심첨부에서 전수축

기 심잡음이 들렸고,왼쪽 겨드랑이로 방사되었다.(한 가지)

5 6 위 문제와 증례가 비슷하죠? 전수축기 심잡음 이고 심첨부에서 들리며 숨이 차다는 점, 왼 쪽 겨드랑이로 방사된다는 점 모두 MR을 시 사합니다. ‘만성의 심한 승모판 역류 환자에서 제1심음 은 수축기 잡음에 포함되어 들리거나 감소되 어 있다. 3/6도 이상의 수축기 심잡음은 승모 판 역류의 가장 특징적인 청진소견으로 S !에 서 시작하여 S2까지 연결되는 온수축기 잡음 이다. 높은 음으로 심첨부에서 가장 크게 들 리고,왼쪽 겨드랑이나 등의 견갑골 사이로 방사되는 특징이 있다.’ Harrison 2Q만,pp. 1818〜 1821 심장학교과서,대한순환기학회,pp.346〜 350

r —

57 58세 남자가 1년 전부터 시작된 운동 중 호흡곤란을 주소로 내원하였

다. 흉골 우상연에서 제1심음 이후에 심잡음이 들렸고 이 잡음은 점점 커지다가 작아졌다.(한 가지)

5 7 심잡음의 우ᅵ치가 복장뼈 상부 오른쪽 가장자 리라는 것이 중요합니다. 대동맥 판막에 의한 심음이 들리는 위치이죠. 제1 심음 이후 들렸 으므로 수축기 잡음이고 커졌다 감소했다는 것을 볼 때 대동맥판 협착에 의한 심음입니다. AS에 의한 심음은 Si 직후 시작하여, systolic ejection 중간까지 소리가 커졌다 감소하죠. 오른쪽 두 번째 갈비뼈 人f이 공간(2nd ICS)에 서 가장 잘 들리며 carotid a•를 따라 방人[됩 니다. 50세 남자리는 것도 중요한 정보인데, A S의 가장 흔한 원인은 가령에 의한(age re­ lated) 토 ᅵ 행성 석회호K)|고 남자에 많습니다. Harrison 20]3', pp. 1802〜 1809

순환기

2020 년 대비 PACIFIC KMLE

m \ 5 8 -5 9

QuestiQrL:*^in

문제해설

각 문제에서 가능성이 큰 진단명을 문제마다 지시하는 수만큼 답가지 중 에서 고르시오. 대동맥박리

2) 변이협심증

이 혀□ 시o 즈 1_1■처 o t=i

4) 불안정협심증

■3■ 서 시 그 ;것 새 즈 bd o 口!_ O ~IO

6) 대동맥판 협착증

급성 심장막염(pericarditis)

8) 기슴막염(pleuritis)

갈비연골염(costochondritis)

58 조깅을 할 때 I



흉통이 있었던 63세 남자가 2 일 전부터는 천천히 걸

을 때에도 심한 흉통이 나타나 내원하였다.(한 가지)

5 8 stable anginal pain이 의심되는 고|•거력을 가 지고 있던 환자가 3일 전부터 진행히는 흉통 을 주소로 내원했습니다. progressive, new onset & severe pain은 불안정협심증에 합당 한 소견입니다. Harrison 2ᄋ만,pp. 1866~ 1872

Q

심전도에서 ST절 상승을 관칠할 수 있는 질환을 고르시오.(세 가지)

ST segment가 상승하는 경우

① ischemia/MI • variant angina •A M I • postMI : ventricular aneurysm ② acute pericarditis ③ 정상 variation ④ LVH/LBBB 傘ST segment가 하강히는 질환 ① 발작성 협심증{발작시) ② 심내막하 심근경색(endocardial MI) ③ digitalis 투여 : 이것은 digitalis가 효고® 나타내는 증거이지, digitalis를 중단해야 하는 sign은 아님 ④ hypokalemia

■■QuestLaD^M

문제해설 -c/\>

다음 문제에서 가능성이 큰 진단명을 문제마다 지시하는 수만큼 답가지 중에서 고르시오. 1) 급성 심근경색

2)

acute myopericarditis

3) 비대성 심근병증

4)

버거씨병

5) ^코올 심^■근근병증

6)

확장성 심근병증

7) 대동맥 박리

8)

긴장성 기흉

60

[b] - |3

52세 남자가 가슴이 아파 응급실에 왔다. 수축기 혈압 70mmHg,체온 37도, 맥박 105회/분이고 심전도는 다음과 같았다. 환자 혈압이 없어져

심폐소생술을 시행하였다. 진단은?(두 가지)

6 0 실제 문제에서 I ’ 1 ,aVF, V 2~ V 6에 up­ ward concave ST elevation이 있었고 re­ ciprocal change는 확인할 수 없었습니다. EKG소견상 명백한 급성 심막염으로 진단할 수 있습니다. 나머지 한 보기가 애매합니다. 급성심근경색은 전형적인 경우 특징적인 reciprocal change를 보이며 이는 급성 심막염 과의 중요한 감별점입니다. 하지만 ST elevation의 감별진단으로 생각할 수 있는 가장 적절한 것은 급성심근경색 뿐입니다. HCMP에서는 LVH와 prominent q wave가 나타날 수 있고, DCMP에서는 non-specific ST abnormality가 나타날 수 있습니다. 두 EKG abnormality가 모두 주어진 EKG오 卜맞 지 않습니다. 정답은 비교적 찾기 쉬우나,급성심근경색에서 문제와 같은 EKG가 나온 것을 설명하기는 쉽 지 않습니다. 우선 생각해 볼 수 있는 것은 infero-lateral MI가 발생한 상황입니다. Inferolateral MI에서는 대부분의 경우 n, m ,aVF, V5~ V 6에 STE I, aVL, V r~V 4에 reciprocal 이 나타나고 가끔 V2〜 V4에도 STE가 Lf타날 수 있다고 힙니다. Reciprocal change가 없기는 하지만 문제의 EKG와 어느 정도 비슷한 EKG가 생길 수 있 습니다. 다소 무리한 가정이기는 하지만,또 다른 가능성으로 심근경색 후 수일에서부터 6 주간 심막염이 나타나는 드레슬러(Dressier) 증후군이 생겼다고 추정해 볼 수도 있습니다. 이영우 편저,순환기학,p. 270



2020년 대비 PACIFIC KMLE

순골기

문제해설

9 _______________________________________________Q u e s t i o r b A

각 문제에서 가능성이 큰 진단명을 문제마다 지시하는 수만큼 답가지 중 에서 고르시오. 1) pheochromocytoma

2) hypothyroidism

3) hyperthyroidism

4) renal failure

5) RVH

6) white coat hypertension

7) masked hypertension

61

lA :□

: ■■

53세 여자가 건강검진상 혈압 150/100으로 나와 내원하였다. 집에서 잰

혈압은 110/80 ,내원당일 진료실 앞 자동혈압계로 잰 혈압은 115/80, 진료시 혈압은 150/100 일 때 진단은?(한 가지)

61

병원에서 측정할 때만 혈압0 1 높아지는 white coat hypertension입니다. 고혈압(sustained hypertension)만큼은 아니지만 white coat hypertension에서도 target organ damage가 생 길 확률이 증가합니다. 또한 고혈압(sustained hypertension)으로 진행할 위험도 증가합니다. Masked hypertension은 진료실혈압이 정상 이고 환자가 가정에서 자가 측정을 하든지 활 동혈압측정을 통해서 진료실외혈압이 상승되 어 있을 때로 정의하고 있습니다. Harrison 2 0 ^ , p. 1669 Masked hypertension Thomas G. Pickering et al Hypertension 2002

62

a-Q

28세 남자 환자가 심한 두통과 어지러움으로 내원하였다. 최근 자주 두

근거림과 얼굴이 창백해지면서 발한을 보였다. 고혈압의 가족력은 없었 다. 앉아서 잰 혈압이 190/110 이었고, 일어서서 잰 혈압이 150/60 이었 다. 그 다음날 앉아서 잰 혈압은 140/47 이었다. 진단은?(한 가지)

6 2 ■두5 거림,■두# ,빌^은 pheochromocytoma 의 triad입니다. 이 세 가지 증상을 동반한 고 혈압인 경우 pheochromocytoma일 가능성이 높습니다. 이 환자는 변동이 심한 혈압과 기 립성저혈압을 보이고 있어서 전형적인 pheochromocytoma로 진단할 수 니다. Harrison 2Q만,pp.l890~1906

R-Type

___________________________________ Qu,e.s,tiflri>j&

각 문제에서 가능성이 큰 진단명을 문제마다 지시하는 수만큼 답가지 중 에서 고르시오. 1) 본태성 고혈압

2) 대동맥 축착

3)

갈색세포종

4) 다카야수혈관염

5)

쿠싱증후군

6) 만성신부전

7)

갑상선기능항진증

8) 말단비대증

9)

원발성 알도스테론증

Z O B - l: :

° °

5년 전 고혈압을 진단받은 55세 남자가 내원하였다. 어머니가 50세에

뇌출혈로 사망하였다. BMI 28kg/m2, 혈압은 155/110mmHg이었다. 신 체검진에서 이상소견은 없었다. 혈액 검사는 다음과 같다. 위의 보기 중 가장 가능성 있는 진단을 고르시오.(한 가지) Na/K/Q 147/4.0/106, BUN/Cr 26/1.0, 요단백(± )

63

55세에 진단을 받은 고혈압 환자의 증례입니 다. 혈압이 stage I 에 해당되는군요. 비만과 고혈압' 뇌출혈의 가족력 외 신체검진에서 특 이 소견은 없었고. 혈액, 소변 검사 결과도 큰 이상은 없네요. 본태성(일차성) 고혈압의 가 능성이 가장 크다고 할 수 있어요. Harrison 2Q만,pp. 189ᄋ〜 1906

_J____________________________________ 20세 남자가 신검에서 고혈압으로 판정받고 내원하였다. 가족 중에는

고혈압을 가진 사람은 없었다. BMI 22kg/m2, 혈압은 155/11 OmmHg, 맥박수 78회/ 분, 복부잡음은 없었고 혈액 검사는 다음과 같다. 위의 보 기 중 가장 가능성 있는 진단을 고르시오.(한 가지) Na/K/Cl 147/2.5/106

6 4 30세에 우연히 발견된 고혈압 환자입니다. 155/1 lOmmHg이면 stage II 에 해당되는 고 혈압이네요. 가족력 없고,신체검진 상 이상 소견도 없는데,hypokalemia가 동반되어 있 네요. hyperaldosteronism에 합당한 소견이 에요. hyperaldosteronism은 primary와 secondary 로 나눌 수 있는데,후자의 대표적인 질환이 renovascular hypertension 이죠. 복부잡음이 안 들렸으니 이건 배제할 수 있을 것입니다. Harrison 2Q만,pp. 189ᄋ〜 1906

2020년 대비 PACIFIC KMLE

순환기

w

65 〜 66

Q u e s t io n ^ A

문제해설 -c A ,

각 문제에서 가능성이 큰 진단명을 문제마다 지시하는 수만큼 답가지 중 에서 고르시오. 1) 안정협심증

2) 심장눌림증

3) 폐색전증

4) 폐심^■증

5) 승모판협착증

6) 고혈압성 심장병

7) 급성 심근경색증

8) 승모판역류

9) 대동맥판협착증

10) 대동맥판역류

11) 원발성 폐동맥고혈압

12) 대동맥 박리

0 - 0

30세 여자가 심하게 숨이 차고 붉은 거품가래가 나와서 왔다. 상기 증상

은 기침: 라 발열이 있은 후 심해졌다고 한다. 혈압 100/70mmHg, 맥박 은 불규칙하였고 115회 / 분 이 었 호 흡 26회/ 분, 체온 37.5도였다. 양 쪽 폐 아래에서 거품소리(crackle)가 들렸다. 제2심음 후 높은 심음과 심장끝에서 확장기 잡음이 들렸다.(한 가지)

ᅳ 在 8 )0

a-

6 5 제2심음 후 높은 심음이란 MS에서 볼 수 있 는 opening snap을 으|미하는 것으로 보입니 다. 심장끝에서 들리는 확장기 심잡음도 MS 를 人I사하는 징후입니다. 양쪽 폐에서 거품소 리가 들렸다는 것으로 보아 폐부종이 동반되 었음을 알 수 있습니다. 붉은 거품가래가 동 반된 것 또한 폐부종을 시사하는 증상입니다. 이 환자의 경우 기침고ᅡ 발열이 있은 후 심방 세동이 생기고 이로 인해 MS의 증상이 악화 되어 폐부종까지 생긴 것으로 보입니다. MS 에서는 이완기에 심방에서 심실로의 혈류가 충분하지 않은데 심방세동이 생겨 심박수가 빨라지면 이완기가 더욱 짧아져 심방에서 심 실로 혈류가 이동하지 못하여 심방에 쌓이고 심지어는 폐에까지 쌓여 폐부종을 일으키게 되죠. Harrison 2Q만,pp. 1813〜 1818



R-Type

r

66

B l

50세 여자가 하루 전부터 숨이 차서 왔다. 진행 우I암으로 특별한 치료

없이 집에서 요양 중이었다. 혈압 90/70mmHg,맥박 120회/ 분,호흡 26회/ 분,체온 36.9도였다. 호흡음은 정상이었다. 산소포화도는 90%였

다.(두 가지)

0% 6 6 활력징후가 불안정한 환자입니다. 호흡음은 정상인 것으로 보아 LV failure일 가능성은 떨어져 보이네요. 보기를 하나하나 보면서 r/o 해나가 보도록 합시다. 1) 안정협심증을 의심할만한 전형적인 증상, 즉 운동시 심해지는 가슴 통증,쉬면 나아 지는 양상을 제시하고 있지 않는 것으로 보아 가능성이 떨어집니다. 2) 심장눌림증으로 인해 심박출량이 떨어져 혈압이 떨어지고 이에 대한 보상작용으로 맥박이 상승할 수 있습니다. 문제에 제시 된 증상을 모두 설명할 수 있군요. 참고로 cardiac tamponade의 주요 원인에 neoplasm이 포함되어있습니다. 환^의 경우 진행 위암이 전이되어 pericardial effusion을 만들고 이것이 심해진 결과로 car­ diac tamponade가 생겼을 기능성을 생각 해볼 수 있습니다. 3) 폐색전증으로 인해 RV outflow에 저항이 생기게 도ᅵ면 심박출량이 감소하여 혈압이 떨어자고 맥박수가 상승할 수 있습니다. 산 소포화도가 90%로 감소되어있는 것 또한 폐색전증에서 볼 수 있는 징후입니다. 환자 의 경우 진행 위함으로 인한 trousseau syndrome이 동반되었을 가능성을 생각 해볼 수 있습니다. trousseau syndrome 이란 암과 동반되어 venous thrombosis, hypercoagulability가 생기는 현상을 일 컫습니다. 또는 장기간의 요양으로 인한 DVT가 합병되어 pulmomary embolism 이 생겼을 가능성도 있겠네요. 4) 폐심장증은 폐에 생긴 문제가 1 차적인 원 인이 되어 RV failure를 유발하는 경우를 말하죠. 문제에서는 폐에 이상이 있었다는 증거를 찾을 수 없습니다. 5),8),9),10) 판막질환이라면 각각에 합당 한 심청진 소견이 제시되었겠죠. 6) 환자의 기저질환으로 고혈압이 제시되어 있지 않아 가능성이 떨어집니다. 7) 급성 심근경색증으로 의심할만한 전형적 인 병력이 주어지지 않았죠. 11) 원발성 폐동맥 고혈압 환자는 이 문제의 증례처럼 급성으로 호흡곤란을 호소하지 는 않습니다. R type문제는 제시된 단서가 충분하지 않은 이상 이런 식으로 하나씩 r/o해 가면서 접근 하는 방법이 필요합니다. 따라서 보기에 제시 된 질병« 모두 이해하고 계시는 것이 중요 합니다. 잘 생각이 나지 않으시면 시간을 좀 투자하셔서 각 질병을 다시 한번 복습하고 넘 어가시길 권합니다.AA

广 '

2020년대비 PACIFIC KMLE

순환기



一Q u f is t i o n ^ A

문제해설 -0^0

다음 문제에서 가능성이 큰 진단명을 문제마다 지시하는 수만큼 답가지 중에서 고르시오. 1) 급성 심근경색

2) acute myopericarditis

3) 버거씨병

4) 대동맥 박리

5) 알코올 심장근근병증

6) 확장성 심근병증

7) 비대성 심근병증

8) 긴장성 기흉

r —

67

~

있= 1

a -O -0

40세 남자가 부부 싸움 중 갑자기 발생한 흉통을 주소로 내원하였다. 왼

쪽 팔에서 측정한 혈압은 80/60,오른쪽 팔에서 측정한 혈압은 160/ 100이었고 오른쪽 반신마비가 나타났다. 흉부 X선 사진과 심전도 소견

은 다음과 같다. 진단은?(한 가지)

o

O

6 7 심전도는 normal sinus rhythm이었습니다. CPA는 aorta 음영이 아주 크게 확장되어 있는 전형적인 aortic dissection 소견이었습 니다. Harrison 2 0 ^ , pp. 1919〜 1923

R-Type



68〜 69

Q u e s tio n ^

p

문제해설

-